ICAEW Past Questions Answers March 2008 to March 2015 Summary of Financial Accounting-Application Level.pdf

Share Embed Donate


Short Description

Download ICAEW Past Questions Answers March 2008 to March 2015 Summary of Financial Accounting-Application Level.pdf...

Description

Professional Stage – Financial Accounting – March 2008

PROFESSIONAL STAGE (APPLICATION) - FINANCIAL ACCOUNTING OT EXAMINER’S COMMENTS The following table summarises how well candidates answered each syllabus content area. How well* candidates answered each syllabus area

Syllabus area

Number of questions

Well answered

Poorly answered

LO1

3

3

0

LO2

9

9

0

LO3

3

2

1

Total

15

14

1

*If 50% or more of the candidates gave the correct answer, then the question was classified as ‘well answered’.

Item 1 This item tested simple elimination of inter-company trading, the resulting unrealised profit figure in inventory at the period end and goods in transit. This is an area of the syllabus that is tested frequently but is consistent with candidates’ performance in the written test questions in the examination in this area.

© The Institute of Chartered Accountants in England and Wales 2008,

Page 1 of 15

Professional Stage – Financial Accounting – March 2008

MARK PLAN AND EXAMINER’S COMMENTARY The mark plan set out below was that used to mark these questions. Markers are encouraged to use discretion and to award partial marks where a point was either not explained fully or made by implication. More marks are available than could be awarded for each requirement, where indicated. This allows credit to be given for a variety of valid points, which are made by candidates.

Question 1 General comments: this is a trial balance question with the requirement to prepare a balance sheet and income statement. Adjustments are required for the disposal of an item of plant, the recognition of a provision and a related asset and the revaluation of property, as well as other minor adjustments.

(a) Ginger Ltd - Balance Sheet as at 31 December 2007 £ ASSETS Non-current assets Property, plant and equipment ((4,135,000 -1,696,750) + 4,835,000 + (857,000 – 363,400)) (W3) Current assets Inventories (168,000 – 8,500) Trade receivables Other receivables (200,000 + 800,000 (W5)) Cash and cash equivalents

£

7,766,850

159,500 516,500 1,000,000 22,600 1,698,600 9,465,450

Total assets EQUITY AND LIABILITIES Capital and reserves Ordinary share capital Revaluation reserve (470,000 – 6,750(W2)) Retained earnings (W6)

6,000,000 463,250 700,900

Equity

7,164,150

Non-current liabilities Bank loan Current liabilities Trade and other payables (461,300 + 140,000) Provisions (W5)

700,000

601,300 1,000,000 1,601,300

Total equity and liabilities

© The Institute of Chartered Accountants in England and Wales 2008,

9,465,450

Page 2 of 15

Professional Stage – Financial Accounting – March 2008

Ginger Ltd – Income Statement for year ended 31 December 2007 £ Revenue 6,800,000 Cost of sales (W4) (3,286,900) Gross profit

3,513,100

Administrative expenses (W4) Other operating costs (450,000 – 15,000 (W1)+ 200,000 (W5)) Other income

(1,656,750) (635,000) 31,800 1,253,150

Finance cost Profit before tax Income tax expense

(680,000) 573,150 (140,000)

Profit for the period

433,150

Note: Marks will be awarded if items are included in a different line item in the income statement provided that the heading used is appropriate.

W1 Disposal of piece of machinery Machinery – cost

£200,000

Accumulated depreciation

£200,000 x 20% x 3 years = £120,000

Carrying amount at disposal

Proceeds Less: CA Profit on disposal

£200,000 - £120,000 = £80,000

£ 95,000 (80,000) 15,000

W2 Revaluation reserve

Retail units Accumulated depreciation Carrying amount Valuation at 1 Jan 2007 Revaluation reserve

Depreciation on cost

Property £ 400,000 (200,000) 200,000

Land £ 400,000 400,000

535,000 335,000

535,000 135,000

Total £ 800,000

470,000

400,000 x 5% years = 20,000 pa

Depreciation on revalued amount

535,000 x 5% = 26,750 pa

Transfer between reserves for additional depreciation 26,750 – 20,000 = 6,750

© The Institute of Chartered Accountants in England and Wales 2008,

Page 3 of 15

Professional Stage – Financial Accounting – March 2008

W3 Property, plant and equipment Freehold land and buildings

Cost (4,000,000 – 400,000 (W2)) (4,700,000 – 400,000 (W2)) Valuation

Property £ 3,600,000

Land £

535,000

4,300,000 535,000

At 31 December 2007

4,135,000

4,835,000

Accumulated depreciation Less revalued depreciation (W2) Depreciation in year (5%)

1,690,000 (200,000) 206,750

At 31 December 2007

1,696,750

Plant and equipment Cost Add back disposal proceeds Less disposal cost At 31 December 2007 Accumulated depreciation Less disposal (W1) Depreciation in year (20%) At 31 December 2007

962,000 95,000 (200,000) 857,000 312,000 (120,000) 171,400 363,400

W4 Expenses Cost of sales Trial balance Purchases Opening inventory Depreciation (W3) Land & buildings Plant & machinery Write down (9,000 – 500) Closing inventory

Administrative expenses 1,450,000

2,600,000 675,000 206,750 171,400 8,500 (168,000) 3,286,900

1,656,750

W5 Provision Legal claim Counter claim

£1,000,000 – most likely outcome £1,000,000 x 80% = £800,000 – receivable

Income statement effect: £1,000,000 - £800,000 = £200,000

© The Institute of Chartered Accountants in England and Wales 2008,

Page 4 of 15

Professional Stage – Financial Accounting – March 2008

W6 Retained earnings

Trial balance Transfer from revaluation reserve (W2)

Add: Profit for period

£ 261,000 6,750 267,750 433,150 700,900

Almost all candidates produced a well laid out income statement and balance sheet in appropriate formats, although some candidates lost marks by not adding across numbers in brackets or transferring numbers from workings. Presentation marks were lost by failing to complete the totals on the statements. Most candidates were able to take items from the trial balance and insert them in the correct place in the formats, with most candidates using an efficient matrix style costs working. Marks were awarded where presentation differed to the marking guide but resulted in a reasonable alternative. For example, trade receivables were often combined with other receivables and the £450,000 of operating costs was included in administrative expenses. Candidates generally dealt correctly with the inventory adjustments and completed the double entry for the taxation figure. The effect of the legal claim caused candidates a number of problems, such as, calculating the closing provision on a weighted average basis, as opposed to at its most likely outcome, describing the provision as a contingent liability, but still including it in the balance sheet and failing to recognise that the income statement should show the net effect of the provision and the counter claim of £200,000 (£1,000,000 less £800,000). Other common errors included charging depreciation on the land balance as well as the buildings, calculating the revaluation uplift based on cost as opposed to on carrying amounts and forgetting to include any revaluation uplift in the revaluation reserve on the balance sheet. Total possible marks Maximum full marks

© The Institute of Chartered Accountants in England and Wales 2008,

24½ 22

Page 5 of 15

Professional Stage – Financial Accounting – March 2008

Question 2 General comments: this question focuses on the differences between cash accounting and the accrual basis of accounting. The question develops by considering these concepts in relation to a number of revenue transactions, with extracts from the financial statements required. The question is completed by looking at the differences between a cash flow statement under IFRS and UK GAAP. (a)(i) Cash and accruals accounting Cash accounting only considers the cash impact of a transaction. For example, sales are recorded in the period when the buyer pays for the goods. This may not be the same period in which the buyer takes delivery of the goods if delayed payment terms are provided. The accrual basis of accounting instead records transactions in the period in which they occur, not when the related cash flow arises. Sales are instead recorded when the risks and rewards of ownership pass from the seller to the buyer, not when the buyer pays for the goods or services.

(a)(ii) Revenue is recorded when there is an increase in economic benefits during the period. However, revenue can only be recognised when an entity is sufficiently certain that it will be paid for the goods or services and that payment is for a known amount. The accrual basis of accounting is followed with revenue being recognised in the period in which the associated work is undertaken rather than when cash is received. In making the assessment of the timing of the economic benefits an entity should consider when the following conditions have been met: • • • •

The entity has transferred the significant risks and rewards of ownership of the goods to the buyer. The seller no longer has management involvement or effective control over the goods. The amount of revenue and costs can be measured reliably. It is probable that the economic benefits associated with the transaction will flow to the entity.

When an entity has met all the above conditions it recognises the revenue even though payment may still be outstanding. The answers to this part of the question were disappointing with a significant number of candidates failing to appreciate the difference between cash and accrual accounting. The majority of candidates were able to describe the basic features of cash and accruals accounting. A significant minority were able to back their answers up by providing a well thought out example. In (a)(ii) most candidates gained the marks for when revenue can be recognised per IAS 18 but few were able to link this to the Framework principles for the recognition of income. This was mainly due to candidates copying straight from the open book text without relating the information to the question requirements. Total possible marks Maximum full marks

© The Institute of Chartered Accountants in England and Wales 2008,

6½ 6

Page 6 of 15

Professional Stage – Financial Accounting – March 2008

(b) Extracts from the financial statements for the year ended 31 December 2007 (1) Sale of products on delayed payment terms (i) Cash accounting Nothing will be recorded under the cash accounting basis as no cash has been received at the year end. (ii) Accrual basis Extract from income statement Revenue Investment income (900 x 6/12)

£ 14,100 450

Extract from balance sheet £ Trade and other receivables (14,100 + 450) 14,550

(2) Gardening services (i) Cash accounting Extract from income statement £ Revenue 3,000 Extract from balance sheet £ Cash 3,000 (ii) Accrual basis Extract from income statement Revenue (3,000 x 2/3)

£ 2,000

Extract from balance sheet £ Current liabilities Deferred income (3,000 – 2,000)

1,000

(3) Garden design (i) Cash accounting Extract from income statement At 31 December 2007, although some work has been undertaken on the garden design, the customer has not paid anything. Therefore no revenue would be recognised under cash accounting. £ Cost of sales 4,000

© The Institute of Chartered Accountants in England and Wales 2008,

Page 7 of 15

Professional Stage – Financial Accounting – March 2008

(ii) Accrual basis Extract from income statement Revenue (11,000 x 50%)

£ 5,500

Cost of sales ((4,000 + 4,000) x 50%)

(4,000)

Extract from balance sheet Trade and other receivables

£ 5,500

This part of the question asked for the preparation of extracts from the financial statements in relation to three sales arrangements under both cash accounting and IAS 18. It was often impossible to distinguish between which part of the answer was supposed to be cash accounting and which was IAS 18. Candidates obviously struggled with the concept of both cash and accrual accounting when related to a scenario. Answers were poor in this area and often incomplete. Common errors included failing to complete the double entry with the chosen revenue figure, to show the appropriate receivable figure (under accrual accounting), including deferred income as an asset as opposed to a liability, including a full year’s worth of interest of £900 rather than 6 months and recognising all of the revenue of £11,000 rather than only half which represented how much work had been completed. Total possible marks Maximum full marks (c)

9½ 8 Marks

Cash flow statement – differences between IFRS and UK GAAP Under UK GAAP a cash flow statement is based on the movement in cash rather than cash and cash equivalents. Cash equivalents are instead treated as part of management of liquid resources under UK GAAP. Under UK GAAP a cash flow statement has eight headings rather than the three used under IFRS. UK GAAP includes an exemption for the preparation of a cash flow statement for small entities and also in certain situations where you have a group of companies and consolidated financial statements have been prepared which include a group cash flow statement. No such exemptions are included under IFRS. Total possible marks Maximum full marks

3 3

The identification of the differences between UK GAAP and IFRS in relation to the cash flow statements was answered well. The majority of candidates were able to identify that there were a different number of headings under the two accounting regimes although some candidates wasted time by writing out the headings in details, which was not required. The majority of candidates also stated that UK GAAP uses pure cash rather than cash and cash equivalents. Total possible marks Maximum full marks

© The Institute of Chartered Accountants in England and Wales 2008,

19 17

Page 8 of 15

Professional Stage – Financial Accounting – March 2008

Question 3 General comments: this question considers three specific areas of accounting. The first is the acquisition of a subsidiary, with fair value adjustments required, specifically in relation to intangible assets. The second area considers the accounting required for an associate and the effect of an impairment and the final area is the development of a non-current asset. The question is completed by the preparation of a statement of changes in equity. (a) (i) Goodwill – Hop Ltd Fair value of net assets:

Per question (500+18+250) Staff retraining costs Start up costs Contingent liability

Net assets (FV) £ 768,000 (15,000) (85,000) (75,000)

Adjusted FV

593,000

Goodwill calculation Fair value of consideration Share of net assets acquired: 75% x 593,000 Discount on acquisition

£ 400,000 444,750 44,750

(ii) Associate Year end carrying amount Cost Share of post-acq change in net assets 25% x (300,000)

Impairment in year (bal fig) Recoverable amount

£ 200,000 (75,000) 125,000 (45,000) 80,000

Investment in associate at 31 Dec 2007 is £80,000

(iii) Property, plant and equipment Land cost (£1,200,000 + £20,000)

£1,220,000

Building cost

(£1,600,000 – 72,000 + 56,000)

£1,584,000

Depreciation:

(1,584,000 / 30 x (6/12))

£26,400

© The Institute of Chartered Accountants in England and Wales 2008,

Page 9 of 15

Professional Stage – Financial Accounting – March 2008

Head Office £ Cost At 1 January 2007 Additions At 31 December 2007 Depreciation At 1 January 2007 Charge for the year

1,220,000 1,584,000 2,804,000

26,400

At 31 December 2007

26,400

Carrying amount At 31 December 2006 At 31 December 2007

1,220,000 2,777,600

Total possible marks Maximum full marks

© The Institute of Chartered Accountants in England and Wales 2008,

10 10

Page 10 of 15

Professional Stage – Financial Accounting – March 2008

(b) Consolidated statement of changes in equity Attributable to equity holders of Lovage plc Share Share premiu Retained Total capital m earnings Net profit for the year (W2&W3) Added on acquisition of subsidiary (W3) Dividend declared (W4) Share issue (W1)

Minority interest

-

-

208,600

208,600

-

-

-

-

160,000

240,000

(63,000) -

(63,000) 400,000

-

1,615,00 0

240,000

1,760,600

2,515,00 0 3,060,60 0

Brought forward

900,000

Carried forward

1,060,000

Total

33,750

242,350

148,250

148,250

-

182,000

(63,000) 400,000 2,515,00 0 3,242,600

Workings W1 Share issue 160,000 shares issued: Share capital (160,000 x £1) Share premium (160,000 x £1.50)

£ 160,000 240,000 400,000

W2 Net profit Net profit for period: Draft profit Discount on acquisition (from a(i)) Share of associate’s loss (from a(ii)) Impairment – associate (from a(ii)) Overheads (72,000 – 56,000) (from a(iii)) Depreciation (from a(iii)) Relocation costs Hop Ltd’s profit share (75% x £135,000 (W))

£ 425,000 44,750 (75,000) (45,000) (16,000) (26,400) (200,000) 101,250

Profit for period

208,600

(W) Hop Ltd’s post-acquisition profit

Per question Computer software amortisation

£ 150,000 (15,000) 135,000

Computer software

£90,000 / 3 x 6/12months = £15,000

© The Institute of Chartered Accountants in England and Wales 2008,

Page 11 of 15

Professional Stage – Financial Accounting – March 2008

W3 Minority interest Minority interest profit in year £135,000 x 25% = £33,750 Minority share of b/fwd reserves £593,000 (part a(i)) x 25% = £148,250 W4 Dividend Dividend 7p x 900,000 = £63,000 The majority of candidates prepared the necessary proforma for the consolidated statement of changes in equity but were often then unable to populate it. A common omission from the proforma was the total column immediately before the minority interest column. An average candidate correctly inserted the brought forward figures, calculated the share issue split between share capital and premium and inserted a dividend figure, although this was often an incorrect figure. The profit for the period was usually inserted, although unadjusted, and the minority interest figures, including that added on acquisition. Common errors included not making any adjustments to the parent’s draft profit for the year, if adjustments to the parent’s draft profit for the year were made these were often shown on the face of the SOCIE rather than in a working, failing to adjust the subsidiary’s post-acquisition profits for the amortisation of the computer software and failing to add the group’s share of the subsidiary’s postacquisition profits to the parents draft profit for the year. Total possible marks Maximum full marks

© The Institute of Chartered Accountants in England and Wales 2008,

12½ 12

Page 12 of 15

Professional Stage – Financial Accounting – March 2008

Question 4 General comments: this is a consolidation question. A consolidated income statement is required along with the movement on retained earnings. The parent company has an investment in a subsidiary and an associate. Adjustments are required for FV changes from the acquired subsidiary, a write down of inventory and inter-company trading between the associate and parent company. (a) Hyssop plc Consolidated income statement for the year ended 31 December 2007 £’000 Revenue (W2) 38,000 Cost of sales (W2) (24,075) 13,925 (7,300)

Gross profit Operating expenses (W2) Profit from operations Investment income (W2) Share of profits of associate (W7)

6,625 530 960 8,115 (2,400)

Profit before tax Income tax expense (W2)

5,715

Profit for the period Attributable to: Equity holders of Hyssop plc (Bal) Minority interest (W6)

5,540 175 5,715

(b) Consolidated movement on retained earnings

Balance at 31 Dec 2006 Profit for the period Dividends

Retained earnings £’000 1,242 5,540 (1,200)

Balance at 31 Dec 2007

5,582

Workings (All figures in £’000) W1 Group Structure Hyssop 30% Arnica

80% Sorrel

© The Institute of Chartered Accountants in England and Wales 2008,

Page 13 of 15

Professional Stage – Financial Accounting – March 2008

W2 Consolidation schedule Revenue Cost of sales Per question Impairment – inventory (W4) Fair value adj (W3) Operating expenses Per question Impairment loss Investment income Tax PAT

Hyssop 28,500

Sorrel 9,500

(16,800) (50)

(7,200)

Adjustments

Total 38,000 (24,075)

(25) (6,460) (20) 1,150 (1,800)

(7,300)

(820) 20 (600) 875

*(640)

530 (2,400)

* Elimination of intragroup dividend (80% x 200) + (30% x 1,600)

W3 Fair value adjustment £ Fair value 700,000 Carrying amount 525,000 Difference 175,000 Additional annual depreciation £175,000 / 7 years = £25,000 W4 Inventory adjustment Net realisable value - £2.50 – 50p = £2.00 Carrying amount - £2.20 Inventory write down – (£2.20 - £2.00) x 250,000 = £50,000 W5 Unrealised profit - associate £250,000 (£200,000) £50,000

125% 100% 25%

H’s share - £50,000 x 30% = £15,000 W6 Minority interest Sorrel Ltd (20% x 875 (W2)) = 175 W7 Associate Profit for the year Less: Intragroup trading Hyssop’s share x 30% Less: impairment for year Share of associate’s profit

© The Institute of Chartered Accountants in England and Wales 2008,

£ 3,250 (50) 3,200 960 960

Page 14 of 15

Professional Stage – Financial Accounting – March 2008

W8 Consolidated retained earnings brought forward Hyssop plc – per question Sorrel Ltd (80% x (1,580 – 1,350 – (2 x 25))) Impairment - Sorrel Arnica ((30% x (750 – 240)) – 75(imp))

1,070 144 (50) 78 1,242

W9 Consolidated retained earnings carried forward Hyssop plc – per question Inventory write off Sorrel Ltd (80% x (2,280 – 1,350 – (3 x 25))) Impairment (50 + 20) Arnica ((30% x (2,400 – 240 - 50)) – 75(imp))

4,460 (50) 684 (70) 558 5,582

Note – this working is for tutorial purposes and is not required to obtain full marks.

A common failing on this question was time wasting by candidates who prepared standard balance sheet workings which were simply not required. In part a) almost all candidates produced an income statement in the correct format, with a split between the group and the minority interest. However, many could not deal with the more complex adjustments such as the trading between the parent entity and its associate. Candidates answers were strong in preparing the inventory adjustment and the basic consolidation schedule. Common errors in answers included adjusting for the goodwill impairment against the subsidiary’s column in the consolidation schedule rather than against the parent’s column, treating the mark-up on the inter-company sales as a margin and adjusting for the whole unrealised profit rather than only the associate’s share. Other common errors included adjusting for three year’s worth of depreciation, i.e the accumulated depreciation, instead of just one year, and then including the adjustment in the parent’s column instead of the subsidiary’s column and not eliminating the subsidiary’s and associate’s dividends. Part b) required the movement on retained earnings to be shown. This illustrated that many candidates did not appreciate the link between the income statement and the balance sheet. Candidates often wasted time by including a column for the minority interest which was not required. A common mistake was to calculate the brought forward figures using the carried forward figures and to include the group’s share of the subsidiary’s and / or associate’s dividends in the dividends shown as a reduction to retained earnings. Total possible marks Maximum full marks

© The Institute of Chartered Accountants in England and Wales 2008,

20 19

Page 15 of 15

Financial Accounting Professional Stage (New Syllabus)- June 2008

PROFESSIONAL STAGE FINANCIAL ACCOUNTING – OT EXAMINER’S COMMENTS The performance of candidates in the June 2008 objective test section of the Professional Stage Financial Accounting paper was good. Candidates performed well across all syllabus areas, although slightly less well at this session on the preparation of consolidated financial statements than on the other two syllabus areas. Care should always be taken to ensure that the principles underlying any particular item are understood rather than the answer learned from previous experience. In particular, candidates should ensure that they read all items very carefully. The following table summarises how well* candidates answered each syllabus content area. Syllabus area

Number of questions

Well answered

Poorly answered

LO1

4

4

0

LO2

5

4

1

LO3

6

4

2

Total

15

12

3

*If 50% or more of the candidates gave the correct answer, then the question was classified as ‘well answered’. Both of the items which were answered poorly on the preparation of consolidated financial statements (LO3) tested the preparation of consolidated cash flow statements. Item 1 This item tested what amount would be shown in respect of the acquisition of an associate during the year in a consolidated cash flow statement. Most candidates incorrectly concluded that the amount shown would be the net of the cash paid for the associate and the cash and cash equivalents held by the associate at the date of acquisition. It may be that candidates misread the question as asking about the impact of the acquisition of a subsidiary during the year, not an associate. Item 2 This item tested what amount would be shown in respect of dividends paid to minority interest in a consolidated cash flow statement. The information given included opening and closing balances on the minority interest “account”, the profit attributable to the minority interest for the year and details of a new subsidiary acquired during the year. The most commonly selected incorrect answer indicated that candidates ignored the impact of the subsidiary acquired during the year on the minority interest account.

© The Institute of Chartered Accountants in England and Wales 2008

Page 1 of 17

Financial Accounting Professional Stage (New Syllabus)- June 2008

MARK PLAN AND EXAMINER’S COMMENTARY The marking plan set out below was that used to mark this question. Markers were encouraged to use discretion and to award partial marks where a point was either not explained fully or made by implication. More marks were available than could be awarded for each requirement. This allowed credit to be given for a variety of valid points which were made by candidates.

Question 1

Total marks 27

General comments Part (a) tested the preparation of financial statements (in this case an income statement and balance sheet) from a trial balance plus a number of adjustments. Adjustments included write-downs to trade receivables and inventories, a lease of land and buildings, and movements on non-current assets, including an asset held for sale. Part (b) tested differences between the financial statements prepared under IFRS in Part (a) and those which would have been prepared under UK GAAP, including marks for recognising that the lease of land and buildings would be treated differently under the two bases. Thirsk Ltd (a) Income statement for the year ended 31 March 2008

Revenue (1,403,000 – 5,000) Cost of sales (W1) Gross profit Distribution costs (W1) Administrative expenses (W1) Profit from operations Finance cost (W3) Profit before tax Income tax expense Profit for the period

© The Institute of Chartered Accountants in England and Wales 2008

£ 1,398,000 (680,900) 717,100 (286,100) (323,100) 107,900 (3,000) 104,900 (26,500) 78,400

Page 2 of 17

Financial Accounting Professional Stage (New Syllabus)- June 2008

Balance sheet as at 31 March 2008 £ Assets Non-current assets Property, plant and equipment (W2) Current assets Inventories (W1) Trade and other receivables (419,200 – 30,000 – 5,000) Cash and cash equivalents Non-current asset held for sale (W6)

1,299,300

529,000 384,200 15,900 929,100 9,500 938,600 2,237,900

Total assets Equity and liabilities Capital and reserves Ordinary share capital Preference share capital (irredeemable) Retained earnings (W4) Equity Non-current liabilities

500,000 100,000 1,052,000 1,652,000

Finance lease liabilities (W3) Current liabilities Trade and other payables Taxation Dividends payable (W4) Finance lease liabilities (W3) Total equity and liabilities

© The Institute of Chartered Accountants in England and Wales 2008

£

195,500

348,900 26,500 5,000 10,000 390,400 2,237,900

Page 3 of 17

Financial Accounting Professional Stage (New Syllabus)- June 2008

Workings (1) Allocation of expenses Cost of sales £ Per Q Movement on bad debt provision (36,000 – 30,000) Opening inventories Adj re lease (20,000 x ½) Closing inventories (525,000 + 4,000) Depreciation/impairment charges (W5)

675,400

Administrative expenses £ 316,600 (6,000)

Distribution costs £ 286,100

415,000 (10,000) (529,000) 119,500 680,900

22,500 323,100

286,100

(2) Property, plant and equipment

Cost b/f Leased building (W3) Machine held for sale (31 December 2007) Acc dep b/f Eliminated on machine held for sale (W6) Depreciation/impairment charges for year (W5)

Land and buildings £ 1,300,000 212,500 (280,000) (22,500)

Plant and equipment £ 545,000 (50,000) (326,700) 40,500 (119,500)

Total

1,210,000

89,300

1,299,300

£

(3) Lease of buildings SOD = (24 x 25)/2 = 300 £ 250,000 (212,500) 37,500

Total payments (20,000 x ½ x 25) Fair value (425,000 x ½) Finance charge Year ended 31 March

B/f

Payment

Capital

Interest

C/f

2008

£ 212,500

£ (10,000)

£ 202,500

£ 205,500

2009

205,500

(10,000)

195,500

£ (24/300 x 37,500) 3,000 (23/300 x 37,500) 2,875

© The Institute of Chartered Accountants in England and Wales 2008

198,375

Page 4 of 17

Financial Accounting Professional Stage (New Syllabus)- June 2008

(4) Retained earnings £ At 1 April 2007 Preference dividend (100,000 x 5%) Profit for the period At 30 September 2007

978,600 (5,000) 78,400 1,052,000

(5) Depreciation and impairment charges Existing buildings ((1,300,000 – 600,000) ÷ 50) Leased building (212,500 ÷ 25)

Plant held throughout year ((545,000 – 50,000) x 20%) On plant held for sale (13,000 + 7,500) (W6)

£ 14,000 8,500 22,500 99,000 20,500 119,500

(6) Plant held for sale Cost Acc dep to 31 March 2007 (50,000 x 20% x 2) Dep for year (50,000 x 20% x 9/12) Carrying amount at classification as held for sale Fair value less costs to sell (10,000 – 500) Impairment

© The Institute of Chartered Accountants in England and Wales 2008

£ 50,000 (20,000) (7,500) 22,500 (9,500) 13,000

Page 5 of 17

Financial Accounting Professional Stage (New Syllabus)- June 2008

As in previous sittings, candidates were clearly well-prepared for this type of question. Almost all candidates produced a well-laid out income statement and balance sheet in appropriate formats, although some lost presentation marks by not adding across numbers in brackets or transferring numbers from workings. Others lost presentation marks by failing to complete the sub-totals and/or totals on their statements or by having incomplete or abbreviated narrative or no heading. Candidates should remember that this type of question requires financial statements to be in a form suitable for publication. Although many workings, in particular the cost matrix, were clearly laid out some candidates’ workings were disorganised, untidy and therefore hard to follow, making it difficult to establish candidates’ approaches where they had not calculated the correct figure. Most candidates were able to deal with the more straightforward adjustments such as the increase in the bad debt provision, the inventory write-down, the income tax charge/liability and the basic annual depreciation charges. Most correctly classified the irredeemable preference shares as equity but not all treated the dividends as such, instead including them as a finance cost. Errors in dealing with the other adjustments included the following: • Failing to split the lease into a finance lease for the building and an operating lease for the land. • Depreciating the leased buildings over 50 years (the useful life for other buildings) instead of 25 years (the lease term) or not depreciating them at all. • Arriving at the incorrect sum of the digits figure. • Although candidates often correctly calculated the impairment on the plant held for sale few followed this through fully by charging it to the income statement, including it in the depreciation/impairment charge for the year in the property, plant and equipment working and taking it out of that working when adjusting for the asset held for sale. • Failing to disclose the asset held for sale correctly on the balance sheet (within current assets after a separate sub-total for all other current assets). • Treating the debit cash balance as an overdraft. • Showing the impairment amount as a liability rather than as a deduction from receivables. • Failing to properly account for the returned goods by reducing the revenue figure but not making the corresponding entry to receivables. However, most candidates correctly adjusted closing inventory for the cost of these goods. Total possible marks Maximum full marks

25 23

(b) Differences between IFRS and UK GAAP Under UK GAAP, where formats are laid down by the Companies Act, the terminology in the balance sheet would differ. For example, non-current assets would be referred to as “fixed assets”, trade and other receivables would become “trade and other debtors”. Under UK GAAP, the balance sheet would also usually be prepared on a net assets basis whereby the “top half” of the balance sheet would show all assets and liabilities and the “bottom half” would show just equity (capital and reserves) although other formats are in fact permissible. Under UK GAAP, the income statement would be referred to as a “profit and loss account” and a sub-total for operating profit would be required (FRS 3). Although this is not prohibited by IAS 1 it is not explicitly required. Specifically with regard to Thirsk Ltd, a difference between an IFRS and a UK GAAP balance sheet might arise with regard to the treatment of the lease of land and buildings. Under IFRS (IAS 17) there is a specific requirement to split leases of land and buildings at inception into a separate lease of land and a separate lease of buildings. These should be classified appropriately, although, usually, the lease of land will be an operating lease, and the lease of buildings, a finance lease. Under UK GAAP (SSAP 21) the lease of land and buildings is considered as one lease and is usually treated as an operating lease. Under UK GAAP therefore the whole of the £20,000 would have been correctly taken to administrative expenses as operating lease rentals.

© The Institute of Chartered Accountants in England and Wales 2008

Page 6 of 17

Financial Accounting Professional Stage (New Syllabus)- June 2008

Answers to Part (b) were mixed. Almost all candidates were able to state that the terminology would differ and to give an example of this, but many answers did not go beyond this. Common failings included the following: • • • • •



Stating that the UK’s version of the income statement is called a “profit and loss” as opposed to a “profit and loss account”. Stating that a UK balance sheet is prepared on a “net assets” basis but being unable to explain this. Giving differences between a UK cash flow statement and a cash flow statement prepared in accordance with IAS 7 when the question was clearly restricted to differences between IAS 1 and UK GAAP. Giving differences between IFRS and UK GAAP in respect of the preparation of consolidated financial statements when the question was clearly restricted to single entity financial statements. A number of candidates referred to the UK having a statement of total recognised gains and losses instead of a statement of changes in equity (although many referred to this as a “STRGL” with no indication that they knew what this stood for) but very few mentioned the reconciliation of movements in shareholders’ funds. Failing to spot that the treatment of the lease of land and buildings would differ under UK GAAP. Of those who did spot this, a number got the treatment the wrong way round.

Total possible marks Maximum full marks

© The Institute of Chartered Accountants in England and Wales 2008

5 4

Page 7 of 17

Financial Accounting Professional Stage (New Syllabus)- June 2008

Question 2

Total marks 16

General comments This question tested the preparation of a single company cash flow statement and supporting note. Missing figures to be calculated included interest paid, tax paid, dividends paid, property, plant and equipment acquired and proceeds from the issue of share capital. A bonus issue of shares and a revaluation of property, plant and equipment during the year also featured. Wetherby plc Cash flow statement for the year ended 31 March 2008 £ Cash flows from operating activities Cash generated from operations (Note) Interest paid (W1) Income tax paid (W2) Net cash from operating activities Cash flows from investing activities Purchase of property, plant and equipment (W3) Proceeds from sales of property, plant and equipment (1,356,000 + 200,500) Net cash used in investing activities Cash flows from financing activities Proceeds from issue of ordinary share capital (500,000 (W4) + 1,650,000 (W5)) Proceeds from issue of borrowings Dividends paid (W7) Net cash from financing activities Net increase in cash and cash equivalents Cash and cash equivalents at beginning of period Cash and cash equivalents at end of period

£

6,341,500 (30,600) (1,789,000) 4,521,900 (7,052,100) 1,556,500 (5,495,600) 2,150,000 50,000 (898,200) 1,301,800 328,100 352,500 680,600

Note: Reconciliation of profit before tax to cash generated from operations Profit before tax Finance cost Depreciation charge Profit on disposal of property, plant and equipment Increase in inventories (1,567,800 – 1,479,600) Increase in trade and other receivables (540,000 – 356,000) Decrease in trade and other payables ((1,678,500 – 4,000) – (1,546,600 – 5,000)) Cash generated from operations

© The Institute of Chartered Accountants in England and Wales 2008

£ 3,355,500 31,600 3,560,000 (200,500) (88,200) (184,000) (132,900) 6,341,500

Page 8 of 17

Financial Accounting Professional Stage (New Syllabus)- June 2008

Workings (1) Interest paid

Cash (β) C/d

£ 30,600 5,000 35,600

B/d IS

£ 4,000 31,600 35,600

£ 1,789,000 1,450,000 3,239,000

B/d IS

£ 1,670,000 1,569,000 3,239,000

(2) Tax paid

Cash (β) C/d

(3) PPE

B/d Revaluation reserve (1,560,000 + 270,000) Additions (β)

£ 15,299,900 1,830,000 7,052,100 24,182,000

Disposal Depreciation C/d

£ 1,356,000 3,560,000 19,266,000 24,182,000

(4) Share capital £

C/d

6,000,000 6,000,000

B/d Share premium (bonus issue) Cash (β)

£ 5,000,000 500,000 500,000 6,000,000

(5) Share premium

Share capital (bonus issue) (W4) C/d

£ 500,000 1,750,000 2,250,000

B/d Cash (β)

£ 600,000 1,650,000 2,250,000

Note: The bonus issue could be taken out of retained earnings since the reserve to be used is not specified by the question. (6) Retained earnings

Dividends in SCE (β) C/d

£ 748,200 8,997,800 9,746,000

B/d Revaluation reserve IS

© The Institute of Chartered Accountants in England and Wales 2008

£ 7,689,500 270,000 1,786,500 9,746,000

Page 9 of 17

Financial Accounting Professional Stage (New Syllabus)- June 2008

(7) Dividends paid £ £ 898,200 B/d 400,000 250,000 SCE (W6) 748,200 1,148,200 1,148,200 This was the first time this topic had been set and candidates were clearly very well prepared for it. Many scored high marks on the reconciliation note, and on the figures for tax paid, interest paid, proceeds from the issue of borrowings, proceeds from the sale of property, plant and equipment and the opening and closing figures for cash and cash equivalents. Presentation was generally good. Most candidates produced workings in the form of T accounts and very few made the mistake of putting opening and closing balances on the wrong side of those T accounts. Generally, candidates showed a good grasp of basic double entry principles, which underpin the preparation of a cash flow statement, whether single company or consolidated. Cash (β) C/d

Where errors were made they included the following: • Failing to adjust the trade payables figure for the opening and closing accrued interest. • Adding the profit on disposal of property, plant and equipment in the reconciliation note instead of deducting it. • Treating the £50,000 proceeds from the issue of borrowings as a cash outflow rather than as a cash inflow. • Failing to account for the bonus issue (or putting through the credit entry but no debit entry – particularly common where candidates combined the share capital and share premium accounts). • Accounting for the reserve transfer in the retained earnings T account but omitting the other side of the entry from the property, plant and equipment T account (or via a revaluation reserve T account). • Failing to adjust dividends for the year as calculated in a retained earnings T account for opening and closing dividends payable in order to arrive at dividends paid. Total possible marks Maximum full marks

© The Institute of Chartered Accountants in England and Wales 2008

16 16

Page 10 of 17

Financial Accounting Professional Stage (New Syllabus)- June 2008

Question 3

Total marks 11

General comments Part (a) required the preparation of a statement of changes in equity for a single company, featuring an adjustment for interest on redeemable preference shares, a revaluation during the year and a share issue. In Part (b) the resultant profit figure had to be combined with figures for a subsidiary acquired during the year and an associate to generate the consolidated profit figure, split between the group and the minority. In arriving at this figure there was also an adjustment to be made for the unrealised profit on the sale of an item of plant within the group. Doncaster plc (a) Statement of changes in equity for the year ended 31 March 2008 Ordinary share capital Recognised directly in equity Revaluation of non-current assets (650,000 – (456,000 – 215,500)) Transfer between reserves (67,800 – 45,600) Profit for the period (526,700 – (1,000,000 x 5%)) Total recognised income and expense for the period Issue of share capital Balance brought forward Balance carried forward

Share premium

Revaluation reserve

Retained earnings

£

£

£

£





409,500







(22,200)

22,200







476,700





387,300

498,900

250,000 250,000 1,000,000 1,250,000

300,000 300,000 – 300,000

– 387,300 – 387,300

– 498,900 2,365,500 2,864,400

This part of the question was very well-answered. Statements of changes in equity were well laid out with, as instructed, no total column, although very few candidates drew a sub-total of “total recognised income and expense for the period”. Where errors were made they included the following: • Failing to adjust the profit for the year by the finance cost on the redeemable preference shares. Some showed this as dividends in the statement, others showed it separately as a finance cost, when it would not appear separately in the statement. • Calculating the revaluation gain arising in the year incorrectly, based on the cost of the asset being revalued as opposed to its carrying amount. • Making an incorrect adjustment for the transfer in respect of the additional depreciation on the revalued asset, when the relevant figures were given in the question. Total possible marks Maximum full marks

© The Institute of Chartered Accountants in England and Wales 2008

5 5

Page 11 of 17

Financial Accounting Professional Stage (New Syllabus)- June 2008

(b) Profit attributable for the year ended 31 March 2008

Net profit per own accounts (a) (457,000 x 6/12) Share of associate’s profits (103,400 x 45%) Non-current asset PURP (40,000 – (30,000 x 4/5)) Additional depreciation on non-current asset (((40,000 ÷ 4) – (30,000 ÷ 5)) x ½) Impairments (50,000 + 10,000) Profit attributable to the minority interest (214,500 x 25%) Profit attributable to the equity holders of Doncaster plc = 624,105

Doncaster plc £ 476,700 46,530 – –

Redcar Ltd £ 228,500 – (16,000) 2,000

(60,000) 463,230 –

– 214,500 (53,625)

463,230

160,875

Candidates continue to struggle with extracts type questions. Layouts were disorganised and difficult to follow which at times made it difficult to award credit. In spite of the fact that the status of the investments (as subsidiary or associate) and the percentage shareholdings were given in the question, the vast majority of candidates wasted time drawing up a group structure diagram for which no marks were available. This indicates candidates’ over-reliance on learnt techniques. Common errors included the following: • Failing to adjust the profit of the subsidiary to allow for the fact that it was acquired half way through the year. • Taking minority interest at the wrong point (for example, correctly calculating the subsidiary’s profit for the year as half of the figure for the whole year but then failing to apply the minority percentage to the net figure). • Although a number of candidates correctly calculated the net adjustment in respect of the plant sold within the group, far less correctly made this adjustment against the subsidiary. Of those who did, less still then calculated the minority interest (see above) on this net figure. • When calculating the additional depreciation on the plant sold within the group a number of candidates failed to base this adjustment on a half year’s charge (as the subsidiary was acquired half way through the year and the transfer made on this date). • A disappointing number of candidates omitted to add the parent’s share of the associate’s profit into the profit attributable to the equity shareholders of the parent. Others omitted to add in the parent’s own profit figure. • A significant minority of candidates calculated a minority interest for the associate as well as for the subsidiary. • Reducing the subsidiary’s and the associate’s profits by the given impairments instead of reducing the parent’s profits. Where candidates did this they often dealt correctly with similar impairments in question four, illustrating yet again that candidates cope less well with topics tested in a less familiar way. Total possible marks Maximum full marks

© The Institute of Chartered Accountants in England and Wales 2008

6 6

Page 12 of 17

Financial Accounting Professional Stage (New Syllabus)- June 2008

Question 4

Total marks 26

General comments Part (a) was a consolidated balance sheet question, featuring one subsidiary and one associate. Adjustments were typical of this type of question and included a fair value adjustment on acquisition, intragroup balances and transactions and impairment write-downs. Part (b) tested an understanding of the concepts underlying the preparation of consolidated financial statements: namely the single entity concept and control versus ownership. York plc (a) Consolidated balance sheet as at 31 March 2008 £ Assets Non-current assets Property, plant and equipment (3,963,900 + 1,686,900) Intangibles (W3) Investments in associates (W7) Current assets Inventories (860,000 + 650,000 – 35,000 (W6) – 30,000 x 40% (W6)) Trade and other receivables (730,000 + 540,000 – 210,000 (W6)) Cash and cash equivalents (29,600 + 15,500)

5,650,800 192,000 261,920 6,104,720 1,463,000 1,060,000 45,100 2,568,100 8,672,820

Total assets Equity and liabilities Capital and reserves Ordinary share capital Share premium account Retained earnings (W5) Attributable to the equity holders of York plc Minority interest (W4) Equity Current liabilities Trade and other payables (878,000 + 546,600 – 210,000 (W6)) Taxation (380,000 + 240,000) Total equity and liabilities

© The Institute of Chartered Accountants in England and Wales 2008

£

2,000,000 1,000,000 3,424,060 6,424,060 414,160 6,838,220 1,214,600 620,000 1,834,600 8,672,820

Page 13 of 17

Financial Accounting Professional Stage (New Syllabus)- June 2008

Workings (1) Group structure

200 York plc

800

500

= 40%

= 80%

1,000

Ripon Ltd

Beverley Ltd

(2) Net assets – Ripon Ltd Balance sheet date Share capital Share premium Retained earnings Per Q PURP (W6) Amortisation adj – intangible FV adj – intangible

Acquisition

Post acq

£ 1,000,000 500,000

£ 1,000,000 500,000

£ -

625,800 (35,000) 30,000 (50,000) 2,070,800

(215,000) (50,000) 1,235,000

835,800 835,800

(3) Goodwill – Ripon Ltd

Cost of investment ((1,000,000 x £1.20) + 400,000) Less Share of FV of net assets acquired (1,235,000 (W2) x 80%) Impairments to date (400,000 + 20,000)

© The Institute of Chartered Accountants in England and Wales 2008

£ 1,600,000 (988,000) 612,000 (420,000) 192,000

Page 14 of 17

Financial Accounting Professional Stage (New Syllabus)- June 2008

(4) Minority interest – Ripon Ltd £ 414,160

Share of net assets (2,070,800 (W2) x 20%) (5) Retained earnings York plc Ripon Ltd (835,800 (W2) x 80%) Beverley Ltd ((210,800 – 56,000 – 30,000 (W6)) x 40%)) Less Impairments to date (420,000 (W3) + 50,000)

3,175,500 668,640 49,920 (470,000) 3,424,060

(6) PURP

SP (210,000/180,000 x ½) Cost (140,000/120,000 x ½) GP

%

Ripon Ltd £

Beverley Ltd £

150 (100) 50

105,000 (70,000) 35,000

90,000 (60,000) 30,000

(7) Investments in associates – Beverley Ltd Cost (200,000 x £1.25) Add: Share of post acquisition increase in net assets ((210,800 – 56,000)) x 40%)) Less: Impairment to date

£ 250,000 61,920 (50,000) 261,920

Note: Candidates who correctly calculated a discount on acquisition of the associate of £12,400 and dealt with it appropriately were also given credit.

© The Institute of Chartered Accountants in England and Wales 2008

Page 15 of 17

Financial Accounting Professional Stage (New Syllabus)- June 2008

Candidates were clearly very well prepared for this question and generally scored highly. Almost all candidates demonstrated a sound technique and most dealt easily with the required adjustments. Errors included the following: • • • • • • •





Failing to adjust both receivables and payables for the invoice value of the sale of goods from the subsidiary to the parent, with a number of candidates making the adjustment at cost. Calculating unrealised profit based on the full invoice value, as opposed to only half of that value, when the question clearly stated that only half of the goods remained in year-end inventory. Taking the cost figures given in the question for the intra-group sale as being the selling price of the goods and hence calculating an incorrect provision for unrealised profit. Treating the pre-acquisition loss of the subsidiary as a pre-acquisition profit. Failing to include the share premium account correctly in the net assets table for the subsidiary. Not adjusting for the accumulated impairments in the group retained earnings working, instead adjusting only for the impairments which had arisen during the current year. Pleasingly, many candidates correctly adjusted for the group share of the provision for unrealised profit arising on goods sold by the associate to the parent against retained earnings and inventory, but many also made an adjustment against the carrying amount of the associate. Others calculated an initial post-acquisition profit figure for the associate less a 40% share of the provision for unrealised profit but then adjusted that total by 40%, consequently scaling down the provision for unrealised profit twice. Only a minority of candidates correctly dealt with the goodwill in the subsidiary’s own books. Others took £20,000 out of the subsidiary’s net assets at both acquisition and at the balance sheet date (instead of £50,000 out at acquisition and £20,000 at the balance sheet date) and some were clearly confused between this goodwill and that arising on consolidation. When calculating the cost of investment in the subsidiary a significant number of candidates failed to allow for an issue price of £1.20 per share, as given in the question, and instead assumed that the shares were issued at par.

A number of candidates failed to provide workings for assets and liabilities on the face of the consolidated balance sheet. Where these balance sheet figures were incorrect no partial marks could then be awarded. Candidates must show their workings in all cases so that partial credit can be given. A number of candidates also failed to complete the consolidated balance sheet, but rather abbreviated line items and/or included partial workings, which were not totalled, although this had improved since a similar question was last set. As the question required the preparation of a consolidated balance sheet, candidates are expected to complete all additions and present a complete balance sheet. Very few candidates gained the presentation marks which were available for clearly disclosing the minority interest as a separate component of equity. Total possible marks Maximum full marks

© The Institute of Chartered Accountants in England and Wales 2008

21 21

Page 16 of 17

Financial Accounting Professional Stage (New Syllabus)- June 2008 (b) Concepts underlying preparation of consolidated financial statements Group accounts are prepared on the basis that the group is a single entity (single entity concept). This reflects the substance of the group arrangement. For example, in the consolidation of the York plc group, all assets and liabilities are added together, as if the group were a single entity (so, for example, trade receivables of £730,000 and £540,000 are added). However, the single entity concept also means that any intra-group transactions and balances need to be eliminated, as otherwise items would be double counted in the context of the group as a single entity. Hence, because Ripon Ltd has sold goods for £210,000 to York plc, that amount needs to be subtracted from York plc’s cost of sales and from Ripon plc’s revenue as if the group were a single entity that transaction would not have occurred. That adjustment cannot be seen in the context of the preparation of a consolidated balance sheet, though it would be seen in the preparation of a consolidated income statement. In the context of the consolidated balance sheet any related intra-group balances need to be eliminated. This amount is included in York plc’s trade payables and Ripon Ltd’s trade receivables as this amount is unpaid at the year end. It needs to be eliminated from both. Any profit made between parent and subsidiary companies also needs to be eliminated where that profit has not yet been realised outside the group. So, for the £210,000 intra-group sale, because half of these goods have not yet been sold outside the group, inventory needs to be reduced by the profit on half that amount, otherwise inventory will be overstated from the point of view of the group as a whole. The adjustment effectively brings inventory back down to what it would have been stated at if the intra-group sale has never taken place. The other principle underlying the preparation of consolidated financial statements is the distinction between control and ownership. Control is reflected by including all of the subsidiary’s assets, liabilities, income and expenses in the consolidated financial statements, even where the parent does not own 100% of that subsidiary. So, for York plc, 100% of Ripon Ltd’s inventories of £650,000 are added in even though, in effect, York plc only owns 80% of those inventories. Ownership is then reflected by showing that part of the subsidiary’s net assets and results included in the consolidation, which is not owned by the parent, as a minority interest. York plc’s consolidated balance sheet shows a minority interest of £414,160, representing that part of Ripon Ltd not owned by York plc. Where an investor (York plc) does not have control but does have significant influence over an investee (Beverley Ltd), line-by-line consolidation is not appropriate, because York plc cannot determine Beverley Ltd’s assets and liabilities. But because York plc has this influence, it should be accountable for the total investment in Beverley Ltd, ie cost plus share of post-acquisition retained earnings (the latter are added to group earnings). As with previous papers, the quality of written answers was disappointing. In common with the other written part of this paper, some candidates made no attempt at this part. Whilst most candidates were able to pick up marks for referring to the single entity concept and substance over form few got beyond this. Almost no candidates scored 5 or even 4 marks on this part of the question, in spite of the number of marks available. Many wasted time discussing the techniques used for consolidating financial statements and/or the factors which might indicate control or significant influence. A number thought the question was about the qualitative characteristics of financial information and based their answer around those or discussed the advantages of consolidated financial statements. Although some of these approaches enabled candidates to pick up the odd extra mark scores were generally low. Very few candidates related their answer to York plc as specified in the requirement. Total possible marks Maximum full marks

© The Institute of Chartered Accountants in England and Wales 2008

8½ 5

Page 17 of 17

Professional Stage – Financial Accounting – September 2008

PROFESSIONAL STAGE FINANCIAL ACCOUNTING – OT EXAMINERS’ COMMENTS

The performance of candidates in the September 2008 objective test questions section for the Application Stage Financial Accounting paper was good. Candidates performed well across all syllabus areas. However, care should always be taken to ensure that the principles underlying any particular item are understood rather than the answer learned from previous experience. In particular, candidates should ensure that they read all items very carefully. The following table summarises how well* candidates answered each syllabus content area. Syllabus area Number of questions Well answered Poorly answered LO1

4

3

1

LO2

5

3

2

LO3

6

5

1

Total

15

11

4

*If 50% or more of the candidates gave the correct answer, then the question was classified as ‘well answered’. The poorly answered questions covered a range of different syllabus areas. Comments on two items of particular note are: Item 1 This item tested the impact that a restoration obligation has on the asset and provisions. There appeared to be a lack of knowledge in this area with candidates either not realising that the provision is added to the cost of the asset or believing that a provision is built up over time. Item 2 This item required a calculation of the minority interest dividend in relation to a consolidated cash flow statement. A subsidiary, with a minority interest, had been acquired during the period; however candidates failed to adjust for this in their workings.

© The Institute of Chartered Accountants in England and Wales 2008

Page 1 of 14

Professional Stage – Financial Accounting – September 2008

MARK PLAN AND EXAMINER’S COMMENTARY The mark plan set out below was that used to mark these questions. Markers are encouraged to use discretion and to award partial marks where a point was either not explained fully or made by implication. More marks are available than could be awarded for each requirement, where indicated. This allows credit to be given for a variety of valid points, which are made by candidates.

Question 1 - Total marks 19 General comments: This wqs a typical question testing the preparation of an income statement and statement of changes in equity from a trial balance plus a number of adjustments. Adjustments included a held for sale asset, the correction of an error, a reorganisation and a share issue.

Agate Ltd – Income Statement for year ended 30 June 2008 £ Revenue (W1) 1,907,500 Cost of sales (W2) (688,250) Gross profit

1,219,250

Administrative expenses (W2) Distribution costs (W2) Net operating costs (6,700 – 30,400)

(1,036,750) (145,000) (23,700) 13,800

Finance costs (19,300 + (30,000 x 5%)) Profit before tax Taxation (5,000 + 2,000)

(20,800) (7,000) (7,000)

Net loss for the period

(14,000)

Note: Marks will be awarded if items are included in a different line item in the income statement provided that the heading used is appropriate. Statement of changes in equity for the year ended 30 June 2008 Ordinary share capital Loss for the period Final dividend ((31,000 + 75,000) x 7p)) Issue of shares (75,000 x £1 / £1.30) Revaluation of noncurrent assets (900,000 – 729,000)

Balance b/fwd As previously stated Correction of error Balance c/fwd

Share premium

Retained earnings

Reval. reserve

Total

-

-

(14,000) (7,420)

-

(14,000) (7,420)

75,000

97,500

-

-

172,500

-

-

-

171,000

171,000

75,000

97,500

(21,420)

171,000

322,080

31,000 -

15,000 -

22,500 (7,350)

-

68,500 (7,350)

106,000

112,500

(6,270)

171,000

383,230

© The Institute of Chartered Accountants in England and Wales 2008

Page 2 of 14

Professional Stage – Financial Accounting – September 2008

W1 Revenue Trial balance Advertising adjustment (30,000 x 14/24 months)

1,925,000 (17,500) 1,907,500

W2 Allocation of expenses Cost of sales

Per question: Purchases Manufacturing costs Opening inventory Impairment (W4) Depreciation (W3) Less: closing inventory Relocation expenses (1,500,000 – 700,000) Patent amortisation (45,000 / 5yrs x 9/12)

Administrative expenses

Distribution costs

230,000

145,000

367,000 291,300 17,000 7,500 24,950 (19,500) 800,000 6,750 688,250

1,036,750

145,000

W3 Plant and equipment Depreciation charge for the year Less: IFRS 5 – classified as held for sale (75,000 x 10% x 6/12)

28,700 (3,750)

As at 30 June 2008

24,950

W4 Held for sale asset Carrying amount

45,000

Fair value Less: costs

39,000 (1,500) 37,500

Impairment (45,000 – 37,500)

7,500

Consistent with previous sittings candidates were well-prepared for this type of question and generally answered it well. This question was slightly different in its requirements compared with previous papers as the preparation of a balance sheet was replaced by the statement of changes in equity. As a result of this change candidates found this more challenging compared with previous sittings. Almost all candidates produced a well laid out income statement. The format of the statement of changes in equity was slightly more varied and often had no total column. A common omission was the date for one or both of the statements. A significant minority of candidates prepared a balance sheet as they clearly had not read the requirement. Candidates should be reminded that question requirements may vary from paper to paper. Most candidates were able to take items from the trial balance and insert them in the correct place in the formats, with most candidates using an efficient matrix style costs working. Marks were awarded where presentation differed to the marking guide but resulted in a reasonable alternative. For example, the patent amortisation being included in costs of sales rather than administrative expenses.

© The Institute of Chartered Accountants in England and Wales 2008

Page 3 of 14

Professional Stage – Financial Accounting – September 2008

Candidates generally dealt with the share issue and the dividend correctly in the statement of changes in equity, although the dividend payment was occasionally also incorrectly shown in the income statement. Common errors included a wrong adjustment to revenue, not appreciating that the amortisation of the patent should have been for less than a year, failing to deal with the prior period error, missing that the coupon on the redeemable preference shares should have been included as part of finance costs and incorrectly calculating the relocation expenses. Total possible marks Maximum full marks

© The Institute of Chartered Accountants in England and Wales 2008

19½ 19

Page 4 of 14

Professional Stage – Financial Accounting – September 2008

Question 2 – Total marks 28 General comments: This question represented a combination of preparing extracts from the financial statements (in this case from both the income statement and balance sheet) together with a written conceptual part (b) and part (c) on the differences between IFRS and UK GAAP.

(a) Extract from income statement for year ended 30 June 2008 £ Operating expenses to include: Depreciation (W7) Management expenses Loss on disposal (W3)

91,200 72,000 4,500

Finance charge (W6)

3,500

Extract from balance sheet as at 30 June 2008 £ Non-current assets Property, plant and equipment (W7)

2,123,300

Non-current liabilities Finance lease liabilities (W6)

27,500

Current liabilities Finance lease liabilities (W6)

4,000

W1 New building Architects fees Legal costs Project management fees Building costs

£ 23,000 7,000 30,000 375,000

Less: lift cost

435,000 (15,000)

As at 1 May 2008

420,000

W2 Plant and machinery Cost

B/fwd Less: scrapped machine Additions - leased plant (W6)

£ 160,000 (18,000) 60,000 202,000

Lift (W1) Scrapped machine (W3) Dep. charge in year C/fwd

15,000

Dep. charge £ X 15% x 5 yrs

X 15% / 10yrs x 2/12 18,000 x 15% x 6/12

© The Institute of Chartered Accountants in England and Wales 2008

£ 48,000 (13,500)

30,300 250 1,350 31,900

217,000

Acc. Dep.

31,900 66,400

Page 5 of 14

Professional Stage – Financial Accounting – September 2008

Carrying amount at 30 June 2008 (217,000 – 66,400) = £150,600 W3 Scrapped machine £ 18,000 (13,500)

Cost Accumulated depreciation (W2) Carrying amount at disposal / loss on disposal

4,500

W4 Fixtures and fittings Cost

B/fwd Less: accumulated depreciation

Dep. charge £

£ 75,000 (36,000)

Carrying amount

39,000

B/fwd - cost Additions: 1 Oct 2007

75,000 20,000

X 25%

9,750

X 25% x 9/12

3,750 13,500

C/fwd

95,000

Acc. dep £ 36,000

13,500 49,500

Carrying amount at 30 June 2008 (95,000 – 49,500) = £45,500 W5 Land and buildings

Land Cost Buildings B/fwd Depreciation (1,075,000 x 4%) Additions (dep 420,000 x 4% x 2/12) Dep. charge in year C/fwd

Cost

Dep. charge

£

£

Accumulated depreciation £

650,000

1,075,000

420,000

172,000 43,000 2,800 45,800

2,145,000

45,800 217,800

Carrying amount at 30 June 2008 (2,145,000 – 217,800) = £1,927,200 W6 Leased asset Deposit Instalments (7,000 x 7)

Fair value of asset Finance charges

£ 25,000 49,000 74,000 (60,000) 14,000

© The Institute of Chartered Accountants in England and Wales 2008

Page 6 of 14

Professional Stage – Financial Accounting – September 2008

SOTD = (7 x 8) = 28 2 B/fwd (60,000 – 25,000 = 35,000) Year end 30 June 2008 30 June 2009

B/fwd £ 35,000 31,500

Interest £ 3,500 3,000

Payment £ (7,000) (7,000)

C/fwd £ 31,500 27,500

31,500

> 1yr

< 1yr

27,500

4,000 (31,500 – 27,500)

Tutorial note: Fixed asset carrying amounts & depreciation charge

Plant & machinery Fixtures & fittings Land & buildings

Depreciation charge £ 31,900 13,500 45,800 91,200

Carrying amounts £ 150,600 45,500 1,927,200 2,123,300

The answers to this question were quite varied. Most candidates made a good attempt at part a) and there had clearly been an improvement in candidates’ ability since December 2007 when a similar question had been set. There was a clear improvement in the calculation of individual balances, however, poor layouts often meant that it was hard to see what exactly had been included in a total and what hadn’t. Often candidates seemed to prepare what looked like a random set of workings with no linkage. Workings must be referenced clearly. Most candidates coped well with the calculations in relation to the leased asset, the elements of the new building, cost calculations and depreciation on the assets held for a complete year. Assets that had been held for less than a full year often saw an incorrect calculation of the number of months that the asset should have been depreciated for. The majority of candidates scored well in the lease calculation, although there were a number of common errors, such as, showing the current lease liability as £7,000 rather than deducting the interest from this amount. Another common mistake was to include the lease payment of £7,000 in the income statement along with the finance charge. These errors show that there is clearly a lack in understanding for accounting for a finance lease. Candidates appear to have simply learnt how to construct the leasing table. A minority of candidates produced a detailed PPE note along with additional notes in relation to the leased asset even though these were not required by the question and therefore wasted time that could have otherwise been spent productively. Other common errors included correctly removing the £72,000 management cost from the cost of the building but then failing to charge this amount in the income statement, including £60,000 as the brought forward figure in the leasing table, an incorrect calculation of the sum-of-the-digits and failing to notice that the fixtures and fittings should have been depreciated on a reducing balance basis. Total possible marks Maximum full marks

© The Institute of Chartered Accountants in England and Wales 2008

21 20

Page 7 of 14

Professional Stage – Financial Accounting – September 2008

(b) Elements of financial statements Assets/liabilities A non-current asset acquired under a finance lease meets the definition of an asset, even though the asset is not legally owned by them, as it is: •

Controlled by the lessee, as they have physical possession of the asset



Results from a past event, the lease was signed at a particular date



Gives rise to future economic benefits, the lessee uses the asset to generate revenue for the company. The lease payments are a liability as the company has an obligation arising from a past event to transfer economic benefits. The economic benefits that the lessee is obliged to transfer are the lease payments. Generally, the lessee will have some kind of legal obligation to make the lease payments. Recognition The asset should be recognised if: •

It is probable that future economic benefits will flow to the company; and



Those benefits can be measured reliably.

Conversely, the liability should be recognised if: •

It is probable that future economic benefits will be made by the company; and



Those benefits can be measured reliably.

The inflows and outflows will be probable as a lease contract agreement has been signed and the benefits can be reliably measured as the lease contract will set out the present value of the minimum lease payments. The answers to part b) were often disappointing with a significant number of candidates explaining the accounting treatment for a finance lease with no reference to the IASB Framework, this approach gained no marks. Candidates often copied text straight from the open book text without explaining further how it related to a finance lease. Many candidates recognised that the signing of the lease was the past event which created the lease obligation but few managed to explain how economic benefits would be obtained. Total possible marks Maximum full marks

7½ 5

(c) Key differences between IFRS and UK GAAP IAS 17 lists a number of factors that would indicate that the risks and rewards of ownership have been transferred to the lessee in order for the lease to be classified as a finance lease. Such as, the lease term is for a major part of the asset’s life and the lessee is responsible for repairs and maintenance. However, under UK GAAP, SSAP 21, there is a rebuttable presumption that if, at the inception of the lease, the present value of the minimum lease payments is at least 90% of the asset’s fair value then there is a finance lease. IAS 17 specifically requires a lease which covers both land and building to be split at inception into two leases, one for the land and one for the buildings. A lease for land will normally be an operating lease since land will normally have an indefinite life. Under UK GAAP there is no requirement to split a lease which covers both land and buildings. Such a lease will therefore normally be recognised as an operating lease.

© The Institute of Chartered Accountants in England and Wales 2008

Page 8 of 14

Professional Stage – Financial Accounting – September 2008

The answers to part c) were mixed. Most candidates wrote something regarding the minimum lease payment being 90% of the fair value of the asset under a finance lease under UK GAAP. This was commonly referred to as the “90% rule” without further explanation as to what exactly this was. A typical answer included some explanation about land and buildings and whether they should be separated or not under IFRS and UK GAAP. However, a number of candidates were clearly confused about which regime separates leases of land and buildings and which does not. The majority of candidates also stated that land would always be an operating lease, with only the very best candidates appreciating that while this might normally be the case it was possible for land to be held under a finance lease. Total possible marks Maximum full marks

© The Institute of Chartered Accountants in England and Wales 2008

4 3

Page 9 of 14

Professional Stage – Financial Accounting – September 2008

Question 3 – Total marks 16 General comments: This question was split into two distinct parts. Part (a) asked for the preparation of a consolidated income statement and related information following the acquisition of a subsidiary and associate. Part (b) asked for calculations of inventory. (a)(i) Consolidated income statement for the year ended 30 June 2008 £’000 Revenue Cost of sales

40,000 (35,975)

Gross profit Operating costs

4,025 (3,010)

Share of profit of associate (W2)

456

Profit before tax Taxation Profit after tax

1,471 (375) 1,096

Attributable to: Equity holders of Spinal plc (1,096 – 85) Minority interest (W4)

1,011 85 1,096

a)(ii) Goodwill calculation: £ 850,000 (681,000)

Consideration Less: share of net assets acquired (908,000 x 75%) Goodwill

169,000

W1 – Consolidation schedule Spinal plc £’000

Revenue Cost of sales PURP (W3) Gross profit Operating costs Management fee Investment income Profit before tax Tax Profit after tax

Carnelian Ltd £’000 (x 3/12)

36,340 (33,920)

(1,980)

3,800 (2,175) (20)

Adj

Consolidated

£’000

£’000

(140) 140

1,605 (1,090)

40,000 (35,975)

4,025 (3,010) 60

60 (200)

© The Institute of Chartered Accountants in England and Wales 2008

(60) 515 (175) 340

1,015 (375)

Page 10 of 14

Professional Stage – Financial Accounting – September 2008

W2 – Associate profit

Share of profit after tax (1,670,000 x 30%) Less: Current year impairment

£ 501,000 (45,000) 456,000

W3 – PURP (140,000 x (0.40 / 1.40)) x ½ = 20,000 W4 - Minority interest Profits attributable to the minority interest in year (340 (W1) x 25%) = £85,000

Almost all candidates made a good attempt at part a), with many obtaining full marks on this part of the question. The most common errors were to not apportion the income statement figures for the subsidiary acquired during the year and to miss the impairment of the associate. A significant number of candidates included drawing a group structure even though the percentages were given in the question. In such circumstances no marks are allocated for such a diagram. Other common errors included failing to account for the intra-group sales or adjusting it in the wrong place and producing balance sheet extracts, including attempting to calculate a balance sheet figure for the associate, when these were not required. Total possible marks Maximum full marks

9½ 9

(b) Raw materials Raw metal in closing inventory = (£1,050 x 50 tonnes) + (£950 x 10 tonnes) = £62,000 Work-in-progress & finished goods Total output

5,000 + (80% x 500) = 5,400 units

Raw material Direct labour

£ 195,000 253,200 448,200

Admin costs Factory overheads

Cost per unit £

448,200 / 5,400

83.00

262,800 / 6,000

43.80

107,000 155,800 262,800

126.80 WIP Inventory = 500 x 80% x £126.80 = £50,720 Finished goods inventory = 350 x £126.80 = £44,380

© The Institute of Chartered Accountants in England and Wales 2008

Page 11 of 14

Professional Stage – Financial Accounting – September 2008

Attempts at part b) were generally disappointing. This was the most commonly missed part of the paper overall, with candidates simply not attempting it. There were also a significant number of candidates who stopped their calculations at raw materials. This highlighted that a worrying number of candidates simply do not have a broad level of knowledge across the syllabus. This part of the question was straight forward in nature and required simple application of inventory accounting. The most common errors included: • not including administrative costs, but including unplanned maintenance costs, in the calculation of a cost per unit; • not basing one or both of administrative costs and factory overheads on budgeted production as opposed to actual production; • basing actual production on the complete number of units produced rather than adding the 500 units that were only 80% complete; • valuing finished goods at NRV even when this was higher than the calculated cost per unit; and • ignoring that the work in progress was only 80% complete. Total possible marks Maximum full marks

© The Institute of Chartered Accountants in England and Wales 2008

7 7

Page 12 of 14

Professional Stage – Financial Accounting – September 2008

Question 4 – Total marks 17 General comments: This question was a consolidated cash flow statement question with the disposal of a subsidiary. (a) Consolidated cash flow statement for the year ended 30 June 2008 £ Note:

Cash flow from operating activities Cash generated from operations (note) Interest paid Tax paid (W2)

£

161,610 (14,400) (57,060) 90,150

Cash flow from investing activities Purchase of property, plant and equipment Proceeds from disposal of subsidiary (W3)

(200,000) 66,000 (134,000)

Cash flows from financing activities Proceeds from issue of ordinary share capital (W4) Borrowings advanced Dividends paid

27,510 25,000 (7,560) 44,950 1,100 400 1,500

Net increase in cash and cash equivalents Cash and cash equivalents at beginning of period Cash and cash equivalents at end of the period Reconciliation of profit before tax to cash generated from operations

Profit before tax (118,320 + 15,800) Finance charge Depreciation charge (W1) Amortisation charge (95,600 – 86,000) Increase in inventories (107,730 – 97,200 + 13,000) Increase in trade and other receivables (56,340 – 53,250 + 9,500) Decrease in trade and other payables (303,000 – 342,190 + 11,700) Cash generated from operations Workings (1) B/fwd Additions

161,610

Property, plant and equipment 948,800 200,000

Depreciation (bal fig) Disposal of sub C/fwd

1,148,800 (2) Cash (bal fig) C/fwd

£ 134,120 14,400 67,100 9,600 (23,530) (12,590) (27,490)

67,100 52,000 1,029,700 1,148,800

Income tax 57,060 30,700

B/fwd IS Disposal of sub

87,760

© The Institute of Chartered Accountants in England and Wales 2008

31,800 51,460 4,500 87,760

Page 13 of 14

Professional Stage – Financial Accounting – September 2008

W3 Disposal of subsidiary

Cash proceeds on disposal Less: cash

£ 68,000 (2,000)

Cash flow on disposal

66,000

W4 Share issue

Share capital (129,000 – 125,000) Share premium (372,210 – 348,700 )

£ 4,000 23,510 27,510

There was a wide range of marks obtained on this question showing that some candidates were clearly well prepared for the inclusion of such a question, whilst others were not. However, weaker candidates were still able to pick up marks for items which would appear in a single company cash flow statement. There were a notable number of arithmetic errors in the answers to this question. Figures in the final cash flow statement were sometimes incorrect, but figures in the T-account workings were correct, with an error being made in calculating the balancing figure. A number of candidates also seemed to be confused as to whether items were an inflow (ie a positive figure) or an outflow (ie a negative figure and therefore shown in brackets). The most common error seen on this question was in relation to the group aspect of the question and included not adjusting for the disposal in the reconciliation, both in the profit before tax figure and the movements on working capital. Instead, candidates often added back the profit on disposal of the subsidiary when it was not included in the profit figure in the first instance. Other common errors included subtracting the disposal figures from the working capital calculations rather than adding the figures, not adjusting for the disposal in T-account workings, such as depreciation, setting out a retained earnings T-account to find dividends paid, when this figure was provided in the question, showing the dividends paid as an inflow rather than an outflow and showing the movement in borrowings as an outflow when it was an inflow. Total possible marks Maximum full marks

© The Institute of Chartered Accountants in England and Wales 2008

17 17

Page 14 of 14

Financial Accounting Professional Stage – December 2008

PROFESSIONAL STAGE FINANCIAL ACCOUNTING – OT EXAMINER’S COMMENTS The performance of candidates in the December 2008 objective test questions section for the Professional Stage Financial Accounting paper was good. Candidates performed well across LOs 1 and 2 but less well, on this occasion, on LO3. However, care should always be taken to ensure that the principles underlying any particular item are understood rather than the answer learned from previous experience. In particular, candidates should ensure that they read all items very carefully. The following table summarises how well* candidates answered each syllabus content area. Syllabus area

Number of questions

Well answered

Poorly answered

LO1

4

4

0

LO2

6

4

1

LO3

5

2

3

Total

15

11

4

*If 50% or more of the candidates gave the correct answer, then the question was classified as ‘well answered’. Three out of the four poorly answered questions were on LO3 (the preparation of consolidated financial statements) although the worst answered question was that from LO2 (the preparation of single company financial statements). Comments on the LO2 item and one other of particular note are: Item 1 This item tested which of four material events should be accounted for as a prior period error in accordance with IAS 8. Although most candidates recognised that misstated opening inventory and misstated opening receivables (due respectively to a computational error and fraud) should be corrected in this way many also thought that the HMRC challenging items in the last period’s tax return (which led to an additional liability) should also be dealt with as a prior period adjustment. The latter, as the normal revision of an accounting estimate, should be dealt with in the income statement for the period. Item 2 This item required a calculation of the amount to be shown in a consolidated cash flow statement in respect of the acquisition of a subsidiary. The subsidiary had a net overdraft and therefore this needed to be added to the cash paid for the subsidiary. Many candidates deducted this net figure instead of adding it.

© The Institute of Chartered Accountants in England and Wales

Page 1 of 16

Financial Accounting Professional Stage – December 2008

PROFESSIONAL STAGE FINANCIAL ACCOUNTING

MARK PLAN AND EXAMINER’S COMMENTARY The marking plan set out below was that used to mark this question. Markers were encouraged to use discretion and to award partial marks where a point was either not explained fully or made by implication. More marks were available than could be awarded for each requirement. This allowed credit to be given for a variety of valid points which were made by candidates.

Question 1 Total marks 24 General comments This is a typical question testing the preparation of full financial statements (in this case an income statement and balance sheet) from a trial balance plus a number of adjustments. The requirement included a specific instruction to analyse expenses by their nature, as opposed to the more usual analysis by function. Adjustments included a write-down to inventories based on an adjusting post balance sheet event, adjusting revenue to reflect the provisions of IAS 18, adjusting the warranty provision and providing for an interim dividend.

Grasmere Ltd Income statement for the year ended 30 September 2008 £ 5,545,700 6,600 (1,157,900) (2,570,000) (87,700) (568,000) 1,168,700 (11,500) 1,157,200 (450,000) 707,200

Revenue (W2) Changes in inventories of finished goods and work in progress (W1) Raw materials and consumables used (W1) Employee benefits expense Depreciation and amortisation expense (W3) Other expenses (W1) Profit from operations Finance cost (1,500 + 10,000 (W1)) Profit before tax Income tax expense Profit for the period

Balance sheet as at 30 September 2008 £ Assets Non-current assets Property, plant and equipment (W3) Current assets Inventories (W1) Trade and other receivables (265,500 + 15,000 – (15,000 + 21,500) Cash and cash equivalents Total assets

© The Institute of Chartered Accountants in England and Wales

£

2,230,800

31,600 244,000 570 276,170 2,506,970

Page 2 of 16

Financial Accounting Professional Stage – December 2008

£ Equity and liabilities Capital and reserves Ordinary share capital Revaluation reserve (W5) Retained earnings (W4)

£

100,000 616,000 803,370 1,519,370

Non-current liabilities Preference share capital (redeemable) Deferred income (105,000 (W2) x 9/21)

200,000 45,000 245,000

Current liabilities Trade and other payables (146,700 + 17,400) Deferred income (105,000 (W2) x 12/21) Taxation Provisions (W6) Dividends payable (W4) Borrowings

164,100 60,000 450,000 26,000 40,000 2,500 742,600 2,506,970

Total equity and liabilities

Workings (1) Allocation of expenses Changes in inventories

Per Q Prepayments and accruals Bad debts Closing inventories (32,000 – (20 x £20)) Adj re redeemable pref dividend (200,000 x 5%) Increase in warranty provision (W6)

£ 25,000

Raw materials and consumables £ 1,140,500 17,400

Other expenses £ 567,500 (15,000) 21,500

(31,600)

(6,600)

1,157,900

(10,000) 4,000 568,000

(2) Revenue Per Q Less After-sales support relating to future years (100,000 x 120% x 21/24)

© The Institute of Chartered Accountants in England and Wales

£ 5,650,700 (105,000) 5,545,700

Page 3 of 16

Financial Accounting Professional Stage – December 2008

(3) PPE £ Property – valuation Plant – cost – acc dep b/f Depreciation for year – property (1,056,000 ÷ 44) – plant ((676,000 – 357,500) x 20%)

£ 2,000,000 676,000 (357,500)

24,000 63,700 (87,700) 2,230,800

(4) Retained earnings £ 132,170 (40,000) 4,000 707,200 803,370

At 30 September 2007 Ordinary dividend (100,000 ÷ 50p x 20p) Transfer from revaluation reserve (W5) Profit for the period At 30 September 2008 (5) Revaluation reserve £ Valuation Carrying amount at 1 October 2007 (1,500,000 – 120,000) Transfer to retained earnings Depreciation charge based on revalued amount (W3) Depreciation charge based on HC (1,000,000 ÷ 50)

£ 2,000,000 (1,380,000) 620,000

24,000 20,000 (4,000) 616,000

(6) Warranty provision At 1 October 2007 Income statement charge At 30 September 2008 (1,300,000 x 2%)

© The Institute of Chartered Accountants in England and Wales

£ 22,000 4,000 26,000

Page 4 of 16

Financial Accounting Professional Stage – December 2008

As in previous sittings candidates were well-prepared for this type of question and generally answered it well. The majority of candidates produced a well laid out income statement and balance sheet in appropriate formats, although some lost presentation marks by not adding across numbers in brackets or transferring numbers from workings. Others lost presentation marks by failing to complete the sub-totals and/or totals on their statements or by having incomplete or abbreviated narrative or no heading. Candidates should remember that this type of question requires financial statements to be in a form suitable for publication. A significant minority of candidates either ignored or were unable to deal with the requirement to analyse expenses by their nature in the income statement and lost presentation marks as a result. Candidates must ensure they are familiar with both income statement formats and should be aware that they are clearly illustrated in the open book text. Although many workings, in particular the cost matrix, were clearly laid out some candidates’ workings were disorganised, untidy and therefore hard to follow, making it difficult to establish candidates’ approaches where they had not calculated the correct figure. Most candidates were able to accurately process given figures but struggled with even some of the more straightforward adjustments to the income statement such as the increase in the bad debt allowance and the inventory write-down. Most dealt accurately with the income tax charge/liability, the annual depreciation charges and the revaluations, with many candidates arriving at the correct figure for property, plant and equipment in the balance sheet and some at the correct figure for the revaluation reserve. Most correctly classified the redeemable preference shares as debt but not all treated the dividends as a finance cost. Other common errors included the following: • Failing to take the (given) increase in the bad debt allowance to the income statement and instead treating this figure as if it were the closing allowance. • Incorrectly calculating the write-down to inventory and/or calculating that there was an overall decrease in inventory over the period instead of an increase. • Although many candidates correctly calculated the closing warranty provision, many were unable to correctly process the movement on this provision (or failed to spot that an opening provision was given) and a significant minority of candidates adjusted revenue by this figure. • The adjustment to revenue for the after-sales support was correctly calculated by only a few candidates, although a good number made a reasonable attempt at this adjustment. However, the corresponding deferred income (balance sheet) adjustment was only attempted by a few candidates, and was generally incorrect when an attempt was made. • Failing to process the transfer between the revaluation reserve and retained earnings even though a number of candidates correctly calculated this adjustment. The adjustment was often made to the revaluation reserve or retained earnings, but not to both. • Incorrectly calculating the ordinary dividend – although most did remember to adjust retained earnings by the calculated figure. Total possible marks Maximum full marks

© The Institute of Chartered Accountants in England and Wales

26.5 24

Page 5 of 16

Financial Accounting Professional Stage – December 2008

Question 2 Total marks 19 General comments This question tested the preparation of a single company cash flow statement and supporting note. Missing figures to be calculated included interest received, tax paid, dividends paid, payment of finance lease liabilities, property, plant and equipment acquired and proceeds from the issue of share capital. A bonus issue of shares and a revaluation of property, plant and equipment during the year also featured.

Coniston plc Cash flow statement for the year ended 30 September 2008 £ Cash flows from operating activities Cash generated from operations (Note) Interest paid Income tax paid (W2) Net cash from operating activities Cash flows from investing activities Purchase of property, plant and equipment (W3) Proceeds from sales of property, plant and equipment (576,700 – 25,600) Interest received (W1) Net cash used in investing activities Cash flows from financing activities Proceeds from issue of ordinary share capital (170,000 (W4) + 200,000 (W5)) Payment of finance lease liabilities (190,300 – 121,000) (W7) Dividends paid Net cash from financing activities Net increase in cash and cash equivalents Cash and cash equivalents at beginning of period Cash and cash equivalents at end of period

£

991,930 (121,000) (226,000) 644,930 (1,151,400) 551,100 23,000 (577,300) 370,000 (69,300) (270,000) 30,700 98,330 3,450 101,780

Note: Reconciliation of profit before tax to cash generated from operations Profit before tax Investment income Finance costs Depreciation charge Loss on disposal of property, plant and equipment Increase in inventories (1,680,220 – 1,188,400) Decrease in trade and other receivables ((556,700 – 6,000) – (543,600 – 7,500)) Increase in trade and other payables (444,100 – 430,950) Increase in warranty provision (420,000 – 385,000) Cash generated from operations

© The Institute of Chartered Accountants in England and Wales

£ 633,300 (24,500) 121,000 665,600 25,600 (491,820) 14,600 13,150 35,000 991,930

Page 6 of 16

Financial Accounting Professional Stage – December 2008

Workings (1) Interest received

B/d IS

£ 6,000 24,500 30,500

Cash (β) C/d

£ 23,000 7,500 30,500

(2) Tax paid

Cash (β) C/d

£ 226,000 275,000 501,000

B/d IS

£ 246,000 255,000 501,000

(3) PPE

B/d Revaluation reserve (W6) Finance leases Additions (β)

£ 3,299,400 133,000 225,000 1,151,400 4,808,800

Disposal Depreciation C/d

£ 576,700 665,600 3,566,500 4,808,800

(4) Share capital £

C/d

1,210,000 1,210,000

B/d Bonus issue Cash (β)

£ 1,000,000 40,000 170,000 1,210,000

(5) Share premium £ C/d

740,000 740,000

B/d Cash (β)

£ 540,000 200,000 740,000

B/d PPE (β)

£ 435,000 133,000 568,000

B/d (500,000 + 67,800) Interest PPE

£ 567,800 121,000 225,000 913,800

(6) Revaluation reserve

SCE C/d

£ 56,000 512,000 568,000

(7) Finance lease liabilities

Cash (β) C/d (600,000 + 123,500)

£ 190,300 723,500 913,800

© The Institute of Chartered Accountants in England and Wales

Page 7 of 16

Financial Accounting Professional Stage – December 2008

This was the second time this topic had been set and some candidates were clearly very well prepared for it. Others did not have a sufficient grasp of the double-entry techniques which underpin the preparation of such a statement to score as well as might have been expected on a question of this type. However, the majority of candidates scored high marks on the reconciliation note, and on the figures for tax paid, proceeds from the sale of

property, plant and equipment and the opening and closing figures for cash and cash equivalents. Presentation was generally good and most candidates produced workings in the form of T accounts. However, a significant number made the mistake of putting opening and closing balances on the wrong side of those T accounts. There were also a minority of candidates that produced no workings for the cash flow statement. This is a risky approach to take as if figures are calculated incorrectly it is not possible to award any partial marks. Candidates generally made a good attempt at the property, plant and equipment T account, with the figures given in the question for depreciation and for the disposal both being correctly used. However, the finance lease adjustment was only included by a minority of candidates. An adjustment for the revaluation during the year was generally made, although the most common figure used was £77,000 being the difference between the opening and closing figures on the revaluation reserve, ie not adjusted for the transfer made during the year shown in the question in the statement of changes in equity. Candidates were clearly confused by interest received and the amounts for the finance lease liabilities, and these amounts were often included in one T account. A calculation of proceeds from the issue of shares was attempted by almost all candidates although it was fairly common to see the bonus issue either missed or included in the wrong T account. Other common errors included the following: • In the reconciliation note, failing to adjust for the increase to the warranty provision, finance costs and investment income or making the adjustment(s) in the wrong direction. • Failing to adjust the trade receivables figure for the opening and closing accrued interest. • Attempting to calculate dividends paid in the year by constructing a retained earnings T account, even though the T account was effectively given in the question as an extract from the statement of changes in equity, and included the dividend figure (this was very common). • In the interest received T account, putting the amounts for opening and closing interest accrued on the wrong side of the T account or posting these to an interest paid T account. • Omission of the figure for interest paid in the cash flow statement – a figure that should simply have been taken from the income statement in the question. Total possible marks Maximum full marks

© The Institute of Chartered Accountants in England and Wales

19.5 19

Page 8 of 16

Financial Accounting Professional Stage – December 2008

Question 3 Total marks 14 General comments This question mixed two discrete topics and contained a conceptual Part (c). Part (a) required extracts from a consolidated cash flow statement and consolidated balance sheet in respect of an associate acquired at the start of the year. Part (b) required the calculation of income statement figures in respect of a machine classified as held for sale during the year. The conceptual Part (c) tested an understanding of the four measurement bases from the IASB’s Framework.

Thirlmere plc (a) Investment in associate – Wastwater Ltd Consolidated cash flow statement for the year ended 30 September 2008 (extracts) £ Cash flows from investing activities Purchase of associate, Wastwater Ltd

(52,000)

Consolidated balance sheet as at 30 September 2008 (extracts) Non-current assets Investments in associates (W)

£ 1,325,100

Working Investment in associate £ Cost ((1,000,000 x £1.20) + 52,000) Share of post acquisition change in net assets Share of post acquisition profits ((210,000 x 40%) Share of additional depreciation based on fair value (((250,000 – 160,000) ÷ 40) x 40%) Less Impairment losses to date

£ 1,252,000

84,000 (900) 83,100 (10,000) 1,325,100

This part of the question was very poorly answered, with a large number of candidates failing to recognise that the acquired entity was an associate, not a subsidiary. Even those who recognised it as an associate seemed unsure as to how to calculate the investment in associate figure for the consolidated balance sheet with many using the old UK GAAP method (which, although it could give the correct answer was a more complex calculation). Some attempted some sort of hybrid calculation falling between the latter method and that used in the learning materials. Even those who were able to calculate the carrying amount for the consolidated balance sheet failed to show this amount as an extract as specified in the requirement (producing only a working) and lost marks because of this. Very few candidates identified the need to adjust the post acquisition retained earnings of the associate for the extra depreciation arising from the fair value adjustment. Few candidates knew that the acquisition of an associate is shown in the consolidated cash flow statement as the amount of cash paid to acquire the shares, not the amount of cash paid less cash and cash equivalents acquired, as for a subsidiary – although most knew that this would be presented under investing activities.

© The Institute of Chartered Accountants in England and Wales

Page 9 of 16

Financial Accounting Professional Stage – December 2008

Those candidates who decided that the acquired entity was a subsidiary wasted significant time by attempting to do unnecessary consolidations/disclosure notes. Few candidates knew that the acquisition of an associate is shown in the consolidated cash flow statement as the amount of cash paid to acquire the shares, not the amount of cash paid less cash and cash equivalents acquired, as for a subsidiary – although most knew that this would be presented under investing activities. Those candidates who decided that the acquired entity was a subsidiary wasted significant time by attempting to do unnecessary consolidations/disclosure notes. Total possible marks Maximum full marks

7 5

(b) Asset held for sale Income statement charges for the year ended 30 September 2008 Depreciation (150,000 x 20% x 10/12)

Cost on 1 April 2005 Depreciation to 30 September 2007 (150,000 x 20% x 2.5) Depreciation for year ended 30 September 2008 Carrying amount at 1 August 2008 Fair value less costs to sell (45,000 – 1,000) Impairment loss on classification as held for sale

£ 25,000 £ 150,000 (75,000) (25,000) 50,000 (44,000) 6,000

Answers to this part were better than those to Part (a) but were still disappointing. The majority of candidates miscalculated both the total number of months the asset had been owned and the number of months in the current year (in order to calculate the current year depreciation charge). Although most candidates recognised that an impairment loss on classification as held for sale would be taken to the income statement for the year relatively few also showed the depreciation expense for the year. In contrast to Part (a) where extracts were not produced when they were required, in this part where extracts were not required, only the calculations (although candidates did need to identify in their answer the two distinct charges being taken to the income statement for the year) many candidates also produced extracts. Total possible marks Maximum full marks

5 4

(c) The four measurement bases Historical cost Assets are recorded at the amount of cash or cash equivalents paid or the fair value of the consideration given to acquire them at the time of their acquisition. At historical cost, the machine was recorded at its total price of £150,000. Current cost Assets are carried at the amount of cash or cash equivalents that would have to be paid if the same or an equivalent asset was acquired currently.

© The Institute of Chartered Accountants in England and Wales

Page 10 of 16

Financial Accounting Professional Stage – December 2008

If the machine had not been held for sale in the current period and was to be measured at its current cost it would have been restated in the balance sheet to £175,000 x 1½ years/5 years = £52,500 – representing an “aged” version of the £175,000 current cost. Realisable (settlement) value Assets are measured at the amount of cash or cash equivalents that could currently be obtained by selling an asset in an orderly disposal. This is effectively the measurement basis that has been adopted for the machine held for sale at the year end – ie measured at £44,000 (or £45,000). Present value Assets are measured at the current estimate of the present discounted value of the future cash flows in the normal course of business. Under this basis, the machine, if retained, would be measured at £40,000, being the present value of future cash flows generated within the business. Many candidates either knew this material or extracted it from their open book text. However, some then lost marks because they failed to explain each of the four bases, as required, with reference to the figures in the question. Others wasted time setting out how the measurement bases relate to liabilities although the question clearly asked the concepts to be explained by reference to the machine in the question (an asset). Other candidates scored poorly as instead of the four measurement bases they wrote about things such as qualitative characteristics or concepts such as accruals/going concern – a worrying indication that they are not as familiar as they should be with the content of the IASB Framework (and their open book text). Total possible marks Maximum full marks

© The Institute of Chartered Accountants in England and Wales

5.5 5

Page 11 of 16

Financial Accounting Professional Stage – December 2008

Question 4 Total marks 23 General comments Part (a) was a consolidated income statement question, featuring two subsidiaries (one fully disposed of within the year) and one associate. Adjustments were typical of this type of question and included a fair value adjustment on acquisition with subsequent impact on the annual consolidated income statement, intra-group trading and unrealised profits and impairment write-downs. The minority interest column from the consolidated statement of changes in equity was also required. Part (b) tested the differences between IFRS and UK GAAP in respect of the preparation of group financial statements, with the treatment of goodwill arising on acquisition specifically excluded.

Windermere plc (a) Consolidated income statement for the year ended 30 September 2008 Revenue (W2) Cost of sales (W2) Gross profit Operating expenses (W2) Profit from operations Share of profit of associates ((1,150 x 30%) – 10) Profit before tax Income tax expense (W2) Profit for the period from continuing operations Profit for the period from discontinued operations (1,240 – 120 (W4)) Profit for the period Attributable to Equity holders of Windermere plc (β) Minority interest (W5)

£’000 59,480 (41,490) 17,990 (9,680) 8,310 335 8,645 (3,300) 5,345 1,120 6,465

5,381 1,084 6,465

Consolidated statement of changes in equity for the year ended 30 September 2008 (extracts) Minority interest £’000 Profit for the year 1,084 Eliminated on disposal of subsidiary (W7) (1,220) (136) Balance at 30 September 2007 (W6) 3,352 Balance at 30 September 2008 (β) 3,216

© The Institute of Chartered Accountants in England and Wales

Page 12 of 16

Financial Accounting Professional Stage – December 2008

Workings (1) Group structure

Windermere plc 0.6

= 30%

2 3.2 4

2.1

= 80% for 6/12

3.5

Rydal Ltd

= 60% Buttermere Ltd

Derwent Ltd

(2) Consolidation schedule

Revenue Cost of sales – per Q – PURP (W3) Op expenses – per Q – GW impairment Tax

Windermere plc £’000 38,700 (26,400)

Derwent Ltd

(7,450) (30) (1,900)

£’000 21,500 (15,750) (60) (2,200)

Adj (W3)

Consol

£’000 (720) 720

£’000 59,480 (41,490) (9,680) (3,300)

(1,400) 2,090

(3) Intra-group sale and PURP

SP Cost GP X½

© The Institute of Chartered Accountants in England and Wales

% 120 (100) 20

£’000 720 (600) 120 60

Page 13 of 16

Financial Accounting Professional Stage – December 2008

(4) Group loss on disposal of Rydal Ltd £’000 Sale proceeds Less: Share of net assets at disposal Net assets at 30 September 2008 Less: Profit since 1 April 2008 (2,480 x 6/12)

Less: Carrying amount of goodwill Cost of investment (3,200 x £2.30) Less: Share of net assets at acquisition ((4,000 + 1,650) x 80%) Less: Impairments to date

7,340 (1,240) 6,100 x 80%

£’000 7,500

(4,880)

7,360 (4,520) (100) (2,740) (120)

(5) Minority interest in year Rydal Ltd (20% x 1,240 (W4)) Derwent Ltd (40% x 2,090 (W2)

£’000 248 836 1,084

(6) Minority interest brought forward Rydal Ltd (20% x (7,340 – 2,480)) Derwent Ltd (40% x (8,100 – 2,150))

£’000 972 2,380 3,352

(7) Minority interest eliminated on disposal of Rydal Ltd

£’000

B/f (W6) Current year

972 248 1,220

(8) Minority interest carried forward (for proof only) Derwent Ltd (40% x (8,100 – 2,150 + 2,090 (W2))

£’000 3,216

The majority of candidates showed that they could construct a consolidated income statement and demonstrated that they understood the underlying principles. However, candidates were less clear on the movement on the minority interest account as demonstrated by the minority interest column from the consolidated statement of changes in equity – although some of the more able candidates scored full marks on this part demonstrating that they understood the link between the minority interest figure in the consolidated income statement and that in the consolidated balance sheet. The most worrying aspect of the workings to the consolidated income statement was the number of candidates who included a column for the subsidiary disposed of during the year in their consolidation schedule (Working 2). This was not necessary, as, per the learning materials, a separate calculation should have been performed to add together the group profit on the disposal of the subsidiary and the subsidiary’s profit for the period. This should then have been presented on the face of the consolidated income statement as the profit for the period from discontinued operations. Those candidates who included a column for the subsidiary disposed of might have been going on to present discontinued operations line-by-line on the face of the consolidated income statement – but none of them did this. Indeed many candidates seemed to be very confused about how to deal with the IFRS 5 disclosures in respect of discontinued operations. Even those who did arrive at a correct figure for discontinued operations often failed to take out the minority interest on this figure at the bottom of the income statement.

© The Institute of Chartered Accountants in England and Wales

Page 14 of 16

Financial Accounting Professional Stage – December 2008

Another common error was to make no attempt to calculate a group profit on the disposal of the subsidiary but to take the parent company profit as given in the question and add this to the subsidiary’s profit for the period to arrive at a figure for profit for the period from discontinued operations. However, the more able candidates did calculate the correct figure and its minority interest. Other common errors included the following: • Incorrectly calculating the subsidiary’s net assets at disposal and/or acquisition in the calculation of group profit on the disposal. • Writing off the impairment on the associate against the parent company instead of against the group share of the associate’s profits. • Reducing the associate’s profits by the impairment before taking the group share of those profits. • Writing off the impairment on the subsidiary against the subsidiary itself instead of against the parent company (or as a consolidation adjustment). • Taking the figure of £600,000 in respect of the intra-group sale as the selling price, when it was clearly stated as being the cost. • Adjusting for the provision for unrealised profits against the parent company instead of against the subsidiary (which was the selling company). • Including a line for dividends in the consolidated statement of changes in equity when the question stated that there were no dividends declared or paid during the period and/or failing to include a line in respect of the disposal of the subsidiary. • When calculating the minority interest brought forward, failing to exclude the profit for the year from the figure for year-end retained earnings given in the question. A worrying minority of candidates completely omitted any figures in respect of the associate and/or made no attempt at the minority interest reconciliation. Total possible marks Maximum full marks

© The Institute of Chartered Accountants in England and Wales

19.5 18

Page 15 of 16

Financial Accounting Professional Stage – December 2008

(b) Differences between IFRS and UK GAAP re preparation of group financial statements Merger accounting is required by UK GAAP (FRS 6) where certain criteria are met. IFRS 3 requires all business combinations to be accounted for using the purchase method. UK GAAP gives specific guidance on fair value measurement in FRS 7. IFRS 3 provides less detailed guidance. Under FRS 7only separable intangible assets are required to be measured at fair value. Under IFRS 3 more intangibles can be recognised as intangible assets recognised under a business combination include separable assets and those arising from contractual or legal rights (regardless of whether those rights are transferable or separable). UK GAAP (FRS 2) includes an exclusion of a subsidiary from consolidation on the grounds of severe longterm restrictions. No such exemption exists under IAS 27 (although control may be lost as a result of the restriction such that the entity will no longer be classified as a subsidiary). UK GAAP (FRS 2) requires the minority interest to be presented separately from shareholders’ funds. IAS 27 requires it to be shown as a separate component of equity. Under UK GAAP no consideration of the existence of potential voting rights is required in the assessment of control. IAS 27 requires the existence of potential voting rights to be considered. In a consolidated cash flow statement presentation under UK GAAP (FRS 1) is different to that under IAS 7, as follows. Item Dividends from associates Dividends paid to minority interest Acquisitions and disposals of subsidiaries/associates

FRS 1 classification Disclosed as a separate caption Included under “return on investments and servicing of finance” Disclosed as a separate caption

IAS 7 classification Included under investing cash flows Included under financing cash flows

Included under investing cash flows

Under UK GAAP (FRS 9) the group share of an associate’s operating profit and the group share of the associate’s interest and tax are brought into the consolidated income statement separately. IAS 1 suggests a single line presentation. UK GAAP (FRS 9) requires the parent company to recognise its share of an associate’s net liabilities. IAS 28 only requires this where there is a legal or constructive obligation to make good those losses. Although the requirement asked for candidates to set out the differences between IFRS and UK GAAP in respect of the preparation of group financial statements, the majority of candidates set out differences which applied to the preparation of single company financial statements. As a result, many candidates scored few marks on this part of the question which should have been an opportunity to score a good number of marks. Those who did address group differences generally scored highly, although a few included differences in respect of the treatment of goodwill, which were specifically excluded from the requirement. Others listed some very worrying incorrect differences – such as the fact that associates are not consolidated in the UK and that in the UK no fair value adjustments are made. This is an important area of the syllabus and candidates should ensure that they know these differences (which are clearly set out in both the learning materials and at the back of the syllabus document) and can apply them to given scenarios. Total possible marks Maximum full marks

© The Institute of Chartered Accountants in England and Wales

10.5 5

Page 16 of 16

Professional Stage – Financial Accounting – September 2009

MARK PLAN AND EXAMINER’S COMMENTARY The mark plan set out below was that used to mark these questions. Markers are encouraged to use discretion and to award partial marks where a point was either not explained fully or made by implication. More marks are available than could be awarded for each requirement, where indicated. This allows credit to be given for a variety of valid points, which are made by candidates.

Question 1

Total marks: 22

General comments This is a trial balance question with the preparation of a statement of financial position and a statement of changes in equity required. Adjustments are required for depreciation, the recognition of a provision and related asset, development expenditure, an operating lease as well as other minor adjustments.

(a) Adeje Ltd – Statement of financial position as at 30 June 2009 £ ASSETS Non-current assets Property, plant and equipment (W2) Intangible assets (120,000 – 40,000 – 25,000)

£

2,175,050 55,000 2,230,050

Current assets Inventories (42,000 + 2,500) Trade & other receivables (32,000 + 10,000)

44,500 42,000 86,500 2,316,550

Total assets EQUITY AND LIABILITIES Equity Ordinary share capital Share premium account 6% Irredeemable preference shares Retained earnings

1,050,000 200,000 60,000 874,050 2,184,050

Current liabilities Trade and other payables (58,000 + 250 (W3)) Bank overdraft Taxation Provisions

58,250 18,250 41,000 15,000 132,500

Total equity and liabilities

© The Institute of Chartered Accountants in England and Wales 2009

2,316,550

Page 1 of 15

Professional Stage – Financial Accounting – September 2009

Statement of changes in equity for the year ended 30 June 2009 Ordinary share capital –



Irredeem. pref shares –





– –

Balance b/fwd Balance c/fwd

Total comprehensive loss for the period (W3) Interim – ordinary dividend Issue of shares Irredeemable dividend (60,000 x 6% x 6/12)

Share premium

Retained earnings

Total

(84,900)

(84,900)



(21,250)

(21,250)

– –

60,000 –

– (1,800)

60,000 (1,800)

– 1,050,000

– 200,000

60,000 –

(107,950) 982,000

(47,950) 2,232,000

1,050,000

200,000

60,000

874,050

2,184,050

W1 Depreciation Depreciation on buildings

(2,500,000 – 700,000) x 3% = 54,000 pa

Depreciation on plant & equipment (351,000 – 1,000 (lease) – 97,000) x 15% = 37,950 pa W2 Property, plant and equipment Land & buildings £ 2,500,000

Cost b/fwd Adjustment

Acc. deprec. b/fwd Charge for year (W1) Acc. deprec. b/fwd Carrying amount

Plant & equipment £ 351,000 (1,000) 350,000

486,000 54,000 540,000

97,000 37,950 134,950

1,960,000

215,050

Total £

2,175,050

W3 Operating lease Deposit 4 instalments of £1,000 Total lease payments

£ 1,000 4,000 5,000

Straight line over two years = £5,000 / 2 years = £2,500 6 months charge in the period = £2,500 x 6/12 months = £1,250 Accrual: 1,250 – 1,000 (paid) = 250

© The Institute of Chartered Accountants in England and Wales 2009

Page 2 of 15

Professional Stage – Financial Accounting – September 2009

W4 Total comprehensive profit/loss for the period £ Trial balance – profit before tax Adjustments: Irredeemable preference dividend Provision Reimbursement Inventory Research & development exp (40,000 + 25,000) Depreciation charge (54,000 + 37,950) Lease charge (W2) Taxation Loss for the period

£ 115,000

1,800 (15,000) 10,000 2,500 (65,000) (91,950) (1,250) (158,900) (41,000) (84,900)

Candidates performance on this question was good and fairly consistent with previous sittings although the preparation of the statement of changes in equity rather than the income statement continues to cause weaker candidates problems. Almost all candidates attempted this question highlighting just how comfortable candidates are with this style of question, which continues to be fundamental to the Financial Accounting syllabus. A good majority of candidates attempted this question first. Almost all candidates produced a well laid out statement of financial position, however the preparation of the statement of changes in equity was often a little haphazard. Candidates seem extremely comfortable when they are asked to prepare the statement of financial position and income statement, but less so when the statement of changes in equity is asked for. Candidates should be reminded that it is likely that any two of these three statements could be, and frequently are, asked for. Candidates generally spent time totalling each line item in their statement of financial position, although subtotals were often missing. Candidates should continue to be reminded that presentation marks are available in this type of question as the requirement asked for statements that are suitable for publication. Workings were generally clearer than in the past with most candidates producing well laid out workings for the property, plant and equipment calculations instead of adding or subtracting figures on the face of the statement which has been a criticism in the past. Most candidates were able to take items from the trial balance and insert them in the correct place in the statement of financial position. Marks were awarded where presentation differed to the marking guide but resulted in a reasonable alternative. A number of candidates produced a working for retained earnings carried forward, seeming not to realise that this simply duplicated the information from one column of their statement of changes in equity – which itself acted as a working for the figure in the statement of financial position. A good number of candidates missed the column for preference shares in the statement of changes in equity and consequently incorrectly showed the share issue as ordinary shares. Most candidates coped well with the goods omitted from the inventory count, the calculation of the appropriate provision, the annual depreciation charges, the issue of irredeemable preference shares and the tax charge for the year. Pleasingly, only a minority of candidates misclassified the irredeemable preference shares as debt, rather than equity. Strong candidates coped well with adjusting the profit before tax figure in the question for each adjustment made. Weaker candidates made the adjustments to statement of financial position figures but failed to make the corresponding adjustment to profit in order to arrive at the correct figure for “total comprehensive income” in the statement of changes in equity.

© The Institute of Chartered Accountants in England and Wales 2009

Page 3 of 15

Professional Stage – Financial Accounting – September 2009

The worse attempted adjustment was that of the lease, with only a minority of candidates arriving at the correct adjustment to profit and the correct closing accrual. Common errors were to treat the operating lease as a finance lease, in spite of the fact that the question specified that this was an operating lease. Some candidates simply treated the £1,000 as an expense, others treated half of it as a prepayment, while others set up the future payments as lease liabilities. A number of candidates failed to deal with this adjustment at all. Other common errors included netting off the £10,000 expected refund against the £15,000 provision and thereby showing only a net £5,000 provision in the statement of financial position, failing to reduce the amount capitalised for research and development expenditure by one of the two amounts that were not allowable under IAS 38, although candidates generally completed their double entry adjustment correctly, calculating the preference dividend based on a whole year rather than six months and reducing trade receivables by £10,000 in respect of the returned goods. Total possible marks Maximum full marks

© The Institute of Chartered Accountants in England and Wales 2009

22 22

Page 4 of 15

Professional Stage – Financial Accounting – September 2009

Question 2

Total marks: 19

General comments This question requires the preparation of a statement of cash flows along with the note reconciling profit before tax to cash generated from operation using the indirect method. A number of adjustments are required, including the disposal of an item of plant with a deferred receipt, two share issues, one for cash and one as part of an acquisition and deferred credit terms on the acquisition of new equipment. Caleta plc Statement cash flows for the year ended 30 June 2009 £ Cash flows from operating activities Cash generated from operations (Note) Interest paid (W1) Income tax paid (W2) Net cash from operating activities Cash flows from investing activities Purchase of property, plant and equipment Purchase of intangible assets Proceeds from sales of property, plant and equipment (35,000 – 11,000) / 2 Net cash used in investing activities Cash flows from financing activities Proceeds from issue of ordinary share capital (80,000 (W5) + 10,000 (W6)) Repayment of loan (600,000 – 350,000) Dividends paid (W7) Net cash from financing activities Net increase in cash and cash equivalents Cash and cash equivalents at beginning of period Cash and cash equivalents at end of period

£

546,680 (165,200) (20,780) 360,700 (50,000) (100,000) 12,000 (138,000)

90,000 (250,000) (34,000) (194,000) 28,700 59,300 88,000

Note: Reconciliation of profit before tax to cash generated from operations Profit before tax Finance costs Depreciation charge (W3) Amortisation charge (W4) Loss on disposal of property, plant and equipment Increase in inventories (123,100 – 106,000) Increase in trade and other receivables ((229,800 – 12,000) – 216,500) Decrease in trade and other payables ((334,800 – 6,300 – 39,000) – (358,600 – 7,500)) Cash generated from operations

© The Institute of Chartered Accountants in England and Wales 2009

£ 133,380 164,000 36,600 281,700 11,000 (17,100) (1,300) (61,600) 546,680

Page 5 of 15

Professional Stage – Financial Accounting – September 2009

Workings (1) Interest paid

Cash (β) C/d

£ 165,200 6,300 171,500

B/d IS

£ 7,500 164,000 171,500

£ 20,780 32,000 52,780

B/d IS

£ 22,000 30,780 52,780

(2) Tax paid

Cash (β) C/d

(3) PPE

B/d Additions

£ 366,500 89,000 455,500

Disposal Depreciation (β) C/d

£ 35,000 36,600 383,900 455,500

Amortisation (β) C/d

£ 281,700 1,163,300 1,445,000

B/d Non-cash issue Cash (β)

£ 550,000 50,000 80,000 680,000

B/d Non-cash issue Cash (β)

£ 110,000 50,000 10,000 170,000

B/d IS

£ 352,700 102,600 455,300

(4) Intangibles

B/d Additions

£ 1,245,000 200,000 1,445,000

(5) Share capital £

C/d

680,000 680,000

(6) Share premium £

C/d

170,000 170,000

(7) Retained earnings

Dividends in SCE (β) C/d

£ 34,000 421,300 455,300

© The Institute of Chartered Accountants in England and Wales 2009

Page 6 of 15

Professional Stage – Financial Accounting – September 2009

Candidates are clearly very comfortable with the preparation of a statement of cash flows. This question was generally completed early on showing that candidates continue to favour this topic. Presentation was generally good and most candidates produced workings in T-accounts. A significant minority of candidates produced T-accounts with the brought forward and carried forward figures the wrong way round and got in-flows and out-flows back to front on the face of the statement of cash flows. The vast majority of candidates arrived at the correct figures for interest paid, tax paid, purchase of property, plant and equipment and intangibles, repayment of loan and dividends paid in the statement of cash flows itself. In the reconciliation most arrived at correct figures (or used their incorrectly calculated adjustments correctly) for all but the changes in working capital, where there were a number of more unusual adjustments to be made. Where errors were made they included adjusting the closing interest accrual by the £1,700, not taking the issue of shares for non-cash consideration into account when calculating the cash proceeds from issue of ordinary share capital, not adjusting the movement on trade payables for the £39,000 due in respect of property, plant and equipment and adjusting the property, plant and equipment T-account by the accumulated depreciation on the asset sold instead of by the carrying amount. Total possible marks Maximum full marks

© The Institute of Chartered Accountants in England and Wales 2009

19 19

Page 7 of 15

Professional Stage – Financial Accounting – September 2009

Question 3

Total marks: 18

General comments This is a mixed topic question covering both revenue recognition and the treatment of an associate acquired during the period in the consolidated statement of financial position. The question also contains a separate element on the different users of financial statements and their information needs. (a) Income statement for year ended 30 June 2009 £ Revenue Fixed price contract (120,000 x 50%) Interest-free credit Commission sales (1,300,000 x 15%) High technology fittings (85,000 – (4,000 x 2 yrs)) Maintenance income (4,000 x 3/12) Cost of sales (45,000 + 35,000) x 50% Finance income (25,000 – 23,500) x 6/12 months

60,000 23,500 195,000 77,000 1,000 40,000 750

Statement of financial position as at 30 June 2009 £ Trade and other receivables Fixed price contract (60,000 – 40,000) Interest-free credit (23,500 + 750)

20,000 24,250

Non-current liabilities Deferred income (4,000 x (21 – 12)/12 months)

3,000

Current liabilities Deferred income

4,000

(It was not necessary to split the deferred income between non-current and current liabilities to gain the marks.) Candidates seem to struggle with extract style questions whether they are single or mixed topic. Candidates are not able to simply follow a process with this style of question and as a result they appear to forget their basic accounting knowledge. This question was often left to the end suggesting that candidates are not comfortable with this style of question. Answers to this question were mixed with the question proving to be a good discriminator between strong and weak candidates. In Part (a) although the majority of candidates arrived at some correct figures, the weaker candidates failed to properly complete their double entry, so whilst they might arrive at, say, a correct figure for revenue they failed to give the correct figure for receivables, or deferred income. A significant number of candidates wasted time providing extracts from the financial statements when all that was required was clearly labelled calculations. A significant number of candidates also made, what appeared to be, correct calculations but there were no clues as to what these figures represented, for example, revenue, cost of sales, receivables etc and therefore lost marks. Comments on specific parts of Part (a) are as follows: • Fixed price contract: Most candidates correctly arrived at revenue of £60,000 but then calculated cost of sales as being £45,000, being the cost incurred to date (instead of adding the costs incurred to date and estimated future costs and then taking 50% of the total as reflecting the stage of completion). Few calculated a closing receivables figure and others described this as deferred income. A minority used the costs basis to calculate cost of sales when a completion basis was specified in the question. • Interest free credit: Many arrived correctly at revenue of £23,500 but gave receivables as £25,000 (or

© The Institute of Chartered Accountants in England and Wales 2009

Page 8 of 15

Professional Stage – Financial Accounting – September 2009

vice versa). A number of candidates calculated finance income based on one year instead of six months and others described this figure as finance cost, as opposed to finance income. Of those who calculated these two income figures few then gave the corresponding total receivable. • Commission sales: the revenue figure was correctly calculated by the majority of candidates. • High technology fittings: attempts at calculating a revenue figure varied with most candidates reducing the £85,000 fee by only one year’s worth of maintenance charges, rather than two. Only the better candidates arrived at the correct figure for revenue and the corresponding figure for deferred income. Total possible marks Maximum full marks

7 6

(b) Extract from consolidated statement of financial position ASSETS Non-current assets Investment in associate (W1) 129,100 Current assets Dividend from Alcala Ltd (W2) (W1) Associate Year end carrying amount Cost Share of post-acq change in net assets 35% x (576,000 – 650,000) Impairment in year Investment in associate

14,000

£ 175,000 (25,900) 149,100 (20,000) 129,100

(W2) Dividend (400,000 / 0.50) x 5p x 35% = £14,000

In Part (b), which did require extracts, a significant number of candidates provided calculations only. Candidates must learn to distinguish between requirements asking for extracts (where marks will be given for presentation and own figure marks from workings) and those requiring calculations only. There was a split between candidates who attempted to calculate a figure for dividend receivable from the associate and those who ignored this totally. Many arrived at an incorrect figure, either missing the fact that these were 50p shares, or that a 35% share was needed. Once calculated, the most common mistake was for this figure to be used to adjust the retained earnings of the associate, with few candidates realising that retained earnings given in the question would already have been reduced by this figure. Disappointingly, it was very rare to see a consolidated statement of financial position extract for the dividend receivable. Most candidates made a reasonable attempt at calculating the carrying amount for the investment in the associate and correctly deducted the impairment of £20,000. Common errors included failing to recognise that the associate had made a post-acquisition loss, as opposed to a post-acquisition profit, consequently adding £25,900 to the cost of the associate rather than deducting it. A worrying minority of candidates treated the associate as a subsidiary and attempted to calculate figures such as goodwill. Even though the (given) 35% shareholding should in itself have indicated an associate, as opposed to a subsidiary, the question also clearly stated that the investment should be treated as an associate. Total possible marks Maximum full marks

© The Institute of Chartered Accountants in England and Wales 2009

5 5

Page 9 of 15

Professional Stage – Financial Accounting – September 2009

(c)(i) Investors – Investors need to decide whether to buy or sell their shares. They analyse information from the financial statements, such as the company’s past dividend policy, the level of profits that the company is making and how the company uses its resources. Employees – financial statement information is needed to assess their employer’s stability and profitability and their ongoing ability to offer further remuneration and career progression. Does the company have plans to expand in the future, is competition fierce or does the company have a niche market? Does the company have share plans for its employees? Lenders - financial statement information is needed to assess whether the company will have sufficient funds available to pay the finance costs and repayments when they fall due. Working capital and cash flow will be of particular interest. Suppliers - financial statement information is needed to assess the company’s ability to pay their debts when they fall due. Working capital and cash flow will be of particular interest. Customers - financial statement information is needed to assess a company’s ability to continue trading, providing continuity of supply for customers. What are the company’s plans for the future, is it planning to expand, diversify or reorganise its business? Government agencies - financial statement information is needed to assess the allocation of resources and therefore the activities of the company. Various different financial and non-financial information will be used by the Government in its collection of national statistics, such as number of employees, level of revenue, geographical locations etc. The general public - financial statement information is needed to assess trends and recent developments in the company’s activities and future trading potential. Expansion plans for the future will be of particular interest as they may provide additional local employment opportunities and are likely to encourage other businesses into the local area. The financial performance of the company may provide some insight into whether the company is likely to remain in the local area. [Note: information presented on the first two user groups only will be marked.] Total possible marks Maximum full marks

5 4

(c)(ii) The financial statements have a number of limitations as set out below: ƒ

Financial statements are prepared to a specific date, the information, when published is therefore historic and backward looking. Although, historic information is useful in assessing how a company has been performing it is limited in the amount of information that it can provide about a company’s future performance.

ƒ

Financial statements are prepared in a standardised manner with much of the information aggregated. While this means that it is easier to compare information between companies because it is presented in a similar manner it also means that the content of standardised and aggregated information may be difficult to identify.

ƒ

Financial statements only contain a limited amount of narrative information about the business which can provide valuable insight into the company’s future, for example, how it is operating, what the company’s plans are for the future, the risks facing the company, such as the number of competitors in the market and how the company is managed.

(Markers were encouraged to use their judgement and award marks where candidates had provided alternative limitations.) Total possible marks Maximum full marks

© The Institute of Chartered Accountants in England and Wales 2009

5½ 3

Page 10 of 15

Professional Stage – Financial Accounting – September 2009

Answers to Part (c) continued the trend of candidates struggling with the written part of questions although there were some good answers from some of the stronger candidates. However, it was pleasing that almost all candidates attempted this part of the question, which was an improvement on previous sittings where weaker candidates often miss out the written elements. Common misconceptions included identifying management/directors and/or auditors as users of the financial statements, stating that banks looking to lend to companies will rely on historic financial statements (as opposed to requiring cash flow forecasts), that financial statements include no narrative information and that immaterial errors matter to users (indicating a lack of understanding of the concept of materiality). Some candidates wasted time discussing the qualitative characteristics of financial information, cost versus benefit, materiality in general and the inherent limitations of an audit. Total possible marks Maximum full marks

© The Institute of Chartered Accountants in England and Wales 2009

22½ 18

Page 11 of 15

Professional Stage – Financial Accounting – September 2009

Question 4

Total marks: 21

General comments This is a consolidation question. A consolidated income statement is required along with an extract from the consolidated statement of financial position showing equity. A subsidiary has been acquired during the period and the consolidation includes an associate. Adjustments are required for differences between the fair value and carrying amount of the subsidiary acquired and inter-company trading has taken place between the parent and both a subsidiary and the associate companies. (a) Galletas plc Consolidated income statement for the year ended 30 June 2009 £ 2,291,300 (1,238,125)

Revenue (W2) Cost of sales (W2)

1,053,175 (263,980)

Gross profit Operating expenses (W2) Profit from operations Share of profits of associate (W6)

789,195 51,383 840,578 (240,685)

Profit before tax Income tax expense (W2)

599,893

Profit for the period Attributable to: Equity holders of Galletas plc (Bal) Non-controlling interest (W5)

517,579 82,314 599,893

Consolidated statement of financial position (extract)

Share capital Retained earnings (W7)

£ 4,000,000 1,879,116

Non-controlling interest (W8)

5,879,116 2,070,600

Total equity

7,949,716

© The Institute of Chartered Accountants in England and Wales 2009

Page 12 of 15

Professional Stage – Financial Accounting – September 2009

Workings (All figures in £’000) W1 Group Structure Galletas

900 / 3,000 = 30%

Arico 2.1 / 3.5 = 60%

85% 1 Apr 09 (3/12 months)

Vilaflor Masca

W2 Consolidation schedule Galletas Revenue Cost of sales Per question PURP (W4) PURP (W4) Operating expenses Per question Fair value adj (dep) (W3) Tax PAT

1,410,500

870,300

Masca 3/12 160,000

(850,000) (9,750) (2,700)

(470,300)

(54,875)

(103,200)

(136,000)

(137,100)

(79,200) 184,800

(23,780) (1,000) (24,385) 55,960

Vilaflor

Total

Adjustments (149,500)

2,291,300

149,500

(1,238,125)

(263,980) (240,685)

W3 Fair value adjustment Additional fair value £320,000 Buildings £320,000 x 50% = £160,000 Additional depreciation charge in year £160,000 / 40 years x 3/12 months = £1,000 W4 Unrealised profit Arico Vilaflor 207,000 180,000 27,000

149,500 130,000 19,500

115% 100% 15%

Vilaflor - £19,500 x ½ = £9,750 Arico - £27,000 x 1/3 = £9,000 Galletas share of Arico PURP - £9,000 x 30% = £2,700 W5 Non-controlling interest Vilaflor Ltd (40% x £184,800 (W2)) = £73,920 Masca Ltd (15% x £55,960 (W2)) = £8,394 Non-controlling interest = £73,920 + £8,394 = £82,314

© The Institute of Chartered Accountants in England and Wales 2009

Page 13 of 15

Professional Stage – Financial Accounting – September 2009

W6 Associate Profit for the year

£ 204,610

Galletas’ share x 30% Less: impairment for year Share of associate’s profit

61,383 (10,000) 51,383

W7 Consolidated retained earnings Galletas plc – c/fwd Less: PURP with Vilaflor (W4) Less: PURP with Arico (W4) Vilaflor Ltd (60% x (580 – 195)) Masca Ltd (85% x 55,960) (W2) Arico Ltd ((30% x (340 – 130)) – 10(imp))

£ 1,560,000 (9,750) (2,700) 231,000 47,566 53,000 1,879,116

W8 Non-controlling interest - SFP Vilaflor Ltd (£4,080,000 x 40%) Masca Ltd Net assets per question Fair value adjustment (increase) Less: extra depreciation on FV adj

£ 1,632,000 2,605,000 320,000 (1,000) 2,924,000

NCI – 2,924,000 x 15%

438,600 2,070,600

The average mark on this question was pleasing with candidates’ performance much improved since the last time a consolidated income statement was set. Almost all candidates made a good attempt at the consolidated income statement itself – although attempts at the extracts from the consolidated statement of financial position varied, showing once again that weaker candidates struggle when required to move away from a pure “learnt technique” style of requirement. The consolidated income statement showed a well-honed exam technique, with candidates clearly understanding the principles of consolidation, adding together the results for the subsidiaries line by line and only including Masca’s results for the three months since acquisition. Presentation was generally good with the majority of candidates showing the two “attributable to” figures although very few struck a sub-total for operating profit before adding the share of the associate’s profits. Several common errors were in respect of the associate, including failing to take 30% of the PURP in respect of sales to the associate, setting the PURP in respect of sales to the associate against the share of the associate’s profits instead of against the parent’s profits, adjusting sales and cost of sales for sales between the associate and the parent and setting the impairment in respect of the associate against the parent’s profits instead of against the parent’s share of the associate’s profits. Other common errors included calculating the additional depreciation on the fair value adjustment based on a whole year instead of just three months, setting the additional depreciation against the parent instead of against the subsidiary and classifying the additional depreciation as a cost of sale instead of as an operating expense (as it related to the company’s head office). Errors on the consolidated statement of financial position extract were more common, with a few candidates making little or no attempt at this. A small minority of candidates produced a consolidated statement of

© The Institute of Chartered Accountants in England and Wales 2009

Page 14 of 15

Professional Stage – Financial Accounting – September 2009

changes in equity instead of an extract from the consolidated statement of financial position. Many candidates produced lengthy (and largely unnecessary) net assets workings in the way they would have practised for a question requiring a complete consolidated statement of financial position. Errors on this part included when calculating consolidated retained earnings, producing a new working to arrive at post acquisition retained earnings for the subsidiary acquired during the year when the figure had already been calculated in the consolidation schedule, omitting to reduce consolidated retained earnings by the PURPs and when calculating non-controlling interest, failing to adjust the net assets of the subsidiary acquired during the year by the fair value adjustment and additional depreciation thereon. Even if those adjustments had been made in a separate net assets working, many candidates failed to use the figure from that working and took the non-controlling interest share of just the net assets at the year end.

Total possible marks Maximum full marks

© The Institute of Chartered Accountants in England and Wales 2009

21 21

Page 15 of 15

Financial Accounting Professional Stage – December 2009

PROFESSIONAL STAGE FINANCIAL ACCOUNTING – OT EXAMINER’S COMMENTS

The performance of candidates in the December 2009 objective test questions section for the Professional Stage Financial Accounting paper was good. Candidates performed well across all syllabus areas. When practising OT items, care should always be taken to ensure that the principles underlying any particular item are understood rather than the answer learned from previous experience. In particular, candidates should ensure that they read all items very carefully. The following table summarises how well* candidates answered each syllabus content area. Syllabus area

Number of questions

Well answered

Poorly answered

LO1

4

4

0

LO2

3

3

0

LO3

8

7

1

Total

15

14

1

*If 50% or more of the candidates gave the correct answer, then the question was classified as ‘well answered’. The only poorly answered question was on LO3 (the preparation of consolidated financial statements). Comments on this item are: Item 1 This item tested what amount would be shown in respect of dividends paid to the non-controlling interest in a consolidated statement of cash flows. The information given included opening and closing balances on the noncontrolling interest “account”, the profit attributable to the non-controlling interest for the year and details of a new subsidiary acquired during the year. The most commonly selected incorrect answer indicated that candidates ignored the impact of the subsidiary acquired during the year on the non-controlling interest account.

© The Institute of Chartered Accountants in England and Wales 2010

Page 1 of 19

Financial Accounting Professional Stage – December 2009

MARK PLAN AND EXAMINER’S COMMENTARY The marking plan set out below was that used to mark this question. Markers were encouraged to use discretion and to award partial marks where a point was either not explained fully or made by implication. More marks were available than could be awarded for most requirements. This allowed credit to be given for a variety of valid points which were made by candidates.

Question 1

Total Marks: 19

General comments This question tested the preparation of single entity financial statements (in this case an income statement and a statement of financial position) from a trial balance plus a number of adjustments. Adjustments included prepayments and accruals, the correction of a suspense account, an adjustment for inventories held by third parties, two provisions and an asset held for sale. Moreton Ltd Income statement for the year ended 30 September 2009

Revenue (2,885,500 – 30,000) Cost of sales (W1) Gross profit Distribution costs (W1) Administrative expenses (W1) Profit from operations Finance cost (200,000 x 5%) Profit before tax Income tax expense Profit for the period

£ 2,855,500 (1,879,900) 975,600 (309,600) (637,400) 28,600 (10,000) 18,600 (4,000) 14,600

Statement of financial position as at 30 September 2009

Assets Current assets Inventories (W1) Trade and other receivables (978,400 + 56,000 – 30,000) Other receivables (W2) Cash and cash equivalents

£ 176,000 1,004,400

Non-current asset held for sale (W4)

45,000 820 1,226,220 21,000

Total assets

1,247,220

© The Institute of Chartered Accountants in England and Wales 2010

Page 2 of 19

Financial Accounting Professional Stage – December 2009

Equity and liabilities Equity Ordinary share capital Share premium account Retained earnings (W3)

£ 100,000 20,000 193,500 313,500

Non-current liabilities Borrowings Current liabilities Trade and other payables (578,620 + 75,000 + (10,000 – 7,500)) Taxation Borrowings Provisions (W2)

200,000 656,120 4,000 13,600 60,000 733,720 1,247,220

Total equity and liabilities

Tutorial note Equal credit was given if candidates assumed that the £7,500 interest paid in the trial balance related to the overdraft and therefore accrued the whole £10,000 interest on the loan. Marks were also awarded if the impairment of £9,000 and/or the provision expense of £15,000 were shown separately on the face of the income statement (instead of within cost of sales and cost of sales or administrative expenses respectively) on the grounds that either of these amounts would have a significant impact on the profit for the period. Workings (1) Allocation of expenses

Per Q Rent (70:30) Opening inventories Prepayments and accruals Closing inventories (156,000 + 20,000) Movement on provision (W2) Movement on provision (W2) Amortisation/impairment charges (2,000 + 9,000) (W4)

Cost of sales Administrative expenses £ £ 1,345,600 456,700 552,300 236,700 134,000 (56,000) (176,000) 15,000 (2,000) 11,000 1,879,900

637,400

Distribution costs £ 234,600

75,000

309,600

(2) Provision for legal claims At 1 October 2008 Settled at Released to IS

£ 27,000 (25,000) 2,000

New provision at 30 September 2009 (most likely outcome) Counter claim @ 75% Charge to IS

60,000 (45,000) 15,000

(3) Retained earnings At 1 October 2008 Profit for the period At 30 September 2009

© The Institute of Chartered Accountants in England and Wales 2010

£ 178,900 14,600 193,500

Page 3 of 19

Financial Accounting Professional Stage – December 2009

(4) Patent held for sale Cost Accumulated amortisation to 30 September 2008 Amortisation to date of classification as held for sale ((40,000 ÷ 10) x 6/12) Carrying amount at classification as held for sale Fair value less costs to sell (22,000 – 1,000) Impairment

£ 40,000 (8,000) (2,000) 30,000 (21,000) 9,000

As in previous sittings, candidates were clearly very well-prepared for this type of question. Almost all candidates produced a well-laid out income statement and statement of financial position with all narrative and sub-totals completed. Although some candidates lost presentation marks for the statement of financial position by not adding across numbers in brackets or failing to complete sub-totals and/or totals on their statements or by having incomplete or abbreviated narrative, presentation for this statement was much improved from previous sittings. Others lost presentation marks for failing to include a sub-total for profit from operations on their income statement. As ever, candidates should remember that this type of question requires financial statements to be in a form suitable for publication. Although many workings, in particular the cost matrix and impairment working, were clearly laid out, a few candidates’ workings were disorganised, untidy and therefore hard to follow, making it difficult to establish candidates’ approaches where they had not calculated the correct figure. It is particularly difficult to follow workings which use little or no narrative, or costs workings done on the face of the income statement. Most candidates were able to deal with the more straightforward adjustments such as the prepayments and accruals, adjusting the closing inventories for goods held by a customer on sale or return, the settlement of the opening provision and the creation of a new provision at the year end and the income tax charge/liability. However, a number of candidates failed to complete the double entry on their adjustments, for example: • adding prepayments to expenses (a debit) and showing them as a current asset (another debit) on the statement of financial position • correctly adjusting trade and other receivables by £30,000 for the goods held by a customer on sale or return but failing to make the corresponding adjustment to revenue • correctly recognising that a provision for £60,000 should be set up and charging the £60,000 to the income statement, but failing to recognise the provision itself within current liabilities • recognising an amount for accrued finance costs in the income statement, but failing to recognise the same amount on the statement of financial position. The majority of candidates arrived at the correct figure for the impairment on the asset classified as held for sale although a few calculated accumulated amortisation incorrectly. However, not all candidates who arrived at an impairment loss then charged this loss to the income statement. Others took the impairment loss to the income statement but failed to take the amortisation charge for the year to the income statement or classified one under cost of sales, the other as an administrative expense. The most disappointing area in candidates’ answers related to the two provisions, with a number of candidates clearly confused about the appropriate double entry. Errors included: • Not releasing the £2,000 over-provision brought forward to the income statement (with a number of candidates instead adding it to the closing provision). • Calculating the closing provision on a weighted average basis rather than taking the most likely outcome (as this was a single obligation). • Not recognising the contingent asset even though it was “virtually certain” to be recovered. • Netting the provision and the contingent asset off on the face of the statement of financial position instead of presenting them separately. • Recognising the provision and contingent asset on the statement of financial position, but ignoring the impact on the income statement.

© The Institute of Chartered Accountants in England and Wales 2010

Page 4 of 19

Financial Accounting Professional Stage – December 2009

Errors in dealing with the other adjustments included the following: • Failing to disclose the asset held for sale correctly on the statement of financial position (within current assets after a separate sub-total for all other current assets). • Offsetting the cash in hand against the overdraft, sometimes even showing a net positive cash balance. • Failing to make the correct (or any) adjustment for the goods held by a customer on sale or return, with a number of candidates deducting these goods from closing inventories rather than adding them. • Adding prepayments to expenses and deducting accruals or adjusting for both in the same direction. • Reflecting the income tax charge for the year in the income statement but not showing the corresponding liability. Total possible marks Maximum full marks

© The Institute of Chartered Accountants in England and Wales 2010

21 19

Page 5 of 19

Financial Accounting Professional Stage – December 2009

Question 2

Total Marks: 25

General comments This question covered various aspects of property, plant and equipment. Part (a) required the preparation of a property, plant and equipment “table”, with the movements on property, plant and equipment during the year including additions, disposals, a self-constructed asset, an impairment and a revaluation. Part (b) required the calculation of the closing balance on the revaluation surplus. Part (c) tested the differences between IAS 16 and UK GAAP. Part (d) covered the information needs of users in the context of property, plant and equipment. Bushley plc (a) Note showing movements on PPE for the year ended 30 September 2009

Cost or valuation At 1 October 2008 Revaluation (1,800,000 + 700,000 – 1,300,000) Additions (56,000 + 24,000) Disposals At 30 September 2009 Depreciation At 1 October 2008 Revaluation Disposals (W3) Impairment losses (W1) Charge for year (W2) At 30 September 2009 Carrying amount At 30 September 2009 At 1 October 2008

Land and buildings £

Plant and machinery £

Under construction £

1,300,000 1,200,000

870,000 -

-

2,500,000

120,000 (56,000) 934,000

80,000 80,000

200,000 (56,000) 3,514,000

423,000 (33,600) 11,200 183,200 583,800

-

603,000 (180,000) (33,600) 11,200 200,700 601,300

180,000 (180,000) 17,500 17,500

2,482,500 1,120,000

350,200 447,000

-

80,000 -

Total £ 2,170,000 1,200,000

2,912,700 1,567,000

Workings (1) Impaired machine

Cost on 1 October 2006 Depreciation to 30 September 2009 (78,000 x 20% x 3) Carrying amount at 30 September 2009 Recoverable amount Impairment

© The Institute of Chartered Accountants in England and Wales 2010

£ 78,000 (46,800) 31,200 (20,000) 11,200

Page 6 of 19

Financial Accounting Professional Stage – December 2009

(2) Depreciation charges for year On plant and equipment On new machine (120,000 x 20% x 6/12) On machine disposed of (56,000 x 20% x 9/12) On machines held for the whole year ((870,000 – 56,000) x 20%)

£ 12,000 8,400 162,800 183,200

On buildings (700,000 ÷ 40)

£ 17,500

(3) Accumulated depreciation on machine disposed of To 30 September 2008 (56,000 x 20% x 3)

£ 33,600

The majority of candidates produced a well-laid out property, plant and equipment “table” showing that they knew what this note would look like in a set of published financial statements, although a number did not finish the note completely. The most common presentation errors were: • Not having a separate total column. • Not showing carrying amounts brought forward and carried forward (or using UK GAAP terminology of “net book value”). • Not having a separate column (or line) for assets under construction. Most candidates dealt correctly with the adjustments to cost, including the revaluation uplift, additions and disposals. Errors were more common in respect of the adjustments to accumulated depreciation, although most candidates arrived at the correct figure for the impairment loss on the plant and the depreciation charge for the year on the revalued buildings. Common errors included the following: • Not including the purchases and wages spent on the self-constructed asset within additions, and if it was included, not separately disclosing that amount as a self-constructed asset. • Omitting to back out the opening accumulated depreciation on the land and buildings revalued during the year. Some candidates added the net surplus of £1,380,000 to cost rather than adjusting both cost and accumulated depreciation. • Using the incorrect fraction (ie number of months) when calculating depreciation charges on the plant purchased and disposed of during the year. • Including the construction costs when calculating the current year depreciation charge. • Omitting to back out the accumulated depreciation on the plant sold during the year. Total possible marks Maximum full marks

© The Institute of Chartered Accountants in England and Wales 2010

14 13

Page 7 of 19

Financial Accounting Professional Stage – December 2009

(b) Balance on the revaluation surplus at 30 September 2009 £ Valuation on 1 October 2008 (1,800,000 + 700,000) Carrying amount of land and buildings at 1 October 2008 Less: Transfer to retained earnings Depreciation based on revalued amount Depreciation based on historic cost ((300,000 – 180,000) ÷ 40)

£ 2,500,000 (1,120,000) 1,380,000

17,500 (3,000) (14,500) 1,365,500

There were very few completely correct answers to this part, although most candidates recognised that a transfer between the revaluation surplus and retained earnings needed to be made, even if they calculated this incorrectly. The most common errors included the following: • Calculating the surplus on initial revaluation as just the cost uplift of £1.2 million, rather than the net uplift of £1,380,000 (sometimes in direct contravention of entries made in Part (a)). • Including only the revaluation surplus on the land and not the building (or vice versa). • Calculating depreciation on historic cost as £6,000, ie as it would have been originally calculated (300,000 divided by the original useful life of 50 years), failing to appreciate that the buildings’ remaining estimated useful life had been reassessed at 40 years. Total possible marks Maximum full marks

© The Institute of Chartered Accountants in England and Wales 2010

3 3

Page 8 of 19

Financial Accounting Professional Stage – December 2009

(c) Differences between IAS 16 and UK GAAP When assets are revalued IAS 16 requires the use of fair values, which will take account of alternative uses. UK GAAP (FRS 15) bases valuations on existing use values. UK GAAP specifies a maximum period of five years between full valuations and an interim valuation every three years. IAS 16 does not specify a maximum period and the timing of revaluations depend on changes in market values. UK GAAP requires impairment losses to be debited first against any revaluation surplus in respect of that asset unless it reflects a consumption of economic benefits. IAS 16 does not include such a limitation. Under UK GAAP when residual values are reassessed they are based on prices prevailing at the date of acquisition. Under IAS 16 reassessment is based on current prices. UK GAAP requires annual impairment reviews for assets which are not depreciated or are depreciated over more than 50 years. IAS 16 does not include such a requirement. There were some good answers to this part, showing that many candidates had spent time learning the differences between IFRS and UK GAAP. Most answers included the difference in relation to the basis of revaluations, although a minority of candidates got this the wrong way round. Candidates who included a discussion about residual values generally missed the point about the differences in prices on which reassessed residual vales are based. Most candidates knew that there was a difference in treatment in relation to impairments but often got confused with whether this was dependent on the impairment being as a result of a consumption of economic benefits or not. Again, some candidates got this difference the wrong way round. Total possible marks Maximum full marks

© The Institute of Chartered Accountants in England and Wales 2010

6½ 4

Page 9 of 19

Financial Accounting Professional Stage – December 2009

(d) How information re PPE meets the needs of users Financial position The financial position of an entity is affected by the economic resources it controls, its financial structure, its liquidity and solvency and its capacity to adapt to changes in the environment in which it operates. Information about the total carrying amount of property, plant and equipment (PPE) as given on the face of an entity’s statement of financial position gives the user an indication of the resources the entity has at its disposal in terms of tangible assets held for long-term use in the business. Revaluation figures are more relevant than cost. That figure will be broken down in the note to the financial statements as produced in (a) above. This indicates the type of PPE held by the entity which may add further to an understanding of resource. This note also shows the changes in financial position in the year For example, land and buildings might be held for its investment potential, as well as being used for office/factory space. Plant will be used to generate future revenues. Equipment could be used for the generation of future revenues or for the entity’s own use, perhaps for administrative purposes. The fact that the amount of leased assets forming part of the total PPE figure is disclosed, shows that these assets have a future cost in terms of lease payments – affecting the liquidity and solvency of the entity. The “capital commitments” note showing the future purchases of PPE to which the entity is committed, indicates a requirement for future finance. The accounting policy note shows the valuation model used and depreciation methods, which allow comparison to other entities. Financial performance Information about financial performance, in particular profitability, is needed in order to assess potential changes in the economic resources that it is likely to control in the future. Disclosure of the annual depreciation charge shows the “cost” of using the assets.. Disclosure of gains/losses on disposal could indicate problems with the depreciation method or where value is greater than carrying amount. Impairment losses may indicate underlying issues, such as underprovision of depreciation, or a downturn in a particular market sector (which might affect future performance). Changes in financial position Changes in financial position are shown in a statement of cash flows. This allows users to assess the ability of the entity to generate cash and its need to use what is generated. Users will be able to see, via the statement of cash flows, PPE purchased during the year and cash inflows from PPE disposed of. If little PPE is purchased and much disposed of the user may be concerned about the future of the entity.

© The Institute of Chartered Accountants in England and Wales 2010

Page 10 of 19

Financial Accounting Professional Stage – December 2009

Answers to Part (d) were the most disappointing and sometimes non-existent. A number of candidates appeared to have rote learnt various information about the Framework (and in particular the qualitative characteristics) and wasted time simply producing a page of irrelevant comments, which gained no marks. Candidates need to be reminded to answer the question set, not the question they wished had been set. A number of candidates focused on what information users might need in respect of property, plant and equipment as proposed to what information is actually provided. Those candidates who demonstrated knowledge of the various disclosures in relation to property, plant and equipment and where they appear in a set of published financial statements, and gave some thought as to how such information might assist users, scored well. As can be seen from the mark plan there were a significant number of marks available, although those for just copying out of the open book text were limited. Total possible marks Maximum full marks

© The Institute of Chartered Accountants in England and Wales 2010

14½ 5

Page 11 of 19

Financial Accounting Professional Stage – December 2009

Question 3

Total Marks: 15

General comments This was a question mixing three topics. Part (a) required the calculation of the profit from discontinued operations in respect of a subsidiary disposed of during the year. Part (b) required extracts from single entity financial statements in respect of debt and equity issued during the year and the correction of a prior period error. Bredon Ltd (a) Profit from discontinued operations for the year ended 30 September 2009 £ Profit on disposal: Sale proceeds Less: Share of net assets at date of disposal (80% x (100,000 + 717,000 + 16,000)) Less: Carrying amount of goodwill at date of disposal Arising on acquisition Impairments to date

Profit for the period (32,000 x 6/12) Profit on discontinued operations

£ 700,000 (666,400)

22,800 (5,000) (17,800) 15,800 16,000 31,800

This part of the question was generally well-answered, with many candidates arriving at the correct answer. Where mistakes were made they included the following: • Not including share capital in the net assets at disposal. • Not including the correct proportion of the profit for the period in the net assets at disposal (or not including it at all). • Failing to also add the above figure to the profit on sale of the shares to arrive at the final profit on discontinued operations. A minority of candidates seemed to have little idea of how to calculate this figure, producing apparently random calculations involving taking 80% of various figures, and scored poorly. A significant number of candidates wasted time drawing a group structure diagram when the percentage holding was clearly given in the question. Total possible marks Maximum full marks

© The Institute of Chartered Accountants in England and Wales 2010

3½ 3

Page 12 of 19

Financial Accounting Professional Stage – December 2009

(b)

(i) Statement of changes in equity for the year ended 30 September 2009

At 1 October 2008 Correction of prior period error (45,000 + 5,000) Restated balance Issue of ordinary shares (200,000 x 0.50) Issue of irredeemable preference shares (50,000 x 0.10) Dividends (700,000 x 10p) (50,000 x 4% x 3/12) Total comprehensive income for the year (W) At 30 September 2009

Ordinary share capital £

Preference share capital (irredeemable) £

500,000

Share premium

Retained earnings

Total

£

£

£

-

-

2,560,000

3,060,000

-

-

-

500,000 200,000

-

100,000

50,000

5,000

-

-

-

(70,500)

(70,500)

-

-

-

563,500

563,500

700,000

50,000

105,000

3,003,000

3,858,000

(50,000) 2,510,000 -

(50,000) 3,010,000 300,000 55,000

Working TCI per Q Finance cost (100,000 x 3% x 6/12) Amortisation added back

© The Institute of Chartered Accountants in England and Wales 2010

£ 560,000 (1,500) 5,000 563,500

Page 13 of 19

Financial Accounting Professional Stage – December 2009

(b) (ii) Extracts from the financial statements for the year ended 30 September 2009 Income statement for the year ended 30 September 2009 Finance cost

£ 1,500

Statement of financial position as at 30 September 2009 £ Equity and liabilities Equity Ordinary share capital Preference share capital (irredeemable) Share premium account Retained earnings Non-current liabilities Preference share capital (redeemable) Current liabilities Dividends payable

700,000 50,000 105,000 3,003,000

100,000

70,000

Statement of cash flows for the year ended 30 September 2009 £ Cash flows from operating activities Interest paid Cash flows from financing activities Proceeds from issue of share capital Proceeds from issue of long-term borrowings Dividends paid

© The Institute of Chartered Accountants in England and Wales 2010

(1,500)

355,000 100,000 (500)

Page 14 of 19

Financial Accounting Professional Stage – December 2009

In Part (i) most candidates produced some kind of table for their statement of changes in equity, although it was rare to see a complete table with all four columns, plus a total column and all amounts carried forward. However, it was surprising how many combined figures for ordinary and irredeemable preference shares in a single column, although most arrived at the correct figures for the shares issued during the year, including the amounts to be posted to the share premium account. Pleasingly, few candidates included the redeemable preference shares in equity. Most candidates remembered to label the profit for the period in the table as “total comprehensive income” but fewer adjusted the original figure of £560,000 given in the question for the unadjusted items of the finance cost on the redeemable preference shares and the backing out of the amortisation for the year on the incorrectly capitalised intangible asset. Other common errors included the following: • Using the incorrect fraction (ie number of months) when calculating the dividends on the preference shares. • Omitting to deal with the prior period error, or deducting £45,000 instead of £50,000. • Failing to show a sub-total for the restated balance after the prior period adjustment. In Part (ii) there were some well laid out extracts, with a number of candidates using the correct subheadings in both the statement of financial position (ie equity, current and non-current liabilities) and the statement of cash flows (ie distinguishing between operating and financing cash flows). In fact the majority of those who attempted this part scored well, as a good proportion of the marks were available for simply taking own figures from Part (i) into well-presented extracts. These were therefore relatively easy marks but were missed by a number of candidates. Common errors included the following: • Failing to distinguish between the dividends payable (in the statement of financial position) and the dividends paid (in the statement of cash flows). • Using the incorrect bracket convention in the statement of cash flows. A minority of candidates wasted significant time by trying to incorporate the impact of the disposal in transaction (1) into this part of the question when the requirement referred only to the transactions set out in (2) and (3). Total possible marks Maximum full marks

© The Institute of Chartered Accountants in England and Wales 2010

14½ 12

Page 15 of 19

Financial Accounting Professional Stage – December 2009

Question 4

Total Marks: 21

General comments This was a typical consolidated statement of financial position question, featuring one subsidiary and one associate (acquired during the year). Adjustments were typical of this type of question and included a fair value adjustment on acquisition, intra-group balances and transactions and impairment write-downs. Stow plc Consolidated statement of financial position as at 30 September 2009 £ Assets Non-current assets Property, plant and equipment (4,175,500 + 2,678,500 + 1,000,000) Intangibles (W3) Investments in associates (W7) Current assets Inventories (1,237,000 + 1,050,000 – 90,000 (W6) – (62,500 x 40%) (W6)) Trade and other receivables (976,500 + 750,000 – 540,000) (W6)) Cash and cash equivalents (9,500 + 1,500)

7,854,000 370,000 909,240 9,133,240 2,172,000 1,186,500 11,000 3,369,500 12,502,740

Total assets Equity and liabilities Equity attributable to owners of Stow plc Ordinary share capital Share premium account Retained earnings (W5) Non-controlling interest (W4) Total equity Current liabilities Trade and other payables (766,000 + 637,800 – 540,000) (W6)) Taxation (280,000 + 140,000) Total equity and liabilities

© The Institute of Chartered Accountants in England and Wales 2010

£

3,000,000 1,000,000 6,296,500 10,296,500 922,440 11,218,940 863,800 420,000 1,283,800 12,502,740

Page 16 of 19

Financial Accounting Professional Stage – December 2009

Workings (1) Group structure

400 Stow plc

1,600

1,000

= 40%

= 80%

2,000

Bourton Ltd

Naunton Ltd

(2) Net assets – Bourton Ltd Year end Share capital Share premium Retained earnings Per Q PURP (W6) FV adj – land

£ 2,000,000 500,000

Acquisition £ 2,000,000 500,000

1,202,200 (90,000) 1,000,000 4,612,200

1,575,000 1,000,000 5,075,000

Post acq £ -

(462,800)

(3) Goodwill – Bourton Ltd

Consideration transferred Non-controlling interest at acquisition (5,075,000 (W2) x 20%) Net assets at acquisition (W2) Impairments to date (50,000 + 20,000)

© The Institute of Chartered Accountants in England and Wales 2010

£ 4,500,000 1,015,000 (5,075,000) 440,000 (70,000) 370,000

Page 17 of 19

Financial Accounting Professional Stage – December 2009

(4) Non-controlling interest – Bourton Ltd £ 922,440

Share of net assets (4,612,200 (W2) x 20%) (5) Retained earnings Stow plc Bourton Ltd ((462,800) (W2) x 80%) Naunton Ltd ((1,298,100 – 875,000 – 62,500) (W6)) x 40%)) Less Impairments to date (70,000 (W3) + 10,000 (W7))

6,602,500 (370,240) 144,240 (80,000) 6,296,500

(6) PURP

% SP Cost GP X½

150 (100) 50

Bourton Ltd £

Naunton Ltd £

540,000 (360,000) 180,000 90,000

375,000 (250,000) 125,000 62,500

(7) Investments in associates – Naunton Ltd Cost Add: Share of post acquisition increase in net assets ((1,298,100 – 875,000)) x 40%)) Less: Impairment to date

© The Institute of Chartered Accountants in England and Wales 2010

£ 750,000 169,240 (10,000) 909,240

Page 18 of 19

Financial Accounting Professional Stage – December 2009

Candidates were clearly very well prepared for this question and generally scored highly. Almost all candidates demonstrated a sound technique, following that set out in the learning materials. The most common errors were in relation to the associate, in either the retained earnings working and/or the investment in associate working itself or in relation to the calculation of the provisions for unrealised profit. Common errors included the following: • • • • • • • • •



• •

In the net assets table for the subsidiary, only including the fair value adjustment in the year-end column and/or deducting the fair value adjustment instead of adding it. Including the fair value adjustment in the net assets table but failing to uplift the value of property, plant and equipment in the consolidated statement of financial position by the same amount. Failing to include the share premium account in the net assets table for the subsidiary. Failing to adjust both receivables and payables (or in some cases, either) for the invoice value of the sale of goods from the subsidiary to the parent, with a number of candidates making the adjustment at cost. Arriving at provisions for unrealised profits in respect of the subsidiary and the associate on different bases when the same cost structure was specified in the question. Failing to calculate any provision for unrealised profit in respect of the goods sold by the associate to the parent. Calculating unrealised profit based on the full invoice value, as opposed to only half of that value, when the question clearly stated that only half of the goods remained in year-end inventory. Taking the cost figures given in the question for the intra-group sales as being the selling price of the goods and hence calculating incorrect provisions for unrealised profit. Pleasingly, many candidates correctly adjusted for the group share of the provision for unrealised profit arising on goods sold by the associate to the parent against retained earnings and inventory, but many also made an adjustment against the carrying amount of the associate. Others calculated an initial post-acquisition profit figure for the associate less a 40% share of the provision for unrealised profit but then adjusted that total by 40%, consequently scaling down the provision for unrealised profit twice. This error occurred most often when candidates produced a(n) (unnecessary) net assets table for the associate. Taking only six-twelfths of the movement on the associate’s profit to retained earnings and the investment in associate working, failing to recognise that although the acquisition of the associate did indeed occur half way through the year the figure given for retained earnings was at the date of acquisition, such that there was no need to time-apportion any of the figures given. Not adjusting for the accumulated impairments in the group retained earnings working, instead adjusting only for the impairments which had arisen during the current year. Having correctly arrived at a post-acquisition loss for the subsidiary in a net assets table, turning this into a post-acquisition profit when taking 80% of this figure to group retained earnings.

A number of candidates failed to provide workings for assets and liabilities on the face of the consolidated statement of financial position. Where these figures were incorrect no partial marks could then be awarded. Candidates must show their workings in all cases so that partial credit can be given. Presentation of the consolidated statement of financial position was generally good, although very few candidates gained the presentation mark which was available for clearly disclosing the non-controlling interest as a separate component of equity. Total possible marks Maximum full marks

© The Institute of Chartered Accountants in England and Wales 2010

21 21

Page 19 of 19

Financial Accounting – Professional Stage – March 2010 PROFESSIONAL STAGE FINANCIAL ACCOUNTING – OT EXAMINER’S COMMENTS

The performance of candidates in the March 2010 objective test questions section for the Professional Stage Financial Accounting paper was good. Candidates performed well across all syllabus areas. When practising OT items, care should always be taken to ensure that the principles underlying any particular item are understood rather than rote learning the answer. In particular, candidates should ensure that they read all items very carefully. The following table summarises how well* candidates answered each syllabus content area. Syllabus area

Number of questions

Well answered

Poorly answered

LO1

2

1

1

LO2

7

6

1

LO3

6

6

0

Total

15

13

2

*If 50% or more of the candidates gave the correct answer, then the question was classified as ‘well answered’. Brief comments on the two poorly answered questions, which covered LO1 (accounting and reporting concepts) and LO2 (preparation of single company financial statements), are below (this paper was marked under the new electronic marking system and no further information regarding responses was available): Item 1 This item asked which roles are undertaken by the International Accounting Standards Committee Foundation (IASCF). Candidates clearly do not understand the structure that surrounds and supports the International Accounting Standards Board. Item 2 This item, required candidates to identify which adjustments should be recognised as a prior period error. Four short scenarios were provided which included a settled legal claim, a computational error, a fraud and a revised tax liability.

© The Institute of Chartered Accountants in England and Wales 2010

Page 1 of 13

Financial Accounting – Professional Stage – March 2010

MARK PLAN AND EXAMINER’S COMMENTARY The mark plan set out below was used to mark these questions. Markers are encouraged to use discretion and to award partial marks where a point was either not explained fully or made by implication. More marks are available than could be awarded for each requirement, where indicated. This allows credit to be given for a variety of valid points, which are made by candidates.

Question 1 Overall marks for this question can be analysed as follows:

Total: 18

General comments This question is a typical question testing the preparation of an income statement and statement of financial position from a trial balance. A number of adjustments were required, including the reversal of a provision, an inventory valuation issue, an adjustment for the over provision of tax and deferred revenue. (a) Karonga plc – Statement of financial position as at 31 December 2009 £ £ ASSETS Non-current assets Property, plant and equipment (W5) 943,435

Current assets Inventories (W3) Trade receivables (1,075,000 – 60,750 (W4)) Cash and cash equivalents

1,161,000 1,014,250 189,500 2,364,750

Total assets EQUITY AND LIABILITIES Equity Ordinary share capital Retained earnings (28,090 + 227,895)

3,308,185

1,325,000 255,985 1,580,985

Non-current liabilities Bank loan Current liabilities Trade and other payables (583,700 + 12,500(W1)) Taxation (W5)

1,025,300

596,200 105,700 701,900

Total equity and liabilities

© The Institute of Chartered Accountants in England and Wales 2010

3,308,185

Page 2 of 13

Financial Accounting – Professional Stage – March 2010 Karonga plc – Income Statement for year ended 31 December 2009 £ Revenue (W1) 6,196,400 Cost of sales (W2) (3,506,501) Gross profit Administrative expenses (W2)

2,689,899 (2,315,434)

Operating profit Finance costs Profit before tax Income tax expense (105,700 – 8,300)

374,465 (49,170) 325,295 (97,400)

Net profit for the period

227,895

Note: Marks will be awarded if items are included in a different line item in the income statement provided that the heading used is appropriate.

W1 Revenue adjustment

Trial balance – revenue Fitness machine deposits (250 x £50)

£ 6,208,900 (12,500) 6,196,400

W2 Expenses

Trial balance Opening inventory Less: closing inventory (W3) Bad debt reversal (W4) Depreciation charge – buildings (12,710 (40% / 60%) Depreciation charge – plant & equipment Provision reversal

Admin expenses £ 2,324,000

(1,650) 5,084 (12,000) 2,315,434

Cost of sales £ 3,553,100 1,093,800 (1,161,000) 7,626 12,975 3,506,501

W3 Inventory adjustment

Closing inventory Net realisable value write down (£20 - £15) x 500 items

£ 1,163,500 (2,500) 1,161,000

W4 Bad debt

Opening allowance Movement in year (balancing figure)

£ 62,400 (1,650)

Closing allowance (53,750 + 7,000)

60,750

© The Institute of Chartered Accountants in England and Wales 2010

Page 3 of 13

Financial Accounting – Professional Stage – March 2010

W5 Property, plant and equipment

Trial balance – L&B Trial balance – P&E

Cost £ 985,500 103,800

Depreciation charge for year (103,800 / 8yrs) Depreciation charge for year ((985,500 – 350,000) / 50yrs) At 31 December 2009

Acc dep £ 88,970 31,210 12,975 12,710

1,089,300

145,865

943,435

As in previous sittings, candidates were clearly very well-prepared for this type of question. Almost all candidates produced a well-laid out income statement and statement of financial position with all narrative and sub-totals completed. Some candidates lost presentation marks for the statement of financial position by not adding across numbers in brackets, failing to complete sub-totals or by having incomplete or abbreviated narrative. On the income statement the most common presentational failing was to not include a sub-total for profit from operations. However, overall presentation is improving with each sitting. As ever, candidates should remember that this type of question requires financial statements to be in a form suitable for publication. Workings generally were set out clearly, with the standard cost matrix generally being produced. Candidates must remember that if they do not provide clear workings for calculations and their final answer is incorrect they risk gaining no marks for a working that may be worth 2 or 3 marks. Clear workings, even if only bracketed will score partial marks for incorrect answers. Most candidates were able to deal with the more straightforward adjustments such as the depreciation charges, closing inventory and adjusting revenue for the payments made in advance, although the corresponding entry in current liabilities was not always included. Common errors included the treatment of the tax figures which seemed to cause some confusion as to how to deal with the over provision from the previous year. Candidates commonly put the same figure in the income statement and statement of financial position, although this was split between whether it was the income tax charge or the liability. Candidates often used the correct brought forward and carried forward figures for the specific bad debt allowance but missed the additional allowance that needed making of £7,000. Other candidates correctly calculated the carried forward figure and hence calculated that an adjustment of £1,650 was needed but then either didn’t recognise this in the income statement or added it to expenses rather than deducting it. Only a few candidates carried the double entry through completely by deducting the full closing allowance from trade receivables, £7,000 was a more common deduction. The treatment of the legal provision also caused a few problems. Very few candidates realised that the provision needed reversing. A mix of treatments were seen with candidates either including the provision in the statement of financial position or providing for it in the current year even though it was a brought forward balance. Total possible marks Maximum full marks

© The Institute of Chartered Accountants in England and Wales 2010

18 18

Page 4 of 13

Financial Accounting – Professional Stage – March 2010

Question 2 Overall marks for this question can be analysed as follows:

Total: 19

General comments This question tested the preparation of a consolidated statement of cash flows and supporting note. A subsidiary was disposed of during the year. Missing figures to be calculated included dividends paid (to the group and to the non-controlling interest), interest paid, tax paid, depreciation and amortisation charge for the year and proceeds from the issue of share capital following a bonus issue during the year. Chitipa plc Consolidated statement of cash flows for the year ended 31 December 2009 £ Cash flows from operating activities Cash generated from operations (Note) 331,900 Interest paid (W1) (73,000) Income tax paid (W2) (76,050) Net cash from operating activities Cash flows from investing activities Purchase of property, plant and equipment (360,000) Disposal of Thyolo Ltd net of cash disposed of (200,000 192,100 – 7,900) Net cash from investing activities Cash flows from financing activities Repayment of borrowings (736,300 – 561,700) (174,600) Proceeds from share issue (W4 & W5) 175,000 Dividends paid (W6) (27,500) Dividends paid to non-controlling interest (W7) (11,850) Net cash used in financing activities Net increase in cash and cash equivalents Cash and cash equivalents at beginning of period Cash and cash equivalents at end of period

£

182,850

(167,900)

(38,950) (24,000) 172,500 148,500

Note: Reconciliation of profit before tax to cash generated from operations Profit before tax (222,000 + 12,600) Finance cost Depreciation charge (W3) Impairment loss on goodwill (373,700 – 364,200) Increase in inventories (401,300 – 393,800) Increase in trade and other receivables (496,300 – 475,200 + 25,400) Increase in trade and other payables ((21,700 – 5,000) – (11,700 – 7,000) + 36,100) Cash generated from operations

© The Institute of Chartered Accountants in England and Wales 2010

£ 234,600 71,000 22,700 9,500 (7,500) (46,500) 48,100 331,900

Page 5 of 13

Financial Accounting – Professional Stage – March 2010

Workings (1) Interest paid

Cash (β) C/d

£ 73,000 5,000 78,000

B/d CIS

£ 7,000 71,000 78,000

(2) Income tax paid

Cash (β) C/d

£ 76,050 33,900 109,950

B/d CIS (69,900 + 3,750)

£ 36,300 73,650 109,950

(3) PPE

B/d Additions

£ 695,000 360,000

Disposal of sub Deprecation charge (β) C/d

1,055,000

£ 308,900 22,700 723,400 1,055,000

(4) Share capital

£

C/d

550,000 550,000

B/d Bonus issue Cash received (β)

£ 400,000 100,000 50,000 550,000

(5) Share premium £ Bonus issue C/d

50,000 215,000 265,000

B/d Cash received (β)

£ 140,000 125,000 265,000

(6) Retained earnings

Dividends in SCE (β) Bonus issue C/d

£ 27,500 50,000 303,140 380,640

B/d CIS

£ 295,100 85,540 380,640

(7) Non-controlling interest

Cash (β) Disposal (306,100 x 20%) C/d

£ 11,850 61,220 448,260 521,330

B/d CIS

© The Institute of Chartered Accountants in England and Wales 2010

£ 490,800 30,530 521,330

Page 6 of 13

Financial Accounting – Professional Stage – March 2010 Candidates generally performed well on this question, adopting a good exam technique that allowed them to gain a good pass in this question but miss out some of the more tricky areas. Presentation of the statement of cash flows was good, although candidates often missed sub-totalling each section and the date for the period for which the cash flow was prepared was missed by a significant minority of candidates. Candidates generally calculated the repayment of borrowings correctly and the purchase of property, plant and equipment, although a minority of candidates showed the latter as an inflow of cash rather than outflow. Interest paid was also generally shown correctly. A common mistake was in relation to the income tax expense where a significant number of candidates missed the tax expense in respect of the discontinued operation. The calculation of the proceeds from the share issue were mixed with candidates gaining the marks for the brought forward and carried forward figures but often getting the entries for the bonus issue back to front. The calculation for the cash flows from the disposal of the subsidiary was one of the most disappointing areas with candidates showing all kinds of long and complicated net assets workings, when a, simple netting off of two figures was required. Candidates seemed happy with the T-account for dividends paid, although the treatment of the bonus issue was not always correctly dealt with, sometimes it was shown on the wrong side of the T-account or missed entirely. However, the calculation of the dividend paid to the non-controlling interest was disappointing with a good majority of candidates simply electing to ignore the calculation entirely. Candidates who did show a working for this generally were unable to calculate the disposal value or simply missed it out. A good attempt at producing the reconciliation of profit before tax to cash generated from operations was made by almost all candidates. Common errors however included not including the profit before tax for the discontinued operation, adding back the loss on disposal even though it was not included in the parent’s profit before tax figure, ignoring the impact of the goodwill impairment and deducting the individual assets and liabilities at disposal in the movements calculations, rather than adding them, or ignoring them completely. Total possible marks Maximum full marks

© The Institute of Chartered Accountants in England and Wales 2010

19 19

Page 7 of 13

Financial Accounting – Professional Stage – March 2010

Question 3 Overall marks for this question can be analysed as follows:

Total: 22

General comments This question required the preparation of a consolidated statement of financial position. The group has an associate, with the acquisition of a subsidiary during the year. A fair value adjustment in relation to a piece of equipment, with depreciation adjustment, was required. Inter-company trading had taken place during the year between the parent and associate company and a suspense account needed eliminating, which was created on the acquisition of property, plant and equipment on deferred payment terms. Rumphi plc (a) Consolidated statement of financial position as at 31 December 2009 £’000 Assets Non-current assets Property, plant and equipment (W8) Intangibles Goodwill Investment in associate (W7) Current assets Inventories Trade and other receivables (120,840 + 945,600) Cash and cash equivalents (72,600 + 189,500)

£’000

1,488,350 36,000 143,723 108,585 1,776,658

52,960 1,066,440 262,100 1,381,500 3,158,158

Total assets Equity and liabilities Equity attributable to Rumphi plc shareholders Ordinary share capital Retained earnings (W5) Attributable to the equity holders of Rumphi plc Non-controlling interest (W4)

930,000 802,840 1,732,840 348,948 2,081,788

Non-current liabilities Deferred payment (W8) Current liabilities Trade and other payables (236,380 + 470,330) Taxation (172,000 + 157,660) Total equity and liabilities

40,000

706,710 329,660 1,036,370 3,158,158

Workings (1) Group structure

Rumphi

245,000 / 350,000 = 70% Luwa Ltd

© The Institute of Chartered Accountants in England and Wales 2010

14,175 / 56,700 = 25% Dedza Ltd

Page 8 of 13

Financial Accounting – Professional Stage – March 2010 (2) Net assets – Luwa Ltd

Share capital Share premium account Retained earnings Goodwill on business PPE FV uplift FV depreciation adjustment (12,000 / 8yrs x 4/12)

31 Dec 2009 £ 350,000 125,000 748,260 (71,600) 12,000 (500) 1,163,160

Acquisition £ 350,000 125,000 600,710 (71,600) 12,000

– 1,016,110

Post acq £

– – 147,550

– – (500) 147,050

(3) Goodwill – Luwa Ltd £ 900,000 (1,016,110) 304,833 188,723 (45,000) 143,723

Consideration transferred Net assets at acquisition (W2) Non-controlling interest at acquisition (1,016,110 (W2) x 30%) Less: Impairment

(4) Non-controlling interest – Luwa Ltd Share of net assets (1,163,160 (W2) x 30%)

£348,948

(5) Retained earnings £ 751,320 (1,700) 102,935 (45,000) 13,285 (10,000) (8,000) 802,840

Rumphi plc Less:PURP (6,800 x 25%) Luwa Ltd (147,050 (W2) x 70%) Less: Impairment Dedza Ltd ((145,695 – 92,555) x 25%)) Less: Impairments to date Machine depreciation adjustment (W8)

(6) PURP % 100 (60) 40

Sale price Cost Gross profit 13,600 x ½ = 6,800

Dedza Ltd £ 34,000 (20,400) 13,600

(7) Investment in associate – Dedza Ltd Original cost Add: Share of post acquisition increase in retained earnings Less: Impairment to date Less: Share of PURP

£ 107,000 13,285 (10,000) (1,700) 108,585

(8) Property, plant and equipment Rumphi plc Luwa Ltd Fair value adjustment FV depreciation adjustment (W2) New machine (80,000 – 40,000) Depreciation adj on new machine (40,000 / 5 yrs)

© The Institute of Chartered Accountants in England and Wales 2010

£ 800,300 644,550 12,000 (500) 40,000 (8,000) 1,488,350

Page 9 of 13

Financial Accounting – Professional Stage – March 2010 The majority of candidate answers to this question were very good. Most notably candidates coped far better with the provision for unrealised profit on sales made by the parent to the associate than they have at previous sittings. Where errors were made in respect of the provision for unrealised profit it was by deducting the full amount of the unrealised earnings from retained earnings and investment in associate rather than the parent’s share. A significant minority of candidates made the adjustment to consolidated inventory instead of to the investment in associate. Presentation was very good, although candidates still seem to not complete statements in some way, most typically by not showing the sub total before the non-controlling interest line and therefore they inevitably lose marks. Workings were generally well laid out, although the property, plant and equipment workings were often squashed on the face of the statement of financial position which made it quite difficult to read. This was compounded by the fact that these scripts were scanned for electronic marking and therefore squashed workings became even harder to read. Candidates should be made aware of this, to try and avoid such an approach in the future. The majority of candidates made a good attempt at the net assets working. However, a number of common errors were made in this area including not deducting the goodwill recognised by the subsidiary, instead candidates included this as part of consolidated intangible assets, and forgetting that the fair value uplift on the equipment meant that additional depreciation needed to be calculated. For candidates that did appreciate that additional depreciation should be recognised they often missed that it was only four months worth, rather than a full year. Another common error was not including the subsidiary’s share premium in the net assets working but instead showing it on the face of the consolidated statement of financial position. One surprising error was that whilst almost all candidates correctly calculated the percentages of the subsidiary and the associate held by the parent, a significant minority of candidates subsequently mixed up the associate percentage (25%) with the non-controlling interest percentage (30%). Other common errors included correctly adding the £40,000 due on the new machine to consolidated property, plant and equipment but either showing the corresponding liability as current or not showing a corresponding liability at all, or simply adding £80,000 to property, plant and equipment rather than only £40,000. A worrying few consolidated either only four-twelfths or 70% of the subsidiary’s assets and liabilities. Total possible marks Maximum full marks

© The Institute of Chartered Accountants in England and Wales 2010

22 22

Page 10 of 13

Financial Accounting – Professional Stage – March 2010

Question 4 Overall marks for this question can be analysed as follows:

Total: 21

General comments The first part of this question is a single topic question focusing on non-current assets, including aspects on leasing. Candidates were required to prepare a finance lease calculation, assess a research and development project and also carry out an impairment review. Parts b) and c) covered concepts issues, with a discussion on substance over form and how the four qualitative characteristics related to lease transactions. Blantyre Ltd (a) Summary of costs included in income statement for the year ended 31 December 2009 £ Administrative expenses: Depreciation (85,000 / 5yrs) 17,000 Amortisation (192,000 / 4yrs x 6/12 months) 24,000 Impairment of know-how (W3) 11,000 Research costs 70,000 Promotional advertising costs 15,000 Staff training costs 13,000 Finance costs (W1) 6,400 Statement of financial position as at 31 December 2009 (extracts) £ Non-current assets Property, plant and equipment (85,000 – 17,000) Intangible assets (120,000 – 15,000 + 157,000)

68,000 262,000

Non-current liabilities Finance lease liabilities (W1)

43,200

Current liabilities Finance lease liabilities (62,400 – 43,200) (W1)

19,200

(1) Finance lease Deposit Instalments (4 x 24,000) Fair value of asset Finance charges

£ 5,000 96,000 (85,000) 16,000

SOTD = (4 x 5) ÷ 2 = 10 B/fwd (85,000 – 5,000) = 80,000 Year ended 31 December 2009 31 December 2010

B/f £ 80,000 62,400

Interest £ (16,000 x 4/10) 6,400 (16,000 x 3/10) 4,800

Payment £ (24,000) (24,000)

C/f £ 62,400 43,200

(2) Technical Know-how Original cost Legal costs Manufacturing supervisors time Testing costs

(3) Impairment Carrying amount at 31 Dec 2009 (192,000 – 24,000) Recoverable amount Impairment

© The Institute of Chartered Accountants in England and Wales 2010

£ 180,000 4,000 3,200 4,800 192,000 £ 168,000 (157,000) 11,000

Page 11 of 13

Financial Accounting – Professional Stage – March 2010

In part (a) most candidates picked up a considerable number of marks for correct calculations, however many lost marks for their statement of financial position extracts, as these were not properly presented. The majority of candidates made a good attempt at the leasing table. The most common error was to not deduct the deposit of £5,000 paid upfront. Candidates however often made a mistake in allocating the lease liability between current and non-current, with a significant minority of candidates allocating the full £24,000 payment as current thereby not understanding that it should only be the capital element of this. Common errors included calculating an incorrect sum-of-digits figure, taking the fair value of £152,000 as the recoverable amount of the technical know-how rather than the estimated future cash flows of £157,000, not excluding £13,000 staff training costs from the amount originally recognised for the technical know-how and failing to compare the amortised carrying amount of the technical know-how to the recoverable amount. Total possible marks Maximum full marks

13 13

(b) Substance over form is an accounting concept that should be applied to all accounting areas in accordance with the IASB Framework. Leasing is an example of the application of this concept. To recognise the substance of a transaction, its economic reality should be reflected rather than merely its legal form. IAS 17, Leases looks at the economic reality of a lease through the assessment of which party carries the risks and rewards of ownership, rather than looking at legal ownership. If the effect of the lease transaction is such that in commercial effect it is similar to borrowing the money and buying the asset outright, both IAS 17 and the IASB Framework require the asset and in effect a related loan to be recognised. Conversely, if the risks and rewards of ownership remain with the lessor, as they do in an operating lease, then in effect the substance of the transaction is the same as its legal form and no asset or corresponding liability should be recognised.

Answers to part (b) were adequate, with most candidates quoting a reasonable definition of substance over form, recognising that the way finance leases are accounted for is an example of this concept and discussing the transfer of risks and rewards. However, most candidates focused on the recognition of the asset with no mention of a corresponding liability. Whilst some candidates discussed the fact that assets held under operating leases are not capitalised because the risks and rewards are not transferred, it was rare to see the point that for operating leases substance is the same as legal form. Total possible marks Maximum full marks

4 3

(c) Qualitative characteristics and IAS 17. Relevance Information is relevant if it can influence the economic decisions of users. By showing the true substance of a finance lease, a company is made to show the debt that it has in its financial statements. This may influence potential lenders in the future. The commitments note in relation to operating leases and the liability note in relation to finance leases will also provide potential lenders essential information on what the company’s commitments and obligations already are. Reliability Information is reliable if it is free from error or bias, complete and portrays events in a way that reflects their reality.

© The Institute of Chartered Accountants in England and Wales 2010

Page 12 of 13

Financial Accounting – Professional Stage – March 2010 To be reliable information must faithfully represent a transaction. IAS 17 does this by following the overriding criteria of substance over form. Comparability Users must be able to compare information with that of previous periods or with that of another entity. Comparability is achieved through consistency and disclosure. IAS 17 does require some subjectivity when a company assesses the risks and rewards of ownership. However, detailed disclosure requirements, including setting out the company’s accounting policies will help with comparability. In addition, IAS 17 ensures that financial statements are comparable between a company that has taken out a loan to acquire an asset or one that has entered into a finance lease. Understandability Information must be readily understandable to users so that they can perceive its significance. It is dependent on how information is presented. There may be some confusion in looking at the non-current assets owned by a company, as these will include those assets that are held under finance leases. However, the accounting policies will explain this and it is assumed that users have a reasonable level of knowledge.

Answers to part (c) were often poor. Most marks were scored from brief general points about the four qualitative characteristics. Reliability was probably the characteristic that was dealt with the best although some candidates strayed into IAS 16 and discussed whether it was more reliable to record an asset at its historic cost or its fair value. Points were often repeated and often placed under the wrong characteristic. With regards to understandability, many cited very complex notes as an example of the application of this concept. Total possible marks Maximum full marks

© The Institute of Chartered Accountants in England and Wales 2010

8 5

Page 13 of 13

Financial Accounting - Professional Stage – June 2010

PROFESSIONAL STAGE FINANCIAL ACCOUNTING – OT EXAMINER’S COMMENTS The performance of candidates in the June 2010 objective test questions section for the Professional Stage Financial Accounting paper was in line with the average performance on this section of the paper over all sittings to date. Candidates performed well across all syllabus areas. When practising OT items, care should always be taken to ensure that the principles underlying any particular item are understood rather than rote learning the answer. In particular, candidates should ensure that they read all items very carefully. The following table summarises how well* candidates answered each syllabus content area. Syllabus area

Number of questions

Well answered

Poorly answered

LO1

5

5

0

LO2

5

4

1

LO3

5

4

1

Total

15

13

2

*If 50% or more of the candidates gave the correct answer, then the question was classified as ‘well answered’. Comments on the two poorly answered questions, which covered LO2 (preparation of single company financial statements) and LO3 (preparation of consolidated financial statements), are below: Item 1 This item required candidates to calculate closing inventories for a manufacturing company, consisting of raw materials, work in progress and finished goods. The finished goods needed to be valued at net realisable value (a discounted selling price less selling costs). The work in progress also needed to be valued at net realisable value (as for the finished goods less estimated costs to completion). The most common errors made were to:  value the work in progress at its cost to date, failing to recognise that the information about selling price and selling costs for finished goods was also relevant to this calculation, or  to ignore the selling costs in the valuation of both finished goods and work in progress. Item 2 This item required candidates to calculate the amount to be shown as trade payables in a consolidated statement of financial position. To arrive at the correct figure candidates needed to adjust for cash-in-transit in the book of the receiving company, and then cancel the intra-group balances. A significant number of candidates failed to recognise that the balance in the paying company’s books would already reflect the cash-intransit.

© The Institute of Chartered Accountants in England and Wales

Page 1 of 17

Financial Accounting - Professional Stage – June 2010

MARK PLAN AND EXAMINER’S COMMENTARY The marking plan set out below was that used to mark this question. Markers were encouraged to use discretion and to award partial marks where a point was either not explained fully or made by implication. More marks were available than could be awarded for some requirements. This allowed credit to be given for a variety of valid points which were made by candidates.

Question 1

Total Marks: 24

General comments This question tested the preparation of an income statement, a statement of comprehensive income and a statement of financial position from a trial balance plus a number of adjustments. Adjustments included a bad debt write off based on an adjusting event after the reporting period, a write-down to inventories, a finance lease taken out during the year, a revaluation during the year and a transfer between the revaluation surplus and retained earnings based on a previous revaluation.

Dashwood Ltd Income statement for the year ended 31 March 2010 £ 945,700 (604,000) 341,700 (97,400) (164,100) 80,200 (6,730) 73,470 (10,000) 63,470

Revenue Cost of sales (W1) Gross profit Distribution costs Administrative expenses (W1) Profit from operations Finance cost (1,230 + 5,000 (OF) (W6) + 500) Profit before tax Income tax expense Profit for the year Statement of comprehensive income for the year ended 31 March 2010

£ 63,470

Profit for the year Other comprehensive income: Gain on property revaluation Total comprehensive income for the year

250,000 313,470

Statement of financial position as at 31 March 2009 £ Assets Non-current assets Property, plant and equipment (W2) Intangibles (75,000 x 4/8) Current assets Inventories (W1) Trade and other receivables (140,950 – 5,500 (OF) (W1)) Total assets

© The Institute of Chartered Accountants in England and Wales

£

2,620,100 37,500 2,657,600 41,000 135,450 176,450 2,834,050

Page 2 of 17

Financial Accounting - Professional Stage – June 2010

Equity and liabilities Equity Ordinary share capital Revaluation surplus (W4) Retained earnings (W3)

£

£ 245,000 835,000 1,329,650 2,409,650

Non-current liabilities Finance lease liability (W6)

138,000

Current liabilities Trade and other payables Finance lease liability (182,000 – 138,000) (W6) Taxation Borrowings (50,700 + 500)

181,200 44,000 10,000 51,200 286,400 2,834,050

Total equity and liabilities

Workings (1) Allocation of expenses Cost of sales Per Q Opening inventories Bad debts (11,000 x ½) Research costs (75,000 x 4/8) Closing inventories Depreciation charges (W2)

£ 392,800 35,600

Distribution costs £ 97,400

Administrative expenses £ 123,600 5,500

37,500 (41,000) 179,100 604,000

97,400

35,000 164,100

(2) PPE

B/f Cost/valuation Finance lease Revaluation (1,000,000 – 750,000) B/f Accumulated depreciation Depreciation – buildings (1,400,000 ÷ 40) Depreciation – plant (895,500 ÷ 5)

Total PPE

© The Institute of Chartered Accountants in England and Wales

Plant and machinery £ 670,500 225,000 895,500 (356,300) (179,100) 360,100

Land and buildings £ 2,150,000

250,000 (105,000) (35,000) 2,260,000 2,620,100

Page 3 of 17

Financial Accounting - Professional Stage – June 2010 (3) Retained earnings £ 1,249,930 16,250 63,470 1,329,650

At 31 March 2009 Transfer from revaluation surplus (W5) Profit for the period At 31 March 2010 (4) Revaluation surplus £ At 31 March 2009 Valuation in the year (W2) Transfer to retained earnings Depreciation charge based on revalued amount (W2) Depreciation charge based on HC (750,000 ÷ 40)

£ 601,250 250,000

35,000 (18,750) (16,250) 835,000

(5) Finance lease £ 240,000 (225,000) 15,000

Instalments (4 x 60,000) Cash price of machine Finance charges SOTD = (5 x 6) ÷ 2 = 15 Year ended

B/f £ 225,000

31 March 2010 31 March 2011

182,000

© The Institute of Chartered Accountants in England and Wales

Interest £ (15,000 x 5/15) 5,000 (15,000 x 4/15) 4,000

Payment £ (48,000)

C/f £ 182,000

(48,000)

138,000

Page 4 of 17

Financial Accounting - Professional Stage – June 2010

As in previous sittings, candidates were clearly very well-prepared for this type of question. Almost all candidates produced a well-laid out income statement and statement of financial position and the standard of presentation was the highest seen to date. Only a small minority of candidates lost presentation marks for the statement of financial position by not adding across numbers in brackets or failing to complete subtotals and/or totals on their statements or by having incomplete or abbreviated narrative. Others lost presentation marks for failing to include a sub-total for profit from operations on their income statement. As ever, candidates should remember that this type of question requires financial statements to be in a form suitable for publication. Presentation of the statement of comprehensive income was also excellent, with the majority of candidates showing this, as required by the question (and as shown in the learning materials) as a separate statement. This was the first time this statement had been examined and the majority of candidates clearly demonstrated an understanding of the relationship between that statement, retained earnings and the revaluation surplus. Although many workings, in particular the cost matrix and the property, plant and equipment working, were clearly laid out, a few candidates’ workings were disorganised, untidy and therefore hard to follow, making it difficult to establish candidates’ approaches where they had not calculated the correct figure. It is particularly difficult to follow workings which use little or no narrative, or costs workings done on the face of the income statement. In particular, it was sometimes difficult to identify the final number for property, plant and equipment on the face of the statement of financial position in the workings. The most sensible layout for a property, plant and equipment working is a table with columns for the different categories which clearly shows brought forward balances and movements in the year (effectively a simplified version of the disclosure note). Most candidates were able to deal with the more straightforward adjustments such as the bad debt writeoff, the write-down of closing inventory from cost to net realisable value and the income tax charge for the year. However, what was pleasing was that many candidates also coped with the more difficult aspects of the question such as the depreciation transfer between reserves. By far the most common error was the failure identify that the bank account balance at the year end was a credit balance with many candidates including this figure in current assets instead of as an overdraft in current liabilities. As always, there was also some failure to complete double entry such as:  including the outstanding bank interest from the bank reconciliation as a finance cost but not adjusting the balance at bank (or vice versa)  using a different figure for depreciation in the property, plant and equipment working and the costs matrix  capitalising some of the research and development costs but not including the balance in expenses. Errors dealing with other adjustments included the following:  Failing to include the leased asset in property, plant and equipment (but then sometimes including depreciation on that asset in expenses).  Treating the finance lease as if payments were in advance rather than in arrears.  Splitting the closing finance lease liability incorrectly between current and non-current.  Including the lease payment in finance costs (sometimes in addition to the interest on the lease).  Not identifying that half of the research and development costs should be capitalised (with a number of candidates either capitalising all the costs or expensing all of the costs).  Allocating expenses to the incorrect income statement category (for example, including the depreciation charge on the leased asset in administrative costs even though it related to plant and machinery).  Treating the revaluation in the year as if it related to buildings rather than to land.  Failing to adjust for the outstanding item in the bank reconciliation.  Deducting the reserves transfer from the revaluation surplus from retained earnings rather than adding it to retained earnings.  Adding total comprehensive income for the year (instead of profit for the year) to opening retained earnings.  In the statement of comprehensive income, showing the net movement on the revaluation surplus for the year, as opposed to the revaluation which took place in the year. Total possible marks Maximum full marks

© The Institute of Chartered Accountants in England and Wales

24½ 24

Page 5 of 17

Financial Accounting - Professional Stage – June 2010

Question 2

Total Marks: 16

General comments This question tested the preparation of a single company statement of cash flows and supporting note. Missing figures to be calculated included interest paid, tax paid, dividends paid, property, plant and equipment acquired and proceeds from the issue of share capital. A bonus issue of shares and the sale of revalued property during the year also featured. Middleton plc Statement of cash flows for the year ended 31 March 2010 £ Cash flows from operating activities Cash generated from operations (Note) Interest paid (W1) Income tax paid (W2) Net cash from operating activities Cash flows from investing activities Purchase of property, plant and equipment (W3) Proceeds from sales of property, plant and equipment Net cash used in investing activities Cash flows from financing activities Proceeds from issue of ordinary share capital (320,000 (W4) + 400,000 (W5)) Proceeds from issue of borrowings Dividends paid (W6) Net cash from financing activities Net decrease in cash and cash equivalents Cash and cash equivalents at beginning of period Cash and cash equivalents at end of period

£

2,457,800 (22,200) (295,000) 2,140,600 (5,578,100) 2,800,000 (2,778,100) 720,000 500,000 (599,400) 620,600 (16,900) 52,500 35,600

Note: Reconciliation of profit before tax to cash generated from operations Profit before tax Finance costs Depreciation charge Amortisation charge (500,000 – 450,000) Profit on disposal of property, plant and equipment (2,800,000 – 2,567,000) Increase in inventories (679,000 – 578,000) Decrease in trade and other receivables (656,800 – 547,500) Decrease in trade and other payables ((657,900 – 5,000) – (567,300 – 6,500)) Decrease in provision (500,000 – 200,000) Cash generated from operations

© The Institute of Chartered Accountants in England and Wales

£ 1,321,900 23,700 1,679,000 50,000 (233,000) (101,000) 109,300 (92,100) (300,000) 2,457,800

Page 6 of 17

Financial Accounting - Professional Stage – June 2010

Workings (1) Interest paid Cash (β) C/d

£ 22,200 6,500 28,700

B/d IS

£ 5,000 23,700 28,700

£ 295,000 270,000 565,000

B/d IS

£ 300,000 265,000 565,000

(2) Tax paid

Cash (β) C/d

(3) PPE

B/d Additions (β)

£ 6,345,400 5,578,100 11,923,500

Disposals Depreciation C/d

£ 2,567,000 1,679,000 7,677,500 11,923,500

(4) Ordinary share capital £

C/d

2,000,000 2,000,000

B/d Bonus issue (1,400,000 ÷ 5) Cash (β)

£ 1,400,000 280,000 320,000 2,000,000

(5) Share premium £ C/d

600,000 600,000

B/d Cash (β)

£ 200,000 400,000 600,000

(6) Retained earnings

Dividends paid (β) Bonus issue (W4) C/d

£ 599,400 280,000 5,252,300 6,131,700

© The Institute of Chartered Accountants in England and Wales

B/d Revaluation surplus IS

£ 3,524,800 1,550,000 1,056,900 6,131,700

Page 7 of 17

Financial Accounting - Professional Stage – June 2010 Candidates were clearly very well prepared for this question. Presentation was generally good and the vast majority of candidates showed a strong grasp of the double-entry techniques which underpin the preparation of a statement of cash flows, although some are still losing marks for failing to show outflows of cash in brackets on the face of the statement. Most candidates produced workings in the form of T accounts with far fewer than usual completing these T accounts with the debits and credits the wrong way round. However, some candidates produced tabular workings or working in brackets on the face of the statement of cash flows. This can make it more difficult to see evidence of correct double entry and to award marks where the final figure is incorrect (or uses the incorrect bracket convention). Pleasingly, very few candidates produced no workings at all – an even riskier approach as if figures are calculated incorrectly it is not possible to award any partial marks. The majority of candidates scored very high marks on the reconciliation note, and on the figures for tax paid, interest paid and the opening and closing figures for cash and cash equivalents. In the reconciliation note most candidates dealt correctly with the opening and closing interest accrual which was included in trade and other payables, an area which has caused problems in the past. Candidates generally made a good attempt at the property, plant and equipment T account, with the figures given in the question for depreciation and for the disposals both being correctly used. Where mistakes were made, the most common error was to include the release of the revaluation surplus during the year on the credit side of this T account instead of on the credit side of the retained earnings T account. The other common mistake which affected the retained earnings T account was to deduct the bonus issue from the share premium account instead of from retained earnings, as specified in the question. Where either the bonus issue or the release of the revaluation surplus was omitted from the retained earnings working, candidates then arrived at a balancing credit balance. Many thought that this indicated dividends received – demonstrating a lack of understanding of how dividends received would be properly accounted for in a set of financial statements. Other common errors included the following:  In the reconciliation note, failing to adjust for the decrease in the warranty provision, or making the adjustment in the wrong direction.  Again in the reconciliation note, failing to adjust for the profit on disposal of property, plant and equipment.  Miscalculating the number of shares in the bonus issue.  Failing to adjust the property, plant and equipment working for the disposals.  Failing to deal with or miscalculating the amortisation charge for the year. Total possible marks Maximum full marks

© The Institute of Chartered Accountants in England and Wales

16 16

Page 8 of 17

Financial Accounting - Professional Stage – June 2010

Question 3

Total Marks: 15

General comments This question mixed two discrete topics. Part (a) required the calculation of specified figures from a consolidated statement of financial position in respect of a subsidiary and an associate, both of which were acquired during the year. A fair value adjustment (with subsequent additional depreciation) had to be made to the subsidiary. Part (b) required the preparation of a provisions note showing both the numerical “table” and the relevant narrative disclosures.

Norland Ltd (a) Figures for the consolidated statement of financial position as at 31 March 2010 (i) Goodwill £ Fair value of consideration Cash Deferred cash Shares (750,000 x £1.20)

200,000 385,500 900,000 1,485,500 230,400 (921,600) 794,300

Non-controlling interest at acquisition (921,600 (W) x 25%) Less: Fair value of net assets at acquisition

(ii) Non-controlling interest (1,086,000 (W) x 25%)

£271,500

(iii) Investment in associate £ 500,000

Cost Share of post acquisition change in net assets Share of post acquisition profits (123,600 x 9/12 x 30%)

27,810 527,810

Working Net assets – Delaford Ltd

Share capital Retained earnings Per Q (741,600 + (9/12 x 235,200)) Fair value adjustment (300,000 – 220,000) Additional depreciation based on fair value ((80,000 (OF) ÷ 5) x 9/12)

At year end £ 100,000

Acquisition £ 100,000

918,000 80,000

741,600 80,000

(12,000) 1,086,000

© The Institute of Chartered Accountants in England and Wales

921,600

Page 9 of 17

Financial Accounting - Professional Stage – June 2010

Candidates generally made a very good attempt at Part (a), with a significant number gaining full marks. This was very encouraging as candidates have historically performed less well when asked to produce extracts from the consolidated financial statements, as opposed to a single consolidated statement. In the calculation of goodwill, almost all candidates correctly calculated the total consideration. A few omitted the shares issued from this calculation or used the £1 nominal value to value this part of the consideration rather than the fair value. A number of candidates wasted time calculating goodwill arising on the acquisition of the associate, when this would not appear as “Goodwill” in the consolidated statement of financial position, but would effectively be subsumed within the “Investment in the associate” figure, which was separately required. The calculation of the investment in the associate was again very pleasing. A significant number of candidates again calculated this correctly, even where they had made mistakes elsewhere in the question. The most common error was to omit to time-apportion the profit for the year to allocate only nine months of the twelve to post acquisition profits. Occasionally, candidates confused the associate percentage holding with the non-controlling interest holding in the subsidiary, an error that was noted at the last sitting – candidates must be careful to use the correct percentages as this loses easy marks. A minority of candidates confused the working by trying to calculate the share of net assets held but adding it to the cost of the associate. The net assets working for the subsidiary was the least well answered part of Part (a) although, again, there were plenty of correct answers. One of the most common errors was to omit share capital from this working. The majority of candidates did adjust net assets at both acquisition and at the year end for the fair value adjustment of £80,000. Pleasingly, most candidates then went on to correctly calculate the additional depreciation arising from this adjustment, although some omitted to pro-rate this for the nine month post-acquisition period and others incorrectly also adjusted net assets at acquisition for this . The most common error in calculating the figure for non-controlling interest, apart from errors in the net assets table, was to calculate the non-controlling interest at acquisition instead of at the year end, or to calculate the non-controlling interest figure for the consolidated income statement instead of for the consolidated statement of financial position.

Total possible marks Maximum full marks

© The Institute of Chartered Accountants in England and Wales

7½ 7

Page 10 of 17

Financial Accounting - Professional Stage – June 2010

(b) Notes to the financial statements for the year ended 31 March 2010 Provisions

At 1 April 2009 Utilised in the year Income statement charge/(credit) (β) At 31 March 2010 (W)

Faulty goods provision £ 10,000 (8,500) 20,900 22,400

Restructuring provision £ 350,000

Provision for fines £ 60,000 (20,000)

350,000

40,000

Total £ 70,000 (8,500) 350,900 412,400

Faulty goods provision The provision in respect of faulty goods relates to the supply of faulty hair straighteners during the year ended 31 March 2010. The provision is based on the cost to the company of repairing or replacing the faulty hair straighteners. All such expenditure is expected to be incurred in the year ended 31 March 2011. Restructuring provision During the year the company publicly announced and then commenced a restructuring of its domestic appliances division. The provision is based on the anticipated further costs of the restructuring, all of which are expected to be incurred in the year to 31 March 2011. Provision for fines The company has not yet fitted smoke filters in its factories as required by legislation which came into force on 1 January 2009. Although the company plans to start the installation in May 2010, companies in similar situations have been fined for such non-compliance. The year-end provision is based on lawyers’ best estimate of the likely amount of such a fine. Working Faulty goods: 800 x 80% = 640 Provide (640 x 50% x £20) + (640 x 50% x £50) = £22,400 Restructuring: Provide for direct expenditure only = 300,000 + 50,000 = £350,000 Fines: Do not provide for costs of fitting smoke filters (no obligating event) but provide for best estimate of fines which are more likely than not (ie 75%) to be imposed

© The Institute of Chartered Accountants in England and Wales

Page 11 of 17

Financial Accounting - Professional Stage – June 2010

Candidates’ answers to Part (b) were mixed. However, there was a marked improvement from candidates’ performance when this topic was set in a previous examination. The majority of candidates clearly understood what the provision table showed and that the year-end position represented the provision required at that date rather than the movement in the year. The faulty goods and restructuring provisions were generally dealt with correctly, although the provision for fines caused more problems. The faulty goods provision using the expected cost was generally calculated correctly. The most common error was not adjusting for the 20% of claims that were not valid and therefore using the whole 800 claims. For the restructuring provision, the majority of candidates recognised that the staff retraining and relocation cost should not be included in the closing provision, with only a few candidates including this. The provision for fines caused the most problems as candidates struggled to distinguish between the fines and the provision for the work. A typical answer included both of these, highlighting that candidates did not appreciate the difference between when an obligation exists and when it does not. A significant number of candidates confused the utilisation of a provision and the movement on the provision in the period. Very few candidates picked up the available presentation mark as they omitted a total column from their provisions “table”. A minority produced a series of tables as opposed to one table and a few produced a series of T accounts. However, the most disappointing aspect to this part of the question was the quality of the narrative disclosures. Candidates still do not appreciate the difference between an explanation of why a provision has been made (which is not required in the Financial Accounting paper) and the information contained in a disclosure note. Good clear narrative disclosures in an appropriate style was only seen in a minority of scripts. Total possible marks Maximum full marks

© The Institute of Chartered Accountants in England and Wales

9 8

Page 12 of 17

Financial Accounting - Professional Stage – June 2010

Question 4

Total Marks: 25

General comments Part (a) was a consolidated income statement question, featuring two subsidiaries (one fully disposed of within the year) and one associate. Adjustments included intra-group trading and unrealised profits and impairment write-downs. Part (b) required the calculation of opening consolidated retained earnings. Part (c) tested an understanding of the concepts underlying the preparation of consolidated financial statements: namely the single entity concept and control versus ownership. Jennings plc (a) Consolidated income statement for the year ended 31 March 2010 £’000 106,500 (62,150) 44,350 (26,600) 17,750 1,500 2,460 21,710 (5,700) 16,010 4,560 20,570

Revenue (W2) Cost of sales (W2) Gross profit Operating expenses (W2) Profit from operations Investment income Share of profit of associates ((6,400 x 40%) – 100) Profit before tax Income tax expense (W2) Profit for the year from continuing operations Profit for the year from discontinued operations (3,900 + 660 ) (W4)) Profit for the year Attributable to Equity holders of Jennings plc (β) Non-controlling interest (W6)

17,825 2,745 20,570

Workings (1) Group structure

Jennings plc 1.6

= 40%

4 6.4 8

= 80% for 6/12

Ferrars Ltd

4.2 6

= 70% Palmer Ltd

Brandon Ltd

© The Institute of Chartered Accountants in England and Wales

Page 13 of 17

Financial Accounting - Professional Stage – June 2010

(2) Consolidation schedule

Revenue Cost of sales – per Q – PURP (W3) Op expenses – per Q – GW impairment Investment income (W5) Tax

Jennings plc £’000 67,600 (43,700) (12,700) (700) 500 (4,000)

Brandon Ltd £’000 42,500 (21,750) (300) (13,200)

Adj (W3) £’000 (3,600) 3,600

Consol £’000 106,500 (62,150) (26,600) 1,500 (5,700)

1,000 (1,700) 6,550

(3) Intra-group sale and PURP

SP Cost GP X½

% 120 (100) 20

£’000 3,600 (3,000) 600 300

£’000

£’000

(4) Group profit on disposal of Ferrars Ltd Carrying amount of net assets at disposal Net assets at 31 March 2009 (8,000 + 2,700) Profit six months to 30 September 2009 (7,800 x 6/12)

10,700 3,900 14,600

Carrying amount of goodwill at disposal Cost of investment Non-controlling interest at acquisition ((8,000 + 550) x 20% Less: Net assets at acquisition (8,000 + 550) Less: Impairments to date Total assets disposed of Less: Attributable to non-controlling interest (14,600 x 20%) Total assets attributable to parent now disposed of Sale proceeds Profit on disposal

10,000 1,710 (8,550) 3,160 (500) 2,660 17,260 (2,920) 14,340 (15,000) 660

(5) Investment income in Jennings plc Total per IS Less: Profit on disposal of Ferrars Ltd (15,000 – 10,000) Share of Brandon Ltd’s ordinary dividend (2,000 x 70%) Share of Palmer Ltd’s ordinary dividend (1,000 x 40%) Other investment income

£’000 7,300 (5,000) (1,400) (400) 500

(6) Non-controlling interest in year Ferrars Ltd (20% x 3,900 (W4)) Brandon Ltd (30% x 6,550 (W2)

© The Institute of Chartered Accountants in England and Wales

£’000 780 1,965 2,745

Page 14 of 17

Financial Accounting - Professional Stage – June 2010

Historically, candidates have performed less well on a consolidated income statement than on a consolidated statement of financial position, but there were some excellent attempts at Part (a) of this question. A good number of candidates calculated the correct profit on disposal of the subsidiary and took this, together with the subsidiary’s pre-disposal profit for the year, to the face of the consolidated income statement. Most candidates also dealt correctly with the impairment in the carrying amount of the associate, whereas this has not been dealt with well in the past. Almost all candidates correctly calculated the percentage holdings for the subsidiaries and the associate. Presentation of the consolidated income statement was also generally good with most candidates gaining some of the available presentation marks. However, those candidates who combined their consolidation schedule with a consolidated income statement were not awarded any presentation marks. Candidates generally made a reasonable attempt at a consolidation schedule, although it was common to see the disposed of subsidiary’s results incorrectly included. This may indicate that some candidates are confused over the difference in treatment between an acquisition and disposal. Candidates generally made an adjustment for the provision for unrealised profit (usually in the correct column) and the related adjustment in revenue and cost of sales, although this was often the incorrect figure of £3 million, with candidates not adjusting for the mark-up on sale. Candidates generally calculated the provision for unrealised profit correctly, with only a minority incorrectly using a gross profit margin instead of a mark-up or forgetting to adjust for only half of the inventories being held at the year end. The two non-controlling interests were generally calculated correctly although a significant minority missed the one for the disposed of subsidiary, not appreciating that this company’s results were included for part of the year. The non-controlling interest was generally separated out on the face of the consolidated income statement, although many abbreviated this to “NCI” which is not acceptable on the face of such a statement. The investment income caused a problem for many candidates although a few did arrive at the correct figure. Candidates often made adjustments for the share of the dividends from the subsidiary and the associate correctly and realised that some kind of adjustment was required for the profit on disposal but generally used the consolidated profit on disposal figure rather than that shown in the parent company financial statements. Although, as discussed above, a good number of candidates calculated the group profit on disposal of the subsidiary completely correctly, other candidates were clearly confused. Most candidates made some attempt at the working, although it was missed out completely by a minority of candidates. Most candidates made a fair attempt at calculating the carrying amount of the goodwill at disposal, although some candidates adjusted for the impairment prior to taking 80% of the net assets at acquisition (or 100% and then adjusting for the non-controlling interest’s 20%). The carrying amount of the net assets at disposal was less well calculated, with the majority of candidates making some miscalculation. Those candidates who attempted to work “backwards” from the year-end net assets to the net assets at disposal generally fared less well than those who worked “forwards” from the opening net assets. A frequent error was to use the number of shares held (6.4 million) as opposed to the issued share capital (10 million). Candidates generally included a figure for the share of the associate’s profits although the figure was sometimes not adjusted for the impairment of £100,000. The profit from discontinued operations on the face of the consolidated income statement was shown by most candidates although it often only included the profit on disposal or the subsidiary’s profit for part (sometimes incorrectly all) of the year rather than a combination of both. Some took only the group share of the subsidiary’s profit into the profit from discontinued operations, instead of taking 100% at this stage and then taking out 20% of this later as part of the non-controlling interest figure. A minority of candidates were clearly confused about where the discontinued operation should be shown and thought that it was instead an “exceptional” type item in the main body of the income statement. Total possible marks Maximum full marks

© The Institute of Chartered Accountants in England and Wales

18 18

Page 15 of 17

Financial Accounting - Professional Stage – June 2010

(b) Consolidated retained earnings brought forward at 1 April 2009

Jennings plc Ferrars Ltd (80% x (2,700 – 550)) Palmer Ltd (40% x (4,550 – 600)) Goodwill impairments to date

£’000 23,800 1,720 1,580 (500) 26,600

The correct figure for opening consolidated retained earnings was correctly calculated by a significant number of candidates. Where mistakes were made they included the following:  Adjusting for the impairment losses which arose in the year in addition to the cumulative impairment losses at the start of the year.  Not including any figure(s) for the associate and/or the disposed of subsidiary (both of which had been acquired several years ago).  Including a figure for the subsidiary acquired on the first day of the current year (and which should not therefore have been included in the retained earnings brought forward on that date (from the last day of the previous year)).  Failing to exclude pre-acquisition profits from the share of retained earnings for the subsidiary and the associate.  Failing to take only the group share of the subsidiary’s and the associate’s retained earnings. Total possible marks Maximum full marks

© The Institute of Chartered Accountants in England and Wales

3 3

Page 16 of 17

Financial Accounting - Professional Stage – June 2010 (c) Concepts underlying the preparation of consolidated financial statements Group accounts reflect the results and net assets of group members to present the group to the parent’s shareholders as a economic single entity (single entity concept). This reflects the substance of the group arrangement as opposed to its legal form, where each group member is a separate legal person. For example, in the consolidation of the Jennings plc group, all revenue and costs are added together, as if the group were a single entity (so, for example, Jennings Ltd’s revenue of £67.6 million and Brandon’s revenue of £42.5 million are added). However, the single entity concept also means that any intra-group transactions need to be eliminated, as otherwise items would be double counted in the context of the group as a single entity. Hence, because Brandon Ltd sold goods for £3.6 million to Jennings plc in the current year, that amount needs to be subtracted from Brandon Ltd’s revenue and from Jennings plc’s cost of sales as if the group were a single entity that transaction would not have occurred. Any profit made between parent and its subsidiary companies also needs to be eliminated where that profit has not yet been realised outside the group. So, for the £3.6 million intra-group sale, because half of these goods have not yet been sold outside the group, closing inventory needs to be reduced (cost of sales increased) by the profit on half that amount (£300,000), otherwise inventory will be overstated from the point of view of the group as a whole. The adjustment effectively brings inventory back down to what it would have been stated at if the intra-group sale had never taken place. The other principle underlying the preparation of consolidated financial statements is the distinction between control and ownership. Control is reflected by including all of the subsidiary’s income and expenses in the consolidated income statement, even where the parent does not own 100% of that subsidiary. So, for Jennings plc, 100% of Brandon Ltd’s income and expenses are added in even though, in effect, Jennings plc only “owns” 70% of those income and expenses. Ownership is then reflected by showing that part of the subsidiary’s results, which is not “owned” by the parent, as a non-controlling interest. Jennings plc’s consolidated income statement shows a non-controlling interest of £2,745,000, representing that part of Brandon Ltd not owned by Jennings plc. Where an investor (Jennings plc) does not have control but does have significant influence over an investee (Palmer Ltd), line-by-line consolidation is not appropriate. But because Jennings has this influence, it is reflected in the consolidated income statement as a single line item – being its share of the associate’s profit for the period. Answers to Part (c) were the most disappointing and sometimes non-existent. Where an answer was attempted most referred to the single entity concept and to substance over form but did not expand on these concepts with examples from the consolidated income statement. Some gave examples from the consolidated statement of financial position, which was not what was required by the question. The distinction between ownership and control was often not mentioned, or if it was, was poorly explained, with the focus often being on how control could be achieved. Comments about the consolidation process were often vague and did not clearly show that candidates understood that 100% of a subsidiary’s figures were added in (control), and then the non-controlling interest share (ownership) taken out later – even if they had just demonstrated this in their answer to Part (a). Many referred to the fact that an associate is “not consolidated” but failed to explain that this is because there is not control, but significant influence, or to explain how the associate is dealt with in the consolidated income statement (again, even if they had just demonstrated this in Part(a)). A few candidates wrote at length about the concepts of accruals and going concern or about the qualitative characteristics and scored few, if any, marks. Total possible marks Maximum full marks

© The Institute of Chartered Accountants in England and Wales

6½ 4

Page 17 of 17

Financial Accounting – Professional Stage – September 2010

PROFESSIONAL STAGE FINANCIAL ACCOUNTING – OT EXAMINER’S COMMENTS The performance of candidates in the September 2010 objective test questions section for the Professional Stage Financial Accounting paper was in line with the average performance on this section of the paper over all sittings to date. Candidates performed well across all syllabus areas. When practising OT items, care should always be taken to ensure that the principles underlying any particular item are understood rather than rote learning the answer. In particular, candidates should ensure that they read all items very carefully. The following table summarises how well* candidates answered each syllabus content area. Syllabus area

Number of questions

Well answered

Poorly answered

LO1

2

1

1

LO2

7

6

1

LO3

6

6

0

Total

15

13

2

If 50% or more of the candidates gave the correct answer, then the question was classified as ‘well answered’. Comments on the two poorly answered questions, which covered LO1 (accounting and reporting concepts) and LO2 (preparation of single company financial statements), are below:

Item 1 This item was a knowledge based question covering the capital maintenance concept and historical cost accounting.

Item 2 This item required candidates to calculate the carrying amount of an asset following an impairment review. A simple comparison between the current carrying amount and recoverable amount was required. The most common error was believing that the asset could simply be restated to an amount higher than its current carrying amount.

© The Institute of Chartered Accountants in England and Wales 2010

Page 1 of 14

Financial Accounting – Professional Stage – September 2010

MARK PLAN AND EXAMINER’S COMMENTARY The mark plan set out below was that used to mark these questions. Markers are encouraged to use discretion and to award partial marks where a point was either not explained fully or made by implication. More marks are available than could be awarded for each requirement, where indicated. This allows credit to be given for a variety of valid points, which are made by candidates.

Question 1 Overall marks for this question can be analysed as follows:

Total: 19

General comments This question was a typical question testing the preparation of an income statement and statement of financial position from a trial balance. A number of adjustments were required to be made, including a calculation of a provision, interest and dividends payable, a prior period error, capitalisation of development expenditure and an adjustment to PPE for incorrect capitalisation. (a) £ ASSETS Non-current assets Property, plant and equipment (W2) Intangible assets (237,600 – 39,600 (W1))

Current assets Inventories Trade receivables Cash and cash equivalents

£

317,560 198,000 515,560

375,600 51,000 21,500 448,100 963,660

Total assets EQUITY AND LIABILITIES Equity Ordinary share capital Irredeemable preference shares Retained earnings (W3)

152,000 180,000 181,260

Equity

513,260

Non-current liabilities Bank loan

150,000

Current liabilities Trade and other payables (123,700 + 95,000) Taxation Provisions (310,000 x 10%) Dividend payable (W3) Interest payable

218,700 35,700 31,000 9,000 6,000 300,400

Total equity and liabilities

© The Institute of Chartered Accountants in England and Wales 2010

963,660

Page 2 of 14

Financial Accounting – Professional Stage – September 2010

Eyam plc – Income Statement for year ended 30 June 2010 £ Revenue (3,973,000 – 31,000) 3,942,000 Cost of sales (W1) (2,260,500) Gross profit Administrative expenses (W1)

1,681,500 (1,556,040)

Operating profit Finance costs (150,000 x 4%) Profit before tax Taxation

125,460 (6,000) 119,460 (35,700)

Net profit for the period

83,760

Note: Marks will be awarded if items are included in a different line item in the income statement provided that the heading used is appropriate.

W1 Expenses

Trial balance Opening inventory Less: closing inventory Operating lease rentals ((1,045,000 + 95,000) / 2) Depreciation charge – fixtures & fittings Research costs Development exp amortisation ((357,600 – 120,000) / 2 yrs x 4/12months)

Cost of sales £ 1,560,000 346,500 (375,600) 570,000

Admin expenses £ 930,000

570,000 56,040

120,000 39,600 2,260,500

1,556,040

W2 Fixtures & fittings adjustment £ Cost Less: repairs (incorrect capitalisation) Accumulated depreciation Depreciation adjustment re repairs (25,000 x 15%)

£ 646,000 (25,000)

(251,150) 3,750 (247,400) 373,600

Depreciation charge for year (373,600 x 15%)

(56,040)

Carrying amount at 30 June 2010

317,560

W3 Retained earnings B/fwd at 1 July 2009 Error re repairs capitalisation (25,000 – 3,750 (W2))

£ 127,750 (21,250)

B/fwd restated Profit in year Preference share dividend (180,000 x 5%)

106,500 83,760 (9,000)

C/fwd at 30 June 2010

181,260

© The Institute of Chartered Accountants in England and Wales 2010

Page 3 of 14

Financial Accounting – Professional Stage – September 2010

As in previous sittings, candidates were clearly very well-prepared for this type of question, which remains fundamental to the Financial Accounting syllabus. Almost all candidates produced a well-laid out income statement and statement of financial position, with a significant number of candidates sub-totalling amounts. The most common missed sub-totals include “operating profit” on the income statement and non-current assets in the statement of financial position. Presentation marks were lost for incorrectly combining some of the amounts presented in current liabilities, for example including taxation and provisions as part of “trade and other payables”. The cost matrix was generally clearly laid out, although a number of candidates were careless as to which column they allocated expenses even where this had been expressly set out in the question text. A few candidates’ workings, however, were disorganised, untidy and therefore hard to follow, making it difficult to establish candidates’ approaches where they had not calculated the correct figure. This was noticeable for the fixtures and fittings working which was often a muddle of figures sometimes resulting in candidates putting the incorrect carrying amount on the face of the statement of financial position even though they had correctly calculated this amount in their workings. Candidates often left out narrative for amounts in their cost matrix, which meant that marking the figures was frequently a challenge! It was pleasing to see that most candidates were able to deal with the majority of adjustments, most of which they would have seen on papers in the past. However, the treatment of returned goods caused a number of problems with candidates not appreciating that this would impact on revenue. Instead candidates recognised the debit side of the provision as an expense (as might be done for, say, a warranty provision). The other adjustment that caused a problem was the incorrect capitalisation of repairs in fixtures and fittings which occurred in the prior period and therefore required an adjustment to brought forward retained earnings. The majority of candidates made the adjustment through current year expenses, or not at all, although most did make the correct adjustment to property, plant and equipment. The most disappointing incorrect treatment was the inclusion of the preference dividend as a finance cost in the Income Statement even where it had already been correctly shown as a deduction in equity. Other common errors included debiting the income tax charge for the period of £35,700 to the income statement, but making no credit entry (as tax payable on the statement of financial position), correctly splitting the accrued rent of £95,000 between administrative expenses and cost of sales, but failing to split the original £1,045,000 and splitting the bank loan between non-current and current liabilities when it was all non-current. A number of common errors were made in relation to non-current assets and these included using an incorrect number of months when calculating the amortisation for the year on the development expenditure, deducting the amortisation from the carrying amount of the intangible asset but failing to show it as an expense and charging depreciation on cost instead of on carrying amount. Total possible marks Maximum full marks

© The Institute of Chartered Accountants in England and Wales 2010

19 19

Page 4 of 14

Financial Accounting – Professional Stage – September 2010

Question 2 Overall marks for this question can be analysed as follows:

Total: 25

General comments The first part of this question was a single topic question focusing on non-current assets. Issues included adjustments for revaluations and the impact on depreciation, what is included in the cost of an asset, component depreciation and the treatment of decommissioning costs. Part b) covered concept issues, with a discussion on the four measurement bases with reference to the measurement of an asset and an explanation of the advantages and limitation of using the historical cost basis. Litton plc (a) Statement of financial position as at 30 June 2010 (extracts) £ Non-current assets Property, plant and equipment (W1)

29,375,200

Non-current assets held for sale

90,000

Non-current liabilities Provision for decommissioning

1,000,000

Equity & liabilities Revaluation surplus (420,000 + 1,292,000)

1,712,000

Summary of amounts included in income statement for the year ended 30 June 2010 £ Administrative expenses: Depreciation ((115,800 – 37,500) + 960,000 + 268,000 + 130,000) Impairment of held for sale asset Allocation of general overheads

1,436,300 22,500 36,000

Other income

45,000

W1 PPE – Carrying amounts £ 7,290,000 351,200 15,040,000 4,824,000 1,870,000 29,375,200

Land (W2) Plant and machinery (W3) Hydro-electric plants (W4) Solar power plants (W5) Wind turbines (W6)

W2 Cost of land £ Cost b/fwd Purchase price Professional fees Site clearance costs Planning application

£ 2,000,000

5,000,000 150,000 125,000 15,000 5,290,000 7,290,000

Carrying amount c/fwd W3 Plant and machinery – Held for sale asset Cost Less: depreciation (300,000 / 8 yrs) x 5yrs Carrying amount at 30 June 2009 FV less costs to sell Impairment

£ 300,000 (187,500) 112,500 (90,000) 22,500

© The Institute of Chartered Accountants in England and Wales 2010

Page 5 of 14

Financial Accounting – Professional Stage – September 2010

Cost b/fwd Less: held for sale asset Cost c/fwd Accumulated depreciation b/fwd Depreciation charge for year Depreciation charged in error (re HFS asset) (300,000 / 8 yrs) Acc dep re HFS asset

£ 950,000 (300,000)

£

650,000 408,000 115,800 (37,500) (187,500) (298,800) 351,200

Carrying amount W4 Hydro-electric plant £ Cost b/fwd Cost Testing - capitalised Decommissioning Acquisition in year Cost c/fwd Accumulated depreciation b/fwd Depreciation charge for year (466,667 + 493,333)

£ 7,000,000

10,000,000 100,000 1,000,000 11,100,000 18,100,000 2,100,000 960,000 (3,060,000) 15,040,000

Carrying amount at y/e

Depreciation on acquisition – 11,100,000 / 15yrs x 8/12 months = 493,333 W5 Solar power plant Cost / valuation b/fwd Less: acc dep

£ 4,200,000 (400,000)

£

Revaluation (5,092,000 – 3,800,000) Cost / valuation c/fwd

3,800,000 1,292,000 5,092,000

Depreciation in yr: 5,092,000 / 19 yrs Carrying amount at y/e

(268,000) 4,824,000

Useful life 25 yrs – 6 yrs (1 July 2003 – 30 June 2009) = 19 yrs W6 Wind turbines Split components: 2,000,000 – 200,000 = 1,800,000 £ Cost Depreciation: 1,800,000 / 20 yrs 90,000 200,000 / 5 yrs 40,000 Carrying amount at y/e

© The Institute of Chartered Accountants in England and Wales 2010

£ 2,000,000

(130,000) 1,870,000

Page 6 of 14

Financial Accounting – Professional Stage – September 2010

There were some good attempts at this question, although workings were often difficult to follow with a haphazard array of numbers with no commentary accompanying them. There has been a marked improvement with candidate responses to property, plant and equipment questions since it was first set in December 2007. The majority of candidates tackled each of the five items of property, plant and equipment separately thereby not confusing the issues. This was the best way to maximise marks. Many candidates produced a property, plant and equipment “table” as a working, which was a reasonable approach to take and clear to mark, although to complete it in its entirety wasted valuable time when the table was not actually required by the question. Others took each type of asset in the question separately and calculated a revised carrying amount and revised depreciation charge, which was another sensible approach. However, other candidates produced a long string of plus and minus numbers, with no narrative, which was almost impossible to mark. The question asked for extracts from the statement of financial position and there were a number of straight forward marks available here based on candidates own figures. However, a number of candidates misinterpreted this as meaning the property, plant and equipment table although the requirement clearly set out that notes to the financial statements were not required. Others produced extracts but lost marks because those extracts were not properly presented (for example, the revaluation surplus was not shown under “Equity”, or “Property, plant and equipment” was abbreviated to “PPE”). Although the income statement impact did not have to be shown as an extract candidates sometimes scattered these amounts throughout their answers, instead of presenting them in a summary. The majority of candidates correctly calculated the impairment for the held for sale asset and made a good attempt at the plant and machinery calculations. One of the most common errors involved the component depreciation calculation, with candidates’ correctly calculating depreciation on the battery storage systems over five years but then adding this cost to the total rather than deducting it to get the cost of the wind turbines themselves. Candidates also often missed the wind turbines out when adding together all the items of property, plant and equipment for the extracts to the statement of financial position. Other common errors for each non-current asset item included: •

Land: deducting the rental income from cost and/or capitalising the general overheads.



Hydro-electric power plants: netting the £1 million restoration costs off the cost of £10 million, instead of adding it. A significant number of candidates showed the provision for restoration costs as an expense rather than capitalising it, or included it as a current, instead of a non-current liability. A significant number of candidates also used the incorrect number of months when calculating the current year depreciation.



Solar power plants: including the £68,000 “additional” depreciation on the surplus in the annual depreciation charge, but not adding in the original £200,000 charge based on cost. Other candidates included the whole £268,000 but also the original £200,000. A few candidates wasted time making a transfer from the revaluation surplus for the additional depreciation, when the question specifically stated that this was not the company’s policy.



Held for sale asset: calculating accumulated depreciation brought forward based on an incorrect number of years.

Total possible marks Maximum full marks

© The Institute of Chartered Accountants in England and Wales 2010

18 18

Page 7 of 14

Financial Accounting – Professional Stage – September 2010

(b)(i) Historical cost – assets are recorded at the cash or fair value of the consideration paid. Current cost – assets are recorded at the amount that it would currently cost to acquire the asset today. Realisable value – the amount that would be received if the asset was sold today in its current condition. Present value – a current estimate of the present discounted value of the future net cash flows in the normal course of business. (ii) Usefulness of historic cost • • •

Historical cost is a known amount, it is a reliable measurement – there is no subjectivity involved unlike the revaluation model where a great deal of judgement is involved. There is no cost involved in valuing historical cost as it is the amount that was paid. Measuring fair value can be extremely costly depending on the nature of the asset. Other measurement bases can be subject to manipulation, as valuation techniques need to be applied.

Limitations of historical cost •



By its very definition it is an historical amount and therefore does not reflect the true value that the asset may be worth unlike revalued amounts which are current at the time of the valuation. For example, property prices generally increase over time, so a property acquired a number of years ago will be shown in the financial statements at a value significantly less than its true value to the business. Historical cost also ignores the effects of inflation.

Responses to part b) were generally disappointing, although there were some very good answers to this part. Most candidates made some attempt at the first part. In (i), most candidates knew where to find an explanation of the four measurement bases in the open book text and used the book to good effect, although a few provided unnecessarily long explanations. Others, however, explained the mechanics of the cost model as opposed to the revaluation model, with much detail given about when and how to revalue. Others confused the four measurement bases with capital maintenance concepts. Answers to the second element were often not thought out with candidates instead writing everything they knew about historical cost and the revaluation model. A significant number of candidates discussed how the cost model suffered from the problems of estimating an accurate useful economic life, seemingly unaware that such a judgement is also needed when using the revaluation model (except for land). Others believed that impairments only needed to be accounted for under the cost model. Although the concepts of relevance and reliability were relevant, these needed to be explained, by reference to the factual nature of historic costs and the subjective nature of revaluations. Those who wrote generally about relevance and reliability scored poorly. A worrying few thought that revaluation made figures more reliable. Total possible marks Maximum full marks

8 7

Total possible marks Maximum full marks

26 25

© The Institute of Chartered Accountants in England and Wales 2010

Page 8 of 14

Financial Accounting – Professional Stage – September 2010

Question 3 Overall marks for this question can be analysed as follows:

Total: 15

General comments This was a mixed topic question covering revenue recognition, inventory valuation and investment in associates. Part a) covered revenue recognition issues which included subscriptions, a service contract, advertising revenues and sale and returns. Part b) required a straight forward calculation of closing inventories based on units of production. Part c) required the calculation of the investment in associate line in the consolidated statement of financial position following a PURP adjustment and the impact of the associate on retained earnings. (a) Calculation of revenue £ Draft revenue Magazine subscriptions adjustment (W1) Promotional advertising (17,500 / 2) Returns (1,500 x £2) News service (W2)

2,176,900 (203,000) 8,750 (3,000) 7,500

Revised revenue

1,987,150

WORKINGS W1 Magazine subscriptions Receipt date

Subscription period

Adjustment

Total £

March 2010 April 2010 May 2010 June 2010

Apr – June May – July June – Aug Jul – Sept

129,000 x 0 84,000 x 1/3 96,000 x 2/3 111,000

– 28,000 64,000 111,000 203,000

W2 Service contract Revenue: Cash received (1 Oct 2009)

(30,000 x 9/12 months)

Additional revenue to recognise

£ 22,500 (15,000) 7,500

Candidates generally made some attempt at this part of the question with the majority of candidates gaining at least half marks in it and many scoring full marks. A common mistake was to calculate total revenue from the items given, as opposed to adjusting the given (total) revenue figure for the extra revenue that needed to be added or deducted. These candidates therefore lost marks for not showing what had been brought in on a cash basis. The most common errors included making adjustments in the wrong direction, taking out the wrong fraction of subscriptions received in April and May and only adjusting for subscriptions received in June.

Total possible marks Maximum full marks

© The Institute of Chartered Accountants in England and Wales 2010

6 6

Page 9 of 14

Financial Accounting – Professional Stage – September 2010

(b) Closing inventories (£1.20 x (25,500 – 2,000)) + (£1 x 2,000) = £30,200 WORKINGS Material cost Variable overheads

Unit cost 600,000 / 800,000 450,000 / 1,000,000

£ 360,000 240,000 600,000

0.75 0.45 £1.20

Answers to this part of the question were the most disappointing on the paper as a whole. The question highlighted a lack of understanding in a very straight-forward part of the syllabus. The most common failing was to include the administrative expenses as part of the inventory unit cost, then spread it either over the 800,000 items produced or over the 1 million budgeted production. Most candidates did, however, recognise that the 450,000 fixed production costs should be spread over the 1 million. Other common errors included calculating a unit cost but then applying it to the whole 25,500 items in yearend inventory, not just the 23,500 which needed to be valued at cost, applying the calculated unit cost to 23,500 items, but failing to add in the other 2,000 items, spreading all costs (with or without the administrative costs) over the 1 million budgeted production and calculating a unit cost but then valuing all inventory at selling price. In addition, a number of candidates failed to calculate a unit cost at all, simply valuing all inventory at selling price (all at £2.50 or 23,500 at £2.50 with the 2,000 correctly valued at £1) or added all costs together (with or without the administrative costs) and valued inventory at that figure, with no attempt to spread that total cost over the number of units produced or budgeted for.

Total possible marks Maximum full marks

3 3

(c)(i) £ 300,000

Cost of investment Share of post-acq retained earnings ((295,000 + 40,000) – 250,000) x 40% Less: Impairment

34,000 (5,000)

Investment in associate

329,000

(ii)

Bretby plc Alport - share of post-acq retained earnings (34,000 – 14,000) Less: Impairment Consolidated retained earnings

£ 1,670,000 20,000 (5,000) 1,685,000

© The Institute of Chartered Accountants in England and Wales 2010

Page 10 of 14

Financial Accounting – Professional Stage – September 2010

W1 Unrealised profit –associate 200,000 (130,000) 70,000

100% 65% 35%

£70,000 x 40% x ½ = £14,000 Answers to this part were quite mixed, with most candidates picking up at least a few marks for stating the cost of the investment and correctly showing the consolidated retained earnings figures without the associate. Candidates clearly are not comfortable with consolidation workings involving an associate, which was a direct contrast to a very good performance on Question 4, which featured two subsidiaries. There were a number of correct answers, but the most common errors were over the treatment of the PURP (which many adjusted for in both calculations, instead of just in retained earnings) or in a lack of consistency between the share of profits taken to the investment in the associate and that taken to retained earnings. Another common error was to adjust for the whole of the £35,000 PURP, as opposed to only 40% of that figure.

Total possible marks Maximum full marks Total possible marks Maximum full marks

© The Institute of Chartered Accountants in England and Wales 2010

6 6 15 15

Page 11 of 14

Financial Accounting – Professional Stage – September 2010

Question 4 Overall marks for this question can be analysed as follows:

Total: 21

This question required the preparation of a consolidated statement of financial position. The group had two subsidiaries, one of which was acquired during the year. The treatment of a gain on bargain purchase was covered and a fair value adjustment in relation to inventories was required. Inter-company trading had taken place during the year. Pinxton plc (a) Consolidated statement of financial position as at 30 June 2010 £ Assets Non-current assets Property, plant and equipment (670,000 + 140,000 + 240,000) Intangibles (265,000 + 20,000) Goodwill (W5)

£

1,050,000 285,000 53,760 1,388,760

Current assets Inventories (135,000 + 60,000 + 65,000 + 3,750 (W3) – 2,000 (W2)) Trade and other receivables (96,400 + 63,000 + 86,400 – 30,000) Cash and cash equivalents (18,900 + 14,000 + 7,950)

261,750 215,800 40,850

Total assets

518,400 1,907,160

Equity and liabilities Equity Ordinary share capital Share premium account Retained earnings (W7) Attributable to the equity holders of Pinxton plc Non-controlling interest (W6) Equity

950,000 310,000 310,550 1,570,550 83,910 1,654,460

Current liabilities Trade and other payables (89,600 + 51,000 + 88,300 – 30,000) Taxation (33,200 + 6,700 + 13,900) Total equity and liabilities

198,900 53,800 252,700 1,907,160

Workings (1) Group structure Pinxton plc

153 / 170 = 90% Hayfield Ltd 192 / 240 = 80% Smisby Ltd

© The Institute of Chartered Accountants in England and Wales 2010

Page 12 of 14

Financial Accounting – Professional Stage – September 2010 (2) Net assets – Hayfield Ltd

Share capital Retained earnings PURP

30 June 2010 £ 170,000 69,300 (2,000)

Acquisition £ 170,000 72,000

Post acq £



(2,700) (2,000)

237,300

242,000

(4,700)

30 June 2010 £ 240,000 30,000 27,150 3,750

Acquisition £ 240,000 30,000 10,300 15,000

Post acq £

300,900

295,300

5,600



(3) Net assets – Smisby Ltd

Share capital Share premium account Retained earnings FV inventory adj (90,000 – 75,000) x 25%

– – 16,850 (11,250)

(4) Goodwill – Hayfield Ltd £ 190,000 (242,000) 24,200

Consideration transferred Net assets at acquisition (W2) Non-controlling interest at acquisition (242,000 (W2) x 10%) Gain on bargain purchase

(27,800)

(5) Goodwill – Smisby Ltd £ 300,000 (295,300) 59,060 63,760 (10,000) 53,760

Consideration transferred Net assets at acquisition (W3) Non-controlling interest at acquisition (295,300 (W3) x 20%) Less: Impairment

(6) Non-controlling interest £ 23,730 60,180 83,910

Hayfield Ltd – share of net assets (237,300 (W2) x 10%) Smisby Ltd – share of net assets (300,900 (W3) x 20%)

(7) Retained earnings £ 292,500 (4,230) 4,480 (10,000) 27,800 310,550

Pinxton plc Hayfield Ltd ((4,700) (W2) x 90%) Smisby Ltd (5,600 x 80%)) Less: Impairments to date Gain on bargain purchase (W4) (8) PURP SP (30,000 x 1/3) Cost (30,000 / 125% x 1/3) GP

© The Institute of Chartered Accountants in England and Wales 2010

% 125 (100) 25

Hayfield Ltd £ 10,000 (8,000) 2,000

Page 13 of 14

Financial Accounting – Professional Stage – September 2010

Answers to this question were very good, showing that candidates are very comfortable with the consolidation of subsidiaries. Presentation was on the whole good, although some candidates did lose presentation marks, often for failing to show the split of total equity between the parent and the non controlling interest. A few candidates lost marks because they failed to show bracketed workings for incorrect lines on their consolidated statement of financial position. It was pleasing to see that a good number of candidates understood that the gain on bargain purchase should be recognised in retained earnings (although a number of candidates then deducted it, instead of adding it) rather than being netted off against the positive goodwill shown in the consolidated statement of financial position. Candidates generally made a good attempt at the two net asset tables, although a common error was to use a positive figure for the movement in post-acquisition retained earnings even though they had calculated one of the subsidiaries to be negative. Workings for this question were generally clearly set out with the standard set of pro-forma workings being used. This makes responses easier to mark with marks being awarded for partially correct answers. One of the most common mistakes on this question in respect of presentation was to try and calculate the goodwill figures in a combined calculation. In this question there was positive and negative (gain on bargain purchase) goodwill and the two amounts should not be netted off (as discussed above). Where errors were made they included the following, not including the share premium account in the calculation of Smisby’s net assets, or including it either at acquisition or at the year end but not at both dates, failing to account for the intangible assets held by Pinxton and Hayfield and treating the information in relation to the fair value adjustment for inventories at acquisition as leading to a reduction to net assets as opposed to a fair value uplift. If this amount was correctly recognised in the net assets table candidates often did not go on to make a corresponding adjustment to inventories on the consolidated statement of financial position. Other common errors included failing to eliminate the £30,000 intra-group balance and using the noncontrolling interest percentages to calculate the share of Hayfield’s and Smisby’s post-acquisition profits to be taken to consolidated retained earnings. Interestingly, after their consolidation, some candidates were left with a figure for “Investments” on their consolidated statement of financial position. Sometimes this was the whole £490,000, sometimes just a part of that figure. It was also interesting to note that a number of candidates made the correct fair value and PURP adjustments in the net assets working but then took the wrong post acquisition figures to retained earnings, for example, just taking the movement on reserves and ignoring the impact of the adjustments they had just made.

Total possible marks Maximum full marks

© The Institute of Chartered Accountants in England and Wales 2010

21 21

Page 14 of 14

Financial Accounting - Professional Stage – June 2013

PROFESSIONAL STAGE FINANCIAL ACCOUNTING – OT EXAMINER’S COMMENTS The performance of candidates in the June 2013 objective test questions section for the Professional Stage Financial Accounting paper was good. Candidates performed better on LO3 (preparation of consolidated financial statements) than they did on the other two syllabus areas. When practising OT items, care should always be taken to ensure that the principles underlying any particular item are understood rather than rote learning the answer. In particular, candidates should ensure that they read all items very carefully. The following table summarises how well* candidates answered each syllabus content area. Syllabus area

Number of questions

Well answered

Poorly answered

LO1

4

3

1

LO2

6

4

2

LO3

5

5

0

Total

15

12

3

*If 50% or more of the candidates gave the correct answer, then the question was classified as ‘well answered’. Comments on the two most poorly answered questions, both in LO2 (preparation of single company financial statements) are given below: Item 1 This item required candidates to calculate closing inventory in accordance with IAS 2, Inventories. The question featured raw materials, work in progress and finished goods. Almost all candidates calculated a net realisable value for work in progress and finished goods which was lower than cost and used that figure in their calculation. However, although most candidates correctly allowed for a discounted selling price and for costs still to be incurred to complete the work in progress, a majority did not reduce the discounted selling price by the selling costs to be incurred to arrive at the correct figure for net realisable value. Item 2 This item tested the calculation of the amount of an intangible asset to be capitalised in accordance with IAS 38, Intangible Assets. Most candidates recognised that initial research costs and the cost of evaluating research findings should not be capitalised and that development costs and patent registration costs should be capitalised. However, a majority of candidates failed to recognise that the depreciation charged in the period on specialised equipment needed for the development process should also have been capitalised.

Copyright © ICAEW 2013. All rights reserved.

Page 1 of 14

Financial Accounting - Professional Stage – June 2013

PROFESSIONAL STAGE FINANCIAL ACCOUNTING MARK PLAN AND EXAMINER’S COMMENTARY The marking plan set out below was that used to mark this question. Markers were encouraged to use discretion and to award partial marks where a point was either not explained fully or made by implication. More marks were available than could be awarded for each requirement. This allowed credit to be given for a variety of valid points which were made by candidates.

Question 1 Overall marks for this question can be analysed as follows:

Total: 30

General comments Part (a) of this question tested the preparation of an income statement (which needed to be split between continuing and discontinued operations) and a statement of financial position from a list of balances plus a number of adjustments. Adjustments included a warranty provision, calculation of the annual depreciation charge, a finance lease taken out during the year, and an adjustment to revenue to reflect IAS 18, Revenue. Part (b) required a discussion of the objective of general purpose financial statements and the purpose of accounting standards, illustrated by reference to the financial statements prepared in Part (a). Falcon Ltd (a) Income statement for the year ended 31 December 2012 £ Continuing operations Revenue (W5) Cost of sales (W1) Gross profit Distribution costs (W1) Administrative expenses (W1) Profit from operations Finance cost (12,600 + 1,000 (W6)) Profit before tax Income tax expense (35,000 – 2,000) Profit for the year from continuing operations

1,264,600 (631,750) 632,850 (38,200) (223,200) 371,450 (13,600) 357,850 (33,000) 324,850

Discontinued operations Loss for the year from discontinued operations (W2) Profit for the year

(164,600) 160,250

Statement of financial position as at 31 December 2012 £ Assets Non-current assets Property, plant and equipment (W3) Current assets Inventories Trade and other receivables Non-current asset held for sale Total assets

Copyright © ICAEW 2013. All rights reserved.

£

543,750 35,600 32,800 68,400 80,000 148,400 692,150

Page 2 of 14

Financial Accounting - Professional Stage – June 2013 Equity and liabilities Equity Ordinary share capital Retained earnings (W4)

200,000 140,550 340,550

Non-current liabilities Finance lease liability (W6) Deferred income (60,000 x 1/3)

103,500 20,000 123,500

Current liabilities Trade and other payables Taxation Deferred income (60,000 x 1/3) Provisions Finance lease liability (W6) Borrowings

78,500 35,000 20,000 55,000 5,000 34,600 228,100 692,150

Total equity and liabilities Workings (1) Allocation of expenses Cost of sales

Per Q Opening inventories Adj re discontinued operations Adj re lease payment Closing inventories Depreciation charges (W3)

£ 744,300 30,200 (160,900) (35,600) 53,750 631,750

Distribution costs £ 43,500 (5,300)

38,200

Administrative expenses £ 235,600 (17,400) (5,000) 10,000 223,200

(2) Loss on discontinued operations £ 114,000 (183,600) (40,000)

Revenue Costs included in TB (160,900 + 17,400 + 5,300) Loss on held for sale asset/depreciation (120,000 – (85,000 – 5,000)) Other discontinued operations costs

(55,000) (164,600)

(3) PPE

B/f Cost B/f Accumulated depreciation Classified as held for sale Leased building (W6) Depreciation – plant @ 25% Depreciation – buildings ((275,000 x 2%) Ab + (112,500 ÷ 25) (OF))

Plant and equipment £ 570,600 (235,600) (120,000)

112,500 215,000 (53,750) (10,000) 161,250

Total PPE

Copyright © ICAEW 2013. All rights reserved.

Land and buildings £ 375,000 (95,000)

382,500 543,750

Page 3 of 14

Financial Accounting - Professional Stage – June 2013

(4) Retained earnings £ (19,700) 160,250 140,550

At 1 January 2012 Profit for the period At 31 December 2012 (5) Revenue

£ 1,418,600 (40,000) (114,000) 1,264,600

Per TB Less: After sales support re future years (60,000 x 2/3) Discontinued operations

(6) Lease of land and buildings SOD = (24 x 25)/2 = 300 £ 125,000 (112,500) 12,500

Total payments (5,000 x 25) Fair value Finance charge Year ended 31 December

B/f

Payment

Capital

Interest

2012

£ 112,500

£ (5,000)

£ 107,500

£ 24/300 x 12,500) 1,000

2013

108,500

(5,000)

103,500

C/f £ 108,500

Tutorial note Credit was also given if candidates depreciated the held for sale asset to the date of classification as held for sale, and then calculated a (smaller) impairment loss. The shortcut taken above recognised the fact that only a single figure for PPE was required for the statement of financial position.

Copyright © ICAEW 2013. All rights reserved.

Page 4 of 14

Financial Accounting - Professional Stage – June 2013

Most candidates made a reasonable attempt at this question with the vast majority preparing a complete statement of financial position and income statement. Presentation was reasonable, although a significant number of candidates lost marks by failing to include appropriate sub-totals on their statement of financial position. The majority of candidates did not appear to have been unnerved by the inclusion of a discontinued operation in this question and, pleasingly, almost all included a figure for loss from discontinued operations on the face of the income statement, although hardly any candidates showed headings for “Continuing operations” and “Discontinued operations” (which was surprising as these were included in Question 2 on the paper). The adjustments to revenue, opening and closing inventory, the finance lease calculations, the asset held for sale and allocation of costs to the correct expense category were all well dealt with, although very many candidates calculated a second year interest charge for the finance lease when this was not needed. By far the most common errors were the failure to correctly split the finance lease liability between current and non-current, the inclusion of the wrong amount on the statement of financial position for income tax payable (including the charge for the year, as opposed to the liability for the year), and an incomplete calculation of the loss from discontinued operations. With regards to the latter, almost all candidates calculated this as the sales of the Scottish operations less its costs, but far less increased this loss by the estimated future costs and/or by the depreciation and/or impairment on the held for sale asset, with many candidates either ignoring these figures or including them in continuing operations. A minority of candidates time-apportioned the figures for the Scottish operations, clearly not understanding how IFRS 5 should be applied. Other common errors included the following. - Failing to deduct the prior year tax overestimate from the current year tax estimate to arrive at the correct current year income statement charge and/or showing the wrong figure in current liabilities. - Not correcting administrative expenses to remove the lease payment incorrectly posted to this account. - Incorrectly calculating the sum-of-the-digits for the finance lease. - Treating the resultant deferred income as a current or non-current asset rather than as a liability or failing to split the liability between current and non-current. - Showing the asset held for sale in non-current as opposed to current assets. - Taking the depreciation on the leased asset to cost of sales instead of administrative expenses. - Adding the opening retained loss to the profit for the year, instead of deducting it. - Failing to include a sub-total for operating profit on the income statement. - Calculating, usually correctly, a carrying amount for the leased asset but then failing to add that figure in to the property, plant and equipment figure for the statement of financial position. - Calculating depreciation and/or impairment on the held for sale asset but failing to remove the cost of the held for sale asset from property, plant and equipment. - Depreciating the leased asset by 2% when it had a shorter useful life of 25 years. Almost all candidates did use the recommended “costs matrix” when allocating costs between the three expense categories. It was very noticeable that those candidates who did not use this format tended to produce disorganised workings (often split between the face of the income statement and /or a number of separate workings) which were difficult to follow and therefore might have lost marks. Far fewer candidates seem capable of producing a clear working for property, plant and equipment which resulted in them repeating calculations and often losing the “connection” between the depreciation expense to go into the costs matrix and the depreciation expense to be added to accumulated depreciation brought forward. It was often impossible to see any “audit trail” to support the final figure for property, plant and equipment on the face of the statement of financial position and many candidates lost potential marks because of this. This is an issue which has been flagged up repeatedly. Total possible marks Maximum full marks

Copyright © ICAEW 2013. All rights reserved.

25 25

Page 5 of 14

Financial Accounting - Professional Stage – June 2013 (b) The objective of general purpose financial reporting The IASB Conceptual Framework states that the objective of general purpose financial reporting is to provide financial information about the reporting entity that is useful to existing and potential investors, lenders and other creditors in making decisions about providing resources to the entity. These decisions involve buying, selling or holding equity and debt instruments and providing or settling loans and other forms of credit. For example, the lessor of the land and buildings may have looked at Falcon Ltd’s previous financial statements in deciding whether or not to extend credit. They would have considered whether Falcon Ltd would be likely to be able to meet the lease repayment terms. Investors would be particularly interested in the information concerning continuing versus discontinued operations – particularly as the continuing operations have made a profit of £324,850, but the discontinued operations have made a loss of £164,600. Without this split it may have seemed that Falcon Ltd was only able to generate profits of £160,250, less than half of its actual continuing profits. Falcon Ltd’s suppliers may look at the financial statements in deciding whether or not to grant credit – they may be concerned that the fact that Falcon Ltd’s current liabilities are way in excess of its current assets may mean that the company could struggle to pay its debts as they fall due. The purpose of accounting standards The purpose of accounting standards is to identify proper accounting practices for the preparation of financial statements. Accounting standards create a common understanding between users and preparers on how particular items are treated. For example, it will be clear from Falcon Ltd’s financial statements that it carries its property, plant and equipment under the cost model. Users will need to then take care if comparing Falcon Ltd’s financial statements with those which use the valuation model. It is IAS 17, Leases, which dictates the correct treatment of finance versus operating leases. Hence the lease of the building was treated as a finance lease. This will be common practice across all entities following IFRS and will make their financial statements comparable with those of other companies. IAS 18, Revenue, dictates that Falcon Ltd only account for revenue on services provided to date. Hence an adjustment was made in Part (a) to remove the revenue relating to after-sales support not yet provided. Again, this will be common practice across all entities following IFRS Attempts at the written part of the paper were, as usual, disappointing, with very few candidates scoring more than one or two marks. Many failed to gain the marks for those parts of the answer that could be taken from the open book text, and only a minority were able to provide examples from Falcon Ltd’s financial statements which were relevant to either the objective of general purpose financial reporting or to the purpose of accounting standards. Whilst most candidates recognised that accounting standards helped to achieve consistency or comparability very few made the point that accounting standards inform the preparers of accounts how to deal with key accounting issues in the financial statements. Many candidates thought that the main purpose of accounting standards is to ensure that financial statements are prepared on the basis of “substance over form” and then proceeded to give examples of accounting for substance over form. Others discussed the qualitative characteristics of financial statements at length and gave examples of how various accounting standards met these qualitative characteristics. Total possible marks Maximum full marks

Copyright © ICAEW 2013. All rights reserved.

8½ 5

Page 6 of 14

Financial Accounting - Professional Stage – June 2013

Question 2 Overall marks for this question can be analysed as follows:

Total: 19

General comments This question tested the preparation of a consolidated statement of cash flows and supporting note, where a subsidiary had been disposed of during the year. Missing figures to be calculated included dividends paid (to the group and to the non-controlling interest), dividends received, tax paid, additions to property, plant and equipment, and proceeds from the issue of share capital. Eagle plc Consolidated statement of cash flows for the year ended 31 December 2012 £ Cash flows from operating activities Cash generated from operations (Note) 495,850 Interest paid (W1) (20,000) Income tax paid (W2) (81,200) Net cash from operating activities Cash flows from investing activities Purchase of property, plant and equipment (W3) (460,200) Proceeds from sale of property, plant and equipment 60,000 Dividends received from associate (W4) 50,600 Disposal of Owl Ltd net of cash disposed of ((194,450 x 80%) + 10,500) – 1,500) 164,560 Net cash used in investing activities Cash flows from financing activities Proceeds from share issues (220,000 + 50,000) – 140,000 110,000 + 20,000)) Repayment of long-term loan (150,000 – 125,000) (25,000) Dividends paid (W5) (266,200) Dividends paid to non-controlling interest (W6) (22,410) Net cash used in financing activities Net increase in cash and cash equivalents Cash and cash equivalents at beginning of period Cash and cash equivalents at end of period

£

394,650

(185,040)

(203,610) 6,000 14,500 20,500

Note: Reconciliation of profit before tax to cash generated from operations Profit before tax (324,100 + 41,400) Share of profits of associate Finance cost Profit on disposal of property, plant and equipment (60,000 – 56,000) Depreciation charge Increase in trade and other receivables ((75,700 + 13,900) – 88,900) Decrease in trade and other payables ((52,800 – 3,000) – (40,500 + 8,450 – 5,000)) Cash generated from operations

Copyright © ICAEW 2013. All rights reserved.

£ 365,500 (56,700) 22,000 (4,000) 175,600 (700) (5,850) 495,850

Page 7 of 14

Financial Accounting - Professional Stage – June 2013

Workings (1) Interest paid

Cash (β) C/d

£ 20,000 5,000 25,000

B/d CIS

£ 3,000 22,000 25,000

(2) Income tax

Cash (β) C/d

£ 81,200 68,000 149,200

B/d CIS (64,800 + 6,400)

£ 78,000 71,200 149,200

(3) Property, plant and equipment

B/d

£ 983,500

Additions (β)

460,200

Disposal of sub Other disposals Depreciation charge C/d

1,443,700

£ 187,500 56,000 175,600 1,024,600 1,443,700

(4) Investment in associate

B/d CIS

£ 179,800 56,700 236,500

Cash received (β) C/d

£ 50,600 185,900 236,500

£ 266,200 663,000 929,200

B/d CIS

£ 675,100 254,100 929,200

(5) Retained earnings

Dividends in SCE (β) C/d

(6) Non-controlling interest

Cash (β) Disposal (194,450 x 20%) C/d

Copyright © ICAEW 2013. All rights reserved.

£ 22,410 38,890 140,200 201,500

B/d CIS

£ 150,800 50,700 201,500

Page 8 of 14

Financial Accounting - Professional Stage – June 2013 Most candidates produced a well-presented statement of cash flows with all the relevant headings, although the headings that continue to let candidates down are “property, plant and equipment” which was all too often shortened to “PPE” and Dividends paid to non-controlling interest” abbreviated to use “NCI”. Such abbreviations, in one of the main financial statements, will cause candidates to lose presentation marks. As usual, many candidates lost marks for use of the incorrect bracket convention and/or including cash flows under the wrong headings. Typically for a question featuring a consolidated statement of cash flows, candidates lost most marks on the groups aspect of the question, in particular failing to adjust accurately for the disposal of the subsidiary and miscalculating the figure for the actual disposal as it should appear in the statement. In the main, workings took the form of T accounts, with very few candidates completing some of their T accounts with all the entries the wrong way round. However, some candidates still insist on producing tabular workings or workings on the face of the statement of cash flows. This can make it more difficult to see evidence of correct double entry and to award marks where the final figure is incorrect (or uses the incorrect bracket convention). Pleasingly, hardly any candidates produced no workings at all – an even riskier approach as if figures are calculated incorrectly it is not possible to award any partial marks. The reconciliation note was generally well done with candidates making a good attempt at the adjustments. However, most candidates failed to add the disposed of subsidiary’s profit before tax to the continuing profit before tax, to form the first figure of the reconciliation note. Most candidates correctly made the adjustments for the associate, the finance costs and the depreciation charge. The profit on disposal of property, plant and equipment was often ignored or added instead of being deducted. It was less common to see the correct adjustments for the movements in trade receivables and payables, with most errors being made over the adjustment for the disposed of subsidiary’s figures. Figures for income tax paid, interest paid, repayment of the long-term loan and purchase of property, plant and equipment were those most commonly calculated correctly and shown correctly on the face of the statement although in a minority of cases a positive figure was shown. However, a significant number of candidates failed to adjust, or adjusted incorrectly for, the tax on the discontinued operations in the income tax T-account. Others omitted to include all three necessary credit entries in the property, plant and equipment T-account. Many candidates also correctly calculated the figure for dividends received from the associate and dividends paid to the parent, although a number misclassified the former as a financing cash flow and others included total profit for the year in retained earnings instead of including only the group share. It was less common to see the correct figure for dividends paid to the non-controlling interest as, once again, many candidates failed to adjust or adjusted incorrectly for the subsidiary disposed of during the period. It was also rare to see the correct figure for the disposal of the subsidiary, with many miscalculating the sale proceeds (showing a lack of understanding as to how the profit on disposal is calculated), although most clearly knew to deduct the cash disposed of from their figure (although a minority deducted this from the closing cash and cash equivalents figure in the statement of cash flows). Only a minority of candidates correctly calculated the proceeds from the share issues, as many failed to appreciate that the bonus issue (which was made out of the share premium account) was effectively a contra entry between the share capital and share premium accounts. Hence all that was needed was a comparison of the opening and closing figures in a combined T-account to arrive at the cash proceeds. Total possible marks Maximum full marks

Copyright © ICAEW 2013. All rights reserved.

19 19

Page 9 of 14

Financial Accounting - Professional Stage – June 2013

Question 3 Overall marks for this question can be analysed as follows:

Total: 22

General comments This question featured a group of companies, comprising parent, two subsidiaries (one acquired during the year) and one associate. Part (a) required the calculation of the goodwill arising on acquisition of the new subsidiary, with fair value adjustments to be made. Part (b) required the preparation of the consolidated income statement. Adjustments included intra-group transactions of both inventory and property, plant and equipment, and impairment write-downs. Kite plc (a) Goodwill arising in the business combination with Vulture Ltd £ Consideration Cash Shares at market value (300,000 x £1.30) Net assets at acquisition Share capital Retained earnings at 1 January 2012 Profit to 31 March 2012 (49,200 x 3/12) Less: Dividends paid Fair value adjustments: Re building (W) Goodwill to be written off (33,000 – (3,600 x 3/12)) Contingent liability

£ 15,000 390,000 405,000

100,000 567,000 12,300 (120,000) 44,000 (32,100) (20,000) (551,200) 165,360 19,160

Non-controlling interest at acquisition x 30% Working – fair value adjustment re building

£ 154,000 (110,000)

Fair value at 1 April 2012 Carrying amount at 1 April 2012 (250,000 – ((250,000 ÷ 25) x 14)) 44,000

Almost all candidates used the correct methodology for calculating goodwill, although some lost marks for not showing the % used to calculate the non-controlling interest share of the net assets at acquisition. It is not sufficient to put “x NCI%”. In calculating the consideration almost all candidates included the cash of £15,000 and the shares at market value. However, many then also included the professional fees that should have been written off to expenses. Others deducted this figure, failing to appreciate that whilst the company had incorrectly included this figure in the cost of investment (which was not given in the question), they themselves had not, having correctly added the cash and the shares. It was therefore rare to see this amount written off to operating expenses in Part (b). In calculating the net assets at acquisition, almost all candidates added share capital, retained earnings brought forwards and three months of the profit for the year. Fewer deducted the dividends paid and fewer still made the correct adjustment for goodwill (although a good number adjusted by the gross figure of £33,000). Where an adjustment was made for the contingent liability, most did use the correct figure of £20,000. A good number of candidates correctly calculated the fair value adjustment in respect of the building, although a significant minority calculated this in Part (b) and did not adjust for it in Part (a). Some credit was given for this. Total possible marks Maximum full marks

Copyright © ICAEW 2013. All rights reserved.

6½ 6

Page 10 of 14

Financial Accounting - Professional Stage – June 2013

(b) Consolidated income statement for the year ended 31 December 2012 £ 2,818,550 (1,850,525) 968,025 (584,000) 384,025 7,080 391,105 (76,125) 314,980

Revenue (W1) Cost of sales (W1) Gross profit Operating expenses (W1) Profit from operations Share of profit of associate ((30,600 x 40%) – 3,000 – 2,160 (W2)) Profit before tax Income tax expense (W1) Profit for the period Profit attributable to Owners of Kite plc (β) Non-controlling interest (W3)

275,500 39,480 314,980

Workings (1) Consolidation schedule

Revenue Cost of sales – per Q – PURP (W2) – PPE PURP ((275,000 – 234,500) ÷ 5) Op expenses – per Q – prof fees re acquisition – additional deprec on building ((44,000 (OF from a) ÷ 11) x 9/12) – adj re GW w/o on acq (3,600 x 9/12) – GW impairment Tax

Kite plc

Harrier Ltd

£

£

1,579,500 (1,050,600)

879,500 (598,700) (13,200) 8,100

491,550 (328,125)

(103,800)

(117,300)

(345,600) (5,000)

Vulture Ltd 9/12 £

Adj

Consol

£

£

(132,000) 132,000

2,818,550

(1,850,525)

(3,000)

2,700 (12,000) (37,500)

(29,400) 142,500

(584,000) (76,125)

(9,225) 36,600

(2) PURPs

Sales Cost of sales GP x½ X 40%

% 100 (80) 20

Harrier Ltd £ 132,000 (105,600) 26,400 13,200

Buzzard Ltd £ 54,000 (43,200) 10,800 5,400 2,160

(3) Non-controlling interest in year Harrier Ltd (20% x 142,500 (W1)) Vulture Ltd (30% x 36,600 (W1))

Copyright © ICAEW 2013. All rights reserved.

£ 28,500 10,980 39,480

Page 11 of 14

Financial Accounting - Professional Stage – June 2013 The consolidation schedule was generally prepared correctly with almost all candidates appreciating that only nine-twelfths of the acquired subsidiary’s results should be included. Candidates who produced a consolidation schedule gained the majority of the more straightforward marks available and then usually went on to prepare a reasonable consolidated income statement which gained a good number of the presentation marks available. The most common loss of presentation marks was for abbreviating noncontrolling interest to “NCI”. Very few candidates did their consolidation workings on the face of the group income statement, which was pleasing. Most candidates correctly reduced group revenue and cost of sales by the sales made between the parent and the subsidiary, but some also made the same adjustment with the sales between the parent and the associate. The majority of candidates correctly calculated the two provisions for unrealised profit on intra-group sales, although some then forgot to reduce the group share of the associate’s profit by only the group share of the associate’s provision for unrealised profit. The non-controlling interest was correctly calculated by the majority of candidates although a small number used the group percentage holding rather than the noncontrolling interest percentage. Others omitted to state what percentage they were using in this calculation (or what figure they were multiplying this percentage by) and so lost marks. Most candidates made a reasonable attempt at the associate calculation, with the most common error being a failure to adjust for the unrealised profit (or forgetting to adjust it for the associate percentage). A few candidates omitted to reduce the group share of the associate’s profit by the impairment loss in respect of the associate, with some charging this instead against the parent’s profits. The most common mistakes were made in the calculation of the adjustments in the consolidation schedule. The provision for unrealised profit in respect of the subsidiary’s sales was generally included correctly along with the goodwill impairment, although a significant number of candidates also deducted the impairment loss in respect of the associate. A minority of candidates included the goodwill impairment in the subsidiary’s column instead of in the parent’s. The intra-group sale of a machine was less well dealt with, with only a minority of candidates getting this completely correct. A significant number of candidates calculated the correct unrealised profit figure but then went on to add it to cost of sales rather than deducting it. It was also common to see this figure in the parent’s column rather than the subsidiary’s (ie seller’s) column. A significant number of candidates failed to appreciate that this transfer had taken place in the previous year and so there was no need to make an adjustment for the original profit on transfer, but just for the difference in subsequent depreciation. The additional depreciation on the fair value adjustment calculated in Part (a) also produced a number of different answers with only a minority of candidates gaining all the marks for the calculation and for dealing with the adjustment correctly. Common mistakes were to either not take nine-twelfths of the of one year’s worth of the adjustment or to use the incorrect adjustment in the first place. The most common error for the write-off of the goodwill on the unincorporated business was to not adjust by nine-twelfths and/or to add the resultant figure to operating expenses rather than deducting it. Total possible marks Maximum full marks

Copyright © ICAEW 2013. All rights reserved.

16 16

Page 12 of 14

Financial Accounting - Professional Stage – June 2013

Question 4 Overall marks for this question can be analysed as follows:

Total: 9

General comments This question required the preparation of a statement of changes in equity for a single entity. Opening balances were provided together with a series of events which occurred during the current year. Matters to be dealt with included the issue of ordinary shares, the payment of ordinary and redeemable preference dividends, a prior period error, a change from the cost model to the revaluation model (with resultant reserve transfer) and a change of depreciation method. Some of the matters also impacted on the draft profit for the period. Hawk Ltd Statement of changes in equity for the year ended 31 December 2012 Ordinary Share premium share capital

Retained earnings Revaluation surplus

At 1 January 2012

£ 500,000

£ 125,000

£ 489,700

£

Correction of error Restated balance

– 500,000

– 125,000

(100,000) 389,700

– –

Issue of ordinary shares Total comprehensive income for the year (Ws 1 and 2) Dividend on ordinary shares (600,000 x 0.20) Transfer to retained earnings (W2)

100,000 –

50,000 –

– 79,300

– 400,000





(120,000)







20,000

(20,000)

At 31 December 2012

600,000

175,000

369,000

380,000



Workings (1) Revised profit for the year Draft profit Less: Dividend on redeemable preference shares (200,000 x 3%) Depreciation on property (3,000,000 ÷ 20) Adj to depreciation re special plant (W3) Add: Error re opening inventory

Copyright © ICAEW 2013. All rights reserved.

£ 137,800 (6,000) (150,000) (2,500) 100,000 79,300

Page 13 of 14

Financial Accounting - Professional Stage – June 2013

(2) Revaluation and depreciation transfer Valuation on 1 January 2012 Carrying amount of property on 1 January 2012 (3,370,000 – 770,000) Annual transfer ÷ 20

£ 3,000,000 2,600,000 400,000 20,000

(3) Depreciation adjustment re special plant Depreciation charged in 2012 (30,000 x 25%) Depreciation on new basis (30,000 ÷ 3) Additional charge needed

£ 7,500 10,000 2,500

Most candidates made some kind of attempt at this question although full presentation marks were only gained by a minority of candidates. The majority of candidates correctly included the brought forward figures for ordinary share capital and share premium. The brought forward figure for retained earnings was sometimes adjusted by the correction of the error rather than showing this adjustment as a separate line on the statement of changes in equity itself. Where the correction of the error was made on the statement it was often added rather than deducted. Only a few candidates showed a restated balance after this adjustment had been made. The majority of candidates showed the correct entries for the share issue in the period, and for the dividend payment made. Candidates generally included the profit figure in the statement although only a minority correctly identified this as being part of “total comprehensive income”. The adjustments to profit were not generally well dealt with, with probably only around half of candidates making some of the adjustments. Of those candidates that did attempt to make adjustments to profit the most common errors were to make the adjustments in the wrong direction (ie added rather than deducted or vice versa). Some candidates made their adjustments on the face of the statement of changes in equity, instead of in a separate working. Only a small minority of candidates included the redeemable preference shares in the statement of changes in equity. The revaluation surplus arising in the year was generally calculated correctly, although this was almost always presented on a separate line to “total comprehensive income” usually being described as a revaluation, highlighting a lack of understanding in this area. Most candidates who arrived at the correct revaluation figure also arrived at the correct transfer between the revaluation surplus and retained earnings, reflecting the “excess” depreciation for the year. Only a minority of candidates correctly calculated the depreciation adjustment for the special plant and went on to adjust profit correctly for it. A number of candidates calculated one or other of the old and new depreciation figures but then often failed to make the resultant adjustment in the correct direction. Total possible marks Maximum full marks

Copyright © ICAEW 2013. All rights reserved.

10 9

Page 14 of 14

Financial Accounting and Reporting – Professional Level – September 2013 MARK PLAN AND EXAMINER’S COMMENTARY The mark plan set out below was that used to mark these questions. Markers are encouraged to use discretion and to award partial marks where a point was either not explained fully or made by implication. More marks are available than could be awarded for each requirement, where indicated. This allows credit to be given for a variety of valid points, which are made by candidates.

Question 1 Overall marks for this question can be analysed as follows:

Total: 31

General comments This question was a question testing the preparation of an income statement and statement of financial position from a trial balance. A number of adjustments were required to be made, including a downward revaluation, share issues, research and development expenditure, foreign exchange, the incorrect treatment of a lease and the clearing of a suspense account. Part b) asked for the UK GAAP differences in relation to the treatment of the revaluation model. Part c) featured the concepts requirement which asked about the two fundamental qualitative characteristics and how they are applied to intangible assets. Temera Ltd – Statement of financial position as at 31 March 2013 £ ASSETS Non-current assets Property, plant and equipment (350,000 + 384,000 (W6)) Intangible assets (63,250 + 6,540)(W3 & W4)

Current assets Inventories (W1) Trade and other receivables Cash and cash equivalents

734,000 69,790 803,790

40,400 17,800 6,900 65,100 868,890

Total assets Equity Ordinary share capital (W2) Revaluation surplus (100,000 – 50,000 (W6)) Retained earnings (W7) Equity Current liabilities Trade and other payables (30,450 – 200 (W7)) Taxation (56,000 + 11,500)

£

400,000 50,000 321,140 771,140

30,250 67,500 97,750

Total equity and liabilities

Copyright © ICAEW 2013. All rights reserved

868,890

Page 1 of 21

Financial Accounting and Reporting – Professional Level – September 2013

Temera Ltd – Income statement for the year ended 31 March 2013 Revenue Cost of sales (W1)

£ 912,500 (372,550)

Gross profit Administrative expenses (W1) Other operating costs (W1) Property impairment (W6)

539,950 (197,300) (92,310) (57,500)

Profit before tax Income tax

192,840 (67,500)

Profit for the period

125,340

W1 Expenses

Trial balance Opening inventories Closing inventories (36,200 + 4,200(W7)) R&D expenditure (W3) R&D amortisation (W3) Patent amortisation (W4) Profit on sale of patent Depreciation charge – property (W6) Exchange difference adj (W7) Building lease

Cost of sales

Admin expenses

£ 381,250 31,700 (40,400)

£ 181,300

46,000 5,750 2,760 (6,500) 16,000 (200) 10,000 372,550

Copyright © ICAEW 2013. All rights reserved

Other operating costs £ 34,500

197,300

92,310

Page 2 of 21

Financial Accounting and Reporting – Professional Level – September 2013 W2 Share capital & premium

Share capital £ 323,000 (3,000) 320,000 80,000 400,000

Trial balance Share issue adjustment (10,000 x 30p) Bonus issue (320,000 / 4) At 31 March 2013

Share premium £ 67,500 3,000 70,500 (70,500) –

W3 Research & development expenditure £ Trial balance Less amounts charged to profit & loss Prior to 1 July 2012 Staff training Promotional spend

£ 115,000

28,000 8,000 10,000 (46,000) 69,000 (5,750) 63,250

Intangible asset at 30 November 2012 Amortisation (69,000 / 4yrs x 4/12)

W4 Patents Cost – b/fwd Disposed in year

£

£ 15,000 (2,400) 12,600

Accumulated amortisation b/fwd

(4,500)

Amortisation charge for year On patents held all year (12,600 / 5yrs) On patent disposed of (2,400 / 5yrs x 6/12)

2,520 240 (2,760)

Acc. amortisation on patent disposed of (2,400 / 5yrs x 30/12) Carrying amount at 31 March 2013

1,200 6,540

W5 Suspense account Trial balance Disposed of patent (6,500 + (2,400 – 1,200)) Reverse lease liability Reverse lease finance charge Reverse lease payment At 31 March 2013

£ 67,300 7,700 (70,250) 5,250 (10,000) –

W6 Property, plant and equipment Land & buildings b/fwd Accumulated depreciation b/fwd Valuation at 1 April 2012 Revaluation Balance on revaluation surplus re buildings (100,000 / 2) Excess to profit or loss

Copyright © ICAEW 2013. All rights reserved

Buildings £ 630,000 (122,500) 507,500 400,000 (107,500) 50,000 57,500 –

Page 3 of 21

Financial Accounting and Reporting – Professional Level – September 2013

£ 400,000 (16,000) 384,000

Valuation at 1 April 2012 Depreciation charge for year (400,000 / 25yrs)

W7 Foreign exchange Translation at 1 January 2013 (5,000 x 0.84) Translation at 31 March 2013 (5,000 x 0.80)

£ 4,200 (4,000) 200

W8 Retained earnings Trial balance Bonus issue (80,000 – 70,500) (W2) Profit for the year

£ 205,300 (9,500) 125,340 321,140

Most candidates made a reasonable attempt at this part of the question, with the vast majority preparing a complete statement of financial position and income statement. Presentation was reasonable, although a significant number of candidates lost marks by failing to include appropriate sub-totals on their statement of financial position, which is a common omission in the Financial Accounting paper also. Depreciation on the revalued building was generally correctly calculated by most candidates as was the adjustment to closing inventory, although this wasn’t always correctly carried through to both the income statement and statement of financial position. The allocation of costs spent on research and development was generally attempted by most candidates, although a common error was to assume that the staff training could be capitalised. The number of months that amortisation on the capitalised development expenditure should have been charged seemed to cause a significant number of candidates a problem. Almost all candidates did use the recommended “costs matrix” when allocating costs between the three expense categories and, on the whole, it was possible to match figures on the face of the financial statements to workings. It was very noticeable that those candidates who did not use this format tended to produce disorganised workings (often split between the face of the income statement and /or a number of separate workings) which were difficult to follow and therefore might have lost marks. Far fewer candidates seemed capable of producing a clear working for intangible assets which resulted in them repeating calculations. A significant number of candidates would produce a working but then do nothing with the figures, or only carry them through to one side of the double entry, for example show in the cost matrix, but not include them in the asset carrying amount. Very many candidates dealt correctly with the share issues during the year. The most common error made was to credit share premium and debit share capital with the nominal value of the cash issue of shares, as opposed to the premium. Most candidates then reduced their share premium account to zero and charged the remainder of their (own figure) bonus issue to retained earnings and provided a clear working for retained earnings carried forward, tying in to their statement of financial position figure. Most candidates correctly calculated that the buildings had suffered a downwards revaluation of £107,500 (although a few omitted to take into account opening accumulated depreciation in their calculations). Many then correctly charged only £50,000 of that to the revaluation surplus (recognising that the other £50,000 in the revaluation surplus related to land), but others used the whole revaluation surplus. Almost all candidates then charged the balance to the income statement. Other common errors included the following. 

Incorrect calculation of the carrying amount of the patent by failing to calculate correctly, or omitting completely, the accumulated amortisation of the disposed asset for the 2.5 years held.



Failing to calculate the profit on sale of the patent, or when it was calculated either forgetting to include it in the statement of comprehensive income or including it in the wrong place (for example as part of retained earnings).

Copyright © ICAEW 2013. All rights reserved

Page 4 of 21

Financial Accounting and Reporting – Professional Level – September 2013



Failing to include the additional £11,500 tax charge within the statement of comprehensive income.



Continuing to treat the lease as a finance lease, instead of as an operating lease (even though in Question 3 almost all candidates recognised that where the length of the lease is significantly less than the useful life of an asset the lease will be an operating lease). It was therefore very common to see a finance charge of £5,250 on the face of the income statement and a lease liability on the statement of financial position.



Correctly calculating a foreign exchange gain of £200, but valuing closing inventory in relation to this at closing rate instead of at historic rate.

Despite the specific requirement to provide a working showing how the suspense account had been cleared, this appeared only rarely, with many candidates providing journal entries relating to this scattered throughout their answer, which were not asked for. Total possible marks Maximum full marks

26 23

(b) UK GAAP values PPE based on a current value model (existing use model), under IFRS fair value includes current market value which is often higher than EUV as it considers alternative uses. FRS 15 under UK GAAP specifies that the maximum period between valuations should be five years and interim valuations should be three year. IFRS does not specify a maximum period. UK GAAP requires any consumptions of economic benefit to be recognised straight in the income statement rather than against a previous revaluation surplus. It is not clear from the scenario whether the downward valuation of Temera Ltd’s land and buildings are caused by a consumption of economic benefits. However, if this is the conclusion then the full £107,500 would be recognised in profit or loss for the period. IFRS has no such restriction. This was a very straightforward requirement but very few candidates gained a reasonable mark in it. Answers to this were disappointing, indicating that many candidates had not committed these differences to memory. Those who had learnt these differences scored well easily picking up two or even the maximum of three marks. Worryingly, some candidates thought that impairments were never charged to the revaluation surplus under UK GAAP and a number of candidates had very confused ideas about when reserve transfers are allowed/required for revalued assets. Others thought that assets are not reviewed for impairment under UK GAAP. Some candidates also wasted time by discussing differences that did not specifically relate to revalued assets. Total possible marks Maximum full marks

Copyright © ICAEW 2013. All rights reserved

4 3

Page 5 of 21

Financial Accounting and Reporting – Professional Level – September 2013

(c) Qualitative characteristics The choice of the revaluation model as a measurement model in IAS 38 Intangible Assets provides relevant information by showing up to date values. This will assist users understanding of what the entity’s underlying assets are actually worth. However, to assist with comparability if the revaluation model is applied, all valuations must take place at the same time for a class of intangible assets. However, as not all intangible assets can be revalued because there is no active market for the asset it is not always possible to apply the revaluation model to the entire class of assets and therefore comparability may be compromised. Although the revaluation model provides more relevant information to users this information is generally seen as less faithful than the cost model. The cost model is based on historical costs which are not the most relevant costs on which to base future decisions. However, historic cost is based on fact and is therefore a faithful representation. The strict recognition criteria in IAS 38 sets out what can be included as part of the cost of an intangible asset and this aids verifiability of the final figure. IAS 38 contains rigid and robust rules for the capitalisation of intangible assets which means that financial statements of different companies can be compared as they are prepared on the same basis. IAS 38 also facilitates comparability between companies by requiring disclosure of accounting policies in respect of, for example, amortisation policy and measurement bases. It also requires the disclosure of both brought forward and carried forward figures aiding comparability between consecutive years. IAS 38 allows comparability between the cost and revaluation model, to ensure that companies financial information can be compared no matter which measurement basis is applied. This comparability is achieved by requiring equivalent cost information to be disclosed under the revaluation model. To improve understandability IAS 38 requires disclosures to be provided by each class of intangible asset. This provides information on what types of intangible assets have been purchased or sold during the year. The table format which is required by IAS 38 also assists users’ understandability by showing movements during the year. This part of the question was particularly badly answered by almost all candidates. Too many candidates simply wrote out what they knew, or had looked up in the Open Book text, about relevance and faithful representation without any link to the treatment of intangible assets. Others just wrote about how to account for intangible assets, with no link provided between that and the qualitative characteristics. The majority of candidates struggled to set out more than one or two relevant points and therefore very few candidates even gained half marks in this part of the question. Total possible marks Maximum full marks

Copyright © ICAEW 2013. All rights reserved

7 5

Page 6 of 21

Financial Accounting and Reporting – Professional Level – September 2013

Question 2 Overall marks for this question can be analysed as follows:

Total: 10

General comments This question required the preparation of a statement of cash flows from a draft statement. Movements in relation to property, plant and equipment involving a non-cash asset and impairment, both a share and bonus issue had been made, along with a dividend payment. (a) Radazul plc Statement of cash flows for the year ended 31 March 2013 £ Net cash from operating activities Cash flows from investing activities Purchase of property, plant and equipment (W2) Proceeds from disposal of property, plant and equipment Net cash used in investing activities Cash flows from financing activities Proceeds from issue of ordinary share capital (165,000 x 2) Dividends paid (W4) Net cash from financing activities

£ (11,935)

(333,615) 46,000 (287,615) 330,000 (39,750)

Net decrease in cash and cash equivalents Cash and cash equivalents at beginning of period Cash and cash equivalents at end of period

290,250 (9,300) (24,700) (34,000)

Workings (1) Cash from operations Draft cash generated from operations Equipment impairment (9,200 – 4,700) Adjust proceeds on disposal of machinery by carrying amount (to give profit) Adjustment for trade and other payables ( – 4,900 – 8,000) Cash generated from operations

£ (42,235) 4,500 38,700 (12,900) (11,935)

Deduct ½ for any incorrect bracket convention

Copyright © ICAEW 2013. All rights reserved

Page 7 of 21

Financial Accounting and Reporting – Professional Level – September 2013

(2) PPE B/d Additions – non-cash Additions (β)

£ 621,500 8,000 333,615

Disposal Impairment Depreciation C/d

£ 38,700 4,500 111,475 808,440 963,115

B/d Non-cash issue (β) Cash issue (165,000 x £2)

£ 120,000 57,000 330,000 507,000

963,115 (3) Share capital and premium £

C/d (342,000 + 165,000)

507,000 507,000

(4) Retained earnings Dividends paid (β) Non-cash issue (W3)

£ 39,750 57,000 96,750

Net movement

£ 96,750 96,750

There were some very good attempts at this question, with a significant number of candidates using the correct figures for proceeds from disposals of property, plant and equipment and proceeds from share issue, and correctly calculating cash paid for the purchase of property, plant and equipment and dividends paid (usually via T-accounts). Where marks were lost on the better scripts they were generally lost on presentation (with some candidates producing only extracts, not a full statement of cash flows) or on the calculation of net cash from operating activities. A few candidates made no attempt to calculate a correct figure for dividends paid and left the original figure in their statement of cash flows. The area that most candidates struggled with was the adjustments needed to cash from operations where, although most candidates did adjust for the impairment, far fewer dealt correctly with the disposal and the adjustments relating to the closing interest accrual (which many dealt with by showing interest paid on the face of the statement of cash flows). Many either ignored the adjustment relating to the plant purchased on credit or made the adjustment the wrong way around. A number of candidates lost easy marks by using abbreviations on the face of the statement of cash flows (most commonly PPE and NCI) and by using the wrong bracket convention particularly when dealing with outflows of cash. Some also showed the right figure for the shares issued for cash in their working but then failed to include this figure on the face of the statement. A number of candidates failed to use brackets for opening and closing cash and cash equivalents. Total possible marks Maximum full marks

Copyright © ICAEW 2013. All rights reserved

9 8

Page 8 of 21

Financial Accounting and Reporting – Professional Level – September 2013

(b) UK GAAP (FRS 1) has a narrower definition of “cash” compared with the IFRS term of “cash and cash equivalents”. Instead UK GAAP deals with “cash equivalents” as part of “management of liquid resources”. UK GAAP requires the statement of cash flows to be prepared under nine headings compared with the three under IFRS. UK GAAP is more restrictive on where certain items should be reported as compared with IFRS. For example, interest paid should be reported under “returns on investments and servicing of finance”. IFRS permits interest paid to be reported under any of the three main headings. Answers to this question on UK GAAP differences were much better than those to Part (b) of Question 1, indicating that candidates had learnt these differences. Most candidates knew that an IFRS statement of cash flows used only three headings, with UK GAAP having nine, and that there is more flexibility under IFRS. Hence many candidates scored a maximum of two marks. A worrying minority of candidates thought that “cash” and “cash equivalents” were simply different terms for the same thing – not that “cash” is a narrower definition. Total possible marks Maximum full marks

Copyright © ICAEW 2013. All rights reserved

3½ 2

Page 9 of 21

Financial Accounting and Reporting – Professional Level – September 2013

Question 3 Overall marks for this question can be analysed as follows:

Total: 27

General comments Part (a) of this question required candidates to explain the financial reporting treatment of four accounting issues, given in the scenario. The four issues covered a convertible bond, a jointly controlled entity, a lease incentive and a held for sale asset. Part (b) required candidates to revise two figures extracted from the draft consolidated financial statements for the adjustments needed as a result of their answer to Part (a). Part (c) required an explanation of any ethical issues arising from the scenario. Centellas plc (a) IFRS accounting treatment (1) Convertible bonds The convertible bonds are a compound financial instruments per IAS 32, Financial Instruments: Presentation and have both an equity and a liability component which should be presented separately at the time of issue. IAS 32 requires that the substance of the transaction be reflected, focusing on the economic reality that in effect two financial instruments have been issued. The liability component should be measured first at the present value of the capital and interest payments. The discount rate used should be the effective rate for an instrument with the same terms and conditions except for the ability to convert to shares.

31 March

Cash flow £ 180,000 180,000 3,180,000

2013 2014 2015 Liability component Equity component (Bal fig) Total

DF @ 9% 1/1.09 1/1.09² 1/1.09³

PV £ 165,137 151,502 2,455,543 2,772,182 227,818 3,000,000

The liability should be measured at £2,772,182 and the equity component should be calculated as the residual amount and measured at £227,818. The equity element will remain unchanged. The interest expense should be calculated at 9% of the liability component. 1 Apr 2012 £ 2,772,182

Interest (9%) £ 249,496

Payment (6%) £ (180,000)

31 Mar 2013 £ 2,841,678

An adjustment is required to increase the finance costs by £69,496 (249,496 – 180,000).

Copyright © ICAEW 2013. All rights reserved

Page 10 of 21

Financial Accounting and Reporting – Professional Level – September 2013

(2) Jointly controlled entity Centellas plc and Bermeja Ltd each own 50% of the share capital of Vidrio Ltd which indicates that the investment should be recognised as a jointly controlled entity and not as a subsidiary as Centellas plc does not have control. Both companies hold an equal number of shares and there is contractual agreement in place that they will make all the major operating and financial decisions concerning Vidrio Ltd jointly. IAS 31, Interest in Joint Ventures offers a choice of recognising the share in Vidrio Ltd using equity accounting or proportionate consolidation, however Centellas plc’s accounting policy choice is to apply the equity method. Under equity accounting Centellas plc will initially show its investment in Vidrio Ltd at cost of £100,000. This will then be adjusted each period by Centellas plc’s share of Vidrio Ltd’s retained earnings and reduced by any impairments. The investment in Vidrio Ltd is shown as a single line in both the consolidated statement of financial position and the consolidated income statement. As Vidrio Ltd was newly incorporated the cost of the investment was equal to the fair value and therefore no goodwill arises at acquisition. The investment in Vidrio Ltd should be shown at £220,000 ((200,000 + 240,000) x 50%) in the consolidated statement of financial position and income from joint venture should be £120,000 (240,000 x 50%). As it has currently been treated as a subsidiary 100% of profits will have been included so consolidated profit should be reduced by £120,000. (3) Lease incentive The new lease agreement runs for five years out of the building’s estimated 25-year life. Therefore, it can be assumed that this is an operating, rather than a finance lease. The initial rent-free period appears to constitute an incentive to enter into the agreement and therefore it should be accounted for under SIC 15, Operating Leases – Incentives. The required treatment of the rent-free period by Centellas plc, the lessee, is to recognise the aggregate benefit of the incentives as a reduction of rental expense over the lease term, on a straight-line basis. The total amount payable under the lease agreement of £57,000 ((£1,000 x 12 x 5yrs) – (£1,000 x 3)) should be spread evenly over the 5-year period: a charge of (£57,000/60 months) = £950 per month should be recognised. Therefore, 9 months x £950 = £8,550 should be recognised as an expense in the year ended 31 March 2013. The amount actually paid in the year was £6,000 (ie six payments of £1,000). Therefore, a payable for the difference, £2,550, should be recognised in Centellas plc’s current liabilities at 31 March 2013 and profit should be reduced by £2,550.

Copyright © ICAEW 2013. All rights reserved

Page 11 of 21

Financial Accounting and Reporting – Professional Level – September 2013 (4) Held for sale asset When the carrying amount of a non-current asset will be recovered principally through sale, rather than its continuing use, the asset must be classified as held for sale in accordance with IFRS 5, Non-Current Assets Held for Sale and Discontinued Operations. This reclassification generally occurs at the time the decision has been made to sell the asset. For an asset to be classified as held for sale it must meet detailed criteria:  

The asset must be available for immediate sale in its current condition; and The sale must be highly probable that it’ll take place.

For a sale to be highly probable:     

Management must be committed to a plan to sell the asset, management are advertising the sale and therefore it can be assumed that it is planning to sell the asset. There must be an active programme to find a buyer, here management are actively marketing the asset by advertising in national trade magazines. The asset must be marketed at a price that is reasonable based on its current fair value, here the advertised price is £62,500 which is reasonable since its fair value is £62,000. The sale should be expected to take place within one year of the date of classification. It is unlikely that there will be significant changes to the plan or that the plan will be withdrawn.

As Centellas plc appears to meet the above criteria, it would appear that the machine does meet the held for sale classification criteria. The machine should therefore be measured at the lower of its carrying amount, being £64,750 (W) and its fair value less costs to sell of £60,500 (£62,000 – £1,500). However, as the machine is measured under the valuation model it should be revalued at fair value under IAS 16, Property, Plant and Equipment immediately before its classification to held for sale. Hence, £2,750 (W) should be recognised against the revaluation surplus. Once classified as held for sale the costs to sell should then be recognised as part of profit or loss for the period, ie £1,500. Once the asset is held for sale it is no longer depreciated and should be shown separately in the statement of financial position. Working

Carrying amount at 31 Mar 2010

Cost/valuation £ 72,000

Revalued amount 31 Mar 2010

84,000

Acc dep (84,000/12yrs x 2yrs) 31 March 2012

(14,000) 70,000

Dep in yr (84,000/12yrs x 9/12)

(5,250) 64,750 (62,000)

Fair value

Revaluation surplus £

12,000

(2,750) 9,250

The answers to this part of the question were of a good standard with virtually all candidates including narrative explanations as well as the relevant calculations. Most candidates correctly identified the four underlying issues as a compound financial instrument, a jointly controlled entity, an operating lease and a held for sale asset, and were able to explain how they should be accounted for. Some marks (though not many) were lost on errors in the calculations but more were lost where candidates, after an initial explanation, then reduced their answer to a series of journal entries. Although there were specific marks allocated to key calculations and to the adjustments using those figures in Part (b), there were no marks for journal entries in lieu of narrative explanations. Marks are only ever awarded for journal entries where these are specifically required by the question.

Copyright © ICAEW 2013. All rights reserved

Page 12 of 21

Financial Accounting and Reporting – Professional Level – September 2013 The calculations for the convertible bonds, joint venture and operating lease were generally correct. With regard to the asset held for sale most candidates calculated the carrying amount correctly but very few realised that as the asset was carried at value a final revaluation should have been made prior to calculating the impairment. Almost all candidates calculated a single impairment down to fair value less costs to sell and charged that to the revaluation surplus. Other errors made included the following: 

Miscalculating the present value of the liability component of the convertible bond (a minority of candidates only).



Not recognising that there are two possible methods for accounting for a jointly controlled entity.



Not recognising the rent-free period on the operating lease as an incentive per SIC 15.



Miscalculating the amount already recognised as an expense on the operating lease as nine months at £1,000 per month, as opposed to six months at that amount (even where their answer referred earlier to the three month rent-free period).



Charging depreciation on the held for sale asset for the wrong number of months.



Not stating the net effect of any corrections to be made – so, for example, on the convertible bond, many calculated that a total finance charge of £249,496 was needed but failed to state that therefore an adjustment of £69,496 was required as £180,000 had already been recognised.

Total possible marks Maximum full marks

29 20

(b) Centellas plc Profit before tax

£ As stated (1) Equity element (1) Finance cost adjustment (2) Share in JV (3) Lease incentive (4) Depreciation (4) Costs to sell (4) Revaluation surplus TOTAL

– (69,496) – (2,550) (5,250) (1,500) –

£ 690,000

Equity (before NCI)

£ 1,260,400 227,818

(120,000)

(78,796) 491,204

(2,750) (78,796) 1,406,672

Most candidates did attempt to make the relevant adjustments although a minority just calculated a revised figure for profit and ignored equity. It appeared that most candidates had built up their answer to this part alongside their answers to Part (a), which is by far the most efficient approach. A number of errors were made because candidates failed to read the question to identify what entries had already been made to the draft financial statements. Total possible marks Maximum full marks

Copyright © ICAEW 2013. All rights reserved

4 3

Page 13 of 21

Financial Accounting and Reporting – Professional Level – September 2013

(c) Ethical issues The finance director, Anton Caro, qualified over 40 year ago, however as an ICAEW chartered accountant he is still bound by the ICAEW Code of Ethics. ICAEW chartered accountants must always abide by the spirit of the five fundamental ethical principles. One of these is professional competence and due care. Anton is obliged to maintain his professional knowledge and skills at an appropriate level; as the finance director of a listed company the appropriate level would be an in-depth knowledge of current financial reporting standards. The fact that Anton is due to retire next year is not an excuse not to maintain his professional knowledge and not act in accordance with the Code of Ethics – that fact is irrelevant and appropriate action should be taken by Anton to improve his level of technical knowledge. He should also improve his knowledge of his obligations in respect of the ethical standards. There are a number of errors in the draft financial statements, all of which reduce profit. This may be a coincidence but it seems unlikely that this is the case and instead Anton is under pressure to deliver a healthy profit, and has instead acted without integrity, as well as having a self-interest threat since his bonus is linked to the reported profit. You face a number of ethical issues, not least the question of whether the mistakes were deliberate or a lack of knowledge on Anton’s part. There is also a potential self-interest threat as the post of finance director has been mentioned if the right results are delivered. You should ignore the possibility of self-interest and discuss the adjustments with Anton and remind him of his professional responsibilities to ensure that accounting standards are correctly followed. Amendments must be made to the consolidated financial statements and if Anton refuses to make them, you should discuss the matter with the other board members. If Anton continues to try to dominate and exert influence on you to misstate the consolidated financial statements then it would be appropriate for you to consult the ICAEW ethical handbook and discuss the matter with the ICAEW confidential helpline. Most candidates picked up a good number of the available marks for this part, recognising the self-interest threat to Anton and to themselves (in the role of newly qualified ICAEW Chartered Accountant), the possible intimidation threat to themselves from Anton, and Anton’s possible lack of professional competence and due care. However, a number of candidates failed to recognise that they were acting within a company, as the financial accountant, and not as part of an audit team. It was therefore inappropriate to suggest referring the matter to the ethics partner or to discuss approaching the audit with increased professional scepticism. Most candidates did recognise the possible need to contact the ICAEW confidential helpline if they were unable to resolve the issues via discussion with Anton or with the other directors, but there was a tendency to be very quick to suggest that their own resignation might be the best solution. A worrying small minority of candidates thought the issue was one of money laundering. Total possible marks Maximum full marks

Copyright © ICAEW 2013. All rights reserved

8 4

Page 14 of 21

Financial Accounting and Reporting – Professional Level – September 2013

Question 4 Overall marks for this question can be analysed as follows:

Total: 14

General comments This question mixed several discrete topics. Revised extracts from the consolidated financial statements were required to be prepared following adjustments for the acquisition of a subsidiary, with contingent consideration, an associate and revenue recognition issues. The associate included trading with the parent company. Revenue adjustments were required for contract revenue and a sale and repurchase agreement. Part b) required candidates to distinguish between the single entity concept and the parent’s relationship with an associated company. (a) Gumar Ltd (i) Consolidated statement of financial position at 31 March 2013 (extract) Non-current assets Property, plant and equipment (987,500 + 210,000 + 350,000) Goodwill (27,800 + 54,500(W1)) Investment in associate (W3)

1,547,500 82,300 114,340

Current assets Inventories (62,900 – 510 (W5)) Trade and other receivables (161,300 + 28,200 + 20,000)

62,390 209,500

Non-current liabilities Borrowings (200,000 + 500,000) Contingent consideration (85,000 x 1.04) Contingent liability

700,000 88,400 32,000

Current liabilities Trade and other payable (75,000 + 12,200)

87,200

Workings (1) Goodwill £ Consideration: Cash Contingent consideration at fair value Non-controlling interest at fair value Less: Fair value of net assets (W2) Goodwill

Copyright © ICAEW 2013. All rights reserved

£

100,000 85,000 185,000 35,000 220,000 (165,500) 54,500

Page 15 of 21

Financial Accounting and Reporting – Professional Level – September 2013 (2) Net assets of Abrigo Ltd 31 March 2013

Share capital Retained earnings Goodwill adjustment Contingent liability

£ 165,000 96,000 (35,000) (32,000) 194,000

At acquisition £ 165,000 72,500 (40,000) (32,000) 165,500

Post acquisition £

28,500

(3) Investment in associate – Caleta Ltd Cost of investment Share of post acquisition retained earnings ((63,400 – 25,600) x 30%) Less: impairments to date

£ 105,000 11,340 (2,000) 114,340

(4) Share of profit of associate – Caleta Ltd £ 13,560 (510) (2,000) 11,050

Share of profit for the year (45,200 x 30%) Less: PURP (W5) Less: impairment in year (5) PURP SP Cost GP 1 X /2 Caleta Ltd – 1,700 x 30% = £510

% 100 (80) 20

£ 17,000 (13,600) 3,400 1,700

(a) (ii) Revised figure for consolidated profit Draft profit before tax Abrigo Ltd (W2) Unwinding of discount – contingent consideration (88,400 – 85,000) Share of associate (W4) Reverse profit Contract revenue

£ 589,200 28,500 (3,400) 11,050 (150,000) 20,000 495,350

Most candidates dealt well with the groups aspect of this question, at least as far as the “standard” workings were needed. Almost all candidates produced net assets and goodwill workings for the subsidiary acquired during the year and an investment in associate working (calculating the figure for the consolidated statement of financial position) for the associate. However, many then also felt the need to produce workings for retained earnings and for non-controlling interest when these were not needed. Conversely, many failed to calculate the share of profit of the associate for the consolidated income statement – a figure that was needed in order to revise consolidated profit. Common errors in these calculations included the following: 

Discounting the contingent consideration further even though it was stated at fair value in the question.



Calculating the non-controlling interest at share of net assets at acquisition, instead of at fair value per the group’s stated accounting policy.



Dealing incorrectly with the subsidiary’s own acquired goodwill in the net assets table (often getting the year end and acquisition figures the wrong way round), or failing to adjust for this at all.



Reducing the investment in associate figure by the provision for unrealised profit.



Not taking only the group share of the unrealised profit with the associate to the share of profit of the associate (or directly to the adjusted consolidated profit calculation).

Copyright © ICAEW 2013. All rights reserved

Page 16 of 21

Financial Accounting and Reporting – Professional Level – September 2013

Most candidates therefore included in their corrected extracts from the consolidated statement of financial position figures for goodwill (with most correctly excluding goodwill from the subsidiary’s own statement of financial position) and investment in associate. However, a significant number failed to add in the figures for property, plant and equipment, trade and other receivables and trade and other payables for the subsidiary acquired during the year, not appreciating that these figures had not been included in the draft consolidated statement of financial position. Some who did add these in failed to show their workings for this, so if calculation errors were made it was not possible to give credit for doing this. Others failed to reduce group inventories by the group share of the unrealised profit with the associate, even when this had been calculated. Others reduced inventories by 100% of this adjustment, as opposed to the group share. It was also rare to see a figure for contingent consideration taken to the consolidated statement of financial position, even where this had been included at fair value in the goodwill calculation. With regard to the other adjustments (Items (3) and (4)) most candidates wrote how they would account for these issues (as if this had been an “explain” type of question), but few actually put these adjustments through to their consolidated statement of financial position, although a few more did revise consolidated profit before tax for these. Where the requirement is to “prepare” or to “calculate” candidates need to be aware that no marks are available for narrative explanations. Most candidates attempted to revise consolidated profit before tax but approaches to this were sometimes haphazard. The most efficient approach was to complete this part of the question (Part (ii)) alongside Part (i). The most common “error” therefore was to fail to make a corresponding adjustment in Part (ii) for every item dealt with in Part (i). In particular, many candidates failed to adjust consolidated profit before tax for the subsidiary’s profit for the year, even though almost all candidates had calculated this in a net assets table. Total possible marks Maximum full marks

14 12

(b) An associated company is not part of a group, therefore the single entity concept that applies between a parent and its subsidiaries does not extend to associated companies. This is because the parent entity only has significant influence over an associated company rather than control. Transactions between group companies and an associated company are not cancelled on consolidation as an associated company is not part of the group. However any unrealised profit on these transactions should be eliminated. Answers to this part were very poor, with almost all candidates answering the question in relation to a parent entity and its subsidiary, instead of in relation to a parent entity and its associate. Such answers almost always scored zero marks. Others described how to account for an associate rather than dealing specifically with how to treat trading between a parent and its associate. Total possible marks Maximum full marks

Copyright © ICAEW 2013. All rights reserved

2 2

Page 17 of 21

Financial Accounting and Reporting – Professional Level – September 2013

Question 5 Overall marks for this question can be analysed as follows:

Total: 18

General Comments This question required the preparation of a consolidated income statement and extracts from the consolidated statement of changes in equity (for retained earnings and the non-controlling interest). The group had two subsidiaries, one of which was disposed of during the year. Fair value adjustments were required on acquisition of one of the companies. Inter-company trading took place during the year between the parent and subsidiary. Gaviota plc Consolidated income statement for the year ended 31 March 2013 Revenue (W1) Cost of sales (W1) Gross profit Operating expenses (W1) Profit from operations (W1) Investment income Profit before tax Income tax expense (W1) Profit for the period from continuing operations Profit for the period from discontinued operations (W2) Profit for the period Profit attributable to Owners of Gaviota plc (β) Non-controlling interest (W3)

£ 1,663,170 (594,850) 1,068,320 (248,950) 819,370 40,000 859,370 (260,220) 599,150 165,310 764,460

681,600 82,860 764,460

Consolidated statement of changes in equity for the year ended 31 March 2013 (extract)

Balance at 1 April 2012 (W5) Total comprehensive income for the year Eliminated on disposal of subsidiary (W2) Dividends (350,000 x 60p x 25%) Balance at 31 March 2013 (β)

Copyright © ICAEW 2013. All rights reserved

Noncontrolling interest £ 428,675 82,860 (214,160) (52,500) 244,875

Page 18 of 21

Financial Accounting and Reporting – Professional Level – September 2013 Workings (1) Consolidation schedule Gaviota plc £ 1,265,600

Socorro Ltd £ 482,570

Adj £ (85,000)

£ 1,663,170

Cost of sales – per Q – PURP (W6)

(538,900) (4,250)

(136,700)

85,000

(594,850)

Op expenses – per Q – FV deprec (160,000/40yrs)

(168,500)

(76,450) (4,000)

Revenue

Investment income – Socorro Ltd (350,000 x 60p x 75%) Tax

Consol

(248,950)

197,500

40,000 (157,500)

(192,800)

(67,420) 198,000

(260,220)

(2) Profit from discontinued operations (Ramblo Ltd) £ Sale proceeds Goodwill at acquisition Less: Impairments to date

£ 450,000

66,850 (20,000) (46,850)

Less: Carrying amount of net assets at disposal Net assets at 31 March 2013 Less: Profit since 1 Jan 2013 (111,200 x 3/12)

563,200 (27,800)

Add back: Attributable to non-controlling interest (535,400 x 40%) Profit on disposal Add: Profit for the year (111,200 x 9/12)

(535,400) 214,160 81,910 83,400 165,310

(3) Non-controlling interest in year £ 49,500 33,360 82,860

Socorro Ltd (25% x 198,000 (W1)) Ramblo Ltd (40% x 83,400 (W2)) (4) Socorro Ltd – Net assets

Share capital Retained earnings (W) FV adjustment FV – depreciation (4,000 x 4yrs) Total

(Proof only) 31 Mar 2013 £ 350,000 489,500 160,000 (20,000) 979,500

1 Apr 2012 £ 350,000 497,500 160,000 (16,000) 991,500

At acq £ 350,000 152,400 160,000 – 662,400

W (489,500 – 202,000) + (350,000 x 60p) = 497,500

Copyright © ICAEW 2013. All rights reserved

Page 19 of 21

Financial Accounting and Reporting – Professional Level – September 2013

(5) Non-controlling interest brought forward Socorro Ltd NCI at acquisition (662,400 (W4) x 25%) Post acquisition ((991,500 (W4) – 662,400) x 25%)

£ 165,600 82,275

£

247,875 Ramblo Ltd NCI at acquisition ((300,000 + 59,000) x 40%) Post acquisition ((263,200 – 111,200 – 59,000) x 40%)

143,600 37,200 180,800 428,675

(6) PURP SP Cost GP 1 X /4

% 125 (100) 25

£ 85,000 (68,000) 17,000 4,250

(7) Non-controlling interest carried forward (for proof only) Socorro Ltd NCI at acquisition Post acquisition ((979,500 (W4) – 662,400) x 25%)

165,600 79,275 244,875

Copyright © ICAEW 2013. All rights reserved

Page 20 of 21

Financial Accounting and Reporting – Professional Level – September 2013 Most candidates made a good attempt at preparing the consolidation schedule and correctly excluded the subsidiary sold during the year. Candidates who produced this schedule generally gained most of the straightforward marks for adjustments such as the inter group trading and the related PURP, with many candidates producing a completely correct consolidation schedule. Virtually all candidates also prepared a reasonable consolidated income statement gaining the relevant presentation marks. The most common loss of presentation marks was for abbreviating the non-controlling interest to “NCI” and/or including the profit from discontinued operations before tax or after the non-controlling interest. Very few candidates did their consolidation workings on the face of the income statement which was pleasing. As mentioned most candidates correctly calculated the PURP although occasionally this was deducted from revenue rather than added to cost of sales and sometimes it was included in the wrong column. A pleasing number of candidates also calculated the adjustment to depreciation correctly although slightly fewer made the correct adjustment to remove inter group dividends. Virtually all candidates attempted to calculate the profit on disposal although few managed to arrive at the correct figure. The most common errors here were deducting the total goodwill rather than the unimpaired goodwill, failing to adjust closing net assets for the profit made after the disposal (candidates frequently used the wrong number of months and/or added the profit rather than deducting it) and multiplying the profit up to disposal by the parent company’s percentage holding. As expected the extract to the consolidated statement of changes in equity was not as well dealt with and few candidates gained the relevant presentation mark. However most candidates did attempt to include at least some of the relevant figures for the disposal adjustment, the non-controlling interest’s share of profit and the dividend. However a number of candidates lost easy markings for lack of consistency ie by not taking the figures already calculated in the consolidated income statement and the disposal calculation. Only a minority of candidates calculated the brought forward non-controlling interest figure correctly and few produced clear, structured workings to support this figure. However, most candidates made some attempt to calculate either non-controlling interest brought forward or carried forward and earned some marks for this. Unfortunately, candidates too often produced messy and unstructured workings with little or no audit trail, which made awarding marks almost impossible. Few candidates seemed to realise that the easiest way to arrive at this figure was by calculating the net assets of the two subsidiaries at the start of the year. Even candidates who did attempt this rarely made the right adjustments to back out current year profit and dividends and recognise the fair value uplift and related impact on accumulated depreciation. Total possible marks Maximum full marks

Copyright © ICAEW 2013. All rights reserved

18½ 18

Page 21 of 21

Financial Accounting and Reporting - Professional Level – December 2013

MARK PLAN AND EXAMINER’S COMMENTARY The marking plan set out below was that used to mark this question. Markers were encouraged to use discretion and to award partial marks where a point was either not explained fully or made by implication. More marks were available than could be awarded for each requirement. This allowed credit to be given for a variety of valid points which were made by candidates.

Question 1 Overall marks for this question can be analysed as follows:

Total: 30

General comments Part (a) of this question required candidates to revise a draft income statement and statement of financial position for a number of adjustments. The amendments were in relation to the receipt of a government grant, a held for sale asset, the recoverability of receivables, irredeemable preference shares and dividend thereon, an adjusting subsequent event, an overprovision of income tax from the previous year, a provision for warranty costs and an accrual. Part (b) required an explanation of any ethical issues arising from the scenario and the action to be taken. Part (c) required candidates to identify and describe the elements of the financial statements which are relevant to the statement of financial position, with reference to the treatment of the irredeemable preference shares and the provision. Dedlock Ltd (a) Revised financial statements Statement of financial position as at 30 June 2013 £ ASSETS Non-current assets Property, plant and equipment (567,800 – (20,000 – 8,500)) Current assets Inventories Trade and other receivables (105,200 – 55,700 – 990 (W2)) Cash and cash equivalents Non-current asset held for sale

556,300

278,500 48,510 15,800 342,810 7,550 350,360 906,660

Total assets Equity Ordinary share capital Share premium Retained earnings (W1) Equity Non-current liabilities Preference share capital (irredeemable) Deferred income (W3)

£

200,000 75,000 394,506 669,506

100,000 6,400 106,400

Current liabilities Trade and other payables (82,200 + 5,300) Deferred income (W3) Provisions (W4) Taxation Total equity and liabilities

Copyright © ICAEW 2014. All rights reserved.

87,500 1,600 15,654 26,000 130,754 906,660

Page 1 of 17

Financial Accounting and Reporting - Professional Level – December 2013 Income statement for the year ended 30 June 2013 £ 2,863,325 (1,998,504)

Revenue (2,876,500 – 10,000 – 3,175) Cost of sales (W2) Gross profit Administrative expenses (W2) Other operating costs (W2)

864,821 (584,500) (241,990)

Operating profit Finance costs (200,000 x 50p x 5% x 6/12) Profit before tax Income tax (26,000 – 3,175) Profit for the year

38,331 (2,500) 35,831 (22,825) 13,006

Workings (1) Retained earnings Per draft Less Draft profit for the year Add Revised profit for the year Add back finance costs from SCE (already taken off as dividend)

£ 484,100 (105,100) 13,006 2,500 394,506

(2) Expenses Cost of sales

Per draft Government grant (W3) Loss on held for sale asset (11,500 – (8,000 – 450)) Bad debt written off Bad debt allowance ((105,200 – 55,700) x 2%) Warranty provision (W4) Accrual

£ 1,980,900 (2,000) 3,950

Other operating costs £ 185,300

Admin expenses £ 579,200

55,700 990 15,654 1,998,504

241,990

5,300 584,500

Note: Marks were awarded if items were included in different line items in the income statement provided that the heading used was appropriate. (3) Government grant Grant as received Taken to cost of sales y/e 30 June 2013 x 20% = At 30 June 2013 Within one year x 20% = After one year (β)

£ 10,000 (2,000) 8,000 (1,600) 6,400

(4) Warranty provision Number to repair or replace = 1,000 x 5% x ½ = 25 Repaired (25 x £190)/1.07 Replaced (600,000/1,000 = £600 x 25 x 80%)/1.07

Copyright © ICAEW 2014. All rights reserved.

£ 4,439 11,215 15,654

Page 2 of 17

Financial Accounting and Reporting - Professional Level – December 2013 Tutorial note Credit was also given if candidates stated that general provisions are not allowed in respect of receivables (IAS 39). Candidates generally performed well on this part of the question. Presentation of the two statements was generally of a sufficient standard to collect the presentation marks. Candidates should ensure they transfer their figures into final totals for individual line items in the financial statements. The majority of candidates identified that there was a non-current asset held for sale and that it should be separately analysed, although a small minority thought that it should still be considered to be a non-current asset. Of those candidates who correctly identified that it should be presented as part of current assets, only a minority presented it separately from current assets generally. A good number of candidates correctly calculated the relevant figure in both statements. The two adjustments to trade receivables for bad and doubtful debts were generally dealt with correctly, with almost all candidates correctly deducting the amount for the customer who had gone into liquidation before calculating the closing allowance. However, a worrying number of candidates presented the closing allowance as a liability in the statement of financial position, as opposed to netting it off trade and other receivables. Property, plant and equipment was stated correctly by a much smaller number of candidates, with various different adjustments being made to the draft figure. The classification and valuation of the preference shares proved a particular challenge. A number of candidates treated this as equity or as a hybrid financial instrument, split between non-current liabilities and equity. Some even treated this as equity but then went on in Part (b) of the question to state that it should be treated as a liability. The related finance costs also caused a significant number of candidates an issue, with only a minority getting the correct figure in the income statement and even less going on to add this figure back to the profit figure. Where the adjustment was made to retained profits it was more often than deducted, instead of being added. Deferred income in respect of grants of £8,000 was correctly calculated by the majority of candidates, although the split between current and non-current liabilities was often incorrect and sometimes the adjustment in the income statement was omitted or incorrectly added to expenses, instead of being deducted. The most common error was to reduce revenue by only the deferred part of the grant, instead of recognising that the whole grant needed to be removed from revenue and dealt with either as other income or offset to cost of sales. Weaker candidates tried to apply the netting off method to the grant and make depreciation adjustments for the asset. Although the income statement figure for taxation was usually correct, some candidates also showed this figure as the closing liability, ignoring the overprovision from the previous year. The warranty provision caused most candidates a problem. Where candidates did attempt a calculation the figure for the repaired element was usually correct. However, the figure for the replaced element was only calculated correctly by a minority of candidates (the most common error being not taking into account the profit margin on the goods under warranty when they were expected to be replaced). Even fewer candidates then went on to discount the total. A significant number of candidates went on to deduct their calculated warranty provision from revenue rather than showing it (separately) as a current liability. Total possible marks Maximum full marks

Copyright © ICAEW 2014. All rights reserved.

21½ 20

Page 3 of 17

Financial Accounting and Reporting - Professional Level – December 2013

(b) Ethical issues Richard has omitted to adjust for a number of issues, all of which could be said to have a negative effect on Dedlock Ltd’s financial statements for the year. The correct treatment of the overprovision of last year’s income tax charge reduces revenue, the reclassification of the irredeemable preference shares increases debt and all of the other adjustments reduce the profit for the year. Profit for the year before the adjustments was £105,100. However, after adjustments it has fallen by 80%, to £13,006. Richard is the finance director of the company, and these are all matters of which he was, or should have been, generally aware. This calls into question whether Richard has failed to make these adjustments as he is influenced by the fact that he may get a better price for his shares if the company’s profit is higher, and its debt lower. This is a self-interest threat and calls Richard’s integrity into question. Alternatively, if it is that Richard does not understand how to make these adjustments, or that these adjustments were necessary, then that calls his professional competence into question. ICAEW Chartered accountants have an obligation to maintain their continuing professional development and they should ensure that their technical knowledge and professional skills are kept up to date. Clara faces a number of ethical issues, not least the question of whether the mistakes were deliberate or a lack of knowledge on Richard’s part. Clara also faces a self-interest threat as she may be offered a permanent position at Dedlock Ltd if she “turns a blind eye” to Richard’s failings. Clara should ignore the possibility of self-interest and discuss the adjustments with Richard and remind him of his professional responsibilities to ensure that accounting standards are correctly followed. Amendments must be made to the financial statements and if Richard refuses to make them, Clara must discuss the matter with the managing director. If Richard continues to try to dominate and exert influence on Clara then it would be appropriate for Clara to consult the ICAEW ethical handbook and discuss the matter with the ICAEW confidential helpline. Almost all candidates made a reasonable attempt at this part of the question, with a good number obtaining full marks. Candidates should remember that to gain the most marks their answer should be tailored to the question scenario. Most candidates correctly identified that there was a self-interest threat for both Richard and Clara, explained how these threats arose and suggested appropriate courses of action. A minority of candidates answered as if Clara was an external auditor, as opposed to an independent consultant. A few felt there were money laundering issues at play. Total possible marks Maximum full marks

Copyright © ICAEW 2014. All rights reserved.

8½ 4

Page 4 of 17

Financial Accounting and Reporting - Professional Level – December 2013

(c) Elements of financial statements, irredeemable preference shares and warranty provision The three elements of financial statements relevant to the statement of financial position are assets, liabilities and equity. Irredeemable preference shares IAS 32 classifies financial instruments as financial assets, financial liabilities or equity. The irredeemable preference shares are an example of a (financial) liability. Although the irredeemable preference shares take the legal form of equity they are liabilities in substance as they include contractual obligations to transfer economic benefits to the holder (fixed preference (ie preferential) dividends). They arise from a past event (the issue of the shares Warranty provision A provision is a liability of uncertain timing or amount and should be recognised if there is a present obligation from a past event, it is probable that an outflow of economic benefits will be needed to settle the obligation and that a reliable estimate can be made of that amount. If one or more of these requirements are not met then a provision should not be recognised as it is not a liability. Probable means that it is more likely than not to occur or >50%. If it is not probable that an outflow of economic benefits will be needed to settle the obligation or the amount of the settlement cannot be measured reliably then it does not meet the definition of a liability and instead the amount may need to be disclosed as a contingent liability. In conclusion, Dedlock Ltd has a present obligation (its contractual obligation to repair or replace any faulty products under a two year warranty), as a result of past events (the sale of the goods). There is a probable outflow and a reliable estimate can be made (based on the number and amount of past claims under warranties). The estimation of the amount of the liability is made using expected values. Richard should therefore have recognised a provision as a liability exists. Answers to this part of the question were very mixed, with only a minority of candidates showing a good understanding of the elements of financial statements. Far too many candidates reproduced text from the open book, which was not required. A significant number of candidates instead discussed the qualitative characteristics of financial information, which gained no marks. Others did write about the elements of financial statements, but failed to relate these to the preference shares and warranty provision. Total possible marks Maximum full marks

Copyright © ICAEW 2014. All rights reserved.

8½ 6

Page 5 of 17

Financial Accounting and Reporting - Professional Level – December 2013

Question 2 Overall marks for this question can be analysed as follows:

Total: 17

General comments This question tested the preparation of a consolidated statement of cash flows and supporting reconciliation note, where a subsidiary had been disposed of during the year. Missing figures to be calculated included dividends paid (to the group and to the non-controlling interest), dividends received, tax paid, additions to property, plant and equipment, and proceeds from the issue of share capital. Chuzzlewit plc Consolidated statement of cash flows for the year ended 31 December 2012 £ Cash flows from operating activities Cash generated from operations (Note) 875,600 Interest paid (W2) (46,400) Income tax paid (W3) (157,400) Net cash from operating activities Cash flows from investing activities Purchase of property, plant and equipment (W4) (965,200) Proceeds from sale of property, plant and equipment 117,000 Dividends received from associate (W5) 104,700 Disposal of Gradgrind Ltd net of cash disposed of (W1) 335,050 Net cash used in investing activities Cash flows from financing activities Proceeds from share issues (W6) Repayment of long-term loan (300,000 – 250,000) Dividends paid (W8) Dividends paid to non-controlling interest (W9) Net cash used in financing activities Net increase in cash and cash equivalents Cash and cash equivalents at beginning of period Cash and cash equivalents at end of period

£

671,800

(408,450) 200,000 (50,000) (401,400) (1,950) (253,350) 10,000 31,500 41,500

Note: Reconciliation of profit before tax to cash generated from operations Profit before tax (635,700 + 82,300) Share of profits of associate Finance cost Profit on disposal of property, plant and equipment (117,000 – 102,000) Depreciation charge Impairment of goodwill (W7) Increase in inventories ((292,900 + 56,400) – 198,100) Increase in trade and other receivables (177,800 – (151,800 + 26,800)) Decrease in trade and other payables ((105,800 – 3,100) – (82,500 + 12,200 – 2,200)) Cash generated from operations

£ 718,000 (102,800) 45,500 (15,000) 351,600 40,500 (151,200) (800) (10,200) 875,600

Workings (1) Net cash inflow on disposal of Gradgrind Ltd

Net assets disposed of (388,500 x 70%) Add: Unimpaired goodwill (56,000 – 10,000) Profit on disposal Less: Cash and cash equivalents at disposal

Copyright © ICAEW 2014. All rights reserved.

£ 271,950 46,000 20,600 (3,500) 335,050

Page 6 of 17

Financial Accounting and Reporting - Professional Level – December 2013

(2) Interest paid Cash (β) C/d

£ 46,400 2,200 48,600

B/d CIS

£ 3,100 45,500 48,600

(3) Income tax Cash (β) C/d

£ 157,400 135,000 292,400

B/d CIS (128,000 + 4,400)

£ 160,000 132,400 292,400

(4) Property, plant and equipment B/d Additions (β)

£ 1,549,000 965,200

Disposal of sub Other disposals Depreciation charge C/d

2,514,200

£ 314,000 102,000 351,600 1,746,600 2,514,200

(5) Investment in associate B/d CIS

£ 287,800 102,800 390,600

Cash received (β) C/d

£ 104,700 285,900 390,600

(6) Share capital and premium £ B/d (300,000 + 40,000) Cash received (β) C/d (450,000 + 90,000)

540,000 540,000

£ 340,000 200,000 540,000

(7) Intangibles B/d

£ 289,500

Impairments (β) Disposal of sub (56,000 – 10,000) C/d

289,500

£ 40,500 46,000 203,000 289,500

(8) Retained earnings Dividends in SCE (β) C/d

£ 401,400 1,435,000 1,836,400

B/d CIS

£ 1,326,100 510,300 1,836,400

(9) Non-controlling interest Cash (β) Disposal (388,500 x 30%) C/d

Copyright © ICAEW 2014. All rights reserved.

1,950 116,550 279,200 397,700

B/d CIS

£ 301,800 95,900 397,700

Page 7 of 17

Financial Accounting and Reporting - Professional Level – December 2013

Most candidates made some attempt at this question, although performance overall was disappointing on what should have been a welcomed straightforward “processing” style question. The presentation of the statement was generally good with most candidates gaining the full presentation mark. Most candidates dealt reasonably well with those aspects of the statement of cash flows which would appear in a single entity statement; it was the consolidation issues which caused the most problems. For example, only a minority of candidates correctly added both the continuing and discontinued profit before tax figures in the reconciliation and correctly made the adjustments for the discontinued operation to the movement in inventories, trade receivable and trade payables. A good majority of candidates correctly calculated both the purchase cost and disposal proceeds for property, plant and equipment. Dividends received from the associate was also a figure which was commonly seen as both calculated correctly and presented in the correct place within the statement. The repayment of the loan was also commonly seen as correct, although significantly less candidates managed to correctly calculate the proceeds from the share issue, with the most common error being the omission of the movement on share premium. Most candidates made some adjustments to profit before taxation in the reconciliation. The most common errors were using the incorrect bracket convention (ie deducting instead of adding or vice versa), omitting the profit on disposal of property, plant and equipment or the impairment figure. Some candidates also made incorrect adjustments in the reconciliation by including items that were not required such as revaluations and the profit on disposal of the subsidiary. The dividend paid to the non-controlling interest was fairly well attempted although candidates occasionally included it in the incorrect section of the statement of cash flows or forgot about the adjustment required for the discontinued operation. The disposal proceeds for the discontinued operation was often missed from the statement, although where candidates did include it a reasonable attempt was made at the calculation, the most common error being to use the whole of the subsidiary’s net assets in the calculation instead of just the group share. Where a calculation was provided almost all candidates correctly deducted the cash balance on the discontinued operation. Total possible marks Maximum full marks

Copyright © ICAEW 2014. All rights reserved.

17½ 17

Page 8 of 17

Financial Accounting and Reporting - Professional Level – December 2013

Question 3 Overall marks for this question can be analysed as follows:

Total: 31

General comments Part (a) of this question required candidates to explain the financial reporting treatment of four accounting issues, given in the scenario. These covered a finance lease, borrowing costs in respect of a selfconstructed asset, a foreign exchange transaction and revaluations of property, plant and equipment (both upwards and downwards). Part (b) required an explanation of any UK GAAP differences in respect of the financial reporting treatment of the four issues. Nickleby plc (a) IFRS accounting treatment (1) Finance lease Under IAS 17, Leases, the machine will be classified as a finance lease as Nickleby plc is leasing the machine for the whole of its useful life and is responsible for the maintenance and insurance of the machine during that period. The machine is also specialised in nature which increases the likelihood of it being a finance lease. Therefore, per IAS 17, the risks and rewards of ownership are deemed to have passed to the lessee. On the basis of substance over form an asset will be recognised with a corresponding liability. The finance lease should have been capitalised at the lower of the fair value of £17,500 and the present value of the minimum lease payments and the lease liability set up. The present value of the minimum lease payments is: Present value calculation 1 July 2012 4,000 30 June 2013 4,000 / 1.15 2 30 June 2014 4,000 / 1.15 3 30 June 2015 4,000 / 1.15 4 30 June 2016 4,000 / 1.15 Present value of the minimum lease payments

£ 4,000 3,478 3,025 2,630 2,287 15,420

The present value of the minimum lease payments is the lower figure, so the journal entry should be: Dr: Non-current assets – cost Cr: Lease liability

£15,420 £15,420

The asset should then be depreciated over the shorter of its useful life and the lease term, ie its four year useful life giving a depreciation charge of £3,855 (15,420 ÷ 4), and a resultant carrying amount of £11,565. Dr: Income statement: Depreciation charge Cr: Non-current assets – accumulated depreciation

£3,855 £3,855

The lease liability should then have been reduced by payments made and increased by interest – spreading the total finance charge of £4,580 (20,000 – 15,420) over the period of the lease using the interest rate implicit in the lease of 15%. The table below illustrates the entries which should have been made. Year ended 30 June 2013 (15,420 – 4,000) 30 June 2014

Copyright © ICAEW 2014. All rights reserved.

B/f £

Interest @15% £

Payment £

C/f £

11,420 9,133

1,713 1,370

(4,000) (4,000)

9,133 6,503

Page 9 of 17

Financial Accounting and Reporting - Professional Level – December 2013 The lease liability at 30 June 2013 is therefore £6,503 non-current and £2,630 current (9,133 – 6,503). However, the £8,000 which should have been used to reduce the lease liability for 2013 has already been debited to the income statement. Only interest of £1,713 should have been charged. The correcting journal entry is: Dr: Lease liability (8,000 – 1,713) Dr: Income statement: Finance costs Cr: Income statement: Cost of sales

£ 6,287 1,713

£

8,000

(2) Borrowing costs IAS 23, Borrowing Costs, requires that borrowing costs that are directly attributable to the acquisition, construction or production of a qualifying asset form part of the cost of that asset. A qualifying asset is one that takes a substantial period of time to get ready for its intended use. The construction of the building is expected to take 12 months so would be a qualifying asset. Because the funds have been borrowed specifically for the construction then the borrowing costs are directly attributable. If surplus funds are invested the borrowing costs capitalised are to be reduced by the investment income received on the excess funds. Capitalisation commences when the entity incurs expenditure on the asset, is incurring borrowing costs and is undertaking activities to prepare the asset for use. All of these conditions are met. Borrowing costs can only be capitalised for the period of construction, of which six months fall into the current year. Therefore in the current year £7,100 ((£500,000 x 5% x 6/12) – 5,400) should be capitalised. To correct the entries made by the financial controller: £ £ Dr: Property, plant and equipment (asset in course of 7,100 construction) – cost Dr: Income statement: Other income 5,400 Cr: Income statement: Finance costs 12,500 As part of the cost of the asset, the borrowing costs will ultimately be depreciated over the asset’s estimated useful life, once depreciation commences. (3) Foreign exchange transaction IAS 21, The Effects of Changes in Foreign Exchange Rates, requires a foreign currency transaction to be recorded on initial recognition in the “functional currency” (ie that of the primary economic environment in which the entity operates – so here £) using the exchange rate at the date of the transaction. The financial controller should therefore have recorded the transaction at the delivery date of 10 June 2013, using a rate of €1: £0.82, as that is when the risks and rewards of ownership pass. Dr: Income statement: Purchases (€101,000 x 0.82) Cr: Trade payables

£82,820 £82,820

At the year end IAS 21 requires monetary items (units of currency held and assets and liabilities to be received or paid in a fixed or determinable number of units of currency) to be retranslated at the closing exchange rate. So, at the year end, the liability (ie trade payable) in respect of this transaction should be restated using the closing rate – ie to £75,750 (€101,000 x 0.75). A retranslation gain of £7,070 (82,820 – 75,750) has been made and should be recognised in profit or loss. The journal entry should be: Dr: Trade payables Cr: Income statement

Copyright © ICAEW 2014. All rights reserved.

£7,070 £7,070

Page 10 of 17

Financial Accounting and Reporting - Professional Level – December 2013

(4) Revaluations Nickleby plc uses the revaluation model per IAS 16, Property, Plant and Equipment, so the valuation on 1 July 2012 needs to be recognised. The increase in the revaluation surplus will be disclosed in other comprehensive income. Both the land and buildings increase in value from their previous carrying amounts (W) so journal entries are: £ £ Dr: Property, plant and equipment – land (1,000,000 – 800,000) 200,000 Dr: Property, plant and equipment – buildings* (2,500,000 – 1,906,000) 594,000 Cr: Revaluation surplus 794,000 The plant falls in value from a carrying amount on 1 July 2012 of £815,700 (W) to a valuation of £450,000 – a fall in value of £365,700. £150,400 of this decrease reverses a previous revaluation so that amount is charged to the revaluation surplus and disclosed in other comprehensive income. The remaining £215,300 (365,700 – 150,400) is recognised as an expense in profit or loss. The journal entry is: £ 150,400 215,300

Dr: Revaluation surplus Dr: Income statement: cost of sales Cr: Property, plant and equipment – plant and machinery

£

365,700

Nickleby plc also needs to recognise the depreciation charges for the year, based on the new valuations (see W). The journal entry is:

Dr: Income statement: administrative expenses Dr: Income statement: cost of sales Cr: Property, plant and equipment – buildings Cr: Property, plant and equipment – plant and machinery

£ 62,500 112,500

£

62,500 112,500

Final carrying amounts are £1,000,000 for the land, £2,437,500 for the buildings and £337,500 for plant and machinery (W). Nickleby plc has a policy of making an annual transfer between the revaluation surplus and retained earnings, so that needs to be made. The transfer is the difference between depreciation charges based on historic cost and those based on carrying amounts. However, this will only be in respect of the buildings as there is no longer any balance in the revaluation surplus in respect of plant and machinery. The journal entry is: Dr: Revaluation surplus (62,500 (W) – 21,500) Cr: Retained earnings

£41,000 £41,000

Working Valuation on 1 July 2012 £ Land Buildings Plant

1,000,000 2,500,000 450,000

Depreciation charge for year £ (÷ 40) 62,500 (÷ 4) 112,500

Carrying amount £ 1,000,000 2,437,500 337,500

*Tutorial note This would be Dr to Valuation and Cr to Accumulated depreciation but the split of the carrying amount was not given so this detail could not be provided. The opposite applies to plant and machinery.

Copyright © ICAEW 2014. All rights reserved.

Page 11 of 17

Financial Accounting and Reporting - Professional Level – December 2013 Generally this part of the question was well answered with the majority of candidates responding to all four issues and providing both explanations and calculations, although a minority of candidates failed to set out the numerical adjustments in the form of journals. Others gave a stream of journal entries with little narrative by way of explanation, and therefore limited the number of marks they could obtain. Journals were set out in a number of different ways, with some candidates setting out several simple journals, and others combining several transactions into a single journal. All of these were given credit where appropriate, but it is important to realise that if many transactions are combined an “audit trail” must be provided. In issue (4) a number of candidates combined all of the parts of the scenario into one journal, with only a single (net) credit to the revaluation surplus and/or to property, plant and equipment, with no supporting workings, which meant that partial marks could not always be awarded. Other candidates wasted time by setting out in journal entry form the entries which had already been made. Issue (1): Virtually all candidates identified this issue as a finance lease but very few calculated the present value of the minimum lease payments to determine the amount at which the initial asset and liability should have been recognised. Another extremely common error was a lack of consistency between the amount recognised as a liability and the amount initially recognised in the finance lease table (although most students did deduct the deposit from whatever figure they used in the leasing table). A further common inconsistency was making a statement that the amount capitalised should be the lower of the asset’s fair value and the present value of the minimum lease payments and then proceeding to capitalise the higher figure. Issue (2): The capitalisation of borrowing costs was also dealt with well with the majority of candidates recognising that interest earned needed to be deducted from the interest paid to arrive at the correct figure for capitalisation. Most candidates also identified the correct period for capitalisation as being six months only. Issue (3): The foreign exchange transaction was not as well dealt with. A surprising number of candidates stated that the liability should be recognised when the goods were ordered rather than when received (ie when the risks and rewards of ownership transferred) although most did re-translate the liability using the year-end rate. As commented on above, a significant number of candidates failed to deal separately with the initial recognition of the liability and its retranslation at the year end, producing a combined journal entry and thereby losing marks. Issue (4): The final issue relating to revaluations was also well dealt with, with most candidates clearly understanding the correct double entry for revaluations and the impact on subsequent depreciation. However, few candidates made the point that the valuations needed to be incorporated into the financial statements because the company had adopted the revaluation model. A pleasing number also showed the correct double entry for the reserves transfer even if the figure was not always correctly calculated. Most candidates also identified that the downwards revaluation for the plant and machinery needed to be split between the revaluation surplus and income statement. Other common errors not referred to above included the following:  Failing to adjust cost of sales for the full £8,000 incorrectly charged re the finance lease.  Drawing up the finance lease table in the wrong “order” ie treating the lease as if payments were in advance rather than in arrears.  Discounting the deposit paid.  Failing to explain what a qualifying asset is and therefore not relating the definition to the information given in the question ie that the building was expected to take 12 months to complete.  Dividing rather than multiplying when translating euros into sterling.  Failing to explain where the foreign currency should be recognised ie in the income statement.  Calculating the revaluation gain on the building incorrectly by reducing the opening carrying amount by current year depreciation.  Making the initial debit on recognition of the liability for the foreign exchange transaction to inventories instead of to purchases.  Calculating the reserves transfer incorrectly by dividing the revaluation surplus by remaining life (which does not work here as some of the surplus related to the land).  Suggesting a reserves transfer for the plant and machinery even though the balance in the revaluation surplus had been eliminated by the downwards revaluation.  Making comments which were relevant to the next financial year, rather than to this one (eg calculating a further foreign exchange loss/gain when the invoice was received after the year end). Total possible marks Maximum full marks

Copyright © ICAEW 2014. All rights reserved.

36 26

Page 12 of 17

Financial Accounting and Reporting - Professional Level – December 2013 (b) UK GAAP differences (1) Finance lease IAS 17 lists a number of factors which would indicate that the risks and rewards of ownership have been transferred to the lessee – indicating that the lease should be classified as a finance lease. However, under UK GAAP there is a rebuttable presumption that if, at the inception of the lease, the present value of the minimum lease payments is at least 90% of the asset’s fair value then there is a finance lease. (2) Borrowing costs IAS 23 requires attributable borrowing costs to be capitalised. UK GAAP (FRS 15) gives entities the choice of whether to capitalise borrowing costs or to expense them as incurred. Capitalisation under UK GAAP is limited to the finance costs incurred on the expenditure incurred. IAS 23 limits the amount capitalised to the borrowing costs on the total related funds less the investment income from any temporary investment of those funds. (4) Revaluations Where assets have been revalued UK GAAP (FRS 15) requires the use of existing use value rather than fair value UK GAAP requires impairment losses to be debited first against any revaluation surplus in respect of the asset unless it reflects a consumption of economic benefits. IAS 16 does not include such a limitation. So, under UK GAAP, the whole downwards revaluation would have been debited to the profit and loss account. Under UK GAAP a maximum period of five years between full valuations and interim valuations every three years is prescribed. No maximum period is specified by IAS 16 – the timing depends on changes in market values. Most candidates made a reasonable attempt at identifying the differences between IFRS and UK GAAP, showing that candidates realise that these differences will always be tested and that these are relatively easy marks to gain. A number of candidates wasted time by discussing differences that were not relevant to the scenario given. A minority of candidates appeared to simply “invent” differences. The two most common errors were believing that UK GAAP does not permit reserves transfers for revalued assets and that the “90% test” is a comparison between the length of the lease and the useful life of the asset. Candidates also need to be very careful to be precise with their wording in their answers to this type of question. For example, with regards to differences in the capitalisation of borrowing costs a number of candidates stated that under UK GAAP income on surplus funds “does not need to be netted off”, which is not the same as stating that it is not netted off. Total possible marks Maximum full marks

Copyright © ICAEW 2014. All rights reserved.

6 5

Page 13 of 17

Financial Accounting and Reporting - Professional Level – December 2013

Question 4 Overall marks for this question can be analysed as follows:

Total: 22

General comments Part (a) required the redrafting of a consolidated statement of financial position, where one subsidiary and one associate (both acquired during the year) had simply been added into the parent company’s figures, and no consolidation adjustments made. Adjustments included fair value adjustments on acquisition, intragroup sales (with inventory still held at the year end), intra-group balances which did not agree and impairment write-downs. In Part (b) candidates were required to explain and justify the fair value method and the proportionate method of calculating non-controlling interest, using calculations where appropriate. Cratchit plc (a) Consolidated statement of financial position as at 30 June 2013 £ Assets Non-current assets Property, plant and equipment (1,697,700 – 377,500) Intangibles (W3) Investment in associate (W7) Current assets Inventories (770,900 – 246,400 – 4,000 (W6)) Trade and other receivables (293,000 – 99,300 – 25,600) Cash and cash equivalents (23,800 – 800 + 6,900)

1,320,200 237,600 139,600 1,697,400 520,500 168,100 29,900 718,500 2,415,900

Total assets Equity and liabilities Equity attributable to owners of Cratchit plc Ordinary share capital (1,000,000 – 300,000 – 200,000) Shares not yet issued (W3) Revaluation surplus (400,000 – 150,000) Retained earnings (W5) Non-controlling interest (W4) Total equity Current liabilities Trade and other payables (315,200 – 97,400 – 18,700) Contingent liability Taxation (229,000 – 61,000) Total equity and liabilities

Copyright © ICAEW 2014. All rights reserved.

£

500,000 240,000 250,000 917,820 1,907,820 120,980 2,028,800 199,100 20,000 168,000 387,100 2,415,900

Page 14 of 17

Financial Accounting and Reporting - Professional Level – December 2013

Workings (1) Group structure

80 = 40% 200

Arusha plc 240

= 80%

300

Java Ltd

Mocha Ltd

(2) Net assets – Drummle Ltd

Share capital Revaluation surplus Retained earnings Per Q Goodwill re sole trader Contingent liability

Year end £ 300,000 150,000

Acquisition £ 300,000 150,000

224,900 (50,000) (20,000) 604,900

108,000 (60,000) (20,000) 478,000

Post acq £

126,900

(3) Goodwill – Drummle Ltd £ 640,000 95,600 (478,000) 257,600 (20,000) 237,600

Consideration transferred (400,000 + (200,000 x 1.20)) Non-controlling interest at acquisition (478,000 (W2) x 20%) Net assets at acquisition (W2) Impairments to date (4) Non-controlling interest – Drummle Ltd

£ 95,600 25,380 120,980

NCI at acquisition date (478,000 (W2) x 20%) Share of post-acquisition reserves (126,900 (W2) x 20%)

(5) Retained earnings £ 850,700 101,520 (10,400) (4,000) (20,000) 917,820

Cratchit plc (1,441,200 – 224,900 – 365,600) Drummle Ltd (126,900 (W2) x 80%) Gargery Ltd (W7) Less: PURP (W7) Less: Impairments to date (6) Inventory PURP – Drummle Ltd SP Cost GP x 40%

Copyright © ICAEW 2014. All rights reserved.

% 120 (100) 20

£ 60,000 (50,000) 10,000 4,000

Page 15 of 17

Financial Accounting and Reporting - Professional Level – December 2013

(7) Investments in associates – Gargery Ltd £ Cost Less Share of post acquisition decrease in net assets Share of post acquisition losses (22,500 x 40%) Add: Share of additional depreciation based on FV (35,000 ÷ 5 x 6/12 x 40%)

£ 150,000

9,000 1,400 (10,400) 139,600

This part of the question was reasonably well answered with most candidates producing an adequately presented consolidated statement of financial position (although a lack of sub-totals and the use of abbreviations were common). The vast majority of candidates did correctly identify the group structure and realised that the assets and liabilities incorrectly included for the associate needed to be “backed out” and the share capital corrected to be that of the parent company only. Where a draft consolidated statement is provided in the question it is extremely important that candidates read the information provided carefully to ascertain exactly on what basis the “consolidation” has been done. Most candidates calculated the consideration for the subsidiary acquired during the year correctly (using the correct share price) although hardly any then included the shares not yet issued in equity in their consolidated statement of financial position (hence failing to complete the double entry). Even those who did realise that this needed to be recognised in the statement of financial position often included it in liabilities. The other most common error was failing to show the contingent liability recognised as a fair value adjustment in liabilities. Other common errors included the following :  Failing to adjust for the cash in transit correctly by deducting the same figure from payables and receivables and/or deducting the amount (rather than adding it) to cash.  Not adjusting the net assets working for the goodwill held by the subsidiary (or only adjusting for the £10,000 change in value).  Not adjusting the net assets working for the contingent liability (or adding rather than deducting it).  Failing to multiply the PURP by the % of shares held in the associate.  Deducting the above PURP from the investment in the associate rather than from inventories.  Including the fair value excess in the investment in associate working.  Failing to multiply the increase in depreciation by the % held in the associate in the above working.  Not recognising that the same figures re post acquisition adjustments in the investment in associate should also be shown in consolidated retained earnings.  Calculating the non-controlling interest as a % of post-acquisition profits rather than as a % of closing net assets.  Deducting a share of the goodwill impairment in the NCI working even though the proportionate method was being used. As always some candidates lost marks by failing to show an “audit trail” for the basic consolidation on the face of the statement of financial position and/or for the calculation of the non-controlling interest and % of post- acquisition profits. Total possible marks Maximum full marks

Copyright © ICAEW 2014. All rights reserved.

17½ 17

Page 16 of 17

Financial Accounting and Reporting - Professional Level – December 2013

(b) The two methods of calculating goodwill and non-controlling interest IFRS 3 allows two methods of measuring the non-controlling interest (NCI) at the acquisition date: (i) At its fair value (the “fair value method”) (ii) At the NCI’s share of the acquiree’s net assets (the treatment used in (a), ie the “proportionate method”). Method (ii) results in goodwill being, in effect, the difference between the cost of the parent’s investment and its share of the net assets acquired. The rationale behind this is that this market transaction has only provided evidence of the amount of the parent entity’s goodwill – there has been no evidence of the amount of the goodwill attributable to the NCI. However, this method means that only the parent’s share (here 80%) of the goodwill of the subsidiary will be recognised – when for every other line item on a consolidated statement of financial position the parent brings in 100% of the subsidiary’s figures, to reflect the fact that the parent has control over that subsidiary. Method (i), the fair value method, is consistent with the rest of IFRS 3 since IFRS 3 requires both the consideration transferred and the net assets acquired to be measured at fair value. It works on the basis that the goodwill attributable to the NCI can be calculated from the estimate of the fair value of the NCI itself. The fair method usually results in a higher amount for the NCI/goodwill – the difference between this amount and the amount as traditionally measured is effectively added to the goodwill acquired in the business combination and is the goodwill attributable to the NCI at the acquisition date. If NCI had been measured in Part (a) using the fair value method it would have been calculated as follows, resulting in an NCI higher than that under the proportionate method: FV of NCI at acquisition Share of post-acquisition reserves (126,900 (W2) x 20%) Less: Impairment to date (20,000 x 20%)

£ 100,000 25,380 125,380 (4,000) 121,380

As shown above, where NCI has been measured at fair value and there is a subsequent impairment to goodwill, part of that impairment will be charged to the NCI at the end of the reporting period, based on the NCI%. If goodwill had been measured in Part (a) using the fair value method it would have been calculated as follows: £ Consideration transferred (a) 640,000 FV of NCI at acquisition 100,000 Net assets at acquisition (a) (478,000) 262,000 Less: Impairment to date (20,000) 242,000 This part of the question was poorly answered with a significant minority of candidates making no attempt to produce an answer. Those candidates who did attempt this part of the question focused on calculations, with very few showing any understanding of the conceptual issues relating to the two methods. Those who did attempt some narrative tended to describe the underlying mechanics of the calculations as opposed to the principles underlying them. Many candidates wasted time by copying out entire workings for goodwill and the non-controlling interest produced in Part (a) of their answer (for which there were no further marks available) rather than just referring back to the relevant figures and calculating the alternatives using the fair value method. A significant number of candidates clearly did not understand the full double entry for an impairment when using the fair value method and it was common to see just the parent company’s share of the impairment deducted from the carrying value of the goodwill. Total possible marks 8 Maximum full marks 5

Copyright © ICAEW 2014. All rights reserved.

Page 17 of 17

Professional Level - Financial Accounting and Reporting – March 2014 MARK PLAN AND EXAMINER’S COMMENTARY The mark plan set out below was that used to mark these questions. Markers are encouraged to use discretion and to award partial marks where a point was either not explained fully or made by implication. More marks are available than could be awarded for each requirement, where indicated. This allows credit to be given for a variety of valid points, which are made by candidates.

Question 1 Overall marks for this question can be analysed as follows :

Total: 30

General comments This question presented a draft set of financial statements with some adjustments. Candidates were required to prepare the amended statement of profit or loss, statement of financial position and the intangible assets table. A number of adjustments were required to be made, including depreciation, research and development expenditure, revenue adjustments, treasury shares and redeemable preference shares. Part b) required candidates to explain the purpose and objectives of IFRS 7 Financial Instruments; Disclosures. Part c) featured the concepts requirement which asked about the enhancing qualitative characteristics. Alloa Ltd – Statement of financial position as at 30 September 2013 £ ASSETS Non-current assets Property, plant and equipment (W5) Intangible assets (95,700 + 17,025)(note)

Current assets Inventories Trade and other receivables (215,000 + 7,200) Cash and cash equivalents

222,200 13,700 261,400 437,685

200,000 100,000 (26,250) 65,735 339,485

Non-current liabilities Redeemable preference shares Current liabilities Trade and other payables Taxation

63,560 112,725 176,285

25,500

Total assets Equity Ordinary share capital (185,000 + 15,000) Share premium (88,750 + (15,000 x 0.75)) Treasury shares (15,000 x £1.75) Retained earnings (W7) Equity

£

50,400

30,800 17,000 47,800

Total equity and liabilities

Copyright © ICAEW 2014. All rights reserved

437,685

Page 1 of 19

Professional Level - Financial Accounting and Reporting – March 2014

Alloa Ltd – Statement of profit or loss for the year ended 30 September 2013

Revenue (W2) Cost of sales (W1)

£ 890,000 (610,605)

Gross profit Operating expenses

279,395 (312,000)

Operating loss Investment income (71,200 + (48,000 x 15%) + 9,524 (W2)) Finance charges (W6) Profit before tax Income taxation (3,000 – 17,000)

(32,605) 87,924 (2,400) 52,919 (14,000)

Net profit for the period

38,919

Notes to the financial statements as at 30 September 2013 Intangible asset Development costs £ – 127,600 – 127,600

Patents

Amortisation At 1 October 2012 Charge for year (W3 & W4) Disposals At 30 September 2013

– 31,900 – 31,900

11,600 28,875 (1,500) 38,975

Carrying amount At 30 September 2012 At 30 September 2013

– 95,700

47,400 17,025

Cost At 1 October 2012 Additions Disposals At 30 September 2013

£ 59,000 (3,000) 56,000

W1 Expenses Cost of sales Trial balance Opening inventories Closing inventories R&D expenditure (W3) R&D amortisation (W3) Patent amortisation (W4) Disposed of patent (3,000 – (3,000 / 2yrs)) Depreciation charge – plant & machinery (W5)

422,590 23,600 (25,500) 100,400 31,900 28,875 1,500 27,240 610,605

W2 Revenue Trial balance Interest free credit (200,000 – (200,000/1.05)) At 30 September 2013

Copyright © ICAEW 2014. All rights reserved

£ 899,524 (9,524) 890,000

Page 2 of 19

Professional Level - Financial Accounting and Reporting – March 2014

W3 Research & development expenditure £ Trial balance Less amounts charged to profit & loss Staff training Research costs Development of the Brora

£ 228,000

5,900 26,000 68,500 (100,400) 127,600 (31,900) 95,700

Intangible asset at 30 September 2013 Amortisation (127,600 / 2yrs x 6/12)

W4 Patents £ Amortisation charge for year (59,000 – 3,000) / 2yrs Disposed of paten (3,000 / 2yrs x 7/12)

£

28,000 875 (28,875)

W5 Plant and equipment £ 90,800 (27,240) 63,560

Carrying amount at 1 Oct 2012 Depreciation charge for the year (90,800 x 30%)

W6 Redeemable preferences shares

30 Sept 2013

Opening balance £ 50,000

Interest exp (4.8%) £ 2,400

Interest paid (4%) £ (2,000)

Closing balance £ 50,400

W7 Retained earnings Per draft Less: draft profit and loss Add: revised profit and loss Add back preference dividend (50,000 x 4%)

£ 263,950 (239,134) 38,919 2,000 65,735

Presentation was generally good, although the presentation of the statement of profit or loss was almost always better than that of the statement of financial position where sub-totals were, as usual, often missing for one or more categories. Most candidates correctly showed the treasury shares as a “negative” balance under equity. Presentation of the intangible asset note was more varied with candidates often merging the patents and development costs into one column and/or netting off cost and amortisation. Only a very small minority of candidates failed to make any attempt at the note. The vast majority of candidates used a “costs matrix” to calculate the figure for cost of sales and, on the whole, it was possible to match figures on the face of the financial statements to workings. Almost all candidates correctly calculated the depreciation charge on property, plant and equipment and included this figure in cost of sales, and the carrying amount on the statement of financial position. Weaker candidates put the carrying amount both on the statement of financial position and added it to cost of sales. The adjustments for opening and closing inventories were generally dealt with correctly and pleasingly many candidates also calculated the tax charge correctly (although not all then went on to include the correct figure in current liabilities). Disappointingly very few candidates managed to calculate the discount on the deferred revenue correctly and even those who did very rarely then recognised the related financing income (even though this issue was almost identical to worked examples in the study manual). Most candidates included the correct figure for royalty income in the statement of profit or loss, but few completed the double entry by also adding this to trade and other receivables.

Copyright © ICAEW 2014. All rights reserved

Page 3 of 19

Professional Level - Financial Accounting and Reporting – March 2014

Surprisingly many candidates also struggled with the redeemable preference shares. Even those who wrote out the “table” working showing the correct interest expense and cash paid often then went on to put the wrong figures in the statement of profit or loss and/or the statement of financial position. However most did recognise that the transaction should be treated as a liability rather than equity. Strangely a number of candidates treated the shares as convertible debt and wasted significant time discounting the future payments to arrive at separate debt and equity elements. Rather disappointingly relatively few candidates calculated the correct figures for development costs and patents. Candidates were often “inconsistent” such as by including some costs twice (ie both capitalising and expensing them) or by calculating amortisation on a different figure to the one capitalised. Common errors in other areas included the following:        

Deducting the treasury shares elements from share capital and premium, instead of adding them and/or showing the treasury shares themselves as a credit balance, instead of a debit. Failing to reduce retained earnings by the draft profit for the year, having increased it by the profit for the year calculated in the revised statement of profit or loss. Failing to capitalise the correct elements of the research and development expenditure. Basing amortisation for the year on the capitalised development costs on one year instead of six months. Incorrectly calculating accumulated amortisation on the patent disposed of during the year (or failing to charge amortisation on that patent up to the point of disposal). Failing to adjust cost of sales for the proceeds on disposal of the patent or making the adjustment in the wrong direction. In the costs of sales matrix including either the amortisation on the patent or on the capitalised development costs, but not both. Capitalising the Brora development costs, even though the project had not yet met the IAS 38 criteria.

Total possible marks Maximum full marks

25½ 23

(b) IFRS 7 Financial Instruments: Disclosure, was published because the IASB felt that existing standards that covered financial instruments needed to be improved. Improvements were needed to ensure that the disclosure of information on financial instruments provided greater transparency of information so that users could better assess the risks that an entity was exposed to. The objective of IFRS 7 is to require entities to provide disclosures in their financial statements which enable users to evaluate both the significance of financial instruments for the entity’s financial position and performance, and the nature and extent of the risks arising from the financial instruments and how the entity manages those risks. Most candidates who made an effort with this requirement made a reasonable attempt by reciting the objectives of IFRS 7 from their open book text. Few candidates went beyond this. Total possible marks Maximum full marks

Copyright © ICAEW 2014. All rights reserved

3 2

Page 4 of 19

Professional Level - Financial Accounting and Reporting – March 2014

(c) Enhancing qualitative characteristics Usefulness There are four enhancing qualitative characteristics which enhance the usefulness of information that is relevant and faithfully represented. These are: comparability, verifiability, timeliness and understandability. Comparability ensures that users can identify and understand similarities in, and differences among, items. Information about a reporting entity is more useful if it can be compared from one reporting period to the next and with similar information from other entities. Comparability allows this. Consistency, although not an enhancing qualitative characteristic itself is related to comparability. This relates to the same methods being used to report the same item, so consistent accounting policies governed by accounting standards. The disclosure of accounting policies is therefore key to ensure that users can make a valid comparison between items. Verifiability helps assure users that information faithfully represents the information provided – it provides credibility to the financial information. It means that different knowledgeable and independent observers could reach consensus that a particular depiction is a faithful representation. Timeliness is equally important as information becomes less useful the longer the time delay in reporting it. Timeliness means that information is available to investors, lenders and other creditors in time for it to be used in their decision making processes. Finally, the characteristic of understandability means that information that may be difficult to understand is made more useful by presenting and explaining it as clearly as possible. Whilst financial information should be presented clearly and in an understandable manner, it is expected that users of t he financial statements have a reasonable level of knowledge and understanding. It would be misleading to exclude information simply because of its complex nature, as this would lead to incomplete information which would be misleading to users. There is a balance between timeliness and the provision of reliable information. For example, a provision has uncertainty involved in it, if an entity waits to report this information then it may have been settled and therefore the uncertainty over its amount will disappear. This information is therefore more reliable the longer an entity waits to report it. However, if such information is not reported until say six months after the year end then the information is less useful to users. As with Part (b), most candidates picked up some marks by using their open book text, correctly identifying the four enhancing qualitative characteristics and making a brief point about each. The depth of explanation was variable. Others wasted time by also discussing the primary qualitative characteristics or other concepts. Total possible marks Maximum full marks

Copyright © ICAEW 2014. All rights reserved

8½ 5

Page 5 of 19

Professional Level - Financial Accounting and Reporting – March 2014

Question 2 Overall marks for this question can be analysed as follows :

Total: 11

General comments This question required extracts from a consolidated statement of cash flows where a subsidiary had been disposed of during the year. Candidates were required to calculate the cost of additions to revalued property, plant and equipment, dividend payments by the parent and a subsidiary company and the proceeds from the issue of shares (following a bonus issue).

Limerigg plc Statement of cash flows for the year ended 30 September 2013 £ Cash generated from operations Cash flows from investing activities Purchase of property, plant and equipment (W1) Disposal of subsidiary (62,000 – 2,300)

(457,355) 59,700

Cash flows from financing activities Proceeds from issue of ordinary share capital (130,000 + 78,000) (W2 & W3) Non-controlling interest dividend (W5) Dividends paid (W4)

208,000 (43,300) (135,200)

£ 497,675

Working £ 396,675 101,000 497,675

Draft cash generated from operations (continuing & discontinued) Depreciation Cash generated from operations Property revaluation £ Carrying amount at 1 October 2012 (300,000 – ((300,000 / 30yrs) x 5yrs)) Revalued amount Revaluation surplus

Copyright © ICAEW 2014. All rights reserved

250,000 325,000 75,000

Page 6 of 19

Professional Level - Financial Accounting and Reporting – March 2014 Workings (1) PPE B/d Revaluation (W) Additions (β)

£ 506,950 75,000 457,355 1,039,305

Disposal of subsidiary Depreciation C/d

£ 76,900 101,000 861,405 1,039,305

(2) Share capital £

C/d

550,000 550,000

B/d Bonus issue (350,000 / 5) Cash issue (β)

£ 350,000 70,000 130,000 550,000

B/d Cash issue (β)

£ 35,000 78,000 113,000

(3) Share premium Bonus issue C/d

£ 35,000 78,000 113,000

(4) Retained earnings Dividends paid (β) Bonus issue (70,000 – 35,000)

C/d

£ 135,200 35,000

132,130 302,330

B/d Revaluation surplus – transfer (75,000 – 72,000) Profit or loss

£ 96,430 3,000

202,900 302,330

(5) Non-controlling interest Dividends paid (β) Disposal (77,850 x 30%) C/d

£ 43,300 23,355 73,845 140,500

B/d Profit or loss

£ 97,600 42,900 140,500

A number of candidates achieved full marks on this question and a pleasing number calculated the correct figures for the cash inflow from the disposal of the subsidiary, purchase of property, plant and equipment and for the dividend paid to the non-controlling interest. Most correctly adjusted cash generated from operations for the depreciation charge although many often also made other unnecessary adjustments. A significant number of candidates lost marks by failing to show brackets round figures which represent an outflow of cash. Candidates should be aware that this convention is just as important in a question which requires extracts from a statement of cash flows as it is for a complete statement of cash flows. Marks were also lost where items were shown under the incorrect headings – the most common error being to show dividends paid to the non-controlling interest as an investing activity instead of as a financing activity, and this was often also shown as a cash inflow instead of as an outflow. Some also prepared the T account workings correctly but then failed to transfer the final figure to the face of the statement of cash flows. Where errors were made they included the following:      

Omitting one or more of the entries from the property, plant and equipment T-account, most commonly the revaluation figure. Failing to adjust for the transfer between the revaluation surplus and retained earnings in the latter Taccount. Omitting the statement of profit or loss figure from the retained earnings and/ or non-controlling interest T-accounts. Debiting the whole bonus issue to the share premium account, when this should have been restricted to the opening balance on the share premium account, which was lower. Omitting the residual bonus issue from the retained earnings T-account. Failing to adjust the non-controlling interest figure for the disposal of the subsidiary.

Total possible marks Maximum full marks

Copyright © ICAEW 2014. All rights reserved

11 11

Page 7 of 19

Professional Level - Financial Accounting and Reporting – March 2014

Question 3 Overall marks for this question can be analysed as follows :

Total: 28

General comments Part (a) of this question required candidates to explain the financial reporting treatment of four accounting issues, given in the scenario. The four issues covered a government grant, an acquisition of a subsidiary, an asset impairment and a related party transaction. Part (b) required candidates to recalculate consolidated profit for the year for the adjustments needed as a result of their answer to Part (a). Part (c) required a calculation of basic earnings per share following a s hare issue for cash and a bonus issue. Part (d) required candidates to identify any UK GAAP differences for the issues set out in Part a). Melloch plc (a) IFRS accounting treatment (1) Government grant This is an income related grant and in this case it should be recognised over the two year period to match the expenditure for which it has been received to compensate. Even though the directors believe that the grant will not be repayable this is not a reason to recognise it fully upon receipt. As at 30 September 2013 Melloch plc has not satisfied all of the recognition criteria. £225,000 (£540,000 x 10/24) of the grant should be recognised as income in the current period. The remaining grant of £315,000 (£540,000 – £225,000) should be removed from profit or loss and recognised as a liability. The liability should be split between current £270,000 (540,000 x 12/24) and non-current £45,000 (£315,000 – £270,000). The grant should not be recognised as revenue. It could either be shown as “other income” in the statement of profit or loss or it could be netted off against the expenditure to which it relates (probably as part of “operating costs”). (2) Acquisition of Sheardale Ltd Sheardale Ltd should be recognised as a subsidiary of Melloch plc at 1 April 2013, as a controlling interest of 80% has been acquired. Sheardale Ltd should be consolidated in the group financial statements from this date. The consideration should be measured at its fair value of £480,000. The costs of £8,000 should not form part of the consideration but should instead be recognised directly in profit or loss. Intangible assets should be recognised if they are separable or they arise from legal or other contractual rights. These contractual rights should therefore have been recognised and form part of Sheardale Ltd’s net assets. The contractual rights should be recognised separately to the goodwill and amortised over their useful life of three years. The carrying amount of the contractual rights at 30 September 2013 is therefore £62,500 (£75,000 – £12,500) and £12,500 ((£75,000/3yrs) x 6/12) should be recognised in profit or loss as amortisation. As the intangible asset is held by Sheardale Ltd, the amortisation will affect the profit attributable to the non-controlling interest. It will therefore be split £10,000 and £2,500 between the profit attributable to the shareholders of Melloch plc and the non-controlling interest respectively. The non-controlling interest can be measured at fair value or proportion of net assets at the date of acquisition, however here the proportionate method should be used.

Copyright © ICAEW 2014. All rights reserved

Page 8 of 19

Professional Level - Financial Accounting and Reporting – March 2014

Goodwill should be measured at: Fair value of consideration Non-controlling interest – (650,000 x 20%) Net assets acquired (575,000 + 75,000) Goodwill – gain on bargain purchase

£ 480,000 130,000 610,000 (650,000) (40,000)

As a gain on bargain purchase has arisen Melloch plc will need to reassess the identification and measurement of the net assets and the measurement of the consideration, however in this case this is purely cash paid at the date of acquisition. Assuming these calculations are correct the gain of bargain purchase should be recognised as part of profit or loss for the period. Sheardale Ltd’s loss attributable to Melloch plc’s shareholders since acquisition should be recognised in the consolidated statement of profit or loss at £72,000 (£180,000 x 6/12 x 80%) and the non-controlling interest in the statement of profit or loss should be decreased by £18,000 (£90,000 x 20%). Consolidated net assets at 30 September 2013 will also decrease. (3) Impairment of research facility It appears that the research facility has suffered an impairment and therefore its carrying amount may be overstated. Assets should not be carried at more than their recoverable amount. Recoverable amount is the higher of value in use and fair value less costs to sell. The value in use at 30 September 2013 is £1,100,000 and fair value less costs to sell is £1, 245,000 (£1,250,000 – £5,000). The recoverable amount is therefore £1,245,000 and an impairment of £155,000 (£1,400,000 – £1,245,000) should be recognised. £100,000 should therefore be recognised against the balance of the revaluation surplus, to reduce this amount to zero. The remaining £55,000 should be recognised as part of profit and loss for the period. (4) Related party Melloch plc will need to establish whether or not the sale of the vehicle to the marketing director is a related party transaction under IAS 24 Related Party Disclosure. The marketing director is a member of the key management personnel of Melloch plc and therefore he is a related party under IAS 24. Therefore, the sale of the vehicle to the marketing director is a related party transaction. Even though the sale was at full fair value, it should be disclosed. Disclosure should include the nature of the related party relationship, ie one of the directors, and whether there are any outstanding balances at the year end, ie £17,500. If there are any special terms and conditions attached to the balance this should also be disclosed. A statement that the transaction took place on an arm’s length basis could only be made if it can be substantiated. Presumably here an external vehicle guide would show the fair value of the vehicle and assuming it to be in line with the price agreed such a statement could be made.

Answers to this part of the question were good. Most candidates correctly identified three out of the four underlying issues as a revenue grant, the acquisition of a subsidiary and the impairment of an item of property, plant and equipment. The related party transaction was less well dealt with, a significant number of candidates completely missing that this was a related party transaction at all.

Copyright © ICAEW 2014. All rights reserved

Page 9 of 19

Professional Level - Financial Accounting and Reporting – March 2014

Some marks (though not many) were lost on errors in the calculations but more were lost where candidates, after an initial explanation, then reduced their answer to a series of journal entries. Although there were specific marks allocated to key calculations and to the adjustments using those figures in Part (b), there were no marks for journal entries in lieu of narrative explanations. Marks are only ever awarded for journal entries where these are specifically required by the question. (1) Government grant Most candidates correctly described the conditions under which a grant can be fully recognised as revenue but then went on to correctly describe how the income should be deferred. The majority of candidates correctly calculated the amount which could be recognised in income in the current year (with only a minority using the wrong number of months) and correctly split the balance between non-current and current liabilities. (2) Acquisition of subsidiary Almost all candidates correctly recognised this as the acquisition of a subsidiary and that it should therefore be consolidated. Candidates then correctly went on to calculate goodwill, although not all arrived at a gain on bargain purchase (in which case marks were given for describing the correct accounting treatment of goodwill, both in this part and in Part (d)). The most common two errors in this calculation were including the associated costs of acquisition in the fair value of the consideration and/or failing to increase the net assets figure by the fair value of the contractual rights. A good number of candidates then arrived at the correct amortisation charge for the year on these rights, but less went on to split this charge between the parent and the non-controlling interest. Similarly, most recognised that the subsidiary’s loss for the year should be recognised in the consolidated statement of profit or loss, a few less correctly stated that only six -twelfths of this figure should be recognised, with fewer still splitting the resultant figure between the parent and the non-controlling interest. It was rare for candidates to make the point that the non-controlling interest could be measured using the fair value method or the proportionate method, and that the latter was the chosen method. Only a very small minority of candidates made the point, where a gain on bargain purchase had been calculated, that this should be reassessed. A significant number of candidates stated that the gain on bargain purchase should be immediately recognised in retained earnings, rather than making it clear that it should be immediately recognised in the consolidated statement of profit or loss. (3) Impairment of research facility There were some very good answers to this part. Almost all candidates correctly stated the “rules” for calculating the amount of an impairment and calculated the correct figures, setting the impairment firstly against the revaluation surplus for this asset. The most common error was to calculate the impairment as the difference between the carrying amount and the value in use, instead of the fair value less costs to sell (which was higher). A significant minority of candidates discussed the scenario as one of an asset held for sale (and then possibly dealt with the legal costs separately). (4) Related party transaction Answers to this issue were very disappointing with very many candidates not even recognising that the key issue here was the disclosure of a related party transaction. Of those who did identify that it was a related party transaction only a few explained why the director was considered to be a related party and what details needed to be disclosed. Fewer still made the point that, provided t hat fact could be substantiated, the arm’s length nature of the transaction could be disclosed. Frighteningly a very significant number of candidates appeared to believe that transactions should not be recognised until the cash had been received and therefore felt that the sale needed to be derecognised, so it was very common to see an adjustment for the profit on sale of £2,500 in Part (b). Total possible marks Maximum full marks

Copyright © ICAEW 2014. All rights reserved

26 17

Page 10 of 19

Professional Level - Financial Accounting and Reporting – March 2014 (b) Melloch plc Profit attributable to Melloch plc’s shareholders

As stated (1) Government grant (2) Acquisition of Sheardale Ltd: - acquisition costs - intangible amortisation - gain on bargain purchase - Share of Sheardale Ltd’s loss (3) Impairment Restated

£ 978,400 (315,000) (8,000) (10,000) 40,000 (72,000) (55,000) 558,400

It appeared that most candidates had built up their answer to this part alongside their answers to Part (a), which is by far the most efficient approach, with most candidates including all of the relevant adjustments that they had discussed in Part (a). The most common errors were to include the gain on bargain purchase as an expense and the (share) of the subsidiary’s loss as a profit or not at all.

Total possible marks Maximum full marks

3 3

(c) Melloch plc

1 Oct – 30 Nov 1 Dec – issue at MV 1 Dec – 31 Mar Bonus issue – 1 April (350,000 / 5) 1 Apr – 30 Sept

No. Of shares 280,000 70,000 350,000 70,000 420,000

Period in issue 2/12

Bonus factor 6/5

Weighted average 56,000

4/12

6/5

140,000

6/12



210,000 406,000

Basic EPS = 558,400 = £1.38 406,000 Many candidates scored full marks on this part. Those that made a poor attempt at this calculation clearly did not understand the impact of the bonus issue. Where errors were made they included the following:  

Using the wrong fractions for the parts of the year, or for the bonus issue, or both. Applying those fractions the 70,000 increments, instead of to the cumulative number of shares to date.

Total possible marks Maximum full marks

Copyright © ICAEW 2014. All rights reserved

4 3

Page 11 of 19

Professional Level - Financial Accounting and Reporting – March 2014

(d) UK GAAP differences Acquisition of subsidiary The contractual rights are treated differently under UK GAAP as they would not be recognised as these are not separable. Hence the intangible asset would be subsumed as part of the goodwill, rather than separately recognised as per IFRS. The £8,000 acquisition costs associated with the acquisition would be recognised as part of the consideration rather than expensed to profit and loss as per IFRS. There is no option to use fair value to measure the non-controlling interest as per IFRS, instead it would be measured as a proportion of net assets. Negative goodwill (a gain or bargain purchase) is recognised as a separate item within goodwill rather than recognised in profit or loss for the period as per IFRS. The negative goodwill should be split between the fair value of the non-monetary assets and that which is in excess of the fair value of these assets. This determines the period over which the negative goodwill should be recognised in profit and loss. Impairment Under UK GAAP an impairment on a revalued asset would normally be recognised against the balance on the revaluation surplus unless the impairment was as a result of a consumption of economic benefits. It is unlikely that the impairment of the research facility is a result of a consumption of economic benefits and therefore there would be no difference in treatment. Under IFRS there is no such requirement. Related parties Under UK GAAP FRS 8 requires the consideration of materiality to both sides of a related party transaction. IFRS requires no such consideration of materiality. Under FRS 8, the names of the related parties would need to be disclosed, there is no such requirement under IFRS.

Most candidates adopted the columnar approach recommended by the examining team at the recent tutor conference, giving both the IFRS and the UK GAAP treatments and giving only differences which were relevant to the issues in Part (a). It was also clear that more candidates had committed these differences to memory. Those who had learnt these differences scored well easily picking up three or more of the available five marks. The most common mistake was to state that under UK GAAP impairments can never be taken to the revaluation surplus.

Total possible marks Maximum full marks

Copyright © ICAEW 2014. All rights reserved

8½ 5

Page 12 of 19

Professional Level - Financial Accounting and Reporting – March 2014

Question 4 Overall marks for this question can be analysed as follows :

Total: 14

General comments This question was a mixed topic question, covering inventory valuation and a sale and operating leaseback. Part b) required a discussion around the ethical issues. Bainsford plc (i) Statement of financial position at 30 September 2013 (extract) Current assets (275,850 + 9,600(W1) – 3,000(W2))

282,450

Current liabilities (141,700 + 93,750 (W3))

235,450

(ii) Statement of profit or loss Draft profit after tax Increase in raw materials Decrease in finished goods Sale and leaseback adjustment ((375,000 – 93,750) – 250,000) Impairment loss

£ 497,300 9,600 (3,000) 31,250 (125,000) 410,150

Workings (1) Raw materials £ Weighted average (5,000 x £74) + (6,000 x £65) + (4,000 x £80) (5,000 + 6,000 + 4,000)) FIFO £80 x 1,200

x 1,200

(86,400) 96,000 9,600

(2) Finished goods Absorption rate (1.50 – 0.25 – 0.15) = £1.10 Adjustment (£1.50 – £1.10) x 7,500

£ 3,000

(3) Sale and operating leaseback Carrying amount Less: fair value Impairment loss Proceeds Less: fair value Profit Deferred income (375,000 x 3/12)

Copyright © ICAEW 2014. All rights reserved

£ 900,000 (775,000) 125,000 1,150,000 (775,000) 375,000 93,750

Page 13 of 19

Professional Level - Financial Accounting and Reporting – March 2014 Answers to this were quite mixed although most candidates calculated, as required, the three revised figures, and, on the whole, carefully followed their supporting calculations through to these figures. A good number arrived at the correct adjustment to both raw materials and finished goods, although typically whilst the adjustment relating to the raw materials was calculated correctly far fewer candidates could correctly identify which costs should be included in the value of finished goods . The most common errors were mistakes in calculating the weighted average cost of raw materials and failing to exclude the storage costs when calculating the absorption rate for finished goods. Attempts at adjusting for the sale and operating leaseback were very mixed, with many candidates writing at length about the appropriate accounting treatment, when only the calculations were required (no use of the word “explain” in the requirement). Although most candidates who made a reasonable attempt at these calculations did realise that the profit on disposal should be recognised over the lease term rather than recognised immediately few calculated it correctly by failing to account for the impairment first. Often the same figure was used to adjust liabilities and profit rather than recognising that the deferred amount should be added to liabilities and the proportion recognised up until the year-end added to profit.

Total possible marks Maximum full marks

10½ 9

(b) Nia’s concerns about the use of creative accounting may be justified as after the adjustments she made to the draft consolidated profit for the year, profit has fallen by 17.5%. While some of the adjustments may be attributable to Nia’s assistant’s lack of knowledge of accounting standards, the fact that the finance director was on hand to help may call into question the finance directors behaviour and whether the figures have been deliberately inflated. Nia should make the appropriate adjustments to the financial statements and explain to the finance director why profit has fallen. If her adjustments are challenged, she may need to seek advice on how to proceed. In the first instance Nia should speak to the other directors or the audit committee. Much will depend upon the finance director’s attitude and whether Nia is challenged in her adjustments. If Nia is still concerned about the issues not being dealt with correctly she may wish to contact the ICAEW advisory helpline. Nia’s other ethical problem relates, in part, to confidentiality. Confidentiality is one of the five fundamental principles set out in the ICAEW’s ethical Code. Nia is expressly required to respect the confidentiality of information required as a result of professional and business relationships. The information about the competitor, of which she is now aware because of a personal contact, could possibly be of benefit to Bainsford plc, and so Nia might be tempted to discuss this information with her employer as it may impact on their business and the opportunity to gain additional funding. Passing on such information may balance out any ill-feelings as a result of making the adjustments to reduce profit and would show her loyalty to her employer. However, professional accountants should be guided not only by the terms but also by the spirit of the ethical Code. Taking this approach, confidentiality should be maintained. Another of the five fundamental principles is professional behaviour. Professional accountants should avoid any action that discredits the profession. If Nia were to use the information for the benefit of her employers, and if this were subsequently to be made public, it is likely that this would appear discreditable to the profession.

Copyright © ICAEW 2014. All rights reserved

Page 14 of 19

Professional Level - Financial Accounting and Reporting – March 2014

As in previous sittings, many candidates framed their answer as if they were part of an audit team, not employed within industry. It was therefore inappropriate to suggest referring the matter to the ethics partner or to discuss approaching the audit with increased professional scepticism. With regard to the information from Sam, most candidates recognised the need to refer to the fundamental principle of confidentiality and knew that Nia should not repeat this information. Others thought that she should repeat it if it could be substantiated. Few referred to the fundamental principle of professional behaviour, which was also relevant. Almost all candidates did recognise the possible need to contact the ICAEW confidential helpline if they were unable to resolve the issues via discussion with the finance director, or with the other directors or the audit committee, but there was a tendency to be very quick to suggest that their own resignati on might be the best solution. Total possible marks Maximum full marks

Copyright © ICAEW 2014. All rights reserved

9 5

Page 15 of 19

Professional Level - Financial Accounting and Reporting – March 2014

Question 5 Overall marks for this question can be analysed as follows :

Total: 17

General comments This question required the preparation of a consolidated statement of profit or loss and extracts from the consolidated statement of changes in equity (for retained earnings and the non-controlling interest). The group had two subsidiaries, one of which was acquired during the year and a joint venture. Fair value adjustments were required on acquisition of one of the companies. Inter-company trading took place during the year between one of the subsidiary’s and the parent and the other subsidiary. Cambus plc (i) Consolidated statement of profit or loss for the year ended 30 September 2013 £ 2,017,550 (677,050) 1,340,500 (504,700) 835,800 12,850 848,650 (178,650) 670,000

Revenue (W1) Cost of sales (W1) Gross profit Operating expenses (W1) Profit from operations (W1) Share of profit of jointly controlled entity (W4) Profit before tax Income tax expense (W1) Profit for the period Profit attributable to Owners of Cambus plc (β) Non-controlling interest (W2)

613,050 56,950 670,000

(ii) Consolidated statement of changes in equity for the year ended 30 September 2013 (extract) Retained Nonearnings controlling £ interest £ Balance at 1 October 2012 (W6 & W5) 266,515 215,180 Total comprehensive income for the year 613,050 56,950 Added on acquisition of subsidiary (82,500 + 280,000) x 20% – 72,500 Dividends (500,000 x 50p) / (300,000 x 25p x 35%) (250,000) (26,250) Balance at 30 September 2013 (β)

Copyright © ICAEW 2014. All rights reserved

629,565

318,380

Page 16 of 19

Professional Level - Financial Accounting and Reporting – March 2014

Workings (1) Consolidation schedule Cambus plc £ Revenue 1,285,300

Ochill Ltd £ 579,000

Kennet Ltd (6/12) £ 216,250

Adj £ (63,000) 63,000

Cost of sales – per Q – PURP (W7)

(418,200)

(236,200) (3,000)

(82,650)

Op expenses – per Q – FV deprec (100,000/25yrs)

(267,500)

(172,000)

(61,200)

Investment income – Ochill (300,000 x 65% x 25p) Tax

Consol £ 2,017,550 (677,050)

(504,700)

(4,000) 48,750 –

(48,750)

(130,000)

(34,200) 129,600

(14,450) 57,950

(178,650)

(2) Non-controlling interest in year £ 45,360 11,590 56,950

Ochill Ltd (35% x 129,600 (W1)) Kennet Ltd (20% x 57,950 (W1))

(3) Ochill Ltd – Net assets

Share capital Retained earnings (W) PURP adj (W6) FV adjustment FV – depreciation (4,000 x 6 / 5yrs) Total

(Proof only) 30 Sept 2013 £ 300,000 296,400 (3,000) 100,000 (24,000) 669,400

1 Oct 2012 £ 300,000 234,800 100,000 (20,000) 614,800

At acquisition £ 300,000 153,700 100,000 – 553,700

W (296,400 – 136,600 + (300,000 x 25p) = 234,800 (4) Jointly controlled entity – Izat Ltd Share of profit for the year (44,625 x 40%) Less: Impairment

£ 17,850 (5,000) 12,850

(5) Non-controlling interest brought forward – Ochill Ltd At acquisition (553,700 (W3) x 35%) Share of post-acquisition profits ((614,800 – 553,700) x 35%)

£ 193,795 21,385 215,180

(6) Retained earnings brought forward Cambus plc (461,200 – 518,350) Add back dividend (500,000 x 50p) Izat Ltd – post acquisition ((225,500 – 44,625 – 96,000) x 40%) Ochill Ltd – post acquisition ((614,800 – 553,700) x 65%) (W3)

Copyright © ICAEW 2014. All rights reserved

£ (57,150) 250,000 33,950 39,715 266,515

Page 17 of 19

Professional Level - Financial Accounting and Reporting – March 2014

(7) PURP SP Cost GP 1 X /2

% 120 (100) 20

£ 36,000 (30,000) 6,000 3,000

(8) Non-controlling interest carried forward (for proof only) Ochil Ltd At acquisition ((300,000 + 153,700 + 100,000) x 35%) Share of post-acquisition profits ((669,400 – 553,700) x 35%)

193,795 40,495 234,290

Kennet Ltd At acquisition ((280,000 + 82,500) x 20%) Share of post-acquisition profits ((140,450 – 82,500) x 20%)

72,500 11,590 84,090 318,380

(9) Retained earnings carried forward (for proof only) Cambus plc Izat Ltd – post acquisition (225,500 – 96,000) x 40% Less: impairment – Izat Ltd Ochill Ltd - post acquisition ((669,400 – 553,700) x 65%) (W3) Kennet Ltd – post acquisition (140,450 – 82,500) x 80%) (W4)

£ 461,200 51,800 (5,000) 75,205 46,360 629,565

Most candidates produced a well laid out consolidated statement of profit or loss, and showed the split between the profit attributable to the parent and to the non-controlling interest. This was backed up, on the whole, by a well laid out consolidation schedule. Attempts at the consolidated statement of changes in equity were generally less good, both in presentation and in content. Many candidates produced a completely correct consolidation schedule, with figures for the provision for unrealised profit and the additional depreciation, in the appropriate columns. The vast majority of candidates correctly took only six-twelfths of the subsidiary’s figures to their consolidation schedule. The most common omission was not to calculate the parent’s share of the dividend from the subsidiary held throughout the year and realise that it made up the whole of the parent’s investment income and that therefore the two figures should be cancelled out. Other common errors were to include a provision for unrealised profit even where the goods had been sold on to third parties and adjusting the parent’s costs (rather than the subsidiary’s) for the additional depreciation arising from the fair value adjustment. The figure for share of profit of jointly controlled entity was more often than not correctly calculated, with the most common error being to omit the impairment. A minority of candidates attempted to calculate some sort of statement of financial position figure, which they then reduced by the impairment or describe the figure as “share of associate” on the face of the consolidated statement of profit or loss. Unfortunately answers to the second part of the question relating to the consolidated statement of changes in equity extract were far weaker. Although most candidates did enter the relevant figures from the consolidated statement of profit or loss many went no further than this. A significant number of candidates correctly calculated the dividend paid by the subsidiary acquired during the year to the non-controlling interest, with the figure omitted more often than errors were made.

Copyright © ICAEW 2014. All rights reserved

Page 18 of 19

Professional Level - Financial Accounting and Reporting – March 2014

A figure for the non-controlling interest added on acquisition of the subsidiary was not seen very often, but where it was included it was more often than not the correct figure. Only some candidates made some attempt to calculate either non-controlling interest and retained earnings brought forward or carried forward and earned some marks for this, but these figures were rarely completely correct, although candidates did pick up some marks, most commonly for an attempt at a net assets table which they used to arrive at pos t acquisition earnings. No marks were given for a group structure diagram, since the percentage holdings were given in the question, although many candidates did produce such a diagram. Some candidates were, however, careless in their use of these percentages, the most common error being to use the parent’s percentages in calculating the non-controlling interest.

Total possible marks Maximum full marks

Copyright © ICAEW 2014. All rights reserved

19½ 17

Page 19 of 19

Financial Accounting and Reporting – Professional Level – June 2014

MARK PLAN AND EXAMINER’S COMMENTARY The marking plan set out below was that used to mark this question. Markers were encouraged to use discretion and to award partial marks where a point was either not explained fully or made by implication. More marks were available than could be awarded for each requirement. This allowed credit to be given for a variety of valid points which were made by candidates.

Question 1 Total Marks: 31 General comments Part (a) of this question tested the preparation of a statement of profit or loss, a statement of financial position and a statement of changes in equity from a trial balance plus a number of adjustments. Adjustments included the revaluation of property, plant and equipment (with a transfer between the revaluation surplus and retained earnings), the receipt of a government grant, share issues and dividends, a foreign exchange transaction and a prior period adjustment. Part (b) required an explanation and quantification of the alternative treatment of the government grant. Part (c) tested the information needs of users in the context of property, plant and equipment. Tipperary plc (a) Financial statements Statement of profit or loss for the year ended 31 December 2013 Revenue (5,709,600 – 18,000 – 9,200 (SCE)) Cost of sales (W1) Gross profit Distribution costs (W1) Administrative expenses (W1) Profit from operations Finance cost (100,000 x 5%) Profit before tax Income tax expense Profit for the year

£ 5,682,400 (3,976,300) 1,706,100 (562,700) (1,097,000) 46,400 (5,000) 41,400 (10,500) 30,900

Statement of financial position as at 31 December 2013 £ Assets Non-current assets Property, plant and equipment (535,000 (W2) – 21,250 – 17,500 (W1)) Current assets Inventories 192,300 Trade and other receivables 363,750 Total assets

Copyright © ICAEW 2014. All rights reserved

£

496,250

556,050 1,052,300

Page 1 of 16

Financial Accounting and Reporting – Professional Level – June 2014

Equity and liabilities Equity Ordinary share capital Share premium Revaluation surplus Retained earnings

£

£ 230,000 9,200 222,500 176,450 638,150

Non-current liabilities Preference share capital (5% redeemable) Deferred income (13,500 – 4,500) (W3)

100,000 9,000 109,000

Current liabilities Bank overdraft Trade and other payables (233,050 + 5,000 – 5,750 (W4)) Deferred income (W3) Taxation

57,850 232,300 4,500 10,500 305,150 1,052,300

Total equity and liabilities Statement of changes in equity for the year ended 31 December 2013

At 1 January 2013 Prior period error Restated balance Rights issue (230,000 ÷ 5) (x 20p) Total comprehensive income for the year (30,000 – 9,000 (W2)) Transfer to retained earnings (17,500 (W1) – 10,000) Ordinary dividend (184,000 x 10p) At 31 December 2013

Ordinary share capital £ 184,000 184,000 46,000 -

Share premium

Revaluation surplus

£

£ 209,000 209,000 21,000

9,200 -

-

-

230,000

9,200

(7,500) 222,500

Retained earnings £ 256,450 (100,000) 156,450 30,900 7,500 (18,400) 176,450

Workings (1) Costs matrix

Per TB Downwards revaluation (W2) Depreciation (85,000/4) ((450,000 – 100,000)/20) Opening inventories (278,000 – 100,000) Closing inventories Ordinary dividend Release of government grant (W3) Exchange gain (W4)

Cost of sales £ 3,968,600 11,000 21,250 178,000 (192,300)

Distrib costs £ 562,700

Admin expenses £ 1,097,900 17,500

(18,400) (4,500) (5,750) 3,976,300

562,700

1,097,000

Note: Marks were awarded if items were included in different line items than the above provided that the heading used was appropriate.

Copyright © ICAEW 2014. All rights reserved

Page 2 of 16

Financial Accounting and Reporting – Professional Level – June 2014 (2) PPE and revaluation reserve

Carrying amount at 1 January 2013 Valuation on 1 January 2013 Revaluation upwards/(downwards) Revaluation surplus at 1 January 2013 To statement of profit or loss

Land and buildings £ 420,000 450,000 30,000

Plant and equipment £ 105,000 85,000 (20,000) 9,000 (11,000)

£ 535,000 209,000

(3) Government grant Grant received Less: Released in year (18,000/4) Deferred income

18,000 (4,500) 13,500

(4) Forex transaction Translation on 15 November 2013 (€115,000 x 0.90) Translation on 31 December 2013 (€115,000 x 0.85) Exchange gain

103,500 97,750 5,750

Candidates generally performed well on this part of the question. Presentation of the three statements was usually of a sufficient standard to collect the available presentation marks with the presentation and indeed completion of the statement of changes in equity (which candidates often find more challenging) of a pleasing standard. Many candidates, however, failed to take their closing balances from this statement to the equity section of their statement of financial position, thereby letting the statement of changes in equity act as a working for those figures, and instead wasted time by producing other workings for these figures. On occasion, the figures in these additional workings and those in the statement of changes in equity were different. Only a minority of candidates failed to produce a statement of changes in equity. The statement of profit or loss was generally well prepared and completed by the majority of candidates. It was pleasing to see that the majority of candidates also prepared a cost matrix working. By preparing this standard working candidates maximise the number of marks they will be awarded. Haphazard cost workings, or brackets on the face of the statement of profit or loss (which were used by a minority of candidates) often lost marks through missing narrative and no audit trail. The majority of candidates correctly reduced opening inventory for the overvaluation in the cost matrix however a minority instead adjusted closing inventory, which whilst having the same effect in the cost matrix meant that closing inventory in the statement of financial position was incorrect. The overvaluation was reflected in the statement of changes in equity by a significant number of candidates although considerably less showed it in the correct place and then showed a sub-total with a restated balance. Others made the adjustment in the statement of changes in equity but then failed to reduce opening inventories in the cost matrix. The government grant was generally dealt with correctly by the majority of candidates in the statement of financial position, although less showed the correct figure as an adjustment to revenue and/or the release of the grant in the year as a deduction from expenses (or as operating income). Similarly the exchange gain was correctly calculated by almost all candidates and a significant number correctly reduced trade and other payables, but again less went on to reflect the adjustment correctly in the statement of profit or loss. The property, plant and equipment working caused problems for a number of candidates, who omitted to revise the balances in the trial balance for the revaluation. Many candidates, though not all, realised that the downwards revaluation on the plant and equipment needed to be split between the statement of profit or loss and the revaluation surplus (although having recognised this not all of these candidates then followed this through to their cost matrix and the statement of changes in equity). Fewer still realised that, since the latter transfer had wiped out that part of the revaluation surplus which related to plant and equipment, the only transfer that could be made for the additional depreciation was that arising from the revaluation of the buildings.

Copyright © ICAEW 2014. All rights reserved

Page 3 of 16

Financial Accounting and Reporting – Professional Level – June 2014 Other common errors included the following:  Showing the bank balance (which was an overdraft as shown by its inclusion on the credit side of the trial balance in the question) in current assets instead of in current liabilities.  Not showing the net revaluation in the year and the profit for the year on a single line in the statement of changes in equity, described as “total comprehensive income”.  Showing £230,000 as the opening balance on ordinary share capital in the statement of changes in equity, as opposed to the closing balance (and then working backwards from that to adjust for the rights issue).  Failing to split the deferred grant between current and non-current liabilities.  Being careless with the bracket convention in the cost matrix, for example showing closing inventory or the foreign currency gain as increases rather than decreases in costs.  Depreciating the land as well as the buildings. Total possible marks Maximum full marks

24½ 22

(b) Alternative treatment of the government grant The alternative method per IAS 20, Accounting for Government Grants is the netting-off approach. The netting-off approach requires the grant to be deducted in arriving at the carrying amount of the asset. Under the netting-off approach the grant of £18,000 would have been credited to the cost of plant and machinery, giving an initial carrying amount of £12,000 (30,000 – 18,000), compared to an initial carrying amount in Part (a) of £30,000. Depreciation would then have been charged on that net amount, giving a charge for the year of £3,000 (12,000/4) compared to a figure in Part (a) of £7,500 (30,000/4). The final carrying amount would then be £9,000 (12,000 – 3,000) compared to £22,500 (30,000 – 7,500). This decrease of £4,500 in the depreciation charge reflects the fact that under the netting-off method the grant is recognised in profit and loss over the life of the depreciable asset – “replacing” the credit of £4,500 in Part (a) where the grant is released directly into cost of sales. The reduction of £13,500 in the final carrying amount “replaces” the total deferred income on the statement of financial position in Part (a) of £13,500. The net effect on profit of the two methods is in fact the same as the different “treatments” are really a difference of presentation. Almost all candidates knew that the alternative treatment of the government grant was the “netting off method” and correctly calculated the figures (cost, depreciation charge and carrying amount) on that basis. Fewer candidates compared these figures to those they had calculated in Part (a). Most stated that the figures “had the same net effect” but few described why this is in any detail. Total possible marks 5½ Maximum full marks 4

Copyright © ICAEW 2014. All rights reserved

Page 4 of 16

Financial Accounting and Reporting – Professional Level – June 2014 (c) How information re PPE meets needs of users Financial position The financial position of an entity is affected by the economic resources it controls, its financial structure, its liquidity and solvency and its capacity to adapt to changes in the environment in which it operates. Information about the total carrying amount of property, plant and equipment (PPE) as given on the face of an entity’s statement of financial position gives the user an indication of the resources the entity has available to it in terms of tangible assets held for long-term use in the business. Revalued figures are more relevant than cost. That figure will then be broken down further in the notes to the financial statements. This indicates the type of PPE held which may add further to an understanding of resource. This note also shows the changes in financial position in the year. For example, land and buildings might be held for investment potential as well as being used for office/factory space. Plant will be used to generate future revenues. Equipment could be used for the generation of future revenues or for the entity’s own use, perhaps for administrative purposes. The fact that the amount of leased assets forming part of the PPE figure is disclosed shows that these assets have a future cost in terms of lease payments – affecting the liquidity and solvency of the entity. The “capital commitments” note showing the future purchases of PPE to which the entity is committed indicates a requirement for future finance. The accounting policy note shows the valuation model used and depreciation methods, which allow comparison to other entities. Financial performance Information about financial performance, in particular profitability, is needed in order to assess potential changes in the economic resources that the entity is likely to control in the future. Disclosure of the annual depreciation charge shows the “cost” of using the assets. Disclosure of significant gains/losses on disposal could indicate problems with the depreciation method or where value is greater than carrying amount. Impairment losses may indicate underlying issues, such as underprovision of depreciation, or a downturn in a particular market sector (which might affect future performance). Changes in financial position Changes in financial position are shown in the statement of cash flows. This allows users to assess the ability of the entity to generate cash and its need to use what is generated. Users will be able to see, via the statement of cash flows, PPE purchased during the year and cash inflows from PPE disposed of. If little PPE is purchased and much disposed of then the user may be concerned about the future of the entity. This part was dealt with much less well. The majority of candidates clearly struggled with this requirement, with a significant number gaining either one or zero marks (in spite of the fact that there is a very similar question in the revision question bank). A significant number of candidates simply discussed the qualitative characteristics in respect of property, plant and equipment, which was not asked for and gained no marks. Others made a series of “random” comments, with no attempt to link these to “financial position”, “financial performance”, “or “changes in financial position” as represented, per the Conceptual Framework, by the statement of financial position, the statement of profit or loss and the statement of cash flows. Candidates must read requirements carefully and be mindful that unless the requirement is addressed they are wasting their time writing about something that they think might be relevant. It was not uncommon to see a whole page of writing gaining zero marks. Those candidates who scored the highest number of marks set up three headings (ie “financial position”, “financial performance” and “changes in financial position”) and made pertinent comments under each. There was, however, a common misconception, even amongst these candidates, that changes in financial position are shown by the statement of changes in equity. Another common error was to say that the statement of financial position showed cost less accumulated depreciation thereby showing what property, plant and equipment “is worth”. Others referred to the performance of the asset, as opposed to the financial performance of the reporting entity. Total possible marks 9½ Maximum full marks 5

Copyright © ICAEW 2014. All rights reserved

Page 5 of 16

Financial Accounting and Reporting – Professional Level – June 2014

Question 2 Total Marks: 29 General comments Part (a) of this question required candidates to explain the financial reporting treatment of four accounting issues, given in the scenario. The issues covered a lease of land and buildings, decommissioning costs, sale and repurchase and an event after the reporting period. Part (b) required the calculation of revised earnings and basic EPS, having adjusted for errors made by the company as discussed in Part (a), plus an explanation of why the managing director’s calculation was incorrect. Part (c) required an explanation of the ethical issues arising from the scenario and the action to be taken.

Limerick plc (a) IFRS accounting treatment (1) Lease of land and buildings IAS 17, Leases, requires that the land and buildings elements of a single lease are considered separately in order to classify as a finance or an operating lease. A lease is classified as a finance lease if it transfers substantially all the risks and rewards incidental to the ownership of an asset. The managing director (MD) is correct that land has an indefinite economic life. However, given the fact that ownership does pass, this lease is relatively long, and in the case of the buildings is for almost all of the asset’s useful life, and in both cases the present value of the minimum lease payments amount to “substantially all” of the fair value of the asset, the whole lease should be treated as a finance lease. As the MD has treated this as an operating lease then the payment of £120,000 made on 1 January 2013 will have been debited to expenses. This entry will need to be reversed. Per IAS 17, the finance lease should be capitalised at the lower of the fair value of £1.3 million and the present value of the minimum lease payments of £1,290,835 (1,183,265 + 107,570). The table below illustrates the entries which should have been made. Year ended

31 Dec 2013 31 Dec 2014

B/f £ 1,290,835 1,287,919

Payment £ (120,000) (120,000)

Capital £ 1,170,835 1,167,919

Interest at 10% pa £ 117,084

C/f £ 1,287,919

A finance cost of £117,084 should be charged in the statement of profit or loss. The lease liability at 31 December 2013 is therefore £1,167,919 non-current and £120,000 current. Because legal title will pass, the building should be depreciated over its useful life of 42 years, giving a depreciation charge for 2013 of £28,173 (1,183,265 ÷ 42). The land is not depreciated. The carrying amount of the land and buildings in the statements of financial position at as 31 December 2013 will therefore be £1,262,662 (1,290,835 – 28,173). (2) Decommissioning costs Per IAS 37, Provisions, Contingent Liabilities and Contingent Assets, a provision should be recognised where:  there is a present obligation as a result of a past event  an outflow of resources is probable, and  the amount can be estimated reliably. The decommissioning costs meet these recognition criteria as:  there is an obligation to decommission (it was a condition of the sale),  it arose from a past event (the purchase of the plant), and  there is a reliably estimated outflow of resources (the £50,000 that will be paid out).

Copyright © ICAEW 2014. All rights reserved

Page 6 of 16

Financial Accounting and Reporting – Professional Level – June 2014 When the plant was purchased on 1 January 2013, a provision should therefore have been made for the 5 discounted costs of decommissioning the plant in five years’ time, measured as £50,000 x 1/(1.07) = £35,649, adding this amount to the cost of the asset. This would also have had the effect of increasing the depreciation charge for 2013 on the asset by £7,130 (35,649 ÷ 5). A finance cost of £2,495 (35,649 x 7%) should be charged in the year ended 31 December 2013 to reflect the unwinding of the discount and the provision should be increased by the same amount. In the statement of financial position as at 31 December 2013 the provision will be shown as a non-current liability of £38,144 (35,649 + 2,495). (3) Sale and repurchase This is a sale and repurchase agreement. Per IAS 18, Revenue, the terms and conditions of the sale need to be considered to determine whether or not there is a sale in substance. Where legal title has been transferred, but the risks and rewards of ownership (here the right to build on the land and potential gains and losses in market values) have been retained by the “seller” the transaction is treated as a financing arrangement. The fact that Limerick plc is likely to repurchase the land and at a price which is below the current market price adds weight to this conclusion. The profit on the “sale” of the land of £250,000 (750,000 – 500,000) should therefore be derecognised. A loan of £750,000 and accrued finance cost of £52,500 (750,000 x 7%) should be recognised. (4) Event after the reporting period Per IAS 10, Events After the Reporting Period, the determination of the court case is an adjusting event as it provides evidence of conditions that existed at the end of the reporting period (ie of the court case that was already in progress). The financial statements should therefore be adjusted to include an accrual for the total due of £125,000 and the note re the contingent liability removed. There is no specific requirement to disclose the adjusting event. Candidates generally performed well overall on this part of the question, although some issues were dealt with better than others. The majority of candidates responded to all four issues and provided both explanations and supporting calculations. Issue (1): Most candidates made a good attempt at the lease of land and buildings, although there was clearly some confusion on this topic. Candidates generally understood that to assess which lease is present for land and buildings they needed to make the assessment separately. However, many candidates incorrectly identified the land as being an operating lease, even where they had noted that legal title passed. Where candidates did realise that both elements were finance leases they often split them out and produced two lease tables, which was unnecessary. However, this sometimes followed on from a statement that the two leases were to be “treated” separately, as opposed to “considered” separately when classifying them and this may have been where the confusion arose. Many answers lacked consistency. For example, the land would be identified as being an operating lease but then the full lease payment was added back and used in the finance lease table. Land was identified as having an indefinite life but then the total, including land, was used for the depreciation working. The finance lease table itself was generally correctly done, although the opening figure was often incorrect and a minority of candidates treated the payments as made in arrears rather than in advance. The majority of candidates incorrectly identified that the depreciation on the building should have been over 40 years rather than 42 years, even where they had identified that ownership passed. Almost all candidates stated that the closing liability needed to be split into current and non-current but a significant number gave an incorrect split of the total figure. Issue (2): Most candidates correctly identified that the decommissioning costs should have been added to the asset’s carrying amount, but fewer identified that a provision should be set up to complete the double entry. Of those that did, only a minority set out the IAS 37 conditions for the recognition of a provision and fewer still applied these conditions to the scenario. The majority of candidates correctly identified that the amount should be discounted although a minority used the incorrect discount rate. It was pleasing to see that the majority of candidates also correctly depreciated the revised carrying amount of the asset and realised that they needed to do some unwinding of the provision (even where they hadn’t identified that a provision should be recognised). The main concern with this issue was a lack of supporting narrative with many answers containing little more than a series of numbers. Issue (3): The answers for the sale and repurchase were mixed with the majority of candidates concluding that this was a sale and leaseback rather than a sale and repurchase. However, a number of marks were still available for a good discussion centred around the principles of substance over form and the nontransference of risks and rewards.

Copyright © ICAEW 2014. All rights reserved

Page 7 of 16

Financial Accounting and Reporting – Professional Level – June 2014 Many calculated an accrued finance cost based on the repurchase price less the sale price, failing to recognise that the repurchase was in two years’ time, not one. Issue (4): The final issue concerned an adjusting event after the reporting period, with most candidates correctly concluding that a provision needed to be made. However, around half of the candidates simply seemed to miss that this should have been a discussion about events after the reporting date and new information concerning a condition that existed at that date, rather than a simple assessment of a provision. Therefore a number of easy marks were lost through lack of narrative. Total possible marks Maximum full marks

26½ 18

(b) Revised earnings and basic EPS £ 500,500 120,000 (117,084) (28,173) (7,130) (2,495) (250,000) (52,500) (125,000) 38,118

Earnings per draft financial statements Add back: Operating lease rental (1) Less: Finance cost re leased asset (1) Depreciation on leased asset (1) Depreciation on decommissioning costs (2) Finance cost re decommissioning costs (2) Profit on “sale” of land (3) Finance cost re land “sold” (3) Damages/costs in court case Revised earnings figure Weighted average number of ordinary shares: Number of shares 1 January 2013 100,000 Bonus issue (1 for 4) 25,000 1 October 2013 125,000 Issue at full market price 80,000 31 December 2013 205,000 EPS (38,118 ÷ 145,000)

x 6/12 x 5/4 x 3/12 x 3/12

Weighted average 62,500 31,250 93,750 51,250 145,000 26.3p

Per IAS 33, Earnings per Share, the calculation of basic earnings per share should be based on the weighted average number of ordinary shares outstanding during the period. So where there have been share issues during the period, as here, it is incorrect to use the opening (or indeed the closing) number of shares. Where shares have been issued at market price, those shares should only be included in the shares in issue for part of the period – ie the period in which the proceeds from that share issue have generated earnings. Conversely, because bonus shares have not generated any cash/earnings they are dealt with in the calculation by IAS 33 by assuming that the shares have always been in issue. The majority of candidates made a good attempt at adjusting the “earnings” given in the question by their figures calculated in Part (a). It was common to see this as the first page of the answer to Question 2, showing that candidates had heeded advice from the examining team about building up this part of an answer as they went along. A few candidates, however, disadvantaged themselves by combining various figures from Part (a) into a “net” adjustment for each issue – all well and good if an audit trail was provided, but if not marks could well have been lost. The calculation of the weighted average number of shares was, however, disappointing, compared to when an EPS calculation was set in a previous paper. A significant number of candidates were unable to correctly calculate this figure. The most common errors were to use the wrong number of months or to incorrectly adjust for the bonus issue. The final element of this part of the question was to explain why the managing director was incorrect in basing his EPS calculation on the opening number of ordinary shares. It was disappointing that few candidates went beyond saying that this was wrong and that the managing director should have used a weighted average number of shares. Very few made any link between the issue of shares and the earnings those shares might or might not generate depending on whether the issue was for cash or not. Total possible marks 8½ Maximum full marks 6

Copyright © ICAEW 2014. All rights reserved

Page 8 of 16

Financial Accounting and Reporting – Professional Level – June 2014 (c) Ethical issues The MD has given plausible reasons for the accounting treatment of the issues identified. Each issue is technical in nature and the treatments may appear reasonable to a business manager with a general appreciation of accounting principles but not a detailed awareness of current reporting standards. However, the MD appears to be applying pressure to have his treatments confirmed by offering incentives for compliance with his wishes (intimidation threat). I should not be swayed by the thought of being made the new FD (self-interest threat). Furthermore, all the treatments adopted by him have the effect of increasing the EPS figure to above that of the previous year, which is said to be a key criteria for the board. Once the correct treatments are adopted basic EPS in fact falls back to below the level of the previous year to 26.3p compared to 70.3p. Even if last year’s EPS is restated for the bonus issue to 56.2p (70.3p x 4/5) this is still a fall in EPS – not the “significant improvement” that the board is looking for. The finance director (FD) left under suspicious circumstances, which need to be confirmed. It may be that he too was put under pressure to adopt incorrect accounting treatments and found the situation untenable. IFRS is quite clear on the appropriate treatment of these four issues. There is little, if any, choice or judgement on any of the matters. I should not give in to the MD’s wishes or prepare financial statements that are contrary to IFRS. I should apply the ICAEW Code of Ethics, with the following programme of actions:  Explain matters to the MD with supporting evidence so that the matters can be corroborated.  If resolution cannot be achieved, discuss the matters with the other directors to explain the situation and obtain support. Consider also discussing the issues with the external auditors/audit committee.  Obtain advice from the ICAEW helpline or local members responsible for ethics. During the resolution process it would be useful to keep a written record of all discussions, who else was involved and the decisions made. Almost all candidates made a reasonable attempt at this part of the question, with a good number obtaining three or four marks, although five marks was rare. Candidates should remember that to gain the most marks their answer should be tailored to the question scenario. Most candidates correctly identified that the departure of the finance director was suspicious and that there was a self-interest threat and an intimidation threat for the financial controller. They then went on to explain how these threats arose and to suggest appropriate courses of action. A minority of candidates answered as if this was a problem facing an external auditor, not an accountant in a company. Others were concerned about the managing director’s lack of technical competence and adherence to the Code of Ethics when he was not a qualified accountant. As ever, a few felt there were money laundering issues at play. Total possible marks 8½ Maximum full marks 5

Copyright © ICAEW 2014. All rights reserved

Page 9 of 16

Financial Accounting and Reporting – Professional Level – June 2014

Question 3 Total Marks: 19 General comments This question required the preparation of a consolidated statement of financial position. The group had two subsidiaries, one of which was acquired at the start of the year. The question featured contingent consideration, both goodwill and a gain on bargain purchase, fair value adjustments on acquisition, and inter-company trading between the two subsidiaries, with adjustments needed to reconcile the intra-group balances and deal with goods in transit. Laois plc Consolidated statement of financial position as at 31 December 2013 Assets Non-current assets Property, plant and equipment (2,687,000 + 2,196,000 + 591,800 + (200,000 – 40,000 (W1)) Goodwill (W3) Current assets Inventories (193,200 + 53,700 + 159,000 – 5,000 (W7) + 12,000 (W7)) Trade and other receivables (288,000 + 92,300 + 207,000 – 50,000 – 15,000 (W7)) Cash and cash equivalents (15,800 + 12,400 + 1,100)

£

£ 5,634,800 168,000 5,802,800

412,900 522,300 29,300 964,500 6,767,300

Total assets Equity and liabilities Equity Ordinary share capital Share premium account Retained earnings (W6) Attributable to the equity holders of Laois plc Non-controlling interest (W5)

2,000,000 750,000 1,992,480 4,742,480 649,520 5,392,000

Current liabilities Trade and other payables (398,600 + 220,800 + 436,400 – 50,000) Deferred consideration Taxation (150,000 + 105,000 + 10,500)

1,005,800 104,000 265,500 1,375,300 6,767,300

Total equity and liabilities Workings (1) Net assets – Carlow Ltd

Share capital Share premium Retained earnings FV adj Deprec on FV adj (200,000/25 years x 5) PURP (5,000 + 3,000) (W7)

Copyright © ICAEW 2014. All rights reserved

Year end £ 650,000 300,000 1,078,600 200,000 (40,000) (8,000) 2,180,600

Acquisition £ 650,000 300,000 592,000 200,000 – – 1,742,000

Post acq £

438,600

Page 10 of 16

Financial Accounting and Reporting – Professional Level – June 2014 (2) Net assets – Kerry Ltd

Share capital Retained earnings Goodwill adj

Year end £ 360,000 176,000 (24,000) 512,000

Acquisition £ 360,000 240,000 (30,000) 570,000

Post acq £

(58,000)

(3) Goodwill – Carlow Ltd £ 1,560,000 350,000 (1,742,000) 168,000

Consideration transferred Non-controlling interest at acquisition at fair value Less: Net assets at acquisition (W1) (4) Gain on bargain purchase – Kerry Ltd

£ Consideration transferred Cash Deferred consideration at present value (104,000/1.04) Non-controlling interest at acquisition at fair value Less: Net assets at acquisition (W2)

200,000 100,000 235,000 (570,000) (35,000)

(5) Non-controlling interest £ Carlow Ltd Fair value at acquisition Share of post-acquisition reserves (438,600 (W1) x 20%) Kerry Ltd Fair value at acquisition Share of post-acquisition reserves ((58,000) (W2) x 40%)

£

350,000 87,720 437,720 235,000 (23,200) 211,800 649,520

(6) Retained earnings £ 1,645,400 (4,000) 350,880 (34,800) 35,000 1,992,480

Laois plc Less: Unwinding of discount (104,000 – 100,000 Carlow Ltd (80% x 438,600 (W1)) Kerry Ltd (60% x (58,000) (W2)) Gain on bargain purchase (W4)

(7) PURP

Sales in year

Goods in transit £ 15,000 (12,000) 3,000

% £ SP 125 50,000 Cost (100) (40,000) GP 25 10,000 x½ 5,000 Generally candidates performed well on this question with a reasonable number achieving full marks. Nearly all candidates produced the expected standard workings (which are to be strongly encouraged) and a significant number arrived at the correct figures for the goodwill and gain on bargain purchase. Most then correctly took the gain to retained earnings although a minority netted it off against the goodwill figure. The fair value adjustment to property was well dealt with (although a number used the incorrect number of years when calculating the depreciation adjustment) as was the calculation of the unrealised profit on the goods held at the year end. Most candidates also correctly followed these adjustments through to property, plant and equipment and inventory respectively. However, the adjustments for the goods in transit were not well dealt with and few candidates dealt correctly with all aspects of this (although many did at least calculate the adjustment for unrealised profit). A good number of candidates failed to increase inventories by the cost of the goods in transit between the two subsidiaries at the year end. Others failed to reduce trade and other receivables by the selling price of these goods, to reflect the fact that the receivable for the goods in transit had already been accounted for in the selling subsidiary’s own financial statements.

Copyright © ICAEW 2014. All rights reserved

Page 11 of 16

Financial Accounting and Reporting – Professional Level – June 2014

The goodwill held within the subsidiary also caused problems and a considerable number of students completely ignored it in the net assets working, with a minority adding rather than deducting it. Many also did not understand the impact of the impairment that had been recognised within the subsidiary in relation to this goodwill and went on to make incorrect adjustments to the discount on acquisition calculated for this subsidiary and/or to retained earnings. The aspect of the question that was least well dealt with was the deferred consideration. Although virtually all candidates used the correct figure to add to consideration very few then charged the unwinding of the discount to retained earnings. Even fewer showed anything in liabilities and even when they did it was often the wrong number. Other common errors included the following:  Making adjustments for unrealised profits in the wrong place (ie against the net assets of the subsidiary buying the inventory or in retained earnings or in both).  Omitting the balance on the share premium account from the net assets table.  Entering figures such as the fair value adjustment in one column of the net assets table rather than in both.  Attempting to calculate the non-controlling interest by taking a percentage of closing net assets (which would work for the proportionate method) when this is clearly wrong if the fair value method is being used.  Adjusting trade receivables for the cost of the inventory in transit rather than for the sales price.  Adjusting trade payables for goods in transit when no liability had been recognised.  Adding (rather than deducting) post-acquisition losses to the non-controlling interest and retained earnings workings. It is disappointing that a good number of candidates still lose marks for failing to show an “audit trail”, particularly for the share of post- acquisition profit or loss to be taken to the non-controlling interest and retained earnings workings. To ensure they get the relevant marks candidates must show the figure (to check that the correct movement in the net assets working has been picked up) multiplied by the appropriate percentage. Many candidates actually waste time by writing out, for example, “NCI share of post- acquisition profit” when it would be faster and clearer to show, for example, “£58,000 x 20%”. Total possible marks Maximum full marks

Copyright © ICAEW 2014. All rights reserved

20½ 19

Page 12 of 16

Financial Accounting and Reporting – Professional Level – June 2014

Question 4 Total Marks: 21 General comments This question required the redrafting of extracts from the consolidated financial statements. Matters to be adjusted for were the disposal of a subsidiary, the setting up of a joint venture and a development project. Part (b) required an explanation of the differences between IFRS and UK GAAP in respect of these issues. Part (c) required a calculation of distributable profits and explanation thereof. Kildare plc (a) Extracts from the consolidated financial statements for the year ended 31 December 2013 £ Consolidated statement of profit or loss (extracts) Profit attributable to Owners of Kildare plc (W5) Non-controlling interest (256,700 + (37,500 (W5) x 30%))

899,590 267,950

Consolidated statement of financial position (extracts) Non-current assets Property, plant and equipment (2,752,100 + 90,000 – 15,000 (W4)) Intangibles (356,000 + 115,000 + 15,000* (W4)) Investment in joint venture (W2)

2,827,100 486,000 110,480

Consolidated statement of cash flows (extract) Net cash from investing activities (– 50,600 + (300,000 – 1,500)) – (115,000 + 90,000) (W4) + 10,000)

52,900

*Note: Credit was also given if the depreciation on the equipment (W4) was expensed, rather than being recapitalised. Workings (1) Profit on disposal of subsidiary £ Sale proceeds Less: Carrying amount of goodwill at date of disposal Consideration transferred NCI at acquisition (214,900 x 30%) Less: Net assets at acquisition (50,000 + 158,900 + (35,000 – 29,000)) Goodwill at acquisition Less: Impairments to date

£ 300,000

225,000 64,470 (214,900) 74,570 (40,000)

Less: Carrying amount of net assets at date of disposal (50,000 + (275,000 – (75,000 x 6/12))) Add back: NCI at date of disposal (287,500 x 30%) Profit on disposal

(34,570) (287,500) 86,250 64,180

(2) Investment in joint venture Cost of investment Share of post-acquisition increase in net assets ((48,400 + 52,800) x 40%) Less: Dividend received

Copyright © ICAEW 2014. All rights reserved

£ 80,000 40,480 (10,000) 110,480

Page 13 of 16

Financial Accounting and Reporting – Professional Level – June 2014

(3) Share of profits of joint venture £ 19,360 (6,000) 13,360

Share of JV’s PAT (40% x 48,400) Less: Share of PURP (40% x (120,000 x 25% x ½)) (4) R&D expenditure Per Suspense account Less: Carried forward as Intangibles: Qualifying development costs Patent registration costs

£ 275,000 110,000 5,000

Carried forward as PPE Written off (initial research costs + evaluation of research findings) (β) Depreciation on equipment (90,000/3 x 6/12)

(115,000) 160,000 (90,000) 70,000 15,000

(5) Profit attributable to owners Per draft Profit on disposal of subsidiary (W1) Share of profit of subsidiary (75,000 x 6/12 = 37,500 x 70%) Share of profit of JV Research costs etc (W4)

£ 865,800 64,180 26,250 13,360 (70,000) 899,590

Answers to this part were generally disappointing. Although it was the most challenging question on the paper there were many easy marks available for basic consolidation workings (such as the disposal of the subsidiary) and for adjustments to property, plant and equipment and intangible assets. Answers were generally difficult to follow often with lengthy and unnecessary workings. Fewer candidates than normal managed to calculate the profit on disposal correctly although it was more common to see the correct figure for goodwill. Many candidates produced one combined and somewhat “muddled” working here which often resulted in the impairment to goodwill decreasing rather than increasing the profit on disposal. Furthermore, many candidates clearly did not understand that the whole of the profit on disposal should have been allocated to the owners, but that the profit of the subsidiary for the year up to disposal should have been split between the owners and the non-controlling interest. It was extremely disappointing to see how few candidates realised that equipment used for research and development should be included within property, plant and equipment rather than in intangible assets. Candidates also struggled to decide how much of the research and development costs should be capitalised and how much should be expensed. These were very simple decisions that should have been quickly made and the appropriate adjustments taken directly to the extracts. Nearly all candidates calculated an amortisation charge for the capitalised development costs even though the new product was still in the development stage. The joint venture also caused problems, in particular the calculation and treatment of the provision for unrealised profits. Those candidates who did attempt to calculate the latter often failed to multiply it by the relevant percentage. Many candidates did adjust the cash used in investing activities for the proceeds of the disposal (net of the cash held by the subsidiary) and for the dividend received from the associate. However, some also used the former (net) figure when calculating the profit on disposal of the subsidiary. It was, however, rare to see the cash used in investing activities adjusted for the amounts spent on intangibles and property, plant and equipment.

Copyright © ICAEW 2014. All rights reserved

Page 14 of 16

Financial Accounting and Reporting – Professional Level – June 2014 Other specific errors not noted above included the following:  Omitting to time apportion the current year depreciation charge.  Omitting to time apportion the post-acquisition profits to be added to the non-controlling interest and owner’s share of profits (and/or failing to multiply them by the relevant percentage).  Assuming the shares had a nominal value of £1 rather than the 50p given in the question.  Ignoring the fair value adjustment for inventory or including it in net assets at the year end (as well as or instead of at acquisition), even though it had been sold.  Attempting to adjust the year end figure for intangibles by the cumulative impairment losses of £40,000 in respect of the disposed of subsidiary, despite the fact that it had been disposed of by this stage (and it was stated in any case in the question that no amounts in respect of this subsidiary had been consolidated).  Treating the joint venture as if it had been bought on the first day of the current year rather than of the previous year.  Taking the wrong figure for the cost of the joint venture (the most common error being to take the whole of the joint venture’s share capital rather than the 40% which the parent company had purchased).  Taking the total post-acquisition profits of the joint venture to investment in joint venture rather than the appropriate percentage (and/or only taking one year’s profits).  Not showing the investment in the joint venture within the non-current assets section of the extracts. Total possible marks Maximum full marks

16½ 12

(b) IFRS v UK GAAP differences Under IAS 38, Intangibles, development expenditure must be capitalised where the relevant criteria are met. Under UK GAAP (SSAP 13) the capitalisation of development expenditure which meets certain criteria is optional. The development expenditure recognition criteria of SSAP 13 include a requirement to have or a reasonable expectation of future benefits. IAS 38 is more stringent as the requirement is to demonstrate future benefits. UK GAAP (FRS 10) would have required the goodwill arising in the business combination with Sligo Ltd to be amortised over its finite useful life. Under IAS 38 goodwill is tested annually for impairment. Under UK GAAP (FRS 6) minority interest (the non-controlling interest) is always measured at its share of net assets. IFRS 3 allows non-controlling interest to be measured at fair value (the fair value method) or at its share of net assets (the proportionate method as used here). UK GAAP (FRS 9) requires the use of the gross equity method for joint ventures. IAS 28, Investments in Associates and Joint Ventures, requires the use of the equity method. The gross equity method is the same as the equity method except that disclosure is required of the following figure:  in the profit and loss account – the investor’s share of the turnover of its joint venture  in the balance sheet – the investor’s share of the gross assets and liabilities underlying the net equity amount. Answers to this part were mixed but most candidates did manage to pick up at least a couple of marks although very few gained full marks. As in previous sittings the main problem is that candidates include differences that are not relevant to the actual issues given, such as discussing the treatment of “negative” goodwill when there was no negative goodwill in this scenario. Total possible marks Maximum full marks

Copyright © ICAEW 2014. All rights reserved

6 4

Page 15 of 16

Financial Accounting and Reporting – Professional Level – June 2014 (c) Distributable profits and explanation For entities within a group, the calculation of distributable profits must be made for each entity separately, not for the consolidated group. Therefore Kildare plc’s distributable profits are those distributable by the parent company only. The basic rule is that distributable profits are measured as accumulated realised profits less accumulated realised losses. In the case of listed companies, the amount of distributable profits is further reduced by any excess of unrealised losses over unrealised profits. In the case of Kildare plc, insufficient information is available in the scenario to identify any such excess. Assuming that no such excess exist, then distributable profits are calculated as below:  The disposal of the shares in Sligo Ltd affect Kildare plc’s parent company figures by the (as yet unrecorded) parent company profit. This profit is the difference between the cost of the shares (£225,000) and the sale proceeds (£300,000), increasing Kildare plc’s single entity retained earnings by £75,000.  The share of profits in the joint venture only affects the consolidated retained earnings, but Kildare plc’s own financial statements would include the dividend from Mayo Ltd of £10,000. Since this has been credited to a suspense account, Kildare plc’s single entity retained earnings need increasing by £10,000.  The research and development costs were spent by Kildare plc and therefore any adjustments in respect of this affect its individual financial statements and hence distributable profits. Kildare plc’s single entity retained earnings need reducing by £70,000 (W4). As there is no further information on the reserve balances which form part of equity, the distributable profits of Kildare plc are therefore: 109,700 + 75,000 + 10,000 – 70,000 = £124,700 Note: Credit was also given, where the depreciation on the equipment in Part (a) had been expensed, for discussing the impact of this on distributable profits. It was clear that very few candidates had spent any time on understanding distributable profits. A significant number of candidates did not attempt this part of the question and even when they did make some attempt, often achieved no marks at all. Very few candidates knew even the most basic points (such as realised profits less realised losses) or that distributable profits are based on the individual company’s financial statements. Many candidates who did attempt this part of the question wasted time by simply copying out adjustments made in Part (a) of the question that related to the consolidated financial statements Total possible marks Maximum full marks

Copyright © ICAEW 2014. All rights reserved

7 5

Page 16 of 16

Professional Level – Financial Accounting and Reporting – September 2014

MARK PLAN AND EXAMINER’S COMMENTARY The mark plan set out below was that used to mark these questions. Markers are encouraged to use discretion and to award partial marks where a point was either not explained fully or made by implication. More marks are available than could be awarded for each requirement, where indicated. This allows credit to be given for a variety of valid points, which are made by candidates.

Question 1 Overall marks for this question can be analysed as follows:

Total: 20

General comments This question required the preparation of the statement of profit or loss and statement of financial position. A number of adjustments were required to be made, including depreciation, borrowing costs, an inventory write down, a bonus issue and a finance lease.

Barchetta Ltd – Statement of financial position as at 31 March 2014 £ ASSETS Non-current assets Property, plant and equipment (2,087,050 (W4) + 41,570 (W5)) Current assets Inventories (W2) Trade and other receivables Cash and cash equivalents

2,128,620

31,850 85,400 6,800 124,050 2,252,670

Total assets Equity Ordinary share capital (400,000 x 6/5) Retained earnings (W7)

480,000 481,909

Equity Non-current liabilities Bank loan Finance lease (W6)

£

961,909

1,100,000 10,872 1,110,872

Current liabilities Trade and other payables Finance lease (13,161 – 10,872) Taxation

93,100 2,289 84,500 179,889

Total equity and liabilities

Copyright © ICAEW 2014. All rights reserved.

2,252,670

Page 1 of 15

Professional Level – Financial Accounting and Reporting – September 2014

Barchetta Ltd – Statement of profit or loss for the year ended 31 March 2014

Revenue Cost of sales (W1) Gross profit Administrative expenses

£ 4,521,000 (3,409,730) 1,111,270 (804,700)

Operating profit Finance charges (83,060 – 27,600 (W3) + 1,071 (W6)) Profit before tax Income taxation

306,570 (56,531) 250,039 (84,500)

Net profit for the period

165,539

W1 Expenses Trial balance Opening inventories Closing inventories (W2) Depreciation charge – building (W4) Depreciation charge – plant & machinery (W5) Reverse lease payment

Cost of sales 3,379,100 27,640 (31,850) 25,600 12,450 (3,210) 3,409,730

W2 Inventory Closing inventory Inventory write down (200 x (315 – (320 – 25)) At 31 March 2014

£ 35,850 (4,000) 31,850

W3 Borrowing costs Weighted average cost of loans = (600,000 x 6.4%) + (500,000 x 7.5%) = 6.9% 1,100,000 Borrowing costs to be capitalised = (300,000 + (400,000 x 3/12)) x 6.9% = 27,600 W4 PPE – Building £ Cost b/f Depreciation charge for year ((2,230,0000 – 250,000 – 700,000)/ 50 yrs) Accumulated depreciation Borrowing costs (W3) Carrying amount at 31 March 2014

£ 2,230,000 (25,600) (144,950) 27,600 2,087,050

W5 PPE – Plant and equipment £ Cost b/f Depreciation charge for the year (60,500 – 22,000) / 5yrs) Plant – different useful life ((22,000 x 4/5) / 8yrs) Leased equipment (15,300 / 6yrs)

Accumulated depreciation b/f Leased equipment Carrying amount at 31 March 2014

Copyright © ICAEW 2014. All rights reserved.

£ 60,500

(7,700) (2,200) (2,550) (12,450) (21,780) 15,300 41,570

Page 2 of 15

Professional Level – Financial Accounting and Reporting – September 2014

W6 Finance lease

31 March 2014 31 March 2015

Opening balance £ 15,300 13,161

Interest @ 7% £ 1,071 921

Lease payment £ (3,210) (3,210)

Closing balance £ 13,161 10,872

W7 Retained earnings Per draft Add: profit and loss in year Bonus issue (75,000 – (400,000 / 5))

£ 321,370 165,539 (5,000) 481,909

Presentation of the statement of profit or loss and statement of financial position was generally very good with most candidates achieving the maximum presentation marks available. A significant majority of candidates arrived at completely correct figures in respect of closing inventories, the finance lease, and the bonus issue. It was rare to see a completely correct figure for property, plant and equipment, although this was usually due to errors on land and buildings as opposed to plant and machinery. It was less common to see the correct figure for capitalised interest, although almost all candidates made a good attempt at this calculation, with nearly all candidates arriving at the correct effective interest rate. However, most candidates went on to apply that rate to the whole £1,100,000 borrowed or to the whole £700,000 paid to the contractor, instead of taking into account that £400,000 of that amount had only been paid four months before the year end. Most candidates provided relatively clear workings for their property, plant and equipment figure. The most common error was to not remove the amounts paid to the contractor before calculating the depreciation on buildings. The vast majority of candidates used a “costs matrix” to calculate the figure for cost of sales, and many correctly allocated all of the costs to this category. The most common errors were to fail to deduct the finance lease payment incorrectly included (even where the candidate had “used” this figure in their finance lease working) and/or not to include all of the depreciation figures to cost of sales. This is despite the question itself specifying that depreciation should be presented under this expense heading. Weaker candidates often got themselves in a muddle in this working, mixing up their bracket convention.

Total possible marks Maximum full marks

Copyright © ICAEW 2014. All rights reserved.

22½ 20

Page 3 of 15

Professional Level – Financial Accounting and Reporting – September 2014

Question 2 Overall marks for this question can be analysed as follows:

Total: 34

General comments Part (a) of this question required candidates to explain the financial reporting treatment of five accounting issues, given in the scenario. The five issues covered the irredeemable preference shares, research and development, an onerous contract, revenue recognition and a related party transaction. Part (b) required candidates to recalculate profit before tax, equity and liabilities for the adjustments needed as a result of their answer to Part (a). Part (c) required candidates to discuss and compare the accrual basis of accounting with cash accounting with reference to revenue recognition and the Framework’s recognition criteria. Impreza plc (a) IFRS accounting treatment (1) Irredeemable preference shares The irredeemable preference shares provide the investor with the right to receive a fixed (5% pa) amount of annual dividend out of Impreza plc’s profit for the period on a mandatory basis. If the annual dividend is not paid then it is rolled up into the following year’s payment as the dividends are cumulative in nature. Under IAS 32 Financial Instruments: Presentation, these shares should be classified as financial liabilities as there is a contractual obligation to deliver cash. The preference shares should therefore be accounted for at amortised cost using the effective interest rate which is equivalent to the annual dividend rate of 5% as they are not redeemable. This reflects the substance of the share issue. £450,000 should therefore be recognised as part of non-current liabilities and removed from equity and the dividend payment of £18,750 (450,000 x 5% x 10/12) should be accrued for at 31 March 2014 and included within finance costs in the statement of profit or loss. (2) Research and development As per IAS 38, Intangible Assets, distinction needs to be made between research and development expenditure as expenditure incurred during the research phase should be recognised as an expense in profit or loss when it occurs. During the research phase there is insufficient evidence that the expenditure will generate future economic benefits. The first £350,000 of expenditure was incurred during investigation work and is therefore classed as research expenditure and should have been recognised as an expense in the statement of profit or loss. Although expenditure incurred after this initial work is all development work, in order for it to be capitalised as an intangible asset Impreza plc needs to meet strict criteria including:      

The technical feasibility of completing the asset The intention to complete the asset The ability to use the asset Demonstrate the commercial viability of the asset The availability of adequate resources Reliable measurement of expenditure

Copyright © ICAEW 2014. All rights reserved.

Page 4 of 15

Professional Level – Financial Accounting and Reporting – September 2014

Therefore all of the development expenditure incurred up to 31 July 2013, ie £700,000, should be recognised as an expense as part of profit or loss because the asset was not commercially viable until that date. On 1 August 2013 the asset met all the capitalisation criteria and therefore qualifying expenditure should be capitalised from this date. The £200,000 incurred on launch activities is not qualifying expenditure because it does not involve design, construction or testing and this should be expensed when incurred. The remaining balance of £2,320,000 (3,570,000 – 350,000 – 700,000 – 200,000) should be capitalised. Amortisation should commence when the asset is available for use. Although the control system was promoted from 1 February 2014 it was not ready for use until 1 April 2014. Therefore at 31 March 2014, no amortisation should be recognised. However, an impairment review should be carried out to ensure that its recoverable cost is not less than the carrying amount. The £320,000 cash received before the year end for pre-orders is effectively deposits, and at this date the risks and rewards have not transferred to the customers as the control system technology has not been delivered to them. These amounts should therefore not be recognised as part of revenue, but instead should be held as deferred income as part of current liabilities. (3) Onerous contract The contract with Murano Ltd constitutes an onerous contract at 1 March 2014. IAS 37 Provisions, Contingent Liabilities and Contingent Assets, defines an onerous contract as one in which the unavoidable costs of meeting the obligation under the contract exceeds the economic benefits expected to be received under it. The standard requires that where an onerous contract exists, the present obligation under the contract should be recognised and measured as a provision. Imprezo plc has made the decision to terminate the contract with Murano Ltd before the year end and the unavoidable costs of meeting the obligation is the termination payment of £20,000. No benefit is expected under the contract and therefore a provision should be made at 31 March 2014 of £20,000, with the corresponding amount recognised in profit or loss. (4) Related party Imprezo plc will need to establish whether or not the sale of goods to Samuri Ltd is a related party transaction under IAS 24, Related Party Disclosures. Samuri Ltd is controlled by one of the close members (ie his daughter) of the family of a member of Imprezo plc’s key management personnel, so Samuri Ltd is a related party of Imprezo plc under IAS 24. Therefore, the sale of goods is a related party transaction. Disclosure should include the nature of the related party relationship, ie one of the directors daughter’s owns a majority share in Samuri Ltd, the amount of the transaction, ie £50,000, and whether there are any outstanding balances at the year end, ie £30,000 is outstanding. The rate of the discount and the names of the related parties do not need to be disclosed under IAS 24. (5) Revenue recognition Where a combined package of goods and services is sold, the separate components need to be identified, then measured and recognised separately. Where the total of the individual fair values exceed the combined package price then the discount needs to be applied to each component in an appropriate manner. Where there is no evidence of how the discount should be applied then the same discount should be applied to each component.

Copyright © ICAEW 2014. All rights reserved.

Page 5 of 15

Professional Level – Financial Accounting and Reporting – September 2014

The total package price is £440,000 whereas to acquire the components separately it would have cost £550,000 (£1,000 x 110% x 500 units), so a discount of 20% was given. The two components should therefore be measured at:  

Software module – £1,000 x 500 x 80% = £400,000 Technical support – (£1,000 x 10%) x 500 x 80% = £40,000 (or £400,000 x 10%)

The revenue for the software module should be recognised immediately as the goods have been transferred. However, the technical support is for 24 months and therefore should be recognised on a straight-line basis, assuming no other basis is more appropriate, over the 24 months. Therefore, revenue of £10,000 (£40,000 x 6/24) should be recognised in the year ended 31 March 2014 in relation to the technical support, with the remaining £30,000 being recognised as deferred income. The deferred income should be split between current of £20,000 (£40,000 x 12/24) and non-current of £10,000.

Virtually all candidates addressed all five issues and included narrative explanations as well as relevant calculations. However, explanations were often superficial and/or didn’t use all of the information given in the scenarios and hence candidates missed out on available marks as a result. A minority of candidates incorrectly assumed that giving journal entries is a valid alternative to narrative explanations. Most candidates correctly identified the underlying issues. It was particularly pleasing that nearly all candidates identified the related party transaction as an issue as this was overlooked entirely by many candidates in a previous sitting. The calculations for the development costs to be capitalised and the splitting of revenue for the combined sales package were frequently correct. Most candidates also correctly identified that the irredeemable preference shares should be treated as debt in the scenario due to mandatory, cumulative dividends, that the deposits received in advance should not be recognised in revenue, that the onerous contract should be provided for and that related party disclosures were needed. The most common errors were not time apportioning the preference share dividend, allowing the capitalisation of development costs before the relevant criteria were met and incorrectly apportioning the revenue on the combined package being sold (normally by misunderstanding how to deal with the discount given). Total possible marks Maximum full marks

31 24

(b) Imprezo plc Profit before tax £ As stated (1) Irredeemable preference shares (1) Interest - prefs (2) R&D (350,000+700,000) (2) R&D – launch activities (2) Customer deposits (3) Onerous contract (5) Revenue (440,000 – 410,000) TOTAL

Copyright © ICAEW 2014. All rights reserved.

£ 5,349,000

– (18,750) (1,050,000) (200,000) (320,000) (20,000) (30,000) (1,638,750) 3,710,250

Equity £ 6,547,000 (450,000) – – – – – – (1,638,750) 4,458,250

Liabilities £ 2,986,000 450,000 18,750 – – 320,000 20,000 30,000 – 3,824,750

Page 6 of 15

Professional Level – Financial Accounting and Reporting – September 2014

The majority of candidates made a good attempt at making the relevant adjustments to profit, equity and liabilities. A significant number of candidates achieved at least five and often all six marks available. It should be remembered that this part of the question has an own figure rule and therefore candidates can gain full marks on this part of the question regardless of whether their answers to part (a) were totally correct. The most worrying and common error was failing to adjust equity for the net impact of the adjustments to profit, highlighting that candidates continue not to think through or understand properly the link between the various elements of the financial statements. Total possible marks Maximum full marks

6½ 6

(c) IAS 18 Revenue, accruals accounting and the Framework Revenue is recorded when there is an increase in economic benefits during the period and the amount can be measured reliably in accordance with the IASB Conceptual Framework. The Framework states that an entity should assess the degree of certainty that economic benefits will flow to the entity. Hence revenue can only be recognised when an entity is sufficiently certain that it will be paid for the goods or services and that payment is for a known amount. The accrual basis of accounting is followed with revenue being recognised in the period in which the associated work is undertaken rather than when cash is received to provide a faithful representation in accordance with the Framework. IAS 18 provides additional guidance to assess the timing of when the economic benefits will flow to the entity:    

Has the entity transferred the significant risks and rewards of ownership of the goods to the buyer? Does the seller still have management involvement or effective control over the goods? Can the amount of revenue and costs be measured reliably? Has a price been agreed? Is it probable that the economic benefits associated with the transaction will flow to the entity? Has a payment date been agreed, is the customer likely to pay on time?

When an entity has met all the above conditions it recognises the revenue even though payment may still be outstanding. Consistent with previous sittings answers to the “conceptual” requirement were rather disappointing and were often too brief. However, most candidates attempted to answer this and normally gained at least a couple of marks by discussing the recognition criteria in IAS 18 and/or the key recognition criteria from the Conceptual Framework. Total possible marks Maximum full marks

Copyright © ICAEW 2014. All rights reserved.

6 4

Page 7 of 15

Professional Level – Financial Accounting and Reporting – September 2014

Question 3 Overall marks for this question can be analysed as follows:

Total: 17

General comments This question was a mixed topic question, with part (a) covering property, plant and equipment and part (b) covering the revised preparation of a consolidated statement of profit or loss, along with an explain element in relation to inter-company trading. Vitara plc (i) Consolidated statement of financial position as at 31 March 2014 (extracts) Property, plant and equipment (W4)

1,128,800

Total assets (1,673,500 + 16,500 – 27,500 (W4))

1,662,500

Consolidated statement of cash flows for the year ended 31 March 2014 (extract) Net cash from investing activities (316,700 + 280,000 – 184,000 (W5))

412,700

(ii) Profit after tax £ 496,500 (3,500) (3,375) 2,375 (6,500) 485,500

Draft profit after tax Held for sale asset – impairment (W1) Scrapped plant – write-off Depreciation (8,375 – 6000) Impairment loss

Depreciation charge Depreciation Impaired equipment (W3) Held for sale asset – reverse depreciation (56,000 / 7yrs) Scrapped plant – reverse depreciation (15,000 x 10% x 0.25) Depreciation charge

£ 127,200 6,000 (8,000) (375) (8,375) 124,825

Workings (1) Held for sale asset Carrying amount at 1 April 2013 Fair value less costs to sell (17,000 – 500) Impairment

£ 20,000 (16,500) 3,500

(2) Obsolete plant Cost Acc depreciation (15,000 x 10% x 7.75 years)

Copyright © ICAEW 2014. All rights reserved.

15,000 (11,625) 3,375

Page 8 of 15

Professional Level – Financial Accounting and Reporting – September 2014

(3) Impaired equipment £ 24,500 (18,000) 6,500

Carrying amount Recoverable amount Impairment Depreciation in year (18,000 / 3yrs)

6,000

(4) Property, plant and equipment B/fwd (1) Held for sale asset (20,000 – 8,000) (2) Obsolete plant (3,375 – 375) (3) Impaired equipment

-

1,156,300 (12,000) (3,000) (6,500) (6,000)

depreciation

(27,500) 1,128,800 (5) PPE – cash movement £ B/fwd (1,156,300 – 280,000) Acquisition of subsidiary Plant on credit terms Additions (β)

876,300 151,200 72,000 184,000 1,283,500

£

Depreciation C/fwd

127,200 1,156,300 1,283,500

Most candidates appeared to produce a random set of quite messy and unreferenced workings, rather than a methodical approach which would have gained the most marks. This rather scattergun approach to this part of the question meant that candidates often omitted figures in workings. The impairment for the held for sale asset was calculated correctly by many candidates as was the carrying amount for the obsolete plant. The impairment of the equipment and depreciation were again calculated reasonably well by the majority of candidates. The most common error here was to use fair value less costs to sell as the recoverable amount even though value in use was higher. Most candidates made a reasonable attempt at calculating the depreciation charge, with the most common error being to add rather than subtract the depreciation. In most cases candidates made some attempt at the various calculations but then transferring these calculated figures to the correct totals in the financial statements was less well done. The adjustments which seemed to cause candidates the biggest problem were the adjustments to the statement of cash flows. Only a minority of candidates completed this calculation correctly. It was concerning that so many candidates appeared unfamiliar with double entry principles in respect of this issue. A number of candidates also wasted time explaining the treatment alongside each of their workings. Candidates are reminded to read the requirement carefully and if the “explain” verb is not used, then no such explanation is required or will have marks allocated to it. Total possible marks Maximum full marks

Copyright © ICAEW 2014. All rights reserved.

15 12

Page 9 of 15

Professional Level – Financial Accounting and Reporting – September 2014

(b) (i) Extract from consolidated statement of profit or loss for year ended 31 March 2014 £ Revenue 1,527,600 Cost of sales (846,950) Gross profit 680,650 Workings (1) Consolidation schedule

Revenue

Vitara plc £ 1,395,600

Cost of sales – per Q – PURP (W6)

(793,200) (3,000)

6/12 Tredia Ltd £ 178,000 (96,750)

Adj £ (46,000) 46,000

Consol £ 1,527,600 (846,950)

(2) PURP SP Cost GP 1 X /2

% 115 (100) 15

£ 46,000 (40,000) 6,000 3,000

(ii) Intra-group balances IFRS financial reporting treatment When one company in a group sells goods to another group member an identical amount is added to the revenue of the first company and to the cost of sales of the second. Yet as far as the entity’s dealings with third parties are concerned no sale has taken place. Consolidated financial statements are based on the concept of substance over form which means that although Vitara plc and Tredia Ltd are two separate entities they are instead accounted for as a single entity. Substance over form is implied in faithful representation. The consolidated figures for sales revenue and cost of sales should represent sales to and purchases from third parties. An adjustment is therefore necessary to reduce the sales revenue and cost of sales figure by the value of intra-group sales made during the year. An adjustment is therefore required to deduct the intra-group sales from both consolidated revenue and cost of sales. If any of the inventory remains within a group company at the year end its value must be adjusted to the lower of cost and net realisable value to the group, applying the single entity concept. This is because these items have not been sold outside the group and therefore contain unrealised profit, so this element is removed from closing inventory (ie cost of sales). The most common errors were to either not pro-rata Tredia Ltd’s figures for only six months of ownership or to subtract the PURP figures from cost of sales rather than adding it. The explanation of the treatment of inter-company trading between a parent and subsidiary was disappointing. Almost all candidates understood that inter-company trading should be removed from the consolidated financial statements although far fewer candidates were able to explain why this was the case. This suggests a rote learning approach to their studies with insufficient time allocated to understanding the principles that lie behind the treatment of transactions. Total possible marks Maximum full marks

Copyright © ICAEW 2014. All rights reserved.

9½ 7

Page 10 of 15

Professional Level – Financial Accounting and Reporting – September 2014

Question 4 Overall marks for this question can be analysed as follows:

Total: 27

Part (a) of this question required the preparation of a revised consolidated statement of financial position from draft information. The group had two subsidiaries, one of which was acquired during the year, and an associate. Fair value adjustments were required on acquisition of the associate and one of the subsidiaries. Impairments in all three companies had taken place during the period. Inter-company trading took place during the year between the parent and the associate. Part (b) required a comparison between IFRS and UK GAAP in respect of the calculation of goodwill for the subsidiary acquired using the fair value method for calculating goodwill and NCI, including calculations under UK GAAP. Part (c) required a discussion of the ethical issues arising from the scenario. Altima plc Consolidated statement of financial position as at 31 March 2014 Assets £ Non-current assets Property, plant and equipment (2,140,050 + 496,000) Investment in associate (W5) Intangible (150,000 – 120,000) Goodwill (31,640 + 32,450) (W3 & W4) Current assets Inventories (191,300 + 49,700) Trade and other receivables (86,600 + 56,600) Cash and cash equivalents (55,000 + 5,450)

444,650 3,439,310

Ordinary share capital Share premium account Retained earnings (W6) Attributable to the equity holders of Altima plc Non-controlling interest (W5)

1,500,000 500,000 493,810 2,493,810 335,300 2,829,110 610,200 3,439,310

Total liabilities (556,050 + 54,150) Total equity and liabilities

Share capital Share premium Retained earnings Brands – intangible asset Amortisation (150,000/5 yrs x 4)

2,636,050 264,520 30,000 64,090 2,994,660

241,000 143,200 60,450

Total assets Equity and liabilities Equity

Workings (1) Net assets – Fuego Ltd

£

Year end £ 420,000 160,000 371,750 150,000 (120,000) 981,750

Acquisition £ 420,000 160,000 236,700 150,000 – 966,700

Post acq £

Year end £ 400,000 50,000 103,600 553,600

Acquisition £ 400,000 50,000 126,800 576,800

Post acq £

15,050

(2) Net assets – Tacoma Ltd

Share capital Share premium Retained earnings

Copyright © ICAEW 2014. All rights reserved.

(23,200)

Page 11 of 15

Professional Level – Financial Accounting and Reporting – September 2014

(3) Goodwill – Fuego Ltd £ 820,000 193,340 (966,700) 46,640 (15,000) 31,640

Consideration transferred Non-controlling interest at acquisition (966,700 x 20%) Less: Net assets at acquisition (W1) Impairment (4) Goodwill – Tacoma Ltd

£ 480,250 150,000 (576,800) 53,450 (21,000) 32,450

Consideration transferred Non-controlling interest at acquisition at fair value Less: Net assets at acquisition (W2) Impairment (5) Investment in associate – Previa Ltd

£ 283,500

Cost of investment Share of post acquisition increase in net assets ((96,900 – 67,000) x 40%) Share of additional depreciation on FV uplift ((30,000 / 6yrs x 2.75 yrs) x 40%) PURP (W7) Less: Impairment

11,960 (5,500) (1,440) (24,000) 264,520

(6) Non-controlling interest £ Tacoma Ltd At acquisition Share of post-acquisition reserves ((23,200) (W2) x 25%) Impairment (21,000 x 25%) Fuego Ltd At acquisition Share of post-acquisition reserves (15,050 (W1) x 20%)

£

150,000 (5,800) (5,250) 138,950 193,340 3,010 196,350 335,300

(7) PURP Selling price Cost Gross profit

% 100 (85) 15

Sales in year £ 24,000 (20,400) 3,600

Previa Ltd - £3,600 x 40% = £1,440 (8) Retained earnings Altima plc Tacoma Ltd (75% x (23,200) (W2)) Fuego Ltd (80% x (15,050 (W1)) Previa Ltd (W5) PURP – Previa Ltd (W7) FV depreciation – Previa Ltd (W5) Impairment – Previa Ltd (W5) Impairment – Tacoma Ltd (21,000 x 75%) Impairment – Fuego Ltd

Copyright © ICAEW 2014. All rights reserved.

£ 548,900 (17,400) 12,040 11,960 (1,440) (5,500) (24,000) (15,750) (15,000) 493,810

Page 12 of 15

Professional Level – Financial Accounting and Reporting – September 2014

Candidates’ performance on this question was again, excellent, and they were clearly well-prepared. Almost all produced the “standard” workings for net assets, goodwill, non-controlling interest, investment in associate and retained earnings. Presentation of the consolidated statement of financial position was generally good, although there were some messy attempts. Almost all candidates included a figure for non-controlling interest, for which they were rewarded. The most common omission from the more straightforward figures was the figure for total liabilities. Of the other figures, many who adjusted for the brand in the net assets working for Fuego Ltd failed to include the closing balance of that brand in intangibles on the consolidated statement of financial position. Most candidates included a figure for goodwill, but some lost marks where there was no audit trail showing clearly that this was the sum of their two goodwill calculations for the two subsidiaries. In the two sets of net assets workings, almost all candidates dealt correctly with share capital, share premium and retained earnings. This meant that the figures for Tacoma Ltd were generally correct, and most candidates then went on to calculate goodwill for this subsidiary correctly, dealing correctly with noncontrolling interest on the fair value basis. Some, however, then fell down in the retained earnings working, failing to show the downwards movement on post-acquisition profits as a debit, as opposed to a credit (with the same error made in the non-controlling interest working). In the net assets working for Fuego Ltd, a good number of candidates dealt correctly with the fair value adjustment in respect of the brand. Where mistakes were made on this they were generally making the adjustment(s) in the wrong direction, adjusting only for the fair value of the brand at acquisition, but not for the subsequent amortisation, or miscalculating the subsequent amortisation. Most candidates then went on from this to produce correct (own) figures for goodwill and retained earnings. Although most candidates were able to deal with calculating gross goodwill on both a fair value and a proportionate basis, a few allocated only the group share of the impairment on the fair value basis. Most candidates arrived at the correct total provision for unrealised profit, but many then failed to account for only the group share (40%) of that figure. Others credited their figure to inventories instead of to the investment in associate figure. The depreciation adjustment seemed to cause the most problems with few candidates arriving at the correct figure and many adjusting for the increase in fair value on the plant itself instead of just for the additional depreciation. Of those who attempted this figure, the most common errors were to adjust for only one year of depreciation, instead of two years and nine months, and/or to account for the whole figure instead of only the group share.

Total possible marks Maximum full marks

20½ 18

(b) UK GAAP treatment UK GAAP is more restrictive than IFRS in respect of the calculation of goodwill and does not permit a choice to be made. Under UK GAAP, the non-controlling interest, which is known as the minority interest, is always calculated using the share of net assets (ie the proportionate basis). Goodwill calculated using the proportionate basis is usually lower than that under the fair value method. Goodwill – Tacoma Ltd Consideration transferred Minority interest at acquisition (576,800 x 25%) Less: Net assets at acquisition (W2) Impairment

Copyright © ICAEW 2014. All rights reserved.

£ 480,250 144,200 (576,800) 47,650 (21,000) 26,650

Page 13 of 15

Professional Level – Financial Accounting and Reporting – September 2014

Under UK GAAP there would be a decrease in consolidated non-current assets, representing goodwill, of £5,800 (£32,450 – £26,650). Goodwill should be amortised over its estimated useful economic life under UK GAAP and there is a rebuttable presumption that this is not more than 20 years. Although annual impairment reviews are not required under UK GAAP if an impairment was identified this would be recognised as above. As the proportionate method is applied under UK GAAP none of the impairment is allocated to the noncontrolling interest (minority interest). Hence, reporting under UK GAAP will also affect the non-controlling interest and the retained earnings as reported in the consolidated financial statements of Altima plc. £ Tacoma Ltd At acquisition (576,800 x 25%) Share of post-acquisition reserves ((23,200) (W2) x 25%) Fuego Ltd Minority interest Retained earnings (per part (a)) Additional impairment re Tacoma Ltd (21,000 x 25%)

£

144,200 (5,800) 138,400 196,350 334,750 493,810 (5,250) 488,560

Nearly all candidates stated that only the proportionate method is available under UK GAAP and most also attempted to recalculate the goodwill figure. The majority also identified that goodwill would be amortised although a significant number of candidates appear to believe that this should always be over twenty years. Only the stronger candidates understood that impairment reviews might still be needed and could clearly explain the impact of this on retained earnings/ NCI when the proportionate rather than fair value method is used. Many candidates wasted time by writing out other differences that were not relevant in this particular scenario, such as the treatment of any “negative” goodwill arising and of acquisition costs.

Total possible marks Maximum full marks

8½ 5

(c) Chartered accountants must always abide by the spirit of the five fundamental ethical principles. One of these is professional competence and due care. The professional competence of the interim manager should be questioned. He has a responsibility to maintain his continuing professional development to the appropriate level required for his current position. For the interim manager this will include keeping his technical knowledge and skills completely up to date as he is accepting contracts which require him to perform the preparation of consolidated financial statements and therefore his skills in this area should be exemplary. If the interim manager’s technical knowledge and skills are lacking in the area of financial statement preparation it is likely that his general accounting ability should be questioned. This would include whether or not he is capable of carrying out an impairment review in a competent manner.

Copyright © ICAEW 2014. All rights reserved.

Page 14 of 15

Professional Level – Financial Accounting and Reporting – September 2014

However, assuming that there is doubt over the carrying amounts of the three investments, Ciera should carry out her own impairment review to confirm or otherwise the valuations. As an ICAEW Chartered Accountant Ciera needs to ensure that she acts with integrity, demonstrating high standards of both professional behaviour and conduct. There is a self-interest threat here as Ciera’s position in Altima plc may be under threat because impairments appear to have arisen on acquisitions in which she was involved. However, her judgement should not be influenced by the fact that her competence may be questioned if large impairments arise from investment decisions she was involved in, remembering that another of the five fundamental principles is professional behaviour. As in previous sittings, many candidates produced a “stock” answer rather than referring specifically to the scenario. Candidates must look at who they are and what their position is. In this question there was no management pressure and therefore discussions with the ICAEW Ethics Helpline was not seen as appropriate. However, most candidates made a reasonable attempt at this part of the question, with a good number obtaining half marks. Total possible marks Maximum full marks

Copyright © ICAEW 2014. All rights reserved.

6½ 4

Page 15 of 15

Financial Accounting and Reporting – Professional Level – December 2014

MARK PLAN AND EXAMINER’S COMMENTARY The marking plan set out below was that used to mark this question. Markers were encouraged to use discretion and to award partial marks where a point was either not explained fully or made by implication. More marks were available than could be awarded for each requirement. This allowed credit to be given for a variety of valid points which were made by candidates.

Question 1 Total Marks: 27 General comments Part (a) of this question tested the preparation of a statement of profit or loss and a statement of financial position from a set of draft financial statements plus a number of adjustments. Adjustments included borrowing costs and depreciation on property, plant and equipment, a bonus issue of ordinary shares, the issue of redeemable preference shares and dividends on both types of shares. Part (b) required an explanation of the treatment of redeemable preference shares. Part (c) tested the differences between IFRS and UK GAAP in respect of the treatment of borrowing costs. Trakehner Ltd (a) Financial statements Statement of profit or loss for the year ended 30 June 2014 £ 3,879,600 (2,122,025) 1,757,575 (919,200) (387,900) 450,475 (11,200) 439,275 (97,000) 342,275

Revenue Cost of sales (W1) Gross profit Administrative expenses (W1) Distribution costs (W1) Profit from operations Finance cost (W4) Profit before tax Income tax expense (120,000 – (143,000 – 120,000)) Profit for the year Statement of financial position as at 30 June 2014 £ Assets Non-current assets Property, plant and equipment (W2) Current assets Inventories Trade and other receivables (241,200 + 10,500) Cash and cash equivalents

1,861,275 453,700 251,700 14,800 720,200 2,581,475

Total assets Equity and liabilities Equity (W3) Ordinary share capital Retained earnings Non-current liabilities Preference share capital (5% redeemable) (211,200 (W4) – 10,000) Borrowings

£

1,000,000 677,575 1,677,575 201,200 250,000 451,200

Copyright © ICAEW 2015. All rights reserved

Page 1 of 16

Financial Accounting and Reporting – Professional Level – December 2014 Current liabilities Trade and other payables (302,600 + 12,600) Loan interest (W2) Preference dividend (200,000 x 5%) Taxation

315,200 7,500 10,000 120,000 452,700 2,581,475

Total equity and liabilities Workings (1) Costs matrix Cost of sales

Per draft Depreciation (W2) Ordinary dividend (1,000,000 (W3) x 10p) Accrual and prepayment

£ 2,015,300 106,725

2,122,025

Admin expenses £ 987,600 19,000 (100,000) 12,600 919,200

Distribution costs £ 398,400

(10,500) 387,900

(2) PPE £ 1,982,500 4,500 (19,000) (106,725) 1,861,275

Carrying amount per draft Loan interest ((250,000 x 4% x 9/12) – 3,000) Depreciation on other property (950,000/50) Depreciation on plant and equipment (426,900 x 25%)

(3) Equity Ordinary share capital £ 800,000 200,000 – –

Per draft Bonus issue (800,000 ÷ 4) Ordinary dividend (W1) Decrease in profit for the year (361,300 – 342,275)

Share premium £ 125,000 (125,000) – – –

1,000,000

(4) Redeemable preference shares Opening balance Year 30 June 2014

£ 200,000

Copyright © ICAEW 2015. All rights reserved

Interest expense (5.6%) £ 11,200

Interest paid (5%) £ Nil

Retained earnings £ 871,600 (75,000) (100,000) (19,025) 677,575

Closing balance £ 211,200

Page 2 of 16

Financial Accounting and Reporting – Professional Level – December 2014 Generally candidates made a good attempt at this part of the question and nearly all produced complete statements of financial position and profit or loss with many gaining the full marks available for presentation. The bonus issue, calculation of current year depreciation charges and adjustments for prepayments and accruals were almost always dealt with correctly. Most candidates also recognised that the redeemable preference shares should be treated as a liability rather than equity. The capitalisation of interest appeared to cause the most problems. Although most candidates recognised that interest on a qualifying asset should be capitalised many struggled with the calculation. The most common mistakes were basing the amount on the costs incurred rather than the amount borrowed, using the wrong number of months in the calculation, and not netting off the interest received. A number of candidates also depreciated the new asset even though it had not yet been completed. It was also worrying to see a lack of understanding regarding the double entry treatment of interest with a significant number of candidates both capitalising and expensing the figure calculated. As always with property, plant and equipment it was often difficult to find an “audit trail” supporting the final figure taken to the statement of financial position. In contrast, most candidates did use the recommended “costs matrix” when allocating costs for the statement of profit or loss, and entered the adjustments into the correct columns. Occasionally errors were made in terms of whether the adjustment was increasing or decreasing costs particularly with regard to the dividend incorrectly posted to administrative expenses and a minority of candidates posted the accrual and/or prepayment in the wrong (sometimes the same) direction(s). Other common errors included the following:  Adding, rather than deducting, the prior year over provision of income tax to the current year charge (or making no adjustment for it at all).  Using the same income tax figure in both the statement of profit or loss and the statement of financial position (thereby omitting to complete the correct double entry).  Treating the redeemable preference shares as a compound financial instrument (and wasting significant time by discounting future cash flows to calculate the “liability” element).  Failing to realise that the interest on the redeemable preference shares was unpaid at the year end.  Splitting the loan into current and non-current components.  Adding the revised profit for the year to retained earnings but failing to deduct the original profit already included. Total possible marks Maximum full marks

Copyright © ICAEW 2015. All rights reserved

23½ 22

Page 3 of 16

Financial Accounting and Reporting – Professional Level – December 2014 (b) Financial reporting treatment of redeemable preference shares Preference shares give the holder the right to receive an annual dividend (ie mandatory) (usually at a fixed rate), which may be also be cumulative, out of the profits of a company, together with a fixed amount on the ultimate liquidation of the company or at an earlier date if the shares are redeemable. Legally, preference shares are equity. However, IAS 32 treats most preference shares as liabilities. This is because they are, in substance, loans and meet the definition of a liability as there is a present obligation, in the form of both preference dividends and redemption payments, which will lead to a future outflow. The liability is measured at amortised cost using the effective interest rate, so that the premium on redemption is effectively treated as part of the interest expense. The interest is treated as a finance cost in the statement of profit or loss, rather than as a distribution out of retained earnings. Again, this was well answered with most candidates discussing substance over form and explaining why redeemable preference shares should be treated as a liability. Almost all candidates also followed this through by explaining that the resulting “dividend” should be treated as a finance cost. Fewer candidates discussed the use of amortised cost and effective interest rate. Total possible marks Maximum full marks

6½ 3

(c) UK GAAP differences re borrowing costs Under UK GAAP (FRS 15) there is a choice as to whether to capitalise borrowing costs or to recognise them as an expense when incurred. Under IFRS (IAS 23) capitalisation is mandatory. Under UK GAAP the amount capitalised is limited to the finance costs on the expenditure incurred. Under IFRS the amount capitalised is limited to the borrowing costs on the total related funds less the investment income from any temporary investment of those funds. This part was also well answered with a significant number of candidates achieving full marks and nearly all candidates as a minimum flagging up the difference in respect of optional versus mandatory capitalisation of interest costs. However, a few candidates lost marks by being imprecise in their wording – for example saying that under IFRS companies “can” as opposed to “should” capitalise interest, thereby losing half a mark. Other answers failed to make it clear that it is surplus investment income on these particular borrowings which should be offset under IFRS, as opposed to any investment income. Total possible marks Maximum full marks

Copyright © ICAEW 2015. All rights reserved

2 2

Page 4 of 16

Financial Accounting and Reporting – Professional Level – December 2014

Question 2 Total Marks: 30 General comments Part (a) of this question required candidates to explain the financial reporting treatment of four accounting issues, given in the scenario. The issues covered a change in depreciation method of equipment, a government grant, a lease and a potential held for sale asset. Part (b) required the calculation of revised figures for profit before tax, total assets and total liabilities. Part (c) required an explanation of the ethical issues arising from the scenario and the action to be taken.

Holstein Ltd (a) IFRS accounting treatment (1) Change of depreciation method IAS 16, Property, Plant and Equipment, requires companies to reassess the accounting estimates used to calculate depreciation each year. If the reducing balance method is a better reflection of the pattern of consumption of economic benefits then it is correct to change to this method. Ryan is correct that a change of accounting policy is dealt with by making a retrospective adjustment to opening figures. However, per IAS 16, a change to the depreciation method is a change in an accounting estimate, not a change of accounting policy. Changes in accounting estimates are dealt with, per IAS 8 prospectively, not retrospectively, by depreciating the carrying amount of the asset at the date of the change under the new method. Therefore the adjustment of £352,100 must be reversed out, reducing the opening balances of both property, plant and equipment and retained earnings. Ryan must have charged depreciation of 25% on this wrongly inflated carrying amount. Hence, depreciation for the year ended 30 June 2014 is overstated by £88,025 (352,100 x 25%). Property, plant and equipment is therefore understated by the same amount. Overall there is a net overstatement of property, plant and equipment of £264,075 ((352,100 – 88,025) or (352,100 x 75%)). (2) Government grant Per IAS 20, Accounting for Government Grants and Disclosure of Government Assistance, grants should be recognised when there is reasonable assurance that:  the entity will comply with the relevant conditions, and  the entity will receive the grant. Ryan does not expect to have to repay the grant and the grant has been received. Both of these conditions therefore appear to have been met so it is appropriate to recognise the grant. IAS 20 requires grants to be recognised in profit or loss over the periods in which the entity recognises the expenses which the grants are intended to compensate. It is against the accruals principle to recognise a grant on a cash receipts basis, which is what has been done here. Holsten Ltd’s stated accounting policy for government grants is to use the netting-off method. Under this method the grant is deducted from the carrying amount of the related asset. The grant will then be recognised over the life of the related asset by way of a reduced depreciation charge. The cost of the asset will therefore be stated at £200,000 (or for saying Cr £200,000 to PPE) (400,000 – 200,000), with accumulated depreciation of £37,500 (200,000 x 9/48). The carrying amount of the asset at 30 June 2104 is therefore £162,500 (200,000 – 37,500). Because Ryan has already charged depreciation of £75,000 (400,000 x 9/48) and credited the statement of profit or loss with income of £200,000 ie a net credit of £125,000, profit before tax needs to be reduced (Dr) by £162,500 (125,000 + 37,500). The corresponding Cr will reduce total assets in the statement of financial position.

Copyright © ICAEW 2015. All rights reserved

Page 5 of 16

Financial Accounting and Reporting – Professional Level – December 2014

(3) Operating lease Per IAS 17, Leases, this is an operating lease because the risk and rewards of ownership have not passed to the lessee (eg maintenance/insurance, use of the asset over the majority of its useful life, present value of minimum lease payments below fair value of £350,000) The lease payments of £180,000 (3 x £60,000) should be charged on a straight-line basis over the four year lease term, even if the payments are not made on such a basis. This is in accordance with the accruals principle. Hence, £45,000 (£180,000 x 25%) should be charged to the statement of profit or loss in the year ended 30 June 2014. An accrual of £45,000 will be included within current liabilities. (4) Asset held for sale IFRS 5, Non-current Assets Held for Sale and Discontinued Operations, states that an asset should be classified as held for sale when there is intention to recover the carrying amount through resale. However, there are strict criteria which must be satisfied before the IFRS rules can be applied. Although the decision by the board shows intent, the asset was not immediately available for resale as the reconditioning work could not be carried out until August. Also the fact that the machine was advertised at a price significantly above the final sale price means that the sale was not “highly probable”. Therefore the held for sale criteria were not met at the year end and the asset should be removed from this classification. The increase in value of (300,000 – 155,000) £145,000 should be removed from total assets and the revaluation surplus, taking the carrying amount back to £155,000, which correctly includes depreciation to 30 June 2014 (or continue to depreciate). However, as the plant is “surplus to requirements” this is an indication that an impairment review is required under IAS 36, Impairment of Assets. The carrying amount of £155,000 is then compared with the recoverable amount, being the higher of fair value less costs to sell and value in use. As the asset has now been sold/is surplus to requirements its value in use, ignoring discounting, will equal fair value less costs to sell so this figure should be used. This is £94,900 (170,000 – 11,600 – 63,500). Therefore an impairment of £60,100 (155,000 – 94,900) should be recognised in the statement of profit or loss. Even if the held for sale criteria had been met, as Holstein Ltd uses the cost model and not the revaluation model, the asset would not be revalued to fair value immediately before the classification – it would be left at its carrying amount, or written down to fair value less costs to sell, if lower. Answers to Part (a) of this question were very disappointing, although the majority of candidates did attempt all four issues and provide both explanations and supporting calculations. The majority of candidates did not achieve a pass mark on this question. Issue (1): This was probably the worst answered part of the question with many candidates believing that the change in the basis of calculating depreciation constituted a change in accounting policy. As a result they thought the current accounting treatment correct then wasted time writing out at length when accounting policies can be changed and what the disclosure requirements are. Other candidates seemed to think that the company was moving to a revaluation basis. Even those candidates who did recognise this was a change in an accounting estimate could rarely say more than that it should be applied prospectively. However, even those candidates who incorrectly believed this was a change in policy often managed to pick some marks up by stating that the adjustment to opening balances should be reversed out and often managed to calculate the correct adjustment to depreciation. Issue (2): This was better dealt with and many candidates correctly calculated the adjustments required with regard to the government grant (although it was common to see depreciation calculated for an incorrect number of months). However, a significant number of candidates wasted time by discussing at length the two different methods allowed for the treatment of capital grants even though the question clearly stated that the netting off method was to be used. Some then went on to produce the figures under both methods. Almost all candidates recognised that the grant should be reversed out of other income (although some seemed to think that if the deferred income approach had been used it would have been acceptable to recognise the grant in full immediately).

Copyright © ICAEW 2015. All rights reserved

Page 6 of 16

Financial Accounting and Reporting – Professional Level – December 2014

Issue (3): This was the best answered part of this question. Virtually all candidates referred to the relevant information in the question that indicated this was an operating lease. Most also understood that the total cost needed to be spread over the life of the lease although some referred to this but then went on to state that the current year cost should simply be the payment to be made next year. Whilst almost all candidates specifically stated that this was a cost to be recognised in the statement of profit or loss fewer discussed the credit side of the entry and the need for an accrual. A significant minority of candidates again showed a worrying lack of understanding of double entry by recognising the correct expense but then showing as a liability the total outstanding payments. Others also incorrectly described the expense as a finance cost. Only a very small minority believed the lease to be a finance lease. Issue (4): This was also badly answered. Many candidates wasted time by listing out all the criteria to determine if an asset should be treated as held for sale, rather than using the information in the question to demonstrate which of the criteria had not been met. Many candidates did not see the relevance of the reconditioning expenses incurred after the year end to the decision as to whether the asset should be classified as held for sale and as a result concluded that the asset had correctly been classified as held for sale. As a result they did not gain the total marks available for discussing the need for an impairment review on the grounds that the asset had become surplus to requirements, as opposed to on the grounds of it being a held for sale asset. However, even these candidates usually recognised that the revaluation was inappropriate and that the entry in the revaluation surplus needed to be reversed out (although fewer justified why this was). Total possible marks Maximum full marks

31 21

(b) Revised figures

Per draft (1) Change of depreciation method – reverse prior period adjustment – adj to annual deprec charge (2) Government grant (3) Operating lease (4) Asset held for sale – reverse revaluation – impairment

Profit/(loss) before tax £ 135,400

88,025 (162,500) (45,000) (60,100) (44,175)

Total assets £ 1,456,000

Total liabilities £ 874,300

(352,100) 88,025 (162,500) 45,000 (145,000) (60,100) 824,325

919,300

Again, answers to this part were not as good as usual and there was less evidence of candidates setting up the adjustment working up front and entering the figures as they worked through Part (a) of the question. It was often difficult to follow figures from Part (a) to Part (b) and/or adjustments referred to in Part (a) were simply not transferred to the adjustments table. It was also clear that many candidates struggled to understand which adjustments would impact on, for example, both profit and total assets or just profit. Total possible marks Maximum full marks

Copyright © ICAEW 2015. All rights reserved

5 4

Page 7 of 16

Financial Accounting and Reporting – Professional Level – December 2014

(c) Ethical issues Ryan has given reasons for the accounting treatment he has adopted for some of the issues identified. However, although some of these explanations may appear reasonable to a non-accountant, they are incorrect and Ryan, as an ACA, should be aware of this. It therefore seems that either Ryan has not been keeping himself technically up-to-date (which is a requirement of his membership of ICAEW) or he has deliberately misstated these items, possibly so that Holstein Ltd still appears to meet the conditions of its loan, and/or as Ryan holds a significant percentage of shares in the company, so has a vested (ie self-interest) in Holstein Ltd’s profitability. Prior to the adjustments which are needed, assets were 166% of liabilities, so above the required 150%. After the adjustments assets are only 90% of liabilities, which would mean that the bank is likely to call in its loan. This adds weight to the possibility that Ryan has deliberately not followed the correct IFRS financial reporting treatment so as to keep assets above the 150%. IFRS is quite clear on the appropriate treatment of these four issues. Other than the presentational choice with regard to the government grant, there is no choice or judgement on any of the matters. I should not allow myself to be associated with financial statements that are contrary to IFRS. There may also be an intimidation threat since Ryan is my superior and a significant shareholder in the company. I should apply the ICAEW Code of Ethics, with the following programme of actions:  Explain matters to Ryan, with supporting evidence so that the matters can be corroborated.  If resolution cannot be achieved, discuss the matters with the other directors to explain the situation and obtain support.  Obtain advice from the ICAEW helpline or local members responsible for ethics. During the resolution process it would be useful to keep a written record of all discussions, who else was involved and the decisions made. Most candidates picked up a good number of the available marks for this part recognising the self- interest threat to Ryan arising from his significant shareholding in the company and the loan covenant (with a pleasing number attempting to illustrate the impact of the errors made on the requirement to maintain total assets at a minimum of 150% of total liabilities). Fewer picked up on the intimidation threat to the financial controller. Virtually all candidates suggested discussing the issues with Ryan, other directors and the ICAEW helpline. Sometimes suggestions were a little inappropriate such as demanding that Ryan go on a professional update course. As always there were a small minority of candidates who answered the question from the perspective of the external auditors and/or who thought that money laundering was the main issue. Total possible marks Maximum full marks

Copyright © ICAEW 2015. All rights reserved

11½ 5

Page 8 of 16

Financial Accounting and Reporting – Professional Level – December 2014

Question 3 Total Marks: 21 General comments Part (a) of this question required the calculation of the profit on disposal of a subsidiary. Part (b) tested the preparation of a consolidated statement of cash flows and supporting note, including the subsidiary disposed of during the year. Missing figures to be calculated included the profit before tax of the subsidiary, dividends paid (to the group and to the non-controlling interest), finance lease liabilities paid, income tax paid, additions to property, plant and equipment, and proceeds from the issue of share capital. Part (c) required consideration of the different users of the financial statements and the type of decisions they make. Appaloosa plc (a) Profit on disposal of Connemara Ltd £ Selling price Less: Carrying amount of good will at date of disposal Consideration transferred NCI at acquisition ((100,000 + 226,000) x 30%) Less: Net assets at acquisition (100,000 + 226,000)) Goodwill at acquisition Less: Impairment to date Less: Carrying amount of net assets at date of disposal Add back: NCI in net assets at date of disposal (734,200 x 30%)

£ 590,000

350,000 97,800 (326,000) 121,800 (50,000) (71,800) (734,200) 220,260 4,260

A significant number of candidates calculated this figure correctly. Others arrived at the correct figure for goodwill, but made errors in the remainder of the calculation. The most common errors were using incorrect figures for the net assets disposed of and/or acquired. Total possible marks Maximum full marks

Copyright © ICAEW 2015. All rights reserved

3½ 2

Page 9 of 16

Financial Accounting and Reporting – Professional Level – December 2014

(b) Consolidated statement of cash flows for the year ended 30 June 2014 £ £ Cash flows from operating activities Cash generated from operations (Note) 1,535,240 Interest paid (51,300) Income tax paid (W3) (362,600) Net cash from operating activities Cash flows from investing activities 1,121,340 Purchase of property, plant and equipment (W4) (1,168,500) Disposal of Connemara Ltd net of cash disposed of 576,200 (590,000 – 13,800) Net cash used in investing activities (592,300) Cash flows from financing activities Proceeds from share issues (W6) 160,000 Repayment of finance lease liabilities (W2) (467,800) Dividends paid (W7) (100,300) Dividends paid to non-controlling interest (W8) (72,940) Net cash used in financing activities (481,040) Net increase in cash and cash equivalents 48,000 Cash and cash equivalents at beginning of period 53,500 Cash and cash equivalents at end of period 101,500 Note: Reconciliation of profit before tax to cash generated from operations Profit before tax (1,538,300 + 92,840 (W1)) Finance cost Depreciation charge Increase in inventories (1,785,900 – 1,025,100) Increase in trade and other receivables ((725,200 + 57,900) – 699,800) Increase in trade and other payables ((582,500 + 42,700) – 489,800) Cash generated from operations

£ 1,631,140 51,300 561,500 (760,800) (83,300) 135,400 1,535,240

Workings (1) Profit before tax of subsidiary Profit from discontinued operations per Q Add back: Income tax expense Less: Profit on disposal (a)

Copyright © ICAEW 2015. All rights reserved

£ 77,500 19,600 (4,260) 92,840

Page 10 of 16

Financial Accounting and Reporting – Professional Level – December 2014

(2) Finance lease liabilities Cash (β) C/d (350,200 +150,200)

£ 467,800 500,400 968,200

B/d (270,000 +148,200) PPE

£ 418,200 550,000 968,200

£ 362,600 420,000 782,600

B/d CP&L (385,000 + 19,600)

£ 378,000 404,600 782,600

(3) Income tax Cash (β) C/d (4) Non-current assets B/d Revaluation (W5) Finance leases Additions (β)

£ 2,478,000 356,500 550,000 1,168,500 4,553,000

Disposal of sub – PPE Depreciation charge Disposal of sub – GW (W1) C/d

£ 705,200 561,500 71,800 3,214,500 4,553,000

(5) Revaluation surplus £ C/d

779,500 779,500

B/d PPE (β)

£ 423,000 356,500 779,500

B/d (400,000 + 40,000) Cash received (β)

£ 440,000 160,000 600,000

(6) Share capital and premium £ C/d (500,000 + 100,000)

600,000 600,000

(7) Retained earnings Dividends in SCE (β) C/d

£ 100,300 2,279,800 2,380,100

B/d CP&L

£ 1,364,800 1,015,300 2,380,100

(8) Non-controlling interest Cash (β) Disposal (734,200 x 30%) C/d

Copyright © ICAEW 2015. All rights reserved

72,940 220,260 664,900 958,100

B/d CP&L

£ 742,600 215,500 958,100

Page 11 of 16

Financial Accounting and Reporting – Professional Level – December 2014 Candidates performed slightly better than they did last time the preparation of a consolidated statement of cash flows was examined (also featuring the disposal of a subsidiary). Although the disposal element of the question would be expected to cause some problems, at this sitting candidates seemed to struggle with even the basics such as arriving at figures for tax paid and interest paid, calculations which are tested at Certificate Level. Many candidates displayed a poor grasp of the fundamentals of double-entry bookkeeping when calculating individual cash flow figures. Those candidates who did not use a T-account approach tended to produce confusing and less structured workings, which had a detrimental impact on the marks earned. Most candidates set out the statement in a reasonably clear way and therefore gained presentation marks. However, a number of candidates lost marks for not providing sub-totals for the different categories of cash flows. Most candidates made a good attempt at the reconciliation of profit before tax to cash generated from operations. The majority of candidates gained over half marks on this with the most common error being not to add in the profit before tax for the discontinued operation. Other common errors were to not make adjustments for the discontinued operation in the movement in trade receivables and payables. Treatment of the disposal of the subsidiary was mixed, with weaker candidates either omitting the impact of the disposal or adjusting for it in the incorrect direction. Only the very best candidates calculated the profit before tax of the subsidiary then used this figure in their reconciliation note, although some others adjusted for the profits for discontinued operations per the question and/or their profit on disposal from Part (a). The proceeds from the share issue and the net cash impact of the disposal were almost always correctly calculated and a significant majority also correctly calculated the dividend paid by Appaloosa plc. Generally, candidates made a reasonable attempt at the property, plant and equipment T-account and the dividend paid to the non-controlling interest There was no specific recurring error in the property, plant and equipment T-account; it was more that candidates missed one (or more) of the figures. In the noncontrolling interest T-account candidates generally missed the disposal figure. The finance lease liability calculation seemed to cause candidates the most problems (other than adjusting for the disposal of the subsidiary). Total possible marks Maximum full marks

Copyright © ICAEW 2015. All rights reserved

16 14

Page 12 of 16

Financial Accounting and Reporting – Professional Level – December 2014

(c) User groups and the decisions they need to make Present and potential investors - Likely risk and return of investment/potential investment - Ability of entity to pay dividends Employees - Employer’s stability and profitability - Ability of employer to provide remuneration/employment opportunities/retirement and other benefits Lenders - Whether loans and interest can be repaid when due Suppliers and other trade payables - Likelihood of being paid when due Customers - Whether the entity will continue in existence Governments and trade agencies - How to allocate central resources - How best to regulate activities - Taxation due - Basis for national statistics The public - Trends and recent developments in prosperity/activities - Likely impact on local economy Whilst most candidates came up with five user groups, some of them were too similar to warrant separate marks (for example, existing and potential investors were marked as one user group, as were directors and management) and the information given re the decisions these groups might make were too often extremely brief, consisting of two or three words. Other candidates cited decisions which were not likely to be made from the published financial statements (for example, lending banks would be unlikely to be interested in historic, as opposed to prospective, cash flows). Frequently, candidates could have chosen better user groups, in order to allow them to write more about the decisions of those groups. For example, whilst management could be considered a user group it is difficult to see what information they would usefully gain from the financial statements to make decisions when they have full access to management accounts which are already tailored to their needs. Nonetheless the mark plan was flexible, and if sensible comments were made, marks were awarded. Total possible marks Maximum full marks

Copyright © ICAEW 2015. All rights reserved

7½ 5

Page 13 of 16

Financial Accounting and Reporting – Professional Level – December 2014

Question 4 Total Marks: 22 General comments Part (a) of this question required the calculation of a revised gain on bargain purchase where errors had been made in the original calculation. Part (b) required the preparation of a consolidated statement of profit or loss. The group had two subsidiaries, one of which was acquired during the year, and an associate. The question featured fair value adjustments, including some to be made to the gain on bargain purchase, inter-company trading and impairment of goodwill. Part (c) tested the differences between IFRS and UK GAAP with respect to the financial reporting treatment followed in Parts (a) and (b). Oldenburg plc (a) Revised gain on bargain purchase Gain on bargain purchase per Q Add: Professional fees wrongly included in consideration FV adjustment to building (W1) Less: Contingent liability Less:

Adj to NCI (W2)

£ 35,000 8,000 22,000 (36,500) (6,500) (6,250) 22,250

Workings (1) Fair value adjustment to building Fair value on 1 October 2013 Carrying amount at 1 October 2013 (300,000 – ((300,000) ÷ 25) x 14)

£ 154,000 (132,000) 22,000

(2) Adjustment to NCI Original NCI on proportionate basis (500,000 + 35,000) x 20/80)) NCI at FV

£ 133,750 (140,000) (6,250)

This part of the question caused a significant amount of confusion. However, a number of candidates presented clear answers to this part and gained full marks. Candidates seemed to struggle with the concept that they had to unpick the accounting that had taken place. They often presented a random set of calculations which mirrored their thought processes but never arrived at a final figure. For example, candidates often knew that they had to adjust for the professional fees but didn’t know whether they should add or subtract those fees. The calculation could have been attempted in two ways; either by adjusting the calculated figure or starting again, and both approaches were marked in a consistent manner. However, a significant number of candidates used a combination of both approaches and therefore often double counted their adjustments. Candidates generally adjusted for the contingent liability and the fair value adjustment although where these adjustments were made was less clear. The adjustment to the non-controlling interest was often simply not calculated. Many correctly calculated the fair value adjustment to the building but then failed to use that figure. Others also calculated the related depreciation adjustment in this part but then failed to use it in Part (b). Where this was the case later credit was given for that calculation. Total possible marks Maximum full marks

Copyright © ICAEW 2015. All rights reserved

5½ 5

Page 14 of 16

Financial Accounting and Reporting – Professional Level – December 2014 (b) Consolidated statement of profit or loss for the year ended 30 June 2014 £ 5,434,000 (3,671,850) 1,762,150 (1,135,350) 626,800 9,804 636,604 (190,200) 446,404

Revenue (W1) Cost of sales (W1) Gross profit Operating expenses (W1) Profit from operations Share of profit of associate (W4) Profit before tax Income tax expense (W1) Profit for the period Profit attributable to Owners of Oldenburg plc (β) Non-controlling interest (W3)

407,664 38,740 446,404

Workings (1) Consolidation schedule Oldenburg plc

Revenue Cost of sales – per Q – PURP (W2) – PPE PURP ((567,000 – 465,500) x 20%) Op expenses – per Q – prof fees re acquisition – additional deprec on building ((22,000 ÷ 11) x 9/12) – GW impairment – Gain on BP (a) Tax

£ 2,978,500 (2,100,600)

(701,600) (8,000)

Zangersheide Ltd £ 1,759,500 (1,198,500) (23,900) 20,300 (203,500)

Westphalian Adj Ltd 9/12 £ £ 982,800 (286,800) (655,950) 286,800

Consol

£ 5,434,000

(3,671,850) (225,000) (1,500)

(18,000) 22,250 (53,000)

(107,200) 246,700

(30,000) 70,350

(1,135,350) (190,200)

(2) PURPs

Sales Cost of sales GP x½ x 30%

% 120 (100) 20

Zangersheide Ltd £ 286,800 (239,000) 47,800 23,900

Hanoverian Ltd £ 101,040 (84,200) 16,840 8,420 2,526

(3) Non-controlling interest in year Zangersheide Ltd (10% x 246,700 (W1)) Westphalian Ltd (20% x 70,350 (W1))

£ 24,670 14,070 38,740

(4) Share of profit of associate Share of PAT (61,100 x 30%) Less: Impairment PURP (W2)

Copyright © ICAEW 2015. All rights reserved

£ 18,330 (6,000) (2,526) 9,804

Page 15 of 16

Financial Accounting and Reporting – Professional Level – December 2014 Candidates generally made a good attempt at the preparation of the consolidated statement of profit or loss. Most statements were reasonably presented with most candidates gaining some marks for presentation. Candidates usually produced a consolidation schedule as part of their workings and those that did tended to gain the most marks as workings and figures were clear. The two inventory PURP figures were usually correctly calculated, although some candidates forgot that one of these needed adjusting to reflect only the associate share or that it should have been set against the share of profit of associate figure rather than against the consolidated cost of sales figure. The property, plant and equipment PURP was often correctly calculated, but then either not used or adjusted for in the wrong direction or wrong column in the consolidation schedule. It was disappointing to see just how many candidates made the very basic error of using the parent’s percentage rather than the non-controlling interest percentage when calculating the figure for noncontrolling interest in the year. However, most did use the figures from the subsidiaries’ columns in their consolidation schedule in their calculation of this figure, although some used the figures from the question without adjustment or with adjustments which failed to mirror what they had done elsewhere in their answer, thereby failing to gain the marks for this calculation. Candidates generally made a reasonable attempt at the share of profit in the associate, with mnay calculating the correct figure. Where errors were made the most common were not adjusting for the PURP, as highlighted above, or multiplying all figures by the 30% interest (including the impairment and often the PURP figure twice). The three most common errors were to omit the revised gain on bargain purchase, the adjustment for the professional fees and/or the additional depreciation on the building, even where these figures had been calculated in Part (a). Total possible marks Maximum full marks

16 14

(c) IFRS v UK GAAP differences Under UK GAAP (FRS 7) acquisition-related costs are added to the cost of the investment in the subsidiary and therefore affect the calculation of goodwill arising on consolidation. IFRS 3 recognises acquisition-related costs as an expense in profit or loss as incurred. UK GAAP (FRS 10) recognises negative goodwill as a separate item within goodwill. This is subsequently recognised in the profit and loss account in the periods in which the non-monetary assets are recovered, whether through depreciation or sale. IFRS 3 requires immediate recognition of negative goodwill (“gain on bargain purchase”) as a gain in profit or loss. Under UK GAAP (FRS 10) goodwill is amortised over its estimated useful economic life, with a rebuttable presumption that this is not more than 20 years. Under IFRS 3 goodwill is subject to annual impairment reviews. UK GAAP (FRS 9) requires the investor’s share of the associate’s operating results, exceptional items, interest, profit before tax and tax to be separately disclosed. IAS 28, Investments in Associates and Joint Ventures, merely requires the investor’s share of the profit or loss of an associate to be disclosed. Under UK GAAP (FRS 6) the non-controlling interest is always measured using the proportionate (share of net assets) method. IFRS3 allows the proportionate method or the fair value method. Answers to this part of the question were very varied, with many candidates gaining full marks and others failing to attempt this requirement at all. Answers on UK GAAP differences continue to be quite varied. Candidates need to be very careful in these requirements as many simply write something without identifying whether it is the treatment under UK GAAP or IFRS, or explain one treatment and then say this isn’t allowed under the other basis without explaining what the alternative treatment is. A minority of candidates included differences that were of no relevance to the earlier parts of the question. Total possible marks Maximum full marks

Copyright © ICAEW 2015. All rights reserved

6½ 3

Page 16 of 16

Financial Accounting and Reporting – March 2015 MARK PLAN AND EXAMINER’S COMMENTARY The mark plan set out below was that used to mark these questions. Markers are encouraged to use discretion and to award partial marks where a point was either not explained fully or made by implication. More marks are available than could be awarded for each requirement, where indicated. This allows credit to be given for a variety of valid points, which are made by candidates.

Question 1 Total marks: 30 Overall marks for this question can be analysed as follows: General comments This question presented a draft set of financial statements with some adjustments. Candidates were required to prepare the amended statement of profit or loss and statement of financial position. A number of adjustments were required to be made, including depreciation, revenue adjustments, provisions, treasury shares, a lease incentive and a prior year inventory adjustment. Part b) required candidates to explain the concepts of accruals basis of accounting and going concern, with reference to the scenario. Part c) required a discussion on the ethical issues arising from the scenario. Coghlan Ltd – Statement of financial position as at 30 September 2014 £

£

ASSETS Non-current assets Property, plant and equipment (600,000 + 138,260) (W3) Current assets Inventories Trade and other receivables Tax asset Cash and cash equivalents

738,260

98,000 125,400 65,000 1,200 289,600 1,027,860

Total assets Equity Ordinary share capital (294,500 + 85,500) Share premium Treasury shares (45,000 x £1.90) Retained earnings (W4) Equity

380,000 94,000 (85,500) 52,910 441,410

Non-current liabilities Lease incentive Current liabilities Trade and other payables Deferred income (36,000 x 3/12) Provision (W2)

Total equity and liabilities

Copyright © ICAEW 2015. All rights reserved

7,200

31,900 9,000 538,350 579,250 1,027,860

Page 1 of 16

Financial Accounting and Reporting – March 2015

Coghlan Ltd – Statement of profit or loss for the year ended 30 September 2014 £ 3,350,200 (2,744,950) 605,250 (1,418,965) (813,715) 97,800 (715,915)

Revenue (3,359,200 – (36,000 x 3/12)) Cost of sales (W1) Gross profit Administrative expenses (W1) Loss before tax Income taxation (65,000 + 32,800) Net profit for the period W1 Expenses

Administrative expenses £

Cost of sales £ Brought forward Opening inventories adj (114,550 – 79,000) Closing inventories adj (142,100 – 98,000) Provision (W2) Lease incentive (1,200 x 6) Impairment (W3) Depreciation charge (43,750 + 34,565) (W3)

2,198,050 (35,550) 44,100 538,350

1,039,700

7,200 293,750 78,315 2,744,950

1,418,965

W2 Provision Brought forward Lawsuits (50 x 350) Warranties ((65,000 x 20%) + (157,000 x 5%)) At 30 September 2014

£ £ 500,000 17,500 20,850 38,350 538,350

W3 Plant and equipment

Carrying amount at 1 Oct 2013 (1,125,000 – 187,500) / (236,000 – 63,175) Depreciation charge for the year (1,125,000 – 250,000) x 5% 172,825 x 20% Carrying amount at 30 Sept 2014 Recoverable amount Impairment

Land and buildings £ 937,500

Fixtures and fittings £ 172,825

(43,750) 893,750 600,000 293,750

(34,565) 138,260 170,000 –

W4 Retained earnings Per draft Add: draft loss Less: revised profit and loss Dividend paid (380,000 x 10p) Prior year adjustment – inventories

£ 425,825 416,550 (715,915) (38,000) (35,550) 52,910

Presentation of the statement of profit or loss and statement of financial position was generally good. As indicated as acceptable at the tutor conference, most candidates omitted sub-totals on the statement of financial position, but were penalised if they omitted totals for total assets and total equity and liabilities. A minority missed out sub-totals on the statement of profit or loss – this is not considered acceptable and marks were lost for this. However, there were a number of very messy statements, usually the statement of profit or loss, where costs workings were shown on the face of the statement instead of in a recommended “costs Copyright © ICAEW 2015. All rights reserved

Page 2 of 16

Financial Accounting and Reporting – March 2015 matrix” in the workings. Whereas in most recent sittings almost all candidates have used a costs matrix, this was not the case at this sitting. Performance on this question was good, with some high marks achieved. A significant number of candidates arrived at completely correct figures in respect of revenue, cost of sales, closing inventories and the provision. Most candidates also arrived at the correct figures for the two depreciation charges for the year, and correctly presented them in administrative expenses. However, a few candidates calculated depreciation based on the year end recoverable amounts instead of on the opening figures. It was also common to see the fixtures and fittings, which were not impaired, revalued, when no indication was given that the company wished to move to the revaluation model. Pleasingly, most candidates did provide relatively clear workings for their property, plant and equipment figure. The tax refund probably caused the most difficulties, with only a few candidates treating both this and the over-provision from the previous year correctly. A number of candidates showed only the tax refund in the statement of profit or loss, others reduced the tax refund by the over-provision from the previous year, instead of adding it. Many were so confused by the income tax position that they showed no figure for income tax at all in the statement of profit or loss. On the statement of financial position it was common to see the overprovision from the previous year reducing the tax asset. And whatever figure was arrived at this was presented more often as a “negative” current liability than (correctly) as a current asset. Other common errors included the following:  

Errors in adjusting cost of sales for the incorrect inventory valuations – most commonly getting the net adjustment in the wrong direction against the cost of sales figure from the draft financial statements, or making careless errors in the calculations. Calculating the dividend paid during the year on a figure other than the one shown in their own statement of financial position.

Total possible marks Maximum full marks

20½ 19

(b) Accrual basis The accrual basis of accounting records transactions in the period in which they occur, rather than when the cash inflow or outflow arises. Under the accrual basis an entity recognises items as assets, liabilities, equity, income and expenses when they satisfy the definition and recognition criteria for those elements in the Framework An example of this is the treatment of the revenue generated from the magazine subscriptions. These were incorrectly recorded in revenue as the cash had been received, however part of the service delivery, ie the magazines being despatched, arose after the year end and therefore part of the revenue should have been deferred. The recognition of the provisions are another example of the accrual basis, as these are present obligations arising from past events and hence have been recognised as liabilities in the current period, although the cash will be paid out in future periods. Other examples include the charging of depreciation on the property, plant and equipment recognising that the entity is generating economic benefits from these assets over their useful lives and the charging of operating lease rental over the total period of the lease. Going concern basis The going concern basis of accounting assumes that the entity will continue operating in the foreseeable future as a going concern. To operate for the foreseeable future there must be no intention by management, or the need, to liquidate the entity by selling its assets and paying its liabilities. The going concern basis affects the valuation of the company’s assets. It is assumed that non-current assets, for example, will be used in the operation of the entity and therefore the use of historical cost is considered appropriate. However, if the entity ceases in operation then the historical cost basis would no longer be Copyright © ICAEW 2015. All rights reserved

Page 3 of 16

Financial Accounting and Reporting – March 2015 appropriate and instead the assets would be valued based on their recoverable amount at that point in time, this valuation basis is known as the break-up basis. The concept of being “non-current” also would no longer be appropriate as all assets and liabilities would be “current” in nature as the entity would no longer be trading. Coghlan Ltd’s financial statements have been prepared using the going concern basis of accounting. If the break-up basis were appropriate due to the company no longer being a going concern, as a result of the adverse publicity, caused by the unsafe products, assets and liabilities might be different. For example, Coghlan Ltd has five years left on the office lease, if Coghlan Ltd ceased to trade the lease would become an onerous obligation and the full amount would need to be recognised. Coghlan Ltd traded at a large loss during the year, if this performance continues it is unlikely that the company would be a viable trading entity for long. In addition a dividend was paid, presumably to ensure shareholders remained happy, however as a result of this retained earnings and hence distributable profits are virtually zero, so no further dividends could be paid in the future without substantial profits being made. It is therefore questionable whether Coghlan Ltd will remain a going concern for much longer.

This part of the question was reasonably well answered although few candidates scored high marks. Most candidates could give a basic definition of the accruals concept, but the quality of explanation using the subscription revenue and the operating lease varied. Again, most candidates could give a basic definition of the going concern concept, and cite the break-up basis as an alternative, but less candidates went beyond this to explain how going concern financial statements differ from those prepared on a break-up basis. However, a majority of candidates made the point that Coghlan Ltd appeared to be in financial difficulties and that therefore the going concern basis may not be appropriate. Total possible marks Maximum full marks

11 6

(c) Professional accountants are expected to follow the guidance contained in the fundamental principles in all of their professional and business activities. The Code of Ethics has five fundamental principles. The financial statements should be prepared fairly, honestly and in accordance with relevant professional standards. Objectivity is one of the five fundamental principles in the ICAEW’s ethical Code, which means that I should not allow bias, conflict of interest or undue influence of others to override professional or business judgements. I should not let the managing director pressure me into completing the financial statements quickly and not making a satisfactory and thorough job. Intimidation threat exists. Professional behaviour is another principle and hence I should ensure that the relevant laws and regulations are complied with. I should ensure that I act with both professional competence and due care and therefore not be influenced by the pressure that management are putting on me. The financial statements should be prepared by someone who has the relevant expertise and that is unlikely to be someone who is undertaking work experience. I should not allow bias in any way, conflict of interest or undue influence of others override my professional judgement. It is unfair for the managing director to mention my performance appraisal and therefore I need to ensure that this does not affect any decisions I make as a self-interest threat exists. I should explain that the financial statements need additional work to the managing director and explain that they may take longer than he would have ideally liked to ensure that they provide a fair assessment of the facts. If he is unwilling to allow additional time then I should discuss the matter with the other directors and explain that I am being pressured by the managing director. I should keep a record of all discussions and I could discuss the matter confidentially with the ICAEW helpline for advice and support.

Copyright © ICAEW 2015. All rights reserved

Page 4 of 16

Financial Accounting and Reporting – March 2015

The answers to the “ethics” part were mixed, with a significant number of candidates putting themselves in the position of being the external auditor, as opposed to the financial controller, as specified in the question. Most candidates identified self-interest and possible intimidation threats, that the financial controller should uphold the values of professional competence and due care and professional behaviour, and refer continuing difficulties with the managing director to the other directors and then to the ICAEW ethics helpline. Weaker candidates missed the point that all discussions should be documented and spent some time discussing the ethics of the managing director, when we were not told whether he was an ICAEW Chartered Accountant or not. Total possible marks Maximum Marks

Copyright © ICAEW 2015. All rights reserved

8½ 5

Page 5 of 16

Financial Accounting and Reporting – March 2015

Question 2 Total marks: 36 Overall marks for this question can be analysed as follows:

General comments Part (a) of this question required candidates to explain the financial reporting treatment of four accounting issues, given in the scenario. The four issues covered borrowing costs, a compound financial instrument, an intangible asset and a joint venture. Journal entries were also required. Part (b) required candidates to recalculate consolidated profit for the year for the adjustments needed as a result of their answer to Part (a). Part (c) required a calculation of basic earnings per share following a rights issue and explanation of the accounting treatment was also required. Porcaro plc (a) (i) IFRS accounting treatment (1) Borrowing cost Under IAS 23 Borrowing costs, certain borrowing costs form part of the cost of the qualifying asset, and should therefore be capitalised. A qualifying asset is an asset which takes a substantial period of time to get ready for its intended use, or sale. The office block is therefore a qualifying asset as it is not ready for use. Borrowing costs are defined as interest and other costs that an entity incurs in connection with the borrowing of funds. Only borrowing costs that are directly attributable to the acquisition, construction or production of the qualifying asset should be capitalised. These are the borrowing costs which would have been avoided if the expenditure on the qualifying asset had not been incurred. As the loan was specifically taken out for the purpose of funding the construction of the office block use the actual interest rate of 6%. Capitalisation of borrowing costs should commence when the entity meets all three of the following conditions: (1) (2) (3)

It incurs expenditure on the asset (the payment to acquire the land was made on 1 October 2013); It incurs borrowing costs (the loan was taken out on 1 October 2013, from which date interest will start to accrue); It undertakes activities that are necessary to prepare the asset for its intended use (the land was acquired on 1 October 2013 with planning permission which was needed for construction to take place).

Borrowing costs of £36,000 (600,000 x 6%) should therefore be capitalised from 1 October 2013. Where the borrowed funds are not required immediately, so instead are put on deposit, the borrowing costs capitalised should be reduced by the investment income received on the invested funds. Investment income: (600,000 – 200,000 = 400,000) (1 Oct 2013 – 28 Feb 2014) 400,000 x 3% x 5/12 = (1 Mar – 31 Aug 2014) 300,000 x 3% x 6/12 = (1 Sept – 30 Sept 2014) 100,000 x 3% x 1/12 =

£5,000 £4,500 £250 £9,750

Total borrowing costs which should be capitalised are £26,250 (36,000 – 9,750). No depreciation should be recognised on the office block as it’s not ready for use.

Copyright © ICAEW 2015. All rights reserved

Page 6 of 16

Financial Accounting and Reporting – March 2015 The journal entries required are: DR: Property, plant and equipment (SOFP) CR: Net interest (PorL)

£ 26,250

£ 26,250

(2) Convertible bonds The convertible bonds are compound financial instruments per IAS 32 Financial Instruments: Presentation. They have both an equity and a liability component which should be presented separately at the time of issue. IAS 32 requires that the substance of such an instrument be reflected, focusing on the economic reality that in effect two financial instruments have been issued. The liability component should be measured first at the present value of the capital and interest payments. The discount rate used should be the effective rate for an instrument with the same terms and conditions except without the ability to convert it into shares.

1 October 2014 1 October 2015 1 October 2016 1 October 2017 (redemption) Liability component Equity component (bal fig) Total

Cash flow £ 30,000 30,000 30,000 630,000

Discount factor @ 7% 1/1.07 2 1/1.07 3 1/1.07 4 1/1.07

Present value £ 28,037 26,203 24,489 480,624 559,353 40,647 600,000

The liability should initially be measured at £559,353 and the equity component is the residual at £40,647. Once recognised the equity element remains unchanged. However, the liability element should be shown at amortised cost at the end of each year: 1 Oct 2013 £ 559,353

Interest (7%) £ 39,155

Payment (5%) £ (30,000)

30 Sept 2014 £ 568,508

At the year an adjustment should be made to non-current liabilities of £31,492 (600,000 – 568,508), and an additional £9,155 recognised as finance costs as part of profit or loss. The journal entries required are: DR: Non-current liabilities (SOFP) DR: Finance costs (PorL) CR: Equity (SOFP)

£ 31,492 9,155

£

40,647

(3) Intangible asset – licence The licence should be recognised as an intangible asset as it is an identifiable non-monetary asset without physical substance. The licence is identifiable as it arises from contractual or legal rights to use the microchip technology. The licence should initially be recognised at its cost of £72,000. Amortisation of £6,000 ((72,000 / 6yrs) x 6/12) should be recognised as part of profit or loss. The carrying amount of the licence at 30 September 2014 under historical cost accounting is £66,000 (72,000 – £6,000). The licence can continue to be held at cost or may be revalued if the directors can show that an active market exists for it. Although a competitor has offered to buy the licence which suggests that an active market exists, part of the definition also requires the items traded to be homogenous. As it states that the licence is unique it is unlikely that it will meet this definition and therefore should be held at historical cost. The revaluation gain of £18,000 (£90,000 - £72,000) at 30 September 2014 should be reversed.

Copyright © ICAEW 2015. All rights reserved

Page 7 of 16

Financial Accounting and Reporting – March 2015 The journal entries required are: DR: Equity – Revaluation surplus (SOFP) DR: Amortisation (PorL) CR: Non-current assets (SOFP) (18,000 + 6,000)

£ 18,000 6,000

£

24,000

(4) Joint venture Porcaro plc should recognise its investment in Barbarossa Ltd as a joint venture. Four companies have joint control over Barbarossa Ltd and there is a contractual arrangement in place to share profits and losses equally. IFRS 11 Joint Arrangements requires the use of the equity method for joint ventures. The investment should therefore be recognised at cost of £25,000 plus the share of the joint venture’s post acquisition increase in net assets, £32,500 (£130,000 x 25%). The investment in Barbarossa Ltd will be shown as a non-current asset, rather than a current asset in the consolidated statement of financial position, so the £25,000 will need to be reclassified. The share of postacquisition profit of £32,500 should be added to non-current assets, giving a carrying amount of £57,500 and the £32,500 recognised in consolidated profit or loss. The journal entries required are: DR: Non-current assets (SOFP) CR: Current assets (SOFP) CR: Share of joint venture profit (PorL)

£ 57,500

£ 25,000 32,500

Most candidates produced reasonably detailed narrative explanations, melded together with calculations although less went on to produce journal entries. Only the very weakest candidates restricted their answers to predominantly calculations, with little explanation. Answers to Issues (1), (2) and (4) were all reasonably well attempted, with Issue (3) causing some difficulties. Borrowing costs Most candidates set out the appropriate terminology, such as “directly attributable” and “qualifying asset”, and correctly concluded that the office block was a qualifying asset and that interest on the loan should be capitalised. However, a significant number of candidates were careless in their choice of words and stated that borrowing costs “could” be capitalised – implying a choice in the matter (even when in Part (d) they went on to clearly state that under IFRS borrowing costs must be capitalised). Most then listed the IAS 23 criteria for the commencement of capitalisation, but few applied these criteria to this scenario. Of those that did, many concluded, in error, that capitalisation could not commence until 31 December 2013, and hence only capitalised nine months of the annual interest. Almost all candidates stated that the borrowing costs should be reduced by the investment income on surplus funds. Calculations for the investment income often contained errors generally around the number of months. The 6% actual interest rate was used, although only a very small minority explained why this was appropriate. Almost all candidates then set out the correct journal entry for their net figure. Convertible bonds The majority of candidates explained that this was a compound financial instrument and that split accounting was appropriate, with fewer mentioning substance over form. Most of these candidates then produced correct calculations for the split of debt and equity and for the amortised cost of the debt, although less referred to “amortised cost” in their explanation. Journal entries were largely correct, although some candidates took a rather convoluted approach to arriving at the correct net journal. Intangible asset – licence This issue caused the most problems. Most candidates gave some basic definitions and calculated the initial carrying amount of the intangible at cost (although some used the incorrect number of months for the amortisation charge). Answers were then mixed, depending on whether candidates realised that the information in the scenario did not support the existence of an “active market”. Those that saw this quickly concluded their answer by reversing out the revaluation. The ones that did not then wasted time calculating

Copyright © ICAEW 2015. All rights reserved

Page 8 of 16

Financial Accounting and Reporting – March 2015 additional amortisation charges, and sometimes also transfers between the revaluation surplus and retained earnings. Others hedged their bets and set out both accounting treatments without a conclusion, which was time consuming. Joint venture There was a lot of confusion to this issue and candidates seem to struggle between the concept of an associate and a joint venture, with many candidates simply believing they are the same instrument. Although the majority of candidates identified that equity accounting should be applied and recognised the cost correctly, candidates often described the investment as an associate. Journal entries were usually correct, with the most common error being to credit cash instead of current assets. The only real error seen in the calculations was taking the appropriate share of only a fraction of the profit after tax, instead of the appropriate share of the whole figure, which was stated to be the profit for that period. Total possible marks Maximum full marks

36 27

(b) Porcaro plc – Group figures Profit for the year £ As stated Issue (1) Issue (2) Issue (3) Issue (4) Profit adjustment

£ 483,150

26,250 (9,155) (6,000) 32,500 43,595 526,745

Most candidates appeared to adopt the recommended approach of setting up a schedule as the first page of their answer starting with the draft profit from the question, and adjusted this as they wrote their explanation for each issue. Many candidates did therefore score the full two marks for this part, based sometimes on completely correct and sometimes on their “own” figures. Only the very weakest candidates failed to attempt this part of the question. Where marks were lost it was generally where candidates failed to replicate in this part the journal entries set out in their answers to Part (a). Total possible marks Maximum full marks

Copyright © ICAEW 2015. All rights reserved

2 2

Page 9 of 16

Financial Accounting and Reporting – March 2015 (c) Porcaro plc

1 Oct 2013 – 31 Jan 2014 Rights issue 1 for 3 1 Feb – 30 Sept 2014 Theoretical ex-rights price: 3 shares @ £2.10 1 share @ £1.70

No. Of shares 270,000 90,000 360,000

Period in issue 4/12 8/12

Bonus factor 210/200

Weighted average 94,500 240,000 334,500

£ 6.30 1.70 8.00

Theoretical ex-rights price per share £8.00 / 4 = £2.00 Bonus fraction: 210 / 200 Basic EPS = 526,745 = £1.57 334,500 A rights issue is an issue of shares to current shareholders in proportion to their existing holdings at a discount to market price. Because the share issue is below market price, a rights issue is in effect a combination of an issue at full market value and a bonus issue. An adjustment therefore needs to be made to the earnings per share for the bonus element. This is calculated by comparing the pre-rights market value with the theoretical ex-rights price. The theoretical price is the price at which the shares would have traded after the rights issue in theory.

A good number of candidates arrived at the correct weighted average number of shares, and produced an EPS based on that and their own figure for revised profit for the year. However calculations often contained errors in the theoretical ex-rights price per share. Only the very best candidates could explain clearly why the rights issue had been scaled up by a bonus fraction, and many of these candidates achieved full marks for this part of the question. Weaker candidates merely described in words what they had done in their calculation. A minority of candidates described the accounting entries for the rights issue which gained no marks. Total possible marks Maximum full marks

7½ 6

(d) UK GAAP differences Borrowing costs Under UK GAAP Porcaro plc has the choice whether to capitalise borrowing costs. If a policy of capitalisation is chosen then this policy should be applied to the class of qualifying assets. Under IFRS borrowing costs which meet the definition of being directly attributable to the acquisition, construction or production of a qualifying asset must be capitalised.

Most candidates achieved the full one mark for this part, clearly stating that capitalisation is mandatory under IFRS, but optional under UK GAAP. Only the weakest candidates got this the wrong way round, or failed to give both the IFRS and UK GAAP treatments. Total possible marks Maximum full marks

Copyright © ICAEW 2015. All rights reserved

1½ 1

Page 10 of 16

Financial Accounting and Reporting – March 2015

Question 3 Total marks: 11 Overall marks for this question can be analysed as follows:

General comments This question was a mixed topic question, covering the completion of extracts from the statement of cash flows for adjustments to investing and financing activities. Part b) required the preparation of an extract from the consolidated statement of financial position, showing non-current and current assets. Henrit plc (a) Consolidated statement of cash flows (extract) Cash flows from investing activities Purchase of property, plant and equipment (W2) Proceeds from sale of property, plant and equipment (124,000 + 9,500) Cash flows from financing activities Payment of finance lease (15,000 – 7,375) (W3) Proceeds from issue of loan (450,000 – 290,000)

£ (365,450) 133,500

(7,625) 160,000

Workings (1) Interest £ 7,250 11,250 18,500

290,000 x 5% x 6/12 450,000 x 5% x 6/12 (2) PPE B/d Additions – finance lease (W3) Additions – cash (β)

£ 729,400 105,350 365,450 1,200,200

Disposals Depreciation C/d

£ 124,000 113,000 963,200 1,200,200

(3) Finance lease Cash C/d

£ 15,000 97,725 112,725

B/d PPE addition (β) Interest (25,875 – 18,500 (W1))

£ – 105,350 7,375 112,725

Answers to this requirement were quite mixed, with a significant number of candidates achieving full marks. Most candidates successfully calculated the proceeds from the disposal of equipment and also attempted to produce a T-account for property, plant and equipment to identify the cost of additions. Within this working nearly all candidates correctly credited the depreciation charge for the year and the carrying amount of the equipment that had been sold. The majority of candidates also realised that they needed to debit the account with plant acquired under a finance lease but very few candidates calculated this figure correctly. Most simply used the closing balance on the finance lease account given in the question. It was clear that the majority of candidates either do not understand that payments under finance leases need to be split between interest and capital or cannot calculate the split. Many candidates merged the finance lease liability and the bank loan and as a result lost the easy mark available for showing the inflow of cash relating to the bank loan. Some candidates used the information given in the question to calculate the interest relating to the bank loan but then made no use of this information. Copyright © ICAEW 2015. All rights reserved

Page 11 of 16

Financial Accounting and Reporting – March 2015

With regards to presentation nearly all candidates did produce extracts as required and also entered figures under the appropriate headings, although totals were often not seen. As is always the case with questions on the statement of cash flows a significant number of candidates lost marks for failing to put brackets around outflows of cash. Total possible marks Maximum full marks

8½ 6

(b) Statement of financial position at 30 September 2014 (extract) Non-current assets Property, plant and equipment (963,200 + 469,400 + 623,150 – 4,400 (W2)) Goodwill (73,400 + 17,750 (W1)) Current assets Inventory (46,980 + 18,900 + 31,300 – 1,500 (W3))

2,051,350 91,150

95,680

Workings (1) Goodwill – Crago Ltd £ 371,750 261,000 (615,000) 17,750

Consideration transferred (230,000 + (45,000 x 3.15)) Non-controlling interest at acquisition at fair value Less: Net assets at acquisition

(2) Inter-company machine transfer £ 38,000 (42,400) 4,400

Original carrying amount (95,000 – (95,000 x 3/5)) Consideration less depreciation (53,000 – (53,000 x 6/30)) Unrealised profit (3) PURP SP Cost GP

% 115 (100) 15

£ 11,500 (10,000) 1,500

Generally this was well answered with many candidates achieving full marks. A majority of candidates correctly calculated goodwill and the PURP relating to inventory and made the relevant adjustments to the figures given in the question. A minority of candidates used the nominal rather than the market value of the shares to calculate the consideration for the acquisition of the subsidiary and a similar number calculated the PURP using gross margin rather than a mark-up on cost. However only a small minority of candidates correctly calculated the PURP relating to the sale of the machine. Common errors were to calculate the profit on disposal or the difference in the subsequent depreciation and therefore only adjust for part of the difference. As with part (a) nearly all candidates produced extracts but again a number failed to add numbers across so could not be given full credit for presentation. Total possible marks Maximum full marks

Copyright © ICAEW 2015. All rights reserved

6 5

Page 12 of 16

Financial Accounting and Reporting – March 2015

Question 4 Total marks: 23 Overall marks for this question can be analysed as follows: This question required the preparation of a consolidated statement of profit or loss and extracts from the consolidated statement of changes in equity (for retained earnings). The group had two subsidiaries, one of which was disposed of during the year. A fair value adjustment was required on acquisition of one of the companies. Inter-company trading took place during the year between one of the subsidiary’s and the parent. Part (b) required candidates to describe the UK GAAP differences for the acquisition and disposal of a subsidiary. Mantia plc (i) Consolidated statement of profit or loss for the year ended 30 September 2014 £ Continuing operations Revenue (W1) Cost of sales (W1) Gross profit Operating expenses (W1) Profit from operations (W1) Investment income (W1) Profit before tax Income tax expense (W1) Profit for the year from continuing operations Discontinued operations Profit for the year from discontinued operations (300,100 (W2) – 32,715 (W4)) Profit for the period Profit attributable to Owners of Mantia plc (β) Non-controlling interest (W2)

3,722,000 (1,658,500) 2,063,500 (536,055) 1,527,445 17,000 1,544,445 (327,000) 1,217,445 267,385 1,484,830

1,327,451 157,379 1,484,830

(ii) Consolidated statement of changes in equity for the year ended 30 September 2014 (extract) Retained earnings £ Balance at 1 October 2013 (W6) Total comprehensive income for the year Dividends (W6) Balance at 30 September 2014 (β)

Copyright © ICAEW 2015. All rights reserved

227,249 1,327,451 (600,000) 954,700

Page 13 of 16

Financial Accounting and Reporting – March 2015 Workings (1) Consolidation schedule

Revenue Cost of sales – per Q – PURP (W5) Op expenses – per Q – FV deprec (70,000/10yrs) – Impairment of goodwill

Mantia plc £ 2,986,000

Appice Ltd £ 768,000

Adj £ (32,000)

Consol £ 3,722,000

(1,343,700)

(345,600) (1,200)

32,000

(1,658,500)

(419,575)

(84,480) (7,000)

(25,000)

Investment income – Appice (80,000 x 40p x 80%) Tax

(536,055)

42,600 (25,600) (259,000)

(68,000) 261,720

17,000 (327,000)

(2) Non-controlling interest in year £ 52,344 105,035 157,379

Appice Ltd (20% x 261,720 (W1)) Starkey Ltd (35% x 300,100 (600,200 x 6/12)) (3) Goodwill – Starkey Ltd

£ 230,000 105,700 335,700

Consideration transferred Non-controlling interest at acquisition (302,000 x 35%) Less: Net assets at acquisition Share capital (91,000 / 65%) Retained earnings

140,000 162,000 (302,000) 33,700 (18,000) 15,700

Goodwill Impairment brought forward Goodwill at date of disposal (4) Group profit/loss on disposal of Starkey Ltd

£ 427,000 (15,700)

Sale proceeds Less: carrying amount of goodwill at disposal (W3) Carrying amount of net assets at disposal Share capital Retained earnings (243,000 + (600,200 x 6/12))

140,000 543,100 (683,100) 239,085 (32,715)

Add back: Attributable to non-controlling interest (683,100 x 35%) Loss on disposal (5) PURP SP Cost GP 1 X /4

Copyright © ICAEW 2015. All rights reserved

% 100 (85) 15

£ 32,000 (27,200) 4,800 1,200

Page 14 of 16

Financial Accounting and Reporting – March 2015 (6) Retained earnings brought forward £ Mantia plc (596,300 – 1,006,325) Add back dividend (500,000 x £1.20) Appice Ltd – post acquisition change in net assets C/fwd retained earnings Less: retained earnings at acquisition Less: profit for the period Add back dividend (80,000 x 40p) Less: FV adjustment (70,000 / 10yrs) Appice Ltd – 3,280 x 80% Starkey Ltd – post acquisition ((243,000 – 162,000) x 65%) Less: impairment – Starkey Ltd

£ (410,025) 600,000

384,200 (136,000) (269,920) 32,000 (7,000) 3,280 2,624 52,650 (18,000) 227,249

Retained earnings carried forward (for proof only) Mantia plc Appice Ltd – post acquisition (384,200 – 136,000 – 14,000 – 1,200) x 80% Less: impairment – Appice Ltd Profit on disposal of investment in Starkey Ltd (427,000 – 230,000)

£ 596,300 186,400 (25,000) 197,000 954,700

Most candidates made a good attempt at preparing the consolidation schedule and correctly excluded the subsidiary held for sale. Many dealt with the relevant adjustments correctly obtaining all the available marks for this part of the question. Where candidates did make errors it was normally for the following:    

deducting the inventory PURP from revenue rather than adding it to cost of sales or adding it to the cost of sales of the purchasing rather than the selling company. calculating the cumulative adjustment to depreciation arising from the fair value adjustment rather than just the current year adjustment and/or entering this into the parent company rather than the subsidiary’s column. adjusting the subsidiary’s profits for the goodwill impairment. deducting 100% of the subsidiary’s dividend from investment income rather than just the parent company’s share of the dividend.

Virtually all candidates attempted to calculate the profit on disposal and a reasonable number arrived at the correct figure. One common error was using the incorrect share capital figure (the shares bought by the parent company rather than total share capital) or ignoring share capital altogether when calculating net assets. Other errors included:   

failing to deduct the impairment from goodwill (many candidates deducted this from the profit on disposal instead). failing to add 6/12 of current year profit to brought forward retained earnings or deducting it rather than adding it. using retained earnings at acquisition rather than at the date of disposal when calculating net assets at disposal.

A number of candidates produced very disorganised workings for their retained earnings calculation and it was often difficult to understand where numbers had come from and whether they were increasing or decreasing the profit on disposal. Candidates are strongly advised to use the standard pro-forma given in the Learning Material to calculate this figure and label workings appropriately. Most candidates did prepare a consolidated statement of profit or loss and showed a separate figure for the profit from discontinued operations. However this figure often ignored the profit up to disposal or just took the parent company’s share of that profit. Candidates should note that if they only produce the consolidation schedule they will not get the presentation marks available for this statement. As expected the extract to the consolidated statement of changes in equity was not as well dealt with. Most candidates who attempted this statement did insert the “easy” figures ie the profit for the period and the dividends paid. However errors were frequently made even with these figures by taking total profit for the period rather than just the profit attributable to the owners of the parent company and/or also including the Copyright © ICAEW 2015. All rights reserved

Page 15 of 16

Financial Accounting and Reporting – March 2015 subsidiary’s dividend as a deduction from retained earnings. Some candidates also showed dividends as an addition rather than a deduction to retained earnings. Relatively few candidates attempted to calculate retained earnings b/fwd or c/fwd. Where they did, workings were again often confused and difficult to follow. Few candidates appear to understand that they should take the same approach to calculate consolidated retained earnings as they do to calculate the consolidated retained earnings figure for consolidated statement of financial position questions. Total possible marks Maximum full marks

21½ 20

(b) UK GAAP differences Acquisition of Starkey Ltd The calculation for goodwill is the same under UK GAAP as per IFRS, however under IFRS the parent entity has a choice whether to measure the non-controlling interest at fair value or at the proportion of net assets. Under UK GAAP only the proportion of net assets method is permitted. UK GAAP requires goodwill to be amortised over its useful life and there is a rebuttable presumption that this should not exceed five years. Under IFRS amortisation is not permitted and instead annual impairment reviews take place. Disposal of Starkey Ltd UK GAAP requires that a detailed analysis of discontinued operations should be shown on the face of the profit and loss account. However, IFRS only requires a single line to be shown on the face of the statement of profit or loss.

The majority of candidates made a good attempt at this part of the question with many achieving full marks. However a significant number of candidates wasted time by including differences that were not relevant to the scenario such as the treatment of a discount on acquisition. A common misunderstanding is that under UK GAAP goodwill must be amortised over five years rather than it being a maximum useful life. Total possible marks Maximum full marks

Copyright © ICAEW 2015. All rights reserved

3½ 3

Page 16 of 16

View more...

Comments

Copyright ©2017 KUPDF Inc.
SUPPORT KUPDF